Sei sulla pagina 1di 517

LECTURE NOTES

ON

HEAT & MASS TRANSFER


(13 ME 354)
BY

DR. T.R.SEETHARAM
(Chair Professor in Thermal Engineering
PESIT, Bangalore 85)

(Revised for the Academic Year 2019-20)

1
11 ME 354: HEAT TRANSFER (4 – 0 – 0 −4)

Course Objectives: In this course the student will be trained to:

1. Understand the basic laws of heat transfer.


2. Account for the consequence of heat transfer in thermal analyses of engineering systems.
3. Analyze problems involving steady state heat conduction in simple geometries.
4. Develop solutions for transient heat conduction in simple geometries.
5. Obtain numerical solutions for conduction heat transfer problems.
6. Understand the fundamentals of convective heat transfer process.
7. Evaluate heat transfer coefficients for natural convection.
8. Evaluate heat transfer coefficients for forced convection inside ducts.
9. Evaluate heat transfer coefficients for forced convection over exterior surfaces.
10. Apply integral method to obtain velocity and temperature profiles for laminar flow over
a flat plate and hence the drag and heat transfer coefficients
11. Analyze heat exchanger performance by using the method of log mean temperature
difference.
12. Analyze heat exchanger performance by using the method of heat exchanger
effectiveness.
13.Determine the condensation heat transfer for filmwise condensation on vertical and
horizontal surfaces
14. Undestand the phenomenon of pool boiling and determine the heat transfer coefficients
for different regimes of pool boiling.

Learning Outcomes:

Upon successful completion of this course, the student will be able to:

1. Understand and apply the basic laws of heat transfer for systems invoving heat transfer.
2. Account for the consequence of heat transfer in thermal analyses of engineering systems.
3. Analyze problems involving steady state heat conduction in simple geometries.
4. Develop solutions for transient heat conduction in simple geometries.
5. Obtain numerical solutions for conduction heat transfer problems.
6. Understand the fundamentals of convective heat transfer process.
7. Evaluate heat transfer coefficients for natural convection.
8. Evaluate heat transfer coefficients for forced convection inside ducts.
9. Evaluate heat transfer coefficients for forced convection over exterior surfaces.
10. Apply integral method to obtain velocity and temperature profiles for laminar flow over a
flat plate and hence the drag and heat transfer coefficients
11. Analyze heat exchanger performance by using the method of log mean temperature
difference.
12. Analyze heat exchanger performance by using the method of heat exchanger
effectiveness.

2
Course Contents

UNIT – I 12 Hours

Chapter 1: Mechanisms of Heat Transfer- Basic laws governing each mechanism; combined
mechanisms; Illustrative examples
Chapter 2: Conduction Basic Equations and Boundary Conditions: Conduction Basic Equations:
one dimensional conduction equation in rectangular, cylindrical and spherical coordinates; thermal
diffusivity;3-dimensional conduction equation in Cartesian coordinates; boundary conditions of first,
second and third kind; radiation boundary condition ;illustrative examples on formulation of conduction
problems.
Chapter 3: One Dimensional Steady State Conduction: Analysis of one dimensional steady state
conduction in slab, radial conduction in cylinders and spheres without and with heat generation; one-
dimensional steady state conduction in composite medium- concept of thermal potential and thermal
resistance; contact resistance; critical thickness of insulation for cylinders and spheres; analysis of fins
of uniform cross section with different tip conditions; one-dimensional steady state conduction in solids
with variable thermal conductivity. Illustrative examples

UNIT – II 10 Hours

Chapter 4 : Transient Conduction: Lumped system analysis with illustrative examples; Criterion for
neglecting internal temperature gradients in transient conduction analysis ;One dimensional transient
conduction in a slab subjected to convective boundary condition- solution of this problem in the form
of Transient-Temperature Chart. Similar charts for radial transient conduction in an infinite cylinder
and in a sphere; illustrative examples; Use of these charts to solve multi-dimensional transient
conduction problems-illustrative examples. Transient temperature charts for semi-infinite solids;
Conduction shape factor: Illustrative examples
Chapter 5: Finite Difference Methods for Conduction: Finite Difference equations for one
dimensional steady state conduction in slabs, cylinders and spheres; Finite difference equations for two
dimensional steady state conduction; Explicit finite difference equations for one dimensional transient
conduction in slabs, cylinders and spheres; Implicit Scheme for one dimensional transient conduction;
Illustrative examples.

UNIT – III 12 Hours

Chapter 6: Convection-Basic Concepts: Basic concepts for flow over bodies-Velocity boundary
layer, thermal boundary layer, drag coefficient, general expression for heat transfer coefficient in terms
of temperature gradient; illustrative examples. Concepts for flow through duct- hydraulic diameter;
hydrodynamically developing and developed flow; thermally developing and thermally developed
flow; general expression for pressure drop and heat transfer coefficient for flow through ducts;
illustrative examples; Concept of turbulence- Prandtl’s mixing length theory; velocity distribution in
turbulent flow through tubes; Dimensionless Parameters in Forced Convection Flow and their physical
significance.
Chapter 7: Forced convection for flow inside ducts: Analysis of hydro dynamically and thermally
developed laminar flow-COUETTE flow, flow inside a circular tube; expressions for friction factor and
pressure drop for hydro dynamically and thermally developed laminar and turbulent flows;

3
hydrodynamic and thermal entrance lengths; use of correlations to determine pressure drop and heat
transfer coefficient for hydro dynamically and thermally developed flow through tubes; illustrative
examples.
Chapter 8: Forced convection for flow over bodies: Integral method of analysis for laminar
incompressible boundary layer over a flat plate; Correlations for drag coefficient and heat transfer
coefficient for flow over a flat plate, flow across a cylinder, flow across a sphere and flow across tube
bundles

UNIT – IV 08 Hours

Chapter 9: Free convection :- Dimensionless parameters for free convective heat transfer problems;
Correlations for free convection from vertical plane surfaces, vertical cylinders, horizontal and inclined
plane surfaces, horizontal cylinders, enclosed spaces; combined forced and free convection; illustrative
examples
Chapter 10: Boiling & Condensation:- Nusselt’s theory for laminar film condensation on a vertical
plane surface; correlations for determining heat transfer coefficient for laminar and turbulent film
condensation on a plane vertical surface and horizontal tubes. Illustrative examples..

UNIT – V 10 Hours

Chapter 11: Heat Exchangers: Classification of heat exchangers; overall heat transfer coefficient;
expressions for mean temperature difference for parallel flow, counter flow heat exchangers; correction
factors for other type of heat exchangers; Limitations of LMTD method ;Effectiveness-NTU method
for heat exchanger analysis; illustrative examples
Chapter 12: Radiation Heat Transfer Among Surfaces in a Non-Participating Medium: Basic
concepts and terms used in radiation heat exchange analysis; Planck’s law,Stefan-Boltzman law,
Wein’s displacement Law,Kirchoff’s law and Lambert’s Law; Radiation heat exchanger between two
parallel infinite black and gray surfaces; Radiation exchange between two finite surfaces – Concept of
View Factor; View factor algebra; Hottel’s Cross string formula; Net work method for analysis of
radiation heat exchange in an enclosure.

Text Book :
“Fundamentals of Heat & Mass Transfer”, F.P.Incropera, D.P.Dewitt, T.L.Bergman ,A.S.Lavine,
K.N.Seetharamu, T.R.Seetharam,Wiley IndiaPvt.Ltd,New Delhi, Sixth Edition, 2013

Reference Books:
1. “ Heat Transfer – A basic Approach”,M.Necati Ozisik, McGraw Hill International Edition, 1985
2. ”Principles of Heat Transfer”, Frank Kreith & Mark S. Bohn , Sixth Edition, Thomson Learning,
2001

4
HEAT TRANSFER
Subject code: 13ME354 No. of Hours: 26

Chapter Title/ % Portions Covered


Class Portions to be Covered
Reference Literature
Classes Cumulative
Chapter 1 UNIT-1
Introduction Mechanisms of Heat Transfer - Basic
1-2 T1: Page 1-12 laws governing each mechanism; 4 4
Faculty: F1 combined mechanisms; Illustrative
examples
Chapter 2 Conduction Basic Equations : one
Introduction to dimensional conduction equation in
3-4 Conduction rectangular, cylindrical and spherical
4 8
T1: Page 47-59 coordinates; thermal diffusivity; 3-
Faculty: F1 dimensional conduction equation in
Cartesian coordinates
Chapter 2 Boundary conditions of first, second and
Introduction to third kind; radiation boundary condition;
5-6 Conduction illustrative examples on formulation of 4 10
T1: Page 66-70 conduction problems.
Faculty: F1
Chapter 3 One Dimensional Steady State
One Dimensional Conduction in a slab, radial conduction in
Steady State cylinder and sphere with and without heat
7 2 12
Conduction generation
T1: Page 78-121
Faculty: F1
Chapter 3 Concept of thermal resistance,
One Dimensional conduction in composite medium and
Steady State overall heat transfer coefficient
8-9 4 16
Conduction
T1: Page 78-94
Faculty: F1

5
Chapter Title/ % Portions Covered
Class Portions to be Covered
Reference Literature
Classes Cumulative
Chapter 3 Governing differential equations for one
One Dimensional dimensional conduction in fins; solution
Steady State to this equation for different tip
10-11 4 20
Conduction conditions; fin efficiency and fin
T1: Page 121-145 effectiveness
Faculty: F1
Chapter 3 One dimensional steady state conduction
One Dimensional in slabs, cylinders and spheres with
Steady State variable thermal conductivity
12 2 22
Conduction
T1: Page 78-121
Faculty: F1
Chapter 5 UNIT-2
Transient Lumped system analysis with illustrative
Conduction examples; Criterion for neglecting 2 26
13 T1: Page 250-256 internal temperature gradients in transient
Faculty: F1 conduction analysis
Chapter 5 One dimensional transient conduction in
Transient a slab subjected to convective boundary
Conduction condition- solution of this problem in the
14-15 T1: Page 265-277 form of Transient-Temperature Chart. 4 30
Faculty: F1 Similar charts for radial transient
conduction in an infinite cylinder and in a
sphere
Chapter 5 Use of charts to solve multi-dimensional
Transient transient problems; semi infinite solids
16 Conduction 2 32
T1: Page 277-283
Faculty: F1
Chapter 5 Finite Difference equations for one
Transient dimensional steady state conduction in
17-18 Conduction slabs, cylinders and spheres; Finite 4 36
T1: Page 294-310 difference equations for two dimensional
Faculty: F1 steady state conduction

6
Chapter Title/ % Portions Covered
Class Portions to be Covered
Reference Literature
Classes Cumulative
Chapter 5 Explicit finite difference equations for
Transient one dimensional transient conduction in
19 Conduction slabs, cylinders and spheres; Implicit 2 38
T1: Page 294-310 Scheme (Crank-Nicholson Scheme) for
Faculty: F1 one dimensional transient conduction
Chapter 5 Illustrative examples
Transient
20 Conduction 2 40
T1: Page 294-310
Faculty: F1
Chapter 12 UNIT-5
Radiation: Basic concepts and terms used in
21-22 Processes & radiation heat exchange analysis;
properties Planck’s law, Stefan-Boltzman law,
4 44
T1: Page 669-708 Wein’s displacement Law, Kirchoff’s law
Faculty: F1 and Lambert’s Law; Radiation heat
exchange between two parallel infinite
black and gray surfaces
Chapter 13 Radiation exchange between two finite
Radiation exchange surfaces – Concept of View Factor; View
23-24 between surfaces factor algebra; Hottel’s Cross string 4 48
T1: Page 739-750 formula
Faculty: F1
Chapter 13 Network method for analysis of radiation
Radiation exchange heat exchange in two and three zone
25-26 between surfaces enclosures. 2 50
T1: Page 750-764
Faculty: F1
Chapter 6 UNIT-3
Introduction to Basic concepts for flow over bodies-
Convection Velocity boundary layer, thermal
T1: Page 332-343 boundary layer, drag coefficient, general
1 expression for heat transfer coefficient in 2 2
Faculty: F2 terms of temperature gradient; illustrative
examples. Dimensionless Parameters in
Forced Convection Flow and their
physical significance.

7
Chapter Title/ % Portions Covered
Class Portions to be Covered
Reference Literature
Classes Cumulative
Chapter 6 Concepts for flow through duct -
Introduction to hydraulic diameter; hydrodynamically
Convection developing and developed flow;
2 T1: Page 332-343 thermally developing and thermally
2 4
Faculty: F2 developed flow; general expression for
pressure drop and heat transfer coefficient
for flow through ducts; illustrative
examples
Chapter 8 Analysis of hydro dynamically and
Internal flow thermally developed laminar flow -
3 2 6
T1: Page 447-453 COUETTE flow
Faculty: F2
Chapter 8 Flow inside a circular tube; expressions
Internal flow for friction factor and pressure drop for
4-5 T1: Page 447-453 hydro dynamically and thermally
4 10
Faculty: F2 developed laminar and turbulent flows;
hydrodynamic and thermal entrance
lengths
Chapter 8 Use of correlations to determine pressure
Internal flow drop and heat transfer coefficient for
6-7 T1: Page 453-479 hydro dynamically and thermally 4 14
Faculty: F2 developed flow through tubes; illustrative
examples.
Chapter 6 Integral method of analysis forlaminar
Introduction to incompressible boundary layer over a flat
Convection plate
8-9 T1: Page 348-358 4 18
Faculty: F2

Chapter 7 Correlations for drag coefficient and heat


External flow transfer coefficient for flow over a flat
10 T1: Page 380-392 plate 2 20
Faculty: F2

8
Chapter Title/ % Portions Covered
Class Portions to be Covered
Reference Literature
Classes Cumulative
Chapter 7 Flow across a cylinder, flow across tube
External flow bundles
11-12 4 24
T1: Page 400-424
Faculty: F2
Chapter 9 UNIT-4
Free convection Dimensionless parameters for free
13-14 T1: Page 510-531 convective heat transfer problems;
28
Faculty: F2 Correlations for free convection from 4
vertical plane surfaces, vertical cylinders,
horizontal cylinders
Chapter 9 Horizontal and inclined plane surfaces;
Free convection combined forced and free convection;
15-16 4 32
T1: Page 510-531 illustrative examples
Faculty: F2
Chapter 10 Nusselt’s theory for laminar film
Boiling & condensation on a vertical plane surface;
Condensation correlations for determining heat transfer
17-18 T1: Page 581-590 coefficient for laminar and turbulent film 4 36
Faculty: F2 condensation on a plane vertical surface
and horizontal tubes. Illustrative
examples.
Chapter 10 Different regimes of pool boiling;
Boiling & Correlations for pool boiling heat transfer
19-20 Condensation 4 40
T1: Page 562-577
Faculty: F2
Chapter 11 UNIT-5
Heat Exchangers Classification of heat exchangers; overall
21 2 42
T1: Page 603-608 heat transfer coefficient
Faculty: F2
Chapter 11 Expressions for mean temperature
Heat Exchangers difference for parallel flow, counter flow
22-23 4 46
T1: Page 608-619 heat exchangers; correction factors for
Faculty: F2 other type of heat exchangers

9
Chapter Title/ % Portions Covered
Class Portions to be Covered
Reference Literature
Classes Cumulative
Chapter 11 Limitations of LMTD method;
Heat Exchangers Effectiveness-NTU method for heat
24-25 4 50
T1: Page 619-630 exchanger analysis; illustrative examples
Faculty: F2

10
CHAPTER 1

INTRODUCTORY CONCEPTS AND BASIC LAWS OF


HEAT TRANSFER
1.1. Introduction:- We recall from our knowledge of thermodynamics that heat is a form
of energy transfer that takes place from a region of higher temperature to a region of lower
temperature solely due to the temperature difference between the two regions. With the
knowledge of thermodynamics we can determine the amount of heat transfer for any system
undergoing any process from one equilibrium state to another. Thus the thermodynamics
knowledge will tell us only how much heat must be transferred to achieve a specified change
of state of the system. But in practice we are more interested in knowing the rate of heat transfer
(i.e. heat transfer per unit time) rather than the amount. This knowledge of rate of heat transfer
is necessary for a design engineer to design all types of heat transfer equipments like boilers,
condensers, furnaces, cooling towers, dryers etc.The subject of heat transfer deals with the
determination of the rate of heat transfer to or from a heat exchange equipment and also the
temperature at any location in the device at any instant of time.
The basic requirement for heat transfer is the presence of a “temperature
difference”. The temperature difference is the driving force for heat transfer, just as the voltage
difference for electric current flow and pressure difference for fluid flow. One of the
parameters ,on which the rate of heat transfer in a certain direction depends, is the magnitude
of the temperature gradient in that direction. The larger the gradient higher will be the rate of
heat transfer.

1.2. Heat Transfer Mechanisms:- There are three mechanisms by which heat transfer can
take place. All the three modes require the existence of temperature difference. The three
mechanisms are: (i) conduction, (ii) convection and (iii) radiation

1.2.1Conduction:- It is the energy transfer that takes place at molecular levels. Conduction is
the transfer of energy from the more energetic molecules of a substance to the adjacent less
energetic molecules as a result of interaction between the molecules. In the case of liquids and
gases conduction is due to collisions and diffusion of the molecules during their random
motion. In solids, it is due to the vibrations of the molecules in a lattice and motion of free
electrons.
Fourier’s Law of Heat Conduction:- The empirical law of conduction based on experimental
results is named after the French Physicist Joseph Fourier. The law states that the rate of heat
flow by conduction in any medium in any direction is proportional to the area normal to the
direction of heat flow and also proportional to the temperature gradient in that direction. For
example the rate of heat transfer in x-direction can be written according to Fourier’s law as

∂T
Qx α – Ax ( ∂x ) …………….(1.1)

11
∂T
Or Qx = − kx Ax ( ) …………….. ..(1.2)
∂x

In equation (1.2), Qx is the rate of heat transfer in positive x-direction through area Ax (Ax can
vary in the direction of x, for example, radial conduction in cylinders and spheres) of the
𝜕𝑇
medium normal to x-direction, (𝜕𝑥 ) is the temperature gradient and kx is the constant of
proportionality and is a material property called “thermal conductivity” of the material for
conduction in x-direction. Since heat transfer has to take place in the direction of decreasing
∂T
temperature, ( ∂x ) has to be negative in the direction of heat transfer. Therefore negative sign
has to be introduced in equation (1.2) to make Qx positive in the direction of decreasing
temperature, thereby satisfying the second law of thermodynamics. If equation (1.2) is divided
throughout by Ax we have

Q ∂T
qx = (Ax )= − kx ( ) ………..(1.3)
x ∂x
qx is called the heat flux in x-direction.

Thermal Conductivity:- The constant of proportionality in the equation of Fourier’s law of


conduction, kx is a material property called the thermal conductivity for conduction along x-
direction (For isotropic solids thermal conductivity is independent of direction; i.e. kx = ky =
kz = k, where as for an-isotropic solids kx ≠ ky ≠ kz ).The units of thermal conductivity can be
obtained from equation (1.2) as follows:
qx
Solving for kx from Equation (1.3) we have kx = − ∂T
( )
∂x
Therefore units of kx in SI system of units = (W∙m−2 ) × (m∙ K−1) = W∙m−1∙K−1 or
W∙m−1∙℃−1. Thermal conductivity is a measure of a material’s ability to conduct heat. The
thermal conductivities of materials vary over a wide range as shown in Figure 1.1.
It can be seen from this figure that the thermal conductivities of gases such as air vary
by a factor of 10 4 from those of pure metals such as copper. The kinetic theory of gases predicts
and experiments confirm that the thermal conductivity of gases is proportional to the square
root of the absolute temperature, and inversely proportional to the square root of the molar
mass M. Hence, the thermal conductivity of gases increases with increase in temperature and
decrease with increase in molar mass. It is for these reasons that the thermal conductivity of
helium (M=4) is much higher than those of air (M=29) and argon (M=40).For wide range of
pressures encountered in practice the thermal conductivity of gases is independent of pressure.
The mechanism of heat conduction in liquids is more complicated due to the fact that
the molecules are more closely spaced, and they exert a stronger inter-molecular force field.
The values of k for liquids usually lie between those for solids and gases. Unlike gases, the
thermal conductivity for most liquids decreases with increase in temperature except for water.
Like gases the thermal conductivity of liquids decreases with increase in molar mass.

In the case of solids heat conduction is due to two effects: the vibration of lattice
induced by the vibration of molecules positioned at relatively fixed positions , and energy
transported due to the motion of free electrons. The relatively high thermal conductivities of
pure metals are primarily due to the electronic component. The lattice component of thermal

12
conductivity strongly depends on the way the molecules are arranged. For example, diamond,
which is highly ordered crystalline solid, has the highest thermal conductivity at room
temperature.

1000 Silver
Copper

Solid
metals
100 Sodium

Liquid
metals
Steel
Oxides
10
Non-
k [W∙m−1∙K−1] Mercury Metallic
solids
1.0 Water
Fibres
Non- He, H
Metallic Insul 2

liquids ating Non-


0.1 Mate Meta
Plastics rials Evacuated
llic Insulating
Wood
Oils gases materials
Foams
CO
2
0.01
Figure 1.1: Typical range of thermal conductivities of various materials

Unlike metals, which are good electrical and heat conductors, crystalline solids such
as diamond and semiconductors such as silicon are good heat conductors but poor electrical
conductors. Hence such materials find widespread use in electronic industry. Despite their high
price, diamond heat sinks are used in the cooling of sensitive electronic components because
of their excellent thermal conductivity. Silicon oils and gaskets are commonly used in the
packaging of electronic components because they provide both good thermal contact and good
electrical insulation.

One would expect that metal alloys will have high thermal
conductivities, because pure metals have high thermal conductivities. For example one would
expect that the value of the thermal conductivity k of a metal alloy made of two metals with
thermal conductivities k1 and k2 would lie between k1 and k2.But this is not the case. In fact k
of a metal alloy will be less than that of either metal.

13
The thermal conductivities of materials vary with temperature. But for
some materials the variation is insignificant even for wide temperature range.At temperatures
near absolute zero, the thermal conductivities of certain solids are extremely large. For example
copper at 20 K will have a thermal conductivity of 20,000 W / (m-K), which is about 50 times
the conductivity at room temperature. The temperature dependence of thermal conductivity
makes the conduction heat transfer analysis more complex and involved. As a first
approximation analysis for solids with variable conductivity is carried out assuming constant
thermal conductivity which is an average value of the conductivity for the temperature range
of interest.

Thermal Diffusivity: This is a property which is very helpful in analyzing transient heat
conduction problem and is normally denoted by the symbol α . It is defined as follows.

k Heat conducted
α = ρC = Heat Stored per unit volume (1.4a)
p

It can be seen from the definition of thermal diffusivity that the numerator represents the ability
of the material to conduct heat across its layers and the denominator represents the ability of
the material to store heat per unit volume. Hence we can conclude that larger the value of the
thermal diffusivity, faster will be the propagation of heat into the medium. A small value of
thermal diffusivity indicates that heat is mostly absorbed by the material and only a small
quantity of heat will be conducted across the material.

Thermal Effusivity: While thermal diffusivity α, is defined as the ratio between the thermal
conductivity, k, and the volumetric thermal capacity, ρcp, the effusivity, e, is related to their
product, as follows:

e = √(𝑘 ∙ 𝜌 𝐶𝑝 ) (1.4b)

It is important to realize that while the two expressions contain the same parameters, they are
quite different. Diffusivity is related to the speed at which thermal equilibrium can be
reached. Effusivity (sometimes called the heat penetration coefficient) is the rate at which a
material can absorb heat. It is the property that determines the contact temperature of two
bodies that touch each other.
For example, it explains the well-known but often misinterpreted effect that a metal feels
cold to the touch and wool warm, even when both are at room temperature. Our finger is a
very poor temperature sensor (that is, of the temperature of the body it touches, not of the
contact temperature), but a sensitive heat flux sensor. The contact temperature is lowest for
materials with a high thermal conductivity and a high thermal capacity, explaining why metal
feels cold.
While the thermal diffusivity is only related to time-dependent heat transfer, the contact
temperature is time-invariant as long as the two materials may be considered infinite. When
is such the case? This is ‘controlled’ by the thermal penetration depth δ, roughly equal to
2√(a�t), with t the time. It is essentially the distance from the surface at which the
temperature begins to rise at some time t after excitation at the surface.

14
In a number of industrial applications it is the effusivity that is the controlling parameter, not
the diffusivity. One example is injection molding, whereby the contact temperature between
the wall and the hot plastic determines the required pressure (due to solidification at the wall)
and the cycle time. The best solution is to coat the wall with a thin layer of low conductivity
material, thereby raising the contact temperature during injection while hardly influencing
the cooling time. Another example can be found in the pharmaceutical industry where
measuring the effusivity of pills, for example, is an established quality criterion.
Because, for most solids, the thermal capacities lie in a rather narrow range , it might be
concluded that, to a first order, the diffusivity scales with k and the effusivity with √k. From
this perspective, both quantities are related, which becomes obvious from looking at Figure
1, where only the solids from Table 1 are included. The two outliers are the ones with a
relatively low density.

Figure 1.2. Thermal effusivity vs. diffusivity for the solids presented in Table 1.
It is interesting to understand how α and e are measured. As discussed previously, the
diffusivity determines how fast some point away from the temperature excitation is heated,
while the effusivity is related to a temperature at the surface. Measuring both quantities
provides the thermal conductivity without the need to know the thermal capacity. Many
methods are described in the open literature, based on either contact techniques or photo -
thermal methods where a modulated heat source generates temperature excursions of which
the amplitude and phase can be used to extract both α and e at the same time.
Table 1 shows data for a number of typical materials. Also presented is the penetration depth,
calculated 1s after excitation. Please note the units
Table 1. Thermal effusivity (from low to high) for selected materials at room temperature.
The penetration depth δ is tabulated at 1s after excitation.

15
1.2.2. Convection : Convection heat transfer is composed of two mechanisms. Apart from
energy transfer due to random molecular motion, energy is also transferred due to macroscopic
motion of the fluid. Such motion in presence of the temperature gradient contributes to heat
transfer. Thus in convection the total heat transfer is due to random motion of the fluid
molecules together with the bulk motion of the fluid, the major contribution coming from the
latter mechanism. Therefore bulk motion of the fluid is a necessary condition for convection
heat transfer to take place in addition to the temperature gradient in the fluid. Depending on
the force responsible for the bulk motion of the fluid, convective heat transfer is classified into
“forced convection” and “natural or free convection”. In the case of forced convection, the
fluid flow is caused by an external agency like a pump or a blower where as in the case of
natural or free convection the force responsible for the fluid flow (normally referred to as the
buoyancy force) is generated within the fluid itself due to density differences which are caused
due to temperature gradient within the flow field. Regardless of the particular nature of
convection, the rate equation for convective heat transfer is given by

q = h ∆T (1.5)

where q is the heat flux, ∆T is the temperature difference between the bulk fluid and the surface
which is in contact with the fluid, and ‘h” is called the “convective heat transfer coefficient”
or “surface film coefficient”. Equation(1.5) is generally referred to as the Newton’s law of
cooling.If Ts is the surface temperature , Tf is the temperature of the bulk fluid and if Ts > Tf,
then Equation (1.5) in the direction of heat transfer can be written as

q = h [Ts – Tf] (1.6a)

16
and if Ts < Tf, the equation reduces to

q = h [Tf – Ts] (1.6b)

The heat transfer coefficient h depends on (i) the type of flow (i.e. whether the flow is
laminar or turbulent), (ii) the geometry of the body and flow passage area, (iii) the thermo-
physical properties of the fluid namely the density ρ, viscosity μ, specific heat at constant
pressure Cp and the thermal conductivity of the fluid k and (iv) whether the mechanism of
convection is forced convection or free convection or both. The heat transfer coefficient for
free convection will be generally lower than that for forced convection as the fluid velocities
in free convection are much lower than those in forced convection. The heat transfer
coefficients for some typical applications are given in Table 1.2.

Table 1.2: Typical values of the convective heat transfer coefficient h


------------------------------------------------------------------------------------------------------------
Type of flow h ,W∙m−2∙K−1
Free convection
Gases 2 – 25
Liquids 50 – 1000
Forced Convection
Gases 25 – 250
Liquids 50 – 20,000
Convection with change of phase
Boiling or condensation 2500 – 100,000

1.2.3. Thermal Radiation:- Thermal radiation is the energy emitted by matter (solid, liquid
or gas) by virtue of its temperature. This energy is transported by electromagnetic waves (or
alternatively, photons).While the transfer of energy by conduction and convection requires the
presence of a material medium, radiation does not require.Infact radiation transfer occurs most
effectively in vacuum.
Consider radiation transfer process for the surface shown in Figure1.3a.Radiation that
is emitted by the surface originates from the thermal energy of matter bounded by the surface,
and the rate at which this energy is released per unit area is called as the surface emissive power
E.An ideal surface is one which emits maximum emissive power and is called an ideal radiator
or a black body.Stefan-Boltzman’s law of radiation states that the emissive power of a black
body is proportional to the fourth power of the absolute temperature of the body. Therefore if
Eb is the emissive power of a black body at temperature T 0K, then

17
G ρG E
q
s q
surr

Surface of emissivity ε, absorptivity α, and


Surface of emissivity ε,
temperature T
s
area A, and temperature
T
s
Surroundings (black) at T
surr
(a) (b)

Figure1.3: Radiation exchange: (a) at a surface and (b) between a


surface and large surroundings

Eb α T 4

Or: Eb = σ T 4 (1.7)

σ is the Stefan-Boltzman constant (σ = 5.67 × 10 − 8 W∙m −2∙K −4 ). For a non black surface the
emissive power is given by

E=εσT4 (1.8)

where ε is called the emissivity of the surface (0 ≤ ε ≤ 1).The emissivity provides a measure of
how efficiently a surface emits radiation relative to a black body. The emissivity strongly
depends on the surface material and finish.
Radiation may also incident on a surface from its surroundings. The rate at which the
radiation is incident on a surface per unit area of the surface is calle the “irradiation” of the
surface and is denoted by G. The fraction of this energy absorbed by the surface is called
“absorptivity” of the surface and is denoted by the symbol α. The fraction of the incident
energy is reflected and is called the “reflectivity” of the surface denoted by ρ and the
remaining fraction of the incident energy is transmitted through the surface and is called the
“transmissivity” of the surface denoted by τ. It follows from the definitions of α, ρ, and τ that

α+ ρ+τ=1 (1.9)

Therefore the energy absorbed by a surface due to any radiation falling on it is given by

Gabs = αG (1.10)

18
The absorptivity α of a body is generally different from its emissivity. However in many
practical applications, to simplify the analysis α is assumed to be equal to its emissivity ε.

Radiation Exchange:- When two bodies at different temperatures “see” each other, heat is
exchanged between them by radiation. If the intervening medium is filled with a substance like
air which is transparent to radiation, the radiation emitted from one body travels through the
intervening medium without any attenuation and reaches the other body, and vice versa. Then
the hot body experiences a net heat loss, and the cold body a net heat gain due to radiation heat
exchange between the two. The analysis of radiation heat exchange among surfaces is quite
complex which will be discussed in chapter 10. Here we shall consider two simple examples
to illustrate the method of calculating the radiation heat exchange between surfaces.
As the first example let us consider a small opaque plate (for an opaque surface τ =
0) of area A, emissivity ε and maintained at a uniform temperature Ts. Let this plate is exposed
to a large surroundings of area Asu (Asu >> A) whish is at a uniform temperature Tsur as shown
in Figure 1.2b.The space between them contains air which is transparent to thermal radiation.

The radiation energy emitted by the plate is given by

Qem = A ε σ Ts4

The large surroundings can be approximated as a black body in relation to the small plate. Then
the radiation flux emitted by the surroundings is σ Tsur4 which is also the radiaton flux incident
on the plate. Therefore the radiation energy absorbed by the plate due to emission from the
surroundings is given by

Qab = A α σ Tsur4.

The net radiation loss from the plate to the surroundings is therefore given by

Qrad = A ε σ Ts4 − A α σ Tsur4.

Assuming α = ε for the plate the above expression for Qnet reduces to

Qrad = A ε σ [Ts4 – Tsur4 ] (1.11)

The above expression can be used to calculate the net radiation heat exchange between a small
area and a large surroundings.
As the second example, consider two finite surfaces A1 and A2 as shown in Figure 1.4.

19
Large Surroundings

A2, ε2, T2
A1, ε1, T1

Figure1.4: Radiation exchange between surfaces A1 and A2

The surfaces are maintained at absolute temperatures T1 and T2 respectively, and have
emissivities ε1 and ε2. Only part of the radiation leaving A1 reaches A2. Similar considerations
apply for the radiation leaving A2.If it is assumed that the radiation from the surroundings is
negligible when compared to the radiation from the surfaces A1 and A2 then we can write the
expression for the radiation emitted by A1 and reaching A2 as

Q1→2 = F1− 2 A1ε1σ T14 (1.12)

where F1 – 2 is defined as the fraction of radiation energy emitted by A1 and reaching A2.
Similarly the radiation energy emitted by A2 and reaching A1 is given by

Q2→1 = F2− 1 A2 ε2 σ T24 (1.13)

where F2 – 1 is the fraction of radiation energy leaving A2 and reaching A1. Hence the net
radiation energy transfer from A1 to A2 is given by

Q1 – 2 = Q1→2 − Q2→1

= [F1− 2 A1ε1σ T14] − [F2− 1 A2 ε2 σ T24]

F1-2 is called the view factor (or geometric shape factor or configuration factor) of A2 with
respect to A1 and F2 - 1 is the view factor of A1 with respect to A2.It will be shown in chapter
10 that the view factor is purely a geometric property which depends on the relative
orientations of A1 and A2 satisfying the reciprocity relation, A1 F1 – 2 = A2 F2 – 1.

Therefore: Q1 – 2 = A1F1 – 2 σ [ε1 T14 − ε2 T24] (1.14)

20
Radiation Heat Transfer Coefficient:- Under certain restrictive conditions it is possible to
simplify the radiation heat transfer calculations by defining a radiation heat transfer coefficient
hr analogous to convective heat transfer coefficient as

Qr = hrA ΔT

For the example of radiation exchange between a surface and the surroundings
[Equation. (1. 11)] using the concept of radiation heat transfer coefficient we can write

Qr = hrA[Ts – Tsur] = A ε σ [Ts4 – Tsur4 ]

ε σ [T4s − T4sur ] ε σ [T2s + T2sur ][T2s − T2sur ]


Or: hr = [Ts − Tsur ]
= [Ts − Tsur ]

Or hr = ε σ [Ts2 + Tsur2 ][Ts + Tsur] (1.15)

1.3.First Law of Thermodynamics (Law of conservation of energy) as applied to


Heat Transfer Problems : The first law of thermodynamics is an essential tool for solving
many heat transfer problems. Hence it is necessary to know the general formulation of the first
law of thermodynamics.
First law equation for a control volume:- A control volume is a region in space bounded by a
control surface through which energy and matter may pass.There are two options of
formulating the first law for a control volume. One option is formulating the law on a rate
basis. That is, at any instant, there must be a balance between all energy rates. Alternatively,
the first law must also be satisfied over any time interval Δt. For such an interval, there must
be a balance between the amounts of all energy changes.

First Law on rate basis :- The rate at which thermal and mechanical energy enters a control
volume, plus the rate at which thermal energy is generated within the control volume, minus
the rate at which thermal and mechanical energy leaves the control volume must be equal to
the rate of increase of stored energy within the control volume.
Consider a control volume shown in Figure 1.5 which shows that thermal and
mechanical energy are entering the control volume at a rate denoted by Ein, thermal and

Eሶ g Eሶst
Eሶ out
Eሶ in

Figure 1.5: Conservation of energy for a control volume on rate basis

21
mechanical energy are leaving the control volume at a rate denoted by Eout. The rate at
.
which energy is generated within the control volume is denoted by Eg and the rate at
.
which energy is stored within the control volume is denoted by Est. The general form of the
energy balance equation for the control volume can be written as follows:

Eሶin + Eሶg − Eሶout = Eሶst (1.16)


𝐄ሶst is nothing but the rate of increase of energy within the control volume and hence can be
dEs
written as equal to
dt

First Law over a Time Interval Δt : Over a time interval Δt, the amount of thermal and
mechanical energy that enters a control volume, plus the amount of thermal energy generated
within the control volume minus the amount of thermal energy that leaves the control volume
is equal to the increase in the amount of energy stored within the control volume.
The above statement can be written symbolically as

Ein + Eg – Eout = ΔEst …………………………..(1.16)

The inflow and outflow energy terms are surface phenomena. That is they are associated
exclusively with the processes occurring at the boundary surface and are proportional to the
surface area.
The energy generation term is associated with conversion from some other form
(chemical, electrical, electromagnetic, or nuclear) to thermal energy. It is a volumetric
phenomenon.That is, it occurs within the control volume and is proportional to the magnitude
of this volume. For example, exothermic chemical reaction may be taking place within the
control volume. This reaction converts chemical energy to thermal energy and we say that
energy is generated within the control volume. Conversion of electrical energy to thermal
energy due to resistance heating when electric current is passed through an electrical conductor
is another example of thermal energy generation
Energy storage is also a volumetric phenomenon and energy change within the
control volume is due to the changes in kinetic, potential and internal energy of matter within
the control volume.

1.4. Illustrative Examples:

A. Conduction

Example 1.1. Heat flux through a wood slab 50 mm thick, whose inner and outer surface
temperatures are 40 ℃ and 20 ℃ respectively, has been determined to be 40 W∙m−2. What is
the thermal conductivity of the wood slab?

Schematic:

22
T1
T1
q T2
T2

k=?

L
Known Find
Geometry of the problem Temperatures(℃) Heat flux(W∙m- k (W∙m-1∙K-
2) 1
)
L = 0.05 m T1 = 40 q = 40
T2 = 20

Assumptions: (i) steady state conduction; (ii) one dimensional conduction across
thethickness;(iii) solid is of constant thermal conductivity

Solution: For steady state conduction across the thickness of the slab and noting that the slab
is not generating any thermal energy, the first law equation for the slab can be written as :
Rate at which thermal energy (conduction) is entering the slab at the surface x = 0 is equal to
the rate at which thermal energy is leaving the slab at the surface x = L.
That is

Qx|x = 0 = Qx|x = L = Qx = constant

dT
By Fourier’s law we have Qx = − kA ( )
dx

Separating the variables and integrating both sides w.r.t. ‘x’ we have

L L
Qx ∫0 dx = − kA ∫0 dT

kA(T1 − T2 )
Or: Qx =
L

Qx k(T1 − T2 )
Heat flux = q = =
A L

qL 40×0.05
Hence: k= = = 0.1 W∙m−1∙K−1.
(T1 − T2 ) (40−20)

23
Example 1.2. A concrete wall, which has a surface area of 20 m2 and thickness 30 cm,
separates conditioned room air from ambient air.The temperature of the inner surface of the
wall is 25 ℃ and the thermal conductivity of the wall is 1.5 W∙m−1∙K−1.Determine the heat loss
through the wall for ambient temperature varying from ─ 15 ℃ to 38 ℃ which correspond to
winter and summer conditions and display your results graphically.

Schematic:

Known: T1 = 25 ℃ ; A = 20 m2; L = 0.3 m :


T1
k = 1.5 W∙m−1∙K−1;
Q
Find: Q for different values of T2 and plot Q
T2 vs T2 for different values of T2

Assumptions: (i) steady state conduction


(ii) one dimensional conduction
L (iii) solid is of constant thermal conductivity

Solution: By Fourier’s law of conduction we have

kA(T1 − T2 ) 1.5×20× (25− 𝑇1 )


For steady state: Q= =
L 0.3

Or: Q = 2500 – 100 T2 …………………………………...(1)

Heat loss Q for different values of T2 ranging from – 15 ℃ to + 38 ℃ are obtained from
Equation (1) and the results are plotted as shown:

24
Q 1.2 equation: Q= 2500-100T(2)

5000

4000

3000
Q ,watts

2000
Series2
1000

0
1 2 3 4 5 6 7 8 9 10 11 12
-1000

-2000
T(2) , celsius

Scale x-axis : 1cm= 5 ℃ ; y-axis : 1cm =1000W

Example 1.3.What is the thickness required of a masonry wall having a thermal conductivity
of 0.75 W∙m−1∙K−1, if the heat transfer rate is to be 80 % of the rate through another wall
having thermal conductivity of 0.25 W∙m−1∙K−1and a thickness of 100 mm? Both walls are
subjected to the same temperature difference.

Schematic:
Let subscript 1 refers to masonry wall and subscript 2 refers to the other wall.

Known: Q1/Q2 = 0.8 ; ; L2 = 100 mm :


T1
k 1= 0.75 W∙m−1∙K−1; k 2= 0.25 W∙m−1∙K−1;
Q1
Find: L1
T2
Assumptions: (i) steady state conduction
(ii) one dimensional conduction
(iii) solid is of constant thermal conductivity
L1

Solution: By Fourier’s law for steady state conduction we have

k1 A [T1 − T2 ]Masonary wall


For masonary wall Q1 = L1

25
k2 A [T1 − T2 ]Other wall
and for other wall Q2 = L2

Given that (T1 – T2)|masonry wall = (T1 – T2)|other wall , we have

Q1 k L
= k1 L2
Q2 2 1

k Q 1 0.75
Or: L1 = k1 Q2 × L1 = 0.8 × × 100 = 375 mm.
2 1 0.25

Example 1.4.A stainless steel block (k = 14.5 W∙m−1∙K−1) has dimensions of 5 cm in x-


direction,10 cm in y- direction and 5 cm in z- direction(see Figure E1.4)The front and back
surfaces are well insulated.The temperatures in both x and y directions vary
linearly.Determine (i) the heat fluxes and rate of heat transfer in x and y directions and (ii)
The magnitude and direction of the heat flux vector.

15℃ 10℃

Δz = 5 cm

Δy = 10 cm

5℃
0℃

x
Δx = 5 cm

Figure E1.4. Figure for example 1.4.

𝜕𝑇
Known: k = 14.5 W∙m−1∙K−1; 𝜕𝑧 = 0;

Find: (i)qx,Qx,qy,Qy; (ii)𝑞⃗ and direction of 𝑞⃗

Assumptions: (i)Steady state conduction in the solid;(ii) k is same for both conduction in x
and y directions

26
Solution (i):Heat flux of conduction in x- direction is given by,

𝜕𝑇
qx = − k 𝜕𝑥
Since temperature variation in direction is given to be linear we have:

∂T ΔT 0 −5
= Δx = ℃ ∙ m−1 .
∂x 0.05

0 −5
Therefore: qx = − 14.5 × ( 0.05 ) = 1450 W∙m−2.

And rate of heat transfer: Qx = Ax qx = (0.1 × 0.05) × 1450 = 7.25 W.

∂T ΔT 15 −5
Similarly we have: = Δy = = 100 ℃ ∙ m−1 .
∂y 0.10

15 −5
qy = − 14.5 × ( 0.10 ) = −1450 W∙m−2.
And: Qy = Ay qy = (0.05 × 0.05) × (−1450) = 3.625 W.

(ii) The heat flux vector is given by, 𝑞⃗ = qx 𝑖⃗ + qy 𝑗⃗

Where i and j are unit vectors in x and y directions.

Magnitude of 𝑞⃗ = √(𝑞𝑥2 + 𝑞𝑦2 ) = √(14502 + (−14502 ) = 2050.6 W∙m−2.

𝑞𝑦 − 1450
Direction of 𝑞⃗ = θ = tan −1(𝑞 ) = tan −1( ) = − 45 °
𝑥 1450
y

x
θ
qy
𝑞⃗
qx

B. Convection:

Example 1.5. Air at 40 ℃ flows over a long circular cylinder of 25 mm diameter with
an embedded electrical heater. In a series of tests, measurements were made of power
per unit length, P required to maintain the surface temperature of the cylinder at 300 ℃

27
for different stream velocities V of the air. The results are as follows:

Air velocity, V (m∙s−1) : 1 2 4 8 12

Power, P (W∙m−1) : 450 658 983 1507 1963

(a)Determine the convective heat transfer coefficient for each velocity and display your
results graphically.
(b)Assuming the dependence of the heat transfer coefficient on velocity to be of the
form h = CV n , determine the parameters C and n from the results of part (a).

Schematic:

Ts

V,T∞
D

Known: D = 0.025 m : Ts = 300 ℃ ; T∞ = 40 ℃;

Find: (a) Heat transfer coefficient as a function of velocity of the fluid (b) To find the values
of C and n in the equation h = CVn

Assumptions: (i) Heat transfer from the surface of the solid to the flowing fluid is by
convection only.(i.e. radiation heat transfer between the surface of the solid and the fluid is
negligible)

Solution: If h is the surface heat transfer coefficient then the power dissipated by the cylinder
by convection is given by

P = hAs (Ts - T∞)

where As is the area of contact between the fluid and the surface of the cylinder.

Therefore P = h πDL (Ts - T∞)

28
P 𝑃
Or: h = πDL(T = 𝜋 ×0.025 ×1 ×(300−40)
s − T∞ )

𝑃
Or: h = 20.42 W∙m−2K−1 ………………………..(1)

Values of h for different flow velocities are obtained and tabulated as follows:

Air Velocity, V (m∙s−1): 1 2 4 8 12

Power,P (W∙m−1) : 450 658 983 1507 1963

h, (W∙m −2∙ K−1 ) : 22.04 32.22 48.14 73.8 96.13

(a) A graph of h versus V can now be plotted as shown in Figure P 1.4 (a).

Scale: X axis 1cm = 1 m∙s−1; Y axis 1cm= 10 W∙m −2∙ K−1

Q 1.4a

120

100

80

60
h

40

20

0
1 2 3 4 5 6 7 8 9 10 11 12 13
velocity, m/s

(b) h = CVn

Therefore ln h = ln C + n ln V …………………………(2)

If ln h is plotted against ln V it will be straight line and the slope of which will give the value
of n. Also the intercept of this line w.r.t the axis on which ln V is plotted will give the value of
ln C from which C can be determined. The log –log plot is as shown in Figure P 1.4(b).
Scale: X axis 1cm=0.25; Y axis 1cm=0.5

29
1.4b Slope: 0.571

3
ln h

0
1 2 3 4 5 6 7 8 9 10 11 12
ln v

ln C = 3.1 or C = 22.2

(ln ℎ−ln 𝐶) (4.55−3.10)


and: n= = = 0.571
ln 𝑉 2.5

Therefore: h = 22.2 V 0.571 is the empirical relation between h and V.

Example 1.6. A large surface at 50 ℃ is exposed to air at 20 ℃. If the heat transfer coefficient
between the surface and the air is 15 W∙m−2∙K−1, determine the heat transferred from 5 m2 of
the surface area in 7 hours.

Schematic:

Ts
h

Known: Ts =50 0C; T∞ = 20 ℃; h = 15 W∙m−2∙K−1; A = 5 m2 : time = t = 7 h ;

Find: Qtotal

Assumption (i) Steady state heat transfer; (ii)Heat transfer from the surface of the solid to
the surroundings is by convection only.(i.e. radiation heat transfer between the surface of the
solid and the surroundings is negligible)

Solution: Using Newton’s law of cooling we have

Q total = Q×t = hA(Ts - T∞) t = 15 ×5 ×(50 – 20) × (7 ×3600) = 56.7 × 10 6 J = 56.7 MJ

Example 1.7.A 25 cm diameter sphere at 120 ℃is suspended in air at 20 ℃. If the


convective heat transfer coefficient between the surface and air is 15 W∙m−2∙K−1

30
determine the heat loss from the sphere.

Sketch for the example:


h
Ts

D
T∞

Known: Ts = 120 0C;T∞ = 20 0C; h = 15 W∙m−2∙K−1; D = 0.25 m

Find: Q

Assumptions: (i) Heat transfer from the surface of the solid to the surrounding fluid is by
convection only.(i.e. radiation heat transfer between the surface of the solid and the fluid is
negligible)(ii) Steady state heat transfer

Solution:Q = hAs(Ts - T∞) = h 4πR2 (Ts - T∞) = 15 × 4π × (0.25/2)2 × (120 – 20) = 294.52 W

C. Radiation:

Example 1.8. A sphere 10 cm in diameter is suspended inside a large evacuated


chamber whose walls are kept at 300 K. If the surface of the sphere is black and
maintained at 500 K what would be the radiation heat loss from the sphere to the walls
of the chamber?. What would be the heat loss if the surface of the sphere has an
emissivity of 0.8?

Schematic:
Known: T1 = 500 K ; T2 = 300 K ; d1 = 0.10 m
T2

Find: (i) Q assuming the surface to be a black


surface; (ii) Q assuming the surface to have an
T1 emissivity of 0.8
.
Assumptions: (i) Steady state Heat Transfer
d1

0.1 2
Solution: Surface area of the sphere = As = 4πR12 = 4π × ( 2 ) = 0.0314 m2
If the surface of the sphere is black then

Qblack = σ As (T14 – T24)

31
= 5.67 × 10 ─ 8× 0.0314 × (5004 – 3004) = 96.85 W

If the surface is having an emissivity of 0.8 then

Q = 0.8 Qblack = 0.8 × 96.85 = 77.48 W

Example 1.9. A vacuum system as used in sputtering conducting thin films on micro
circuits, consists of a base plate maintained at a temperature of 300 K by an
electric heater and a shroud within the enclosure maintained at 77 K by circulating
liquid nitrogen. The base plate insulated on the lower side is 0.3 m in diameter and has
an emissivity of 0.25.
(a) How much electrical power must be provided to the base plate heater?

(b) At what rate must liquid nitrogen be supplied to the shroud if its latent heat of vaporization
is 125 kJ∙kg−1?

Schematic:

Known: T1 = 300 K ; T2 = 77 K ; d = 0.3 m ; ε1 = 0.25

Find: (a) Qr ; (b) Mass of nitrogen vapourised per unit time, ṁN2

Assumptions: Steady state heat transfer process

𝜋 𝜋
Solution: Surface area of the top surface of the base plate = As = ( 4 )d12 = ( 4 ) × 0.32

= 0.0707 m2
(a) Qr = ε1σ As (T14 – T24)

32
= 0.25 ×5.67 ×10 ─ 8 × 0.0707 × (3004 – 774) = 8.08 W

(b) If mሶN2 = mass flow rate of nitrogen that is vapourised then

Qሶ 8.08
mሶN2 = h r = 125 ×1000 = 6.464 × 10 – 5 kg∙s−1 = 0.233 kg per hour
fg

Example 1.10. A flat plate has one surface insulated and the other surface exposed to the
sun. The exposed surface absorbs the solar radiation at a rate of 800 W∙m−2 and dissipates
heat by both convection and radiation into the ambient at 300 K. If the emissivity of the
surface is 0.9 and the surface heat transfer coefficient is 12 W∙m−2∙K−1, determine the
surface temperature of the plate.

Schematic:
Qsolar
T∞
Qr
Qconv Ts , ε , h

Insulated

Known: T∞ = 300 K ; qsolar = 800 W∙ m −2 ; ε = 0.9 ; h = 12 W∙m−2∙K−1.

Find: Ts

Assumptions: (i) steady state heat transfer;(ii) the emissivity and the surface heat transfer
coefficient is uniform throughout the surface.

Solution: Energy balance equation for the top surface of the plate is given by

Qsolar = Qr + Qconv

qsolar As = ε σ As (Ts4 - T∞4) + h As (Ts - T∞)

Therefore 800 = 0.9 ×5.67 × 10 ─ 8× (Ts4 – 3004) + 12 × (Ts – 300)

On simplifying the above equation we get

33
𝑇 4
𝑠
(100 ) + 2.35 Ts = 943 …………………………(1)

Equation (1) has to be solved by trial and error.

Trial 1: Assume Ts = 350 K. Then LHS of Equation (1) = 972.6 which is more than RHS of
Equation (1). Hence Ts < 350 K.

Trial 2: Assume Ts = 340 K. Then LHS of Equation (1) = 932.6 which is slightly less than

RHS. Therefore Ts should lie between 340 K and 350 K but closer to 340 K.

Trial 3:- Assume Ts = 342.5 K. Then LHS of Equation (1) = 942.5 = RHS of Equation (1).

Therefore Ts = 342.5 K

Example 1.11.The solar radiation incident on the outside surface of an aluminum


shading device is 1000 W∙m− 2. Aluminum absorbs 12 % of the incident solar energy and
dissipates it by convection from the back surface and by combined convection and
radiation from the outer surface. The emissivity of aluminum is 0.10 and the convective
heat transfer coefficient for both the surfaces is 15 W/(m2 –K). The ambient temperature
of air may be taken as 20 ℃. Determine the temperature of the shading device.

Schematic:

q solar qr
qc1

qc2

Known: qsolar = 1000 W∙ m−2 ; absorptivity of aluminum = α = 0.12 ; emissivity

of aluminum = ε = 0.10 ; h = 15 W∙m−2∙K−1; T∞ = 20 + 273 = 293 K ;

Find: Temperature of the shading device, Ts

Assumptions: (i) Steady state heat transfer; (ii) the emissivity and the surface heat transfer
coefficient are uniform throughout the surface

34
Solution: Solar radiation flux absorbed by aluminum = qa = α qsolar

= 0.12 × 1000 = 120 W∙ m−2.

Under steady state conditions,the energy absorbed by aluminum is dissipated by convection


from the back surface and by combined convection and radiation from the outer surface.
Hence the energy balance equation can be written as

qa = qr + qc1 + qc2

Therefore: qa = ε σ (Ts4 – α σT∞4) + h1(Ts - T∞) + h2 (Ts - T∞)

Or: 120 = 5.67 ×10 ─ 8×0.10 (Ts4 – 0.12 × 2934) + (Ts – 293) × (15 + 15)

On simplifying we get,
𝑇 4
𝑠
(100 ) + 53 Ts = 15873 …………………………(1)

Equation(1) has to be solved by trial and error.

Trail 1:- Assume Ts = 300 K. Then LHS = 15981 which is > RHS.

Trail 2 :- Assume Ts = 295 K. Then LHS = 15710.73 which is < RHS. Hence Ts should lie
between 300K and 295 K.

Trial 3 :- Assume Ts = 297 K . Then LHS = 15819 which is almost equal to RHS (Within
0.34 %)

Therefore Ts = 297 K.

Exercise Problems

1.1.The temperature distribution in a solid(k =0.233 W∙m−1∙K−1) is given by

T(x,y,z) = 𝑥 2 −2𝑦 2 +𝑧 2 −xy + 2yz,


Where T is in ℃ and x,y, and z are in m.Find the magnitude of the heat flux vector at the
point (x,y,z) =(0.5,0.0,0.2).

1.2.The temperature distribution in a rod (k = 0.35 W∙m−1∙K−1) is given by,

T(r,θ,z) = r sin θ + 2z.

Where T is in ℃ , r and z are in m and θ is in rad.Find the magnitude of the heat flux vector
at the location (r,θ,z) = (0.1,0.0.5)

35
CHAPTER 2

GOVERNING EQUATIONS OF CONDUCTION


2.1.Introduction: In this chapter, the governing basic equations for conduction in Cartesian
coordinate system is derived. The corresponding equations in cylindrical and spherical
coordinate systems are also mentioned. Mathematical representations of different types of
boundary conditions and the initial condition required to solve conduction problems are also
discussed. After studying this chapter, the student will be able to write down the governing
equation and the required boundary conditions and initial condition if required for any
conduction problem.

2.2. One – Dimensional Conduction Equation in differential form : In order to derive the
one-dimensional conduction equation, let us consider a volume element of the solid of
thickness Δx along x – direction at a distance ‘x’ from the origin as shown in Figure 2.1.Qx
represents the rate

36
A(x) q’’’

Qx Qx + Δx

O
x Δx

Figure 2.1: Nomenclature for one dimensional conduction equation

of heat transfer in x – direction entering into the volume element at x, A(x) area of heat flow
at the section x ,q′′′ is the thermal energy generation within the element per unit volume and
Qx+Δx is the rate of conduction out of the element at the section x + Δx. The energy balance
equation per unit time for the element can be written as follows:

[ Rate of heat conduction into the element at x + Rate of thermal energy generation within
the element − Rate of heat conduction out of the element at x + Δx ]

= Rate of increase of internal energy of the element.

∂E
i.e., Qx + Qg – Qx+Δx = ∂t

∂Q ∂2 Q Δx2
or Qx + q′′′ Ax Δx – {Qx + ( ∂xx ) Δx + ( ∂x2x ) + …….}
2!

𝜕
= 𝜕𝑡 [ρA(x)ΔxCpT]

Neglecting higher order terms and noting that ρ and Cp are constants the above equation
simplifies to

∂Q 𝜕𝑇
Qx + q′′′ Ax Δx – {Qx + ( ∂xx ) Δx} = ρAx ΔxCp ( 𝜕𝑡 )

∂Q 𝜕𝑇
Or: − ( ∂xx ) + q′′′ Ax = ρAx Cp ( 𝜕𝑡 )

𝜕𝑇
Using Fourier’s law of conduction, Qx = − 𝑘𝑥 Ax (𝜕𝑥 ), the above equation simplifies to

37
𝜕 𝜕𝑇 𝜕𝑇
− 𝜕𝑥 {− 𝑘𝑥 Ax (𝜕𝑥 )}+ q′′′ Ax = ρAx Cp ( 𝜕𝑡 )

1 𝜕 𝜕𝑇 𝜕𝑇
Or: {− 𝑘𝑥 Ax (𝜕𝑥 )} + q′′′ = ρ Cp( 𝜕𝑡 ) ……… ……………(2.1)
𝐴𝑥 𝜕𝑥

Equation (2.1) is the most general form of conduction equation for one-dimensional unsteady
state conduction.

2.2.1.Equation for one-dimensional conduction in plane walls : For plane walls, the area of
heat flow Ax is a constant. Hence Equation (2.1) reduces to the form

𝛛 𝜕𝑇 𝛛𝐓
{− 𝑘𝑥 ( )}+ q′′′ = ρ Cp( )
𝛛𝐱 𝜕𝑥 𝛛𝐭
……………………..(2.2)

(i) If the thermal conductivity of the solid is constant then the above equation reduces to

∂2 T 𝑞′′′ 1 ∂T
(∂ x2 ) + ( 𝑘 )= ( ∂t ) ………………………(2.3)
𝑥 𝛼

(ii) For steady state conduction problems in solids of constant thermal conductivity
temperature within the solid will be independent of time [i.e.(∂T/ ∂t) = 0]
and hence Equation (2.3) reduces to

d2 T 𝑞′′′
( ) +( )= 0 ……………………….(2.4)
dx2 𝑘𝑥

(iii) For a solid of constant thermal conductivity for which there is no thermal energy
generation within the solid q’’’ = 0 and the governing equation for steady state conduction is
obtained by putting q’’’ = 0 in Equation (2.4) as

d2 T
(dx2 ) = 0 ………………………(2.5)

2.2.2.Differential Equation for one-dimensional radial conduction in cylinders:

38
R
Qr
r

Qr

For radial conduction in cylinders, by convention the radial coordinate is denoted by ‘r’
instead of ‘x’ and the area of heat flow through the cylinder of length L,at any radius r is
given by Ax = A(r) = 2πrL. Hence substituting this expression for Ax and replacing x by r in
Equation (2.1) we have

1 ∂ 𝛛𝐓
(2πrL) ∂r {𝑘𝑟 2πrL (∂T/∂r)} + q′′′ = ρ Cp ( 𝛛𝐭 )

𝟏 𝛛 𝛛𝐓 𝛛𝐓
Or {𝑘𝑟 𝐫 ( 𝛛𝐫 )} + q′′′ = ρ Cp ( 𝛛𝐭 ) ……….. ……….(2.6)
𝐫 𝛛𝐫

(i) For cylinders of constant thermal conductivity the above equation reduces to

𝟏 𝛛 𝛛𝐓 𝒒′′′ 𝟏 𝛛𝐓
{𝐫 ( 𝛛𝐫 )} + = 𝛂 ( 𝛛𝐭 ) …….…………….(2.7)
𝐫 𝛛𝐫 𝑘𝑟

𝑘
where α = 𝜌 𝐶𝑟 .
𝑝
(ii) For steady state radial conduction [i.e. (∂T/ ∂t) = 0] in cylinders of constant k, the above
equation reduces to

𝟏 𝒅 𝒅𝑻
{𝐫 ( )} + q′′′ / 𝒌𝒓 = 0 …………………….(2.8)
𝒓 𝒅𝒓 𝒅𝒓

(iii) For steady state radial conduction in cylinders of constant k and having no thermal
energy generation (i.e. q’’’ = 0) the above equation reduces to

𝑑 𝑑𝑇
{r ( 𝑑𝑟 )} = 0 ………………………(2.9)
𝑑𝑟

39
2.2.3.Equation for one-dimensional radial conduction in spheres:For one-dimensional radial
conduction in spheres, the area of heat flow at any radius r is given by Ar = 4πr2. Hence
Equation(2.1) for a sphere reduces to

1 ∂ ∂T ∂T
{4πr2 } {𝑘𝑟 4π r 2 ( ∂r )} + q′′′ = ρCp ( ∂t )
∂r

1 ∂ ∂T ∂T
Or: {𝑘𝑟 r 2 ( ∂r )} + q′′′ = ρ Cp ( ∂t ) .…………………(2.10)
r2 ∂r

(i) For spheres of constant thermal conductivity the above equation reduce to

1 ∂ ∂T q′′′ 1 ∂T
{r 2 ( )} + = ( ) ………………..(2.11)
r2 ∂r ∂r 𝑘𝑟 α ∂t

(ii) For steady state conduction in spheres of constant k the above equation further reduce to

1 𝑑 ∂T q′′′
{r 2 ( ∂r )} + =0 …………………(2.12)
𝑟 2 𝑑𝑟 𝑘𝑟

(iii) For steady state conduction in spheres of constant k and without any thermal energy
generation the above equation further reduces to

1 𝑑 ∂T
{r 2 ( ∂r )} =0 …………………(2.13)
𝑟2 𝑑𝑟

Equation in compact form:- The general form of one – dimensional conduction equations
for plane walls, cylinders and spheres {equations (2..2), (2.5) and (2.9)} can be written in a
compact form as follows:

1 ∂ ∂T ∂T
{𝑘𝑟 r n ( ∂r )} + q′′′ = ρ Cp ( ∂t ) …….. ……….(2.14)
rn ∂r

Where n = 0 for plane walls,


n = 1 for radial conduction in cylinders
n = 2 for radial conduction in spheres,
and for plane walls it is customary to replace the ‘r’ variable by ‘x’ variable.

2.3.Three dimensional conduction equations: While deriving the one – dimensional


conduction equation, we assumed that conduction heat transfer is taking place only along one
direction. By allowing conduction along the remaining two directions and following the same
procedure we obtain the governing equation for conduction in three dimenions.

2.3.1. Three dimensional conduction equation in Cartesian coordinate system: Let us


consider a volume element of dimensions Δx, Δy and Δz in x y and z directions respectively.
The conduction heat transfer across the six surfaces of the element is shown in Figure 2.3.

40
z Qz + Δz Qy + Δy
y

Δz
x

Qx + Δx
Qx

Δy

Δx

Qy Qz
Figure 2.3: Conduction heat transfer across the six faces of a volume
element

Net Rate of conduction into the element in x-direction = Qx – Qx + Δx

𝜕𝑄 𝜕2 𝑄 𝛥𝑥 2
= Qx – [Qx + ( 𝜕𝑥𝑥 ) Δx + ( 𝜕𝑥 2𝑥 ) ( ) + ….]
2!

𝜕𝑄
= − ( 𝜕𝑥𝑥 ) Δx (by neglecting higher order terms).

∂ ∂T
= − ∂x [− k x Δy Δz ( ∂x )] Δx

∂ ∂T
= ∂x [ k x (∂x )] Δx Δy Δz

Similarly the net rate of conduction into the element


∂ ∂T
in y – direction = ∂y [ k y ( ∂y)] Δx Δy Δz

∂ ∂T
and in z – direction = ∂z [ k z ( ∂z )] Δx Δy Δz .

Hence the net rate of conduction into the element from all the three directions

∂ ∂T ∂ ∂T ∂ ∂T
Qin = [ k x ( ∂x )] Δx Δy Δz + ∂y [ k y (∂y )] Δx Δy Δz + ∂z [ k z ( ∂z )] Δx Δy Δz
∂x

∂ ∂T ∂ ∂T ∂ ∂T
= {∂x [ k x ( ∂x )] + [ k y (∂y)] + [ k z ( ∂z )]} Δx Δy Δz
∂y ∂z

Rate of heat thermal energy generation in the element = Qg = q’’’ Δx Δy Δz

41
∂E ∂T
Rate of increase of internal energy within the element = = ρ Δx Δy Δz Cp ( ∂t )
∂t

Applying I law of thermodynamics for the volume element we have

∂E
Qin + Qg = ∂t

∂E
Substituting the expressions for Qin, Qg and and simplifying we get
∂t

∂ ∂T ∂ ∂T ∂ ∂T ∂T
{∂x [ k x ( ∂x )] + [ k y (∂y )] + [ k z ( ∂z )]}+ q′′′ = ρ Cp ( ∂t ) …………(2.15)
∂y ∂z

Equation (2.15) is the most general form of conduction equation in Cartesian coordinate
system. This equation reduces to much simpler form for many special cases as indicated
below.

Special cases:- (i) For isotropic solids, thermal conductivity is independent of direction; i.e.,
kx = ky = kz = k. Hence Equation (2.15) reduces to

∂ ∂T ∂ ∂T ∂ ∂T ∂T
{∂x [ k ( ∂x )] + [ k (∂y)] + [ k ( ∂z )]}+ q′′′ = ρ Cp ( ∂t ) …………..(2.16)
∂y ∂z

(ii) For isotropic solids with constant thermal conductivity the above equation further reduces
to

∂2 T ∂2 T ∂2 T q′′′ 1 ∂T
+ ∂y2 + + = α ( ∂t ) …………….(2.17)
∂x2 ∂z2 k

Equation(2.17) is called as the “Fourier – Biot equation” and it reduces to the following
forms under specified conditions as mentioned below:

(iii) Steady state conduction [i.e., (∂T / ∂t) = 0]

∂2 T ∂2 T ∂2 T q′′′
+ ∂y2 + + = =0 …………….(2.18)
∂x2 ∂z2 k

Equation (2.18) is called the “Poisson equation”.

(iv) No thermal energy generation [i.e. q’’’ = 0]:

∂2 T ∂2 T ∂2 T 1 ∂T
+ ∂y2 + = α ( ∂t ) …………………..(2.19)
∂x2 ∂z2

Equation (2.19) is called the “diffusion equation”.

(v) Steady state conduction without heat generation [i.e., (∂T / ∂t) = 0 and q’’’ = 0]:

42
∂2 T ∂2 T ∂2 T
+ ∂y2 + =0 ………………(2.20)
∂x2 ∂z2

Equation (2.20) is called the “Laplace equation”.

2.3.2. Three dimensional conduction equation in cylindrical coordinate system:

It is convenient to express the governing conduction equation in cylindrical coordinate system


when we want to analyse conduction in cylinders. Any point P in space can be located by using
the cylindrical coordinate system r, θ and z and its relation to the Cartesian coordinate system
(See Figure 2.4) can be written as follows:
z

P(x,y,z)

x
θ
r

Figure2.4: Cylindrical coordinate system


y

x = r cos θ ; y = r sin θ ; z = z. Using these transformations and after laborious simplifications


Equation (2.16) reduces to

1 ∂ ∂T 1 ∂ ∂T ∂ ∂T ∂T
(kr )+ (k ) + ∂z (k ) + q’’’ = ρCp …………..(2.21)
r ∂r ∂r r2 ∂θ ∂θ ∂z ∂t

The above equation is valid for only for isotropic solids.The above equation can also be
obtained by writing the energy balance equation for a volume element of dimensions dr, rdθ
and dz in r,θ and z directions respectively.

43
2.3.2. Three dimensional conduction equation in Spherical coordinate system:

For spherical solids,it is convenient to express the governing conduction equation in spherical
coordinate system. Any point P on the surface of a sphere of radius r can be located by using
the spherical coordinate system r, θ and φ and its relation to the Cartesian coordinate system
(See Figure 2.5) can be written as follows:
z
OP’ = r sin φ.Hence

x = r sin φ cos θ ;
P(x,y,z)
y = r sin φ sin θ ;

z = r cos φ
r
φ

O
x
θ

P’
Fig: 2.5: Spherical coordinate system
y

Using the relation between x, y ,z and r, θ and φ, the conduction equation (2.16) can be
transformed into the equation in terms of r, θ and φ as follows.

1 𝜕 𝜕𝑇 1 𝜕 𝜕𝑇 1 𝜕 𝜕𝑇 𝜕𝑇
(𝑘𝑟 2 )+ (𝑘 ) + 𝑟 2 sin 𝜑 (𝑘 sin 𝜑 ) + q’’’ = ρCp
𝑟 2 𝜕𝑟 𝜕𝑟 𝑟 2 (sin 𝜑)2 𝜕𝜃 𝜕𝜃 𝜕𝜑 𝜕𝜑 𝜕𝑡

…………….(2.22).

The above equation can also be obtained by wrting the energy balance equation for volume
element of dimensions dr, rdφ and r sinφ dθ in r,φ and θ directions.

2.4.Boundary and Initial Conditions:

The temperature distribution within any solid is obtained by integrating the above conduction
equation with respect to the space variable and with respect to time.The solution thus obtained
is called the “general solution” involving arbitrary constants of integration. The solution to a
particular conduction problem is arrived by obtaining these constants which depends on the
conditions at the bounding surfaces of the solid as well as the initial condition. The thermal

44
conditions at the boundary surfaces are called the “boundary conditions” . Boundary
conditions normally encountered in practice are:
(i) Specified temperature (also called as boundary condition of the first kind),
(ii) Specified heat flux (also known as boundary condition of the second kind),
(iii) Convective boundary condition (also known as boundary condition of the third kind) and
(iv) radiation boundary condition. The mathematical representations of these boundary
conditions are illustrated by means of a few examples below.

2.4.1. Specified Temperatures at the Boundary:- Consider a plane wall of thickness L whose
outer surfaces are maintained at temperatures T0 and TL as shown in Figure2.6(a). For one-
dimensional unsteady state conduction the boundary conditions can be written as
y
T(x,t)
T0 TL T(x,y)
T = φ(x)

L T2
b
Ψ(y)
a

x
T1
x

(a) plane wall (b) rectangular plate

Figure 2.6: Boundary condition of first kind

(i) at x = 0, T(0,t) = T0 ; (ii) at x = L, T(L,t) = TL.

Consider another example of a rectangular plate as shown in Figure 2.6(b). The boundary
conditions for the four surfaces to determine two-dimensional steady state temperature
distribution T(x,y) can be written as follows.

(i) at x = 0, T(0,y) = Ψ(y) ; (ii) at y = 0, T(x,0) = T1 for all values of y

(iii) at x = a, T(a,y) = T2 for all values of y; (iv) at y = b, T(x,b) = φ(x)

2.4.2. Specified heat flux at the boundary:- Consider a rectangular plate as shown in Figure
2.7 and whose boundaries are subjected to the prescribed heat flux conditions as shown in the
figure. Then the boundary conditions can be mathematically expressed as follows.

45
qb T(x,y)
y

q0 qa
b

a
x
insulated

Figure2.7: Prescribed heat flux boundary conditions

∂T
(i) at x = 0, − k [ ∂x ] = q 0 for 0 ≤ y ≤ b ;
x=0

∂T
(ii) at y = 0 , [∂y ] = 0 for 0 ≤ x ≤ a ;
y=0

∂T
(iii) at x = a, k [ ∂x ] = q a for 0 ≤ y ≤ b ;
x=a

∂T
(iv) at y = b, − k [∂y ] = qb for 0 ≤ x ≤ a ;
y=b

2.4.3. Boundary surface subjected to convective heat transfer:- Figure 2.8 shows a plane
wall whose outer surfaces are subjected to convective boundary conditions. The surface at x =
0 is in contact with a fluid which is at a uniform temperature Ti and the surface heat transfer
coefficient is hi. Similarly the other surface at x = L is in contact with another fluid at a uniform
temperature T0 with a surface heat transfer coefficient h0. This type of boundary condition is
encountered in heat exchanger wherein heat is transferred from hot fluid to the cold fluid with
a metallic wall separating the two fluids. This type of boundary condition is normally referred
to as the boundary condition of third kind. The mathematical representation of the boundary
conditions for the two surfaces of the plane wall can be written as follows.

𝜕𝑇
(i) at x = 0, qconvection = q conduction; i.e., hi[Ti − T|x = 0 ] = − k[𝜕𝑥 ]
𝑥=0

𝜕𝑇
(ii) at x = L, − k[𝜕𝑥 ] = h0 [T|x = L − T0]
𝑥=𝐿

46
T(x)
Surface in contact with
fluid at T0 with surface
heat transfer coefficient h0

L
Surface in contact with fluid
at Ti with surface heat
transfer coefficient h i

x
Figure 2.8: Boundaries subjected to convective heat transfer for a plane wall

2.4.4.Radiation Boundary Condition:Figure 2.9 shows a plane wall whose surface at x =L is


having an emissivity ‘ε’ and is radiating heat to the surroundings at a uniform temperature Ts.
The mathematical expression for the boundary condition at x = L can be written as follows:

T(x,t)
Surface with emissivity ε is
radiating heat to the
surroundings at Ts 0K

x
Figure 2.9: Boundary surface at x = L subjected to radiation heat transfer for a plane wall

𝜕𝑇
at x = L, qconduction = qradiation ; i.e., − k [𝜕𝑥 ] = σ ε [( T| x = L)4 − Ts 4]
𝑥=𝐿

In the above equation both T| x = L and Ts should be expressed in degrees Kelvin.

47
2.4.5. General form of boundary condition (combined conduction, convection and
radiation boundary condition): There are situations where the boundary surface is subjected
to combined conduction, convection and radiation conditions as illustrated in Figure 2.10.It is
a south wall of a house and the outer surface of the wall is exposed to solar radiation. The
interior of the room is at a uniform temperature Ti. The outer air is at uniform temperature T0
. The sky, the ground and the surfaces of the surrounding structures at this location is modeled
as a surface at an effective temperature of Tsky.
x
L

qradiation

qconduction αqsolar

qconvection

Figure 2.10: Schematic for general form of boundary condition

Energy balance for the outer surface [x = L] is given by the equation

qconduction + α qsolar = qradiation + qconvection

𝜕𝑇
− k [𝜕𝑥 ] + αqsolar = ε σ [(T|x = L)4 − Tsky4] + h0[T|x = L − T0]
𝑥=𝐿

48
2.5. Illustrative Examples:

A. Derivation of conduction Equations:

Example2.1.By writing an energy balance for a differential cylindrical volume element in the
‘r’variable (r is any radius), derive the one-dimensional time dependent heat conduction
equation with internal heat generation and variable thermal conductivity in the cylindrical
coordinate system.

Schematic :

r
Qr
dr
Qr

Qg
Qr+dr
Qr+dr

Find: One dimensional time dependent heat conduction equation

Assumptions: One dimensional time dependent radial heat conduction

Solution:
A cylindrical element of thickness dr in the radial direction at a radius r is shown in the figure
above. For unsteady state one dimensional radial conduction with heat generation is given by
∂E
Qr + Qg – Qr+dr = ∂t

𝜕𝑄 ∂E
Or Qr + Qg – [Qr + ( 𝜕𝑟𝑟 ) dr] = ∂t

𝜕𝑄 ∂E
Or ─ ( 𝜕𝑟𝑟 ) dr + Qg = …………………………..(1)
∂t

𝜕𝑇
where Qr is the rate of conduction into the element at radius r = ─ k 2πrL (𝜕𝑟 )

Qg is the rate of heat generation within the element = 2π rL dr q′′′

49
∂E 𝜕𝑇
( ∂t )is the rate of increase of the energy of the element. = 2π rL dr ρCp ( 𝜕𝑡 )
where dV=2πrLdr-------- volume

Substituting these expressions in Equation(1) we get

𝜕 𝜕𝑇 𝜕𝑇
(─ {─ 2π rLk (𝜕𝑟 )} )dr + 2π rL dr q′′′ = 2π rL dr ρCp ( 𝜕𝑡 ).
𝜕𝑟

Simplifying we get

1 𝜕 𝜕𝑇 𝜕𝑇
(𝑟 ) 𝜕𝑟 [k (𝜕𝑟 )] + q′′′ = ρCp ( 𝜕𝑡 )

Example2.2.By writing an energy balance for a differential spherical volume element in the
variable (r is any radius), derive the one-dimensional time dependent heat conduction
equation with internal heat generation and variable thermal conductivity in the spherical
coordinate system.

Sketch:

dr

Qr+dr
Qr

Qg

Find: One dimensional time dependent heat conduction equation

Assumptions: One dimensional time dependent radial heat conduction

Solution: Consider a spherical element of thickness dr at any radius r as shown in the figure
above. The energy balance equation for one – dimensional radial unsteady state conduction
with heat generation is given by

∂E
Qr + Qg – Qr+dr = ∂t

𝜕𝑄 ∂E
Or Qr + Qg – [Qr + ( 𝜕𝑟𝑟 ) dr] = ∂t

𝜕𝑄 ∂E
Or - ( 𝜕𝑟𝑟 )dr + Qg = ……………………(1)
∂t

50
𝜕𝑇
Where Qr = rate of heat conducted in to the element at radius r = - k 4π r2 (𝜕𝑟 ),

Qg = rate of heat generation within the element = (4/3)π [(r + dr)3 – r3 ] q′′′

4 ∂T
(∂E / ∂t) = rate of increase of energy of the element = ρ (3)π [ (r + dr) 3 – r3 ] ( ∂t )

Now (r + dr)3 – r3 = r3 + 3r2dr + 3r(dr)2 + (dr)3 – r3

= 3r2dr + 3r(dr)2 + (dr)3

Neglecting higher order terms like (dr)3 and (dr)2 we have

(r + dr)3 – r3 = 3 r2 dr.

Therefore: Qg = 4 π r2 dr q′′′

∂E ∂T
And: = ρ 4 π r2 dr Cp( ∂t ).
∂t

∂E
Substituting the expressions for Qr, Qg and ( ∂t )in Equation (1) we have

∂ ∂T ∂T
─ [∂r {− k 4π r 2 ( ∂r )}] dr + 4 π r2 dr q′′′ = ρ 4 π r2 dr Cp( ∂t )

Simplifying the above equation and noting that if k is given to be constant we have

∂ ∂T q′′′ ρ r2 Cp ∂T
{ r 2 ( ∂r )} + r2 ( )= ( ) ( ∂t )
∂r k k

1 ∂ ∂T q′′′ 1 ∂T k
Or: { r 2 ( ∂r )} + ( )= ( ∂t ) ; where α = ρC
r2 ∂r k α p

Example2.3.By simplifying the three-dimensional heat conduction equation, obtain one


dimensional steady-state conduction equation with heat generation and constant
thermal conductivity for the following coordinate systems:
(a) Rectangular coordinate in the ‘x’ variable; (b) Cylindrical coordinate in the r variable.
(c) Spherical coordinates in the ‘r’ variable

Find: Differential equation for one dimensional time dependent heat conduction with heat
generation

51
Assumptions: One dimensional time dependent radial heat conduction with heat generation

Solution: (a) The general form of conduction equation for an isotropic solid in rectangular
coordinate system is given by

∂ ∂T ∂ ∂T ∂ ∂T ∂T
(k ) + ∂y (k ) + ∂z (k ) + q′′′ = (ρ Cp) ( ∂t )
∂x ∂x ∂y ∂z
…………..(1)
∂T
For steady state conduction ( ∂t ) = 0 ; For one dimensional conduction in x – direction we have
∂T ∂T ∂T dT
= ∂z = 0 . Therefore ∂x = dx .
∂y

Therefore Equation (1) reduces to


d ∂T
(k ) + q′′′ = 0.
dx ∂x

For constant thermal conductivity the above equation reduces to

d2 T q′′′
+ = 0.
dx2 k

(b) The general form of conduction equation in cylindrical coordinate system is given by

1 ∂ ∂T 1 ∂ ∂T ∂ ∂T ∂T
(k r ) + r2 ∂θ (k ) + ∂z (k ) + q′′′ = ρCp( ∂t )
r ∂r ∂r ∂θ ∂z

∂T
For steady state conduction, ( ∂t )= 0 ; For one-dimensional radial conduction we have
∂T ∂T ∂T dT
= 0 and ∂z = 0. Therefore = dr . With these simplifications the general form of conduction
∂θ ∂r
equation reduces to
1 d dT
(k r ) + q′′′ = 0
r dr dr

For constant thermal conductivity the above equation reduces to

1 d dT q′′′
(r )+ = 0.
r dr dr k

(a)The general form of conduction equation in spherical coordinate system is given by

1 ∂ ∂T 1 ∂ ∂T
(k r 2 ) + {r2 sin2 φ} ∂θ (k )
r2 ∂r ∂r ∂θ

1 ∂ ∂T ∂T
+ {r2 sinφ} ∂φ (k sinφ ) + q′′′ = ρCp ( ∂t ) ………..(1)
∂φ

∂T
For steady state conduction ( ∂t )= 0 ; For one dimensional radial conduction we have

52
∂T 𝜕𝑇 ∂T dT
= 0 and = 0. Therefore = . Substituting these conditions in
∂θ 𝜕𝜑 ∂r dr

1 d dT
Equation (1) we have, (k r 2 ) + q′′′ = 0.
𝑟 2 dr dr

For constant thermal conductivity the above equation reduces to

1 d dT q′′′
( r2 ) + = 0.
𝑟2 dr dr k

B. Mathematical Formulation of Boundary conditions:

Example2.4.A plane wall of thickness L is subjected to a heat supply at a rate of q0 W/m2 at


one boundary surface and dissipates heat from the surface by convection to the ambient
which is at a uniform temperature of T∞ with a surface heat transfer coefficient of
h∞.Write the mathematical formulation of the boundary conditions for the plane wall.

Schematic:

h,T∞

q0

Solution: Boundary conditions are :


𝑑𝑇
(i)at x = 0; ─ k [𝑑𝑥 ] = q0
𝑥=0

𝑑𝑇
(ii) at x = L; ─ k[𝑑𝑥 ] = h(T|x=L - T∞)
𝑥=𝐿

Example2.5.Consider a solid cylinder of radius R and height Z. The outer curved surface of
the cylinder is subjected to a uniform heating electrically at a rate of q0 W / m2.Both the
circular surfaces of the cylinder are exposed to an environment at a uniform temeperature T∞
with a surface heat transfer coefficient h.Write the mathematical formulation of the boundary
conditions for the solid cylinder.

53
Schematic:

h,T∞
z
R

q0
q0
Z

h,T∞
Known: (i) Solid cylinder of radius R; (ii)Surface heat flux at radiyus R = q0 W/m2; (iii) The
two circular faces are exposed to ambient at T∞ with surface heat transfer coefficient h

Solution:
Boundary conditions are:
𝜕𝑇
(i) at r = 0; (𝜕𝑟 ) = 0 (axis of symmetry)

𝜕𝑇
(ii) at r = R; k(𝜕𝑟 ) = q0

𝜕𝑇
(iii) at z = 0; h[T|z=0 - T∞] + k [ 𝜕𝑧 ] =0
𝑧=0

𝜕𝑇
(iv) at z = Z; ─ k(𝜕𝑟 ) = h [ T|z = z ─ T∞]

Example2.6. A hollow cylinder of inner radius ri, outer radius r0 and height H is subjected
to thefollowing boundary conditions.
(a) The inner curved surface is heated uniformly with an electric heater at a constant rate of
q0 W/m2,
(b) the outer curved surface dissipates heat by convection into an ambient at a uniform
temperature, T∞ with a convective heat transfer coefficient,h
(c) the lower flat surface of the cylinder is insulated, and
(d) the upper flat surface of the cylinder dissipates heat by convection into the ambient at T∞
with surface heat transfer coefficient h. Write the mathematical formulation of the boundary
conditions for the hollow cylinder.

54
Schematic:
Solution:
hz,T∞
hr,T∞ Boundary conditions are:
𝜕𝑇
(i) at r = ri, ─ k(𝜕𝑟 ) = q0 for all z;

𝜕𝑇
(ii) at r = ro, ─ k(𝜕𝑟 ) = hr[T|r=r2 - T∞]
q0
H
q0 for all z

𝜕𝑇
(iii) at z = 0, ( 𝜕𝑧 )= 0 for all r.

(iv) at z = H,

𝜕𝑇
─ k [ 𝜕𝑧 ] = hz[T|z=H - T∞]
ri 𝑧=𝐻
Insulated ro for all r

C. Formulation of Heat Conduction Problems:

Example2.7. A plane wall of thickness L and with constant thermal properties is initially
at a uniform temperature Ti. Suddenly one of the surfaces of the wall is subjected to
heatingby the flow of a hot gas at temperature T∞ and the other surface is kept
insulated. Theheat transfer coefficient between the hot gas and the surface exposed to it
is h. There is no heat generation in the wall. Write the mathematical formulation of the
problem to determine the one-dimensional unsteady state temperature within the wall.

Schematic:
T = Ti at t = 0

L
Insulated h∞,T∞
T=
T(x,t)

Solution:
Governing differential equation to determine T(x,t) is given by

55
𝜕2 𝑇 1 𝜕𝑇
= 𝛼 ( 𝜕𝑡 )
𝜕𝑥 2

where α is the thermal diffusivity of the wall.

Initial condition is at time t = 0 T = Ti for all x.

𝜕𝑇
The boundary conditions are : (i) at x = 0, [𝜕𝑥 ] = 0 (Insulated), for all t > 0.
𝑥=0

𝜕𝑇
(ii) at x = L, ─ k [𝜕𝑥 ] = h∞ [T|x=L ─ T∞] for all t > 0.
𝑥=𝐿

Example2.8. A copper bar of radius R is initially at a uniform temperature Ti. Suddenly the
heating of the rod begins at time t=0 by the passage of electric current, which generates
heat at a uniform rate of q’’’ W/m3. The outer surface of the dissipates heat into an
ambient at a uniform temperature T∞ with a convective heat transfer coefficient h.
Assuming that thermal conductivity of the bar to be constant, write the mathematical
formulation of the heat conduction problem to determine the one-dimensional radial
unsteady state temperature distribution in the rod.

Schematic:

h,T∞

T = Ti at t ≤ 0
q’’’ for t ≤ 0
R

Solution:

The governing differential equation to determine T(r,t) is given by

1 𝜕 𝜕𝑇 𝑞0 1 𝜕𝑇
(𝑟 )+ = 𝛼 ( 𝜕𝑡 ).
𝑟 𝜕𝑟 𝜕𝑟 𝑘

𝜕𝑇
Boundary conditions are: (i) at r = 0, (𝜕𝑟 ) = 0 ( Axis of symmetry)

𝜕𝑇
(ii) at r = R, ─ k [𝜕𝑟 ] = h [T |r=R ─ T∞]
𝑥=𝑅

Initial condition is: At t = 0, T = Ti for all r

Example2.9. Consider a solid cylinder of radius R and height H. Heat is generated in the

56
solid at auniform rate of q’’’ W/m3. One of the circular faces of the cylinder is insulated and
theother circular face dissipates heat by convection into a medium at a uniform
temperature of T∞ with a surface heat transfer coefficient of h. The outer curved surface
of the cylinder is maintained at a uniform temperature of T0. Write the mathematical
formulation to determine the two-dimensional steady state temperature distribution
T(r,z) in the cylinder.

Sketch for example 2.9:


z Solution: The governing differential
h, T∞ equation to determine T(r,z) is given by

1 𝜕 𝜕𝑇 𝜕2 𝑇 𝑞′′′
T0 (𝑟 )+ + =0
𝑟 𝜕𝑟 𝜕𝑟 𝜕𝑧 2 𝑘

H Boundary conditions are:


𝜕𝑇
R (i) at r = 0, (𝜕𝑟 ) = 0, for all z (axis
𝑟=0
of symmetry).
(ii) at r = R, T = T0 for all z.

r 𝜕𝑇
(iii) at z = 0 , ( 𝜕𝑧 ) = 0 for all r.
𝑧=0

Insulated 𝜕𝑇
(iv) at z = H, − k ( ) = h (T |z=H ─ T∞)
𝜕𝑧 𝑧=𝐻
for all r.

Example2.10. Consider a rectangular plate as shown in Figure 2.10. The plate is generating
heatat a uniform rate of q’’’ W/m3. Write the mathematical formulation to determine two
dimensional steady state temperature distribution in the plate.

57
h,T∞

T0

bb
a
a
x

Insulated
Figure 2.10 : Schematic for example 2.10

Solution: The governing differential equation to determine T(x,y) is given by

𝜕2 𝑇 𝜕2 𝑇 𝑞′′′
+𝜕𝑦 2 + =0
𝜕𝑥 2 𝑘

Boundary conditions are:

𝜕𝑇
(i) at x=0, ─ k (𝜕𝑥 ) = q0 for all y ; (ii) at x = a, T = T0 for all y
𝑥=0

𝜕𝑇 𝜕𝑇
(iii) at y = 0, (𝜕𝑦) for all x ; (iv) at y = b, ─ k (𝜕𝑦) = h[T |y=b ─ T∞].
𝑦=0 𝑦=𝑏

Example2.11. Consider a medium in which the heat conduction equation is given in its
simple form as

𝜕2𝑇 1 𝜕𝑇
= ( )
𝜕𝑥 2 𝛼 𝜕𝑡

(a) Is heat transfer in this medium steady or transient?


(b) Is heat transfer one-, two- or three-dimensional?
(c) Is there heat generation in the medium?
(d) Is thermal conductivity of the medium constant or variable?

Solution: The given differential equation is

58
∂2 T 1 ∂T
= α ( ∂t )
∂x2

It can be seen from this equation that T depends on one space variable x and the time variable
t. Hence the problem is one dimensional transient conduction problem. No heat generation
term appears in the equation indicating that the medium is not generating any heat.The thermal
conductivity of the medium does not appear within the differential symbol indicating that the
conductivity of the medium is constant.

Example2.12. Consider a medium in which the heat conduction equation is given in its
simple form as
1 𝑑 𝑑𝑇
(𝑘 𝑟 𝑑𝑟 ) + q’’’ = 0.
𝑟 𝑑𝑟

(a) Is heat transfer steady or unsteady?


(b) Is heat transfer one-, two- or three-dimensional?
(c) Is there heat generation in the medium?
(d) Is the thermal conductivity of the medium constant or variable?

Solution: The given differential equation is

1 𝑑 𝑑𝑇
(𝑘 𝑟 𝑑𝑟 ) + q’’’ = 0.
𝑟 𝑑𝑟

It can be seen from this equation that the temperature T depends only on one space variable ‘r’
and it does not depend on time t. Also the heat generation term q’’’ appears in the differential
equation.Hence the problem is a one-dimensional steady state conduction problem with heat
generation. Since the thermal conductivity appears within the differential symbol, it follows
that the thermal conductivity of the medium is not a constant but varies with temperature.

Example2.13. Consider a medium in which heat the heat conduction equation in its simplest
form is given as
1 𝜕 𝜕𝑇 𝜕 𝜕𝑇
(𝑘 𝑟 𝜕𝑟 ) + 𝜕𝑧 (𝑘 𝜕𝑧 ) + 𝑞′′′ = 0
𝑟 𝜕𝑟

Is heat transfer steady state or unsteady state?


(a) Is heat transfer one-, two- or three-dimension?
(b) Is there heat generation in the medium?
(c) Is the thermal conductivity constant or variable?

Solution: The given differential equation is

1 𝜕 𝜕𝑇 𝜕 𝜕𝑇
(𝑘 𝑟 )+ (𝑘 𝜕𝑧 ) + 𝑞′′′ = 0
𝑟 𝜕𝑟 𝜕𝑟 𝜕𝑧

It can be seen from the above equation that the temperature T depends on two space variables
r and z and does not depend on time. There is the heat generation term appearing in the equation
and the thermal conductivity k appears within the differential symbol ∂/∂r and ∂/∂z. Hence

59
the problem is two-dimensional steady state conduction with heat generation in a medium of
variable thermal conductivity.

Example2.14. Consider a medium in which the heat conduction equation is given in its
simplest form as
1 𝜕 𝜕𝑇 1 𝜕2 𝑇 1 𝜕𝑇
(𝑟 2 ) + 𝑟 2 𝑠𝑖𝑛2 𝜑 (𝜕𝜃2 ) = 𝛼 ( 𝜕𝑡 )
𝑟 2 𝜕𝑟 𝜕𝑟

(a) Is heat transfer steady state or unsteady state?


(b) Is heat transfer one-, two- or three-dimension?
(c)Is there heat generation in the medium?
(d)Is the thermal conductivity constant or variable?

Solution: The given differential equation is

1 𝜕 𝜕𝑇 1 𝜕2 𝑇 1 𝜕𝑇
(𝑟 2 ) + 𝑟 2 𝑠𝑖𝑛2 𝜑 (𝜕𝜃2 ) = 𝛼 ( 𝜕𝑡 )
𝑟2 𝜕𝑟 𝜕𝑟

It can be seen from the given equation that the temperature T depends two space variables r
and θ and it also depends on the time variable t. There is no heat generation term appearing in
the given equation . Also the thermal conductivity k do not appear within the differential
symbol. Hence the given equation represents two-dimensional, unsteady state conduction in a
medium of constant thermal conductivity and the medium is not generating any heat.

Example2.15. Consider the north wall of a house of thickness L. The outer surface of the wall
exchanges heat by both convection and radiation.The interior of the house is maintained at a
uniform temperature of Ti, while the exterior of the house is at a uniform temperature T0. The
sky, the ground, and the surfaces of the surrounding structures at this location can be modeled
as a surface at an effective temperature of Tsky for radiation heat exchange on the outer
surface.The radiation heat exchange between the inner surface of the wall and the surfaces of
the other walls, floor and ceiling are negligible.The convective heat transfer coefficient for the
inner and outer surfaces of the wall under consideration are hi and h0 respectively.The thermal
conductivity of the wall material is K and the emissivity of the outer surface of the wall is ‘ε0’.
Assuming the heat transfer through the wall is steady and one dimensional, express the
mathematical formulation (differential equation and boundary conditions) of the heat
conduction problem

60
Schematic:

Surface in
contact with
fluid at Ti x
and surface L T (x)
heat transfer
coefficient hi qradiation
surface in contact with fluid
at T0 and surface heat
qconduction transfer coefficient h0

qconvection
ε0

Solution: The problem is one-dimensional steady state conduction without any heat generation
and the wall is of constant thermal conductivity. Hence the governing differential equation is
𝑑2 𝑇
= 0.
𝑑𝑥 2

The boundary conditions are:

𝜕𝑇
(i) at x = 0, hi [Ti – T |x = 0 ] = − k (𝜕𝑥 ) ;
𝑥=0

(ii) at x = L, qconduction = qconvection + qradiation

𝜕𝑇
Or: − k (𝜕𝑥 ) = h0[T|x = L − T0] + ε0 σ [{T|x = L}4 – Tsky4}
𝑥=𝐿

CHAPTER 3

61
ONE DIMENSIONAL STEADY STATE CONDUCTION
3.1. Introduction:- In this chapter the problems of one-dimensional steady state conduction
without and with thermal energy generation in slabs, cylinders and spheres and subjected to
different types of boundary conditions are analyzed to determine the temperature distribution
and rate of heat flow. The concept of thermal resistance is introduced and the use of this
concept, for solving conduction in composite layers is illustrated. The problem of critical
thickness of insulation for cylinder and sphere are also analyzed. The effects of variable
thermal conductivity on temperature distribution and rate of heat transfer are also studied.
Finally the problems of one dimensional heat conduction in extended surfaces (fins) subjected
to different types of boundary conditions are examined.

3.2. Conduction Without Heat Generation

3.2.1. The Plane Wall (The Infinite Slab):- The statement of the problem is to determine the
temperature distribution and rate of heat transfer for one dimensional steady state conduction
in a plane wall without heat generation subjected to specified boundary conditions.

T = T(x)

T1 T2
Qx
R = L /(Ak)

L
Figure 3.1: One dimensional steady state conduction in a slab

The governing equation for one − dimensional steady state conduction without heat
generation is given by

d2 T
=0 (3.1)
dx2

Integrating Equation(3.1) twice with respect to x we get

T = C1x + C2 (3.2)

62
where C1 and C2 are constants which can be evaluated by knowing the boundary conditions.

Plane wall with specified boundary surface temperatures:- If the surface at x = 0 is


maintained at a uniform temperature T1 and the surface at x = L is maintained at another
uniform temperature T2, then the boundary conditions can be written as follows:
(i) at x = 0, T(x) = T1 ; (ii) at x = L, T(x) = T2.

Condition (i) in Equation(3.2) gives T1 = C2.

Condition (ii) in Equation (3.2) gives T2 = C1L + T1

(T2 − T1 )
Or: C1 =
L

Substituting for C1 and C2 in Equation (3.2), we get the temperature distribution in the plane
wall as
𝑥
T(x) = (T2 – T1) + T1
𝐿

T(x)− T1 x
Or: =L (3.3)
T2 − T1

Expression for Rate of Heat Transfer:

The rate of heat transfer at any section x is given by Fourier’s law as

dT
Qx = − k A(x) ( dx )

For a plane wall A(x) = constant = A. From Equation (3.3), dT/dx = (T2 – T1) / L.

(T1 − T2 )
Hence: Qx = kA (3.4)
L

Concept of thermal resistance for heat flow:

It can be seen from the above equation that Qx is independent of x and is a constant.
Equation(3.4) can be written as:

(T1 − T2 ) (T1 − T2 )
Qx = L = (3.5)
( ) R
kA

L
Where: R = kA (3.5a)

Equation (3.5) is analogous to Ohm’s law for flow of electric current. In this equation

63
(T1 – T2) can be thought of as “thermal potential”, R can be thought of as “thermal
resistance”,so that the plane wall can be represented by an equivalent “thermal circuit” as
shown in Figure3.1.The units of thermal resistance R is K∙W−1.

Plane wall whose boundary surfaces subjected to convective boundary conditions:

Surface in contact with a fluid


at To with heat transfer
coefficient ho

Surface in contact with a fluid


at Ti with heat transfer
x coefficient hi
L
T1 T2
Rci R Rco
Ti
To
Qx Qx Qx Qx

Figure3.2: Thermal Circuit for a plane wall with convective boundary conditions

Let T1 be the surface temperature at x = 0 and T2 be the surface temperature at x = L.


If we assume that Ti > To, then for steady state conduction heat will transfer by convection
from the fluid at Ti to the surface at x = 0, then it is conducted across the
plane wall and finally heat is transferred by convection from the surface at x = L to the fluid
at To.
The expression for rate of heat transfer Qx can be written as follows:

(Ti − T1 )
Qx = hi A [Ti – T1] = 1
hiA

(Ti − T1 )
Or: Qx = (3.6a)
Rci

1
Rci = h is called thermal resistance for convection at the surface at x = 0.
iA

(T1 − T2 )
Similarly: Qx = (3.6b)
R

L
where R = Ak is the thermal resistance offered by the wall for conduction and
(T2 − T0 )
Qx = (3.6c)
Rc0

64
1
Where Rco = is the thermal resistance offered by the fluid at the surface at x = L for
h0A
convection. It follows from Equations (3.6a), (3.6b) and (3.6c) that

(Ti − T1 ) (T1 − T2 ) (T2 − T0 )


Qx = = =
Rci R Rc0

(Ti − To )
Or: Qx = R (3.7)
ci + R+ Rc0

3.2.2. Radial Conduction in a Hollow Cylinder:

The governing differential equation for one-dimensional steady state radial conduction in a
hollow cylinder of constant thermal conductivity and without thermal energy generation is
given by Equation(2.10b) with n = 1: i.e.,

𝑑 𝑑𝑇
[𝑟 ]=0 (3.8)
𝑑𝑟 𝑑𝑟

Integrating the above equation once with respect to ‘r’ we get

r (dT / dr) = C1

Or: (dT / dr) = C1/ r

Integrating once again with respect to ‘r’ we get

T(r) = C1 ln r + C2 (3.9)

where C1 and C2 are constants of integration which can be determined by knowing the
boundary conditions of the problem.

Hollow cylinder with prescribed surface temperatures: Let the inner surface at r = r1 be
maintained at a uniform temperature T1 and the outer surface at r = r2 be maintained at another
uniform temperature T2 as shown in Figure 3.3.

Substituting the condition at r1 in Equation(3.9) we get

T1 = C1 ln r1 + C2 (3.10a)

and the condition at r2 in Equation (3.9) we get

T2 = C1 ln r2 + C2 (3.10b)

Solving for C1 and C2 from the above two equations we get,

65
(T − T ) (T1 − T2 )
C1 = (ln r1 −ln2r ) = r
1 2 ln( 1 )
r2

(T1 − T2 )
And: C2 = T1 − r ln r1
ln( 1 )
r2

Substituting these expressions for C1 and C2 in Equation (3.9) we have,

(T1 − T2 ) (T1 − T2 )
T(r) = r ln r + T1 − r ln r1
ln( 1 ) ln( 1 )
r2 r2

r
(T(r)− T1 ) ln( )
r1
Or: = r (3.11)
(T2 − T1 ) ln( 2 )
r1

T2
T1

r2
r1

Figure3.3: Hollow cylinder with prescribed surface temperatures

Equation (3.11) gives the temperature distribution with respect to the radial direction in a
hollow cylinder. The plot of Equation (3.11) is shown in Figure 3.4.

Expression for rate of heat transfer:- For radial steady state heat conduction in a hollow
cylinder without heat generation, energy balance equation gives

Qr = Qr|r = r1 = Qr|r = r2

dT
Hence: Qr = − k [A(r) dr ] (3.12)
r= r1

Now A(r) |r = r1 = 2 π r1 L .From Equation (3.11) we have

66
𝑑𝑇 (T2 − T1 ) 1
= r ×𝑟
𝑑𝑟 ln( 2 )
r1

𝑑𝑇 (T2 − T1 ) 1
Hence: [ 𝑑𝑟 ] = r ×𝑟
𝑟= 𝑟1 ln( 2 ) 1
r1

Substituting the expressions for A(r)|r = r1 and (dT / dr)|r = r1 in Equation (3.12) we get the
expression for rate of heat transfer as

2πLk (T1 − T2 )
Qr = r (3.13)
ln( 2 )
r1

Thermal resistance for radial conduction in a hollow cylinder: Equation 3.13 can be written
as :

(T1 − T2 )
Qr = (3.14a)
R

(T – T1)
(T2 – T1)

0 r / r1
1.0 r2 / r1

Figure 3.4:Radial temperature distribution for a hollow cylinder with


prescribed surface temperatures at inner and outer radii

r
ln( 2 )
r1
Where: R= (3.14b)
2πLk

Hollow cylinder with convective boundary conditions at the surfaces: Let for the hollow
cylinder, the surface at r = r1 is in contact with a fluid at temperature Ti with a surface heat
transfer coefficient hi and the surface at r = r2 is in contact with another fluid at a
temperature To as shown in Figure3.5.By drawing the thermal circuit (see Figure 3.5(b))for

67
this problem and using the concept of thermal resistance it is easy and straight forward to
write down the expression for the rate of heat transfer as shown below:

(Ti − T1 )
Qr = hiAi(Ti – T1) = 2π r1L hi (Ti – T1) = (3.6a)
Rci

1
where Rci = 2πr
1 Lhi

(T1 − T2 )
Also: Qr = (3.6b)
R

r
ln( 2 )
r1
where: R=
2πLk

(T2 − To )
and: Qr = (3.6c)
Rco

1
where: Rco = 2πr .
2 Lho

Surface in
contact with
fluid at To and Surface in contact With fluid at Ti and
heat transfer Surface heat transfer Coefficient hi
coefficient ho

Figure3.5(a).Schematic for radial conduction in a


hollow cylinder subjected to convective boundsry
condition
r1
r2
Ti To

Qr
Rci R Rco

Figure 3.5(b). Thermal network radial conduction in a hollow cylinder


subjected to convective boundsry condition

It follows from Equations (3.6a),(3.6b) and (3.6c) that,

68
(Ti − T1 ) (T1 − T2 ) (T2 − To )
Qr = = =
Rci R Rco

(Ti − To )
Or: Qr = (R (3.16)
ci + R+ Rco )

3.2.3. Radial Conduction in a Hollow Sphere:

The governing differential equation for one-dimensional steady state radial conduction in a
hollow sphere without thermal energy generation is given by Equation(2.10b) with n = 1: i.e.,

d dT
[r 2 ( dr )] = 0 (3.17)
dr

Integrating the above equation once with respect to ‘r’ we get

dT
r2 = C1
dr

dT 𝐶
Or: = 𝑟 12
dr

Integrating once again with respect to ‘r’ we get


𝐶1
T(r) = − + C2 (3.18)
𝑟

where C1 and C2 are constants of integration which can be determined by knowing the
boundary conditions of the problem.

Hollow sphere with prescribed surface temperatures:

(i) Expression for temperature distribution:-Let the inner surface at r = r1 be maintained at a


uniform temperature T1 and the outer surface at r = r2 be maintained at another uniform
temperature T2 as shown in Figure 3.6.

The boundary conditions for this problem can be written as follows:

(i) at r = r1, T(r) = T1 and (ii) at r = r2, T(r) = T2.


𝐶1
Condition (i) in Equation (3.18) gives T1 = − + C2 (3.19a)
𝑟1

𝐶
Condition (ii) in Equation (3.18) gives T2 = − 𝑟1+ C2 (3.19b)
2

69
Solving for C1 and C2 from Equations (3.19a) and (3.19b) we have,

(𝑇1 − 𝑇2 ) (𝑇1 − 𝑇2 )
C1 = 1 1 and C2 = T1 + 1 1
[ − ] 𝑟1 [ − ]
𝑟2 𝑟1 𝑟2 𝑟1

Surface at temperature T2

Surface at temperature T1
r2

r1

Figure 3.6: Radial conduction in a hollow sphere with prescribed


surface temperatures

Substituting these expressions for C1 and C2 in Equation (3.18) we get,

(𝑇1 − 𝑇2 ) (𝑇1 − 𝑇2 )
𝑟 𝑟1
T(r) = 1 1 + T1 + 1 1
[ − ] [ − ]
𝑟2 𝑟1 𝑟2 𝑟1

1 1
𝑇(𝑟)− 𝑇1 [− ]
𝑟2 𝑟
= 1 1 (3.20)
𝑇2 − 𝑇1 [ − ]
𝑟2 𝑟1

(ii) Expression for Rate of Heat Transfer:- The rate of heat transfer for the hollow sphere is
given by

𝑑𝑇
Qr = −k A(r) ( 𝑑𝑟 ) (3.21)

Now at any radius for a sphere ,A(r) = 4π r2 and from Equation (3.20)

𝑑𝑇 (𝑇1 − 𝑇2 ) 1
(𝑑𝑟 ) = 1 1 × 𝑟2
[ − ]
𝑟2 𝑟1

Substituting these expressions in Equation (3.21) and simplifying we get

70
4 π k r1 r2 [T1 − T2 ]
Qr = (r2 − r1 )
(3.22)

(T1 − T2 )
Equation(3.22) can be written as: Qr = (3.23a)
R

Where R is the thermal resistance for the hollow sphere and is given by
𝒓 −𝒓
R = 4 π𝟐k r 𝟏r (3.23b)
1 2

Hollow sphere with convective conditions at the surfaces:- Figure 3.7 shows a hollow
sphere whose boundary surfaces at radii r1 and r2 are in contact with fluids at temperatures Ti
and T0 with surface heat transfer coefficients hi and h0 respectively.

Surface in contact with


fluid at T0 and surface
heat transfer coefficient h0
r2
Surface in contact with
fluid at Ti and surface heat
transfer coefficient hi
r1

Figure 3.7: Radial conduction in a hollow sphere with convective


conditions at the two boundary surfaces

The thermal resistance network for the above problem is shown in Figure3.8.

Qci = Qr = Qco (3.24)

Where Qci = heat transfer by convection from the fluid at Ti to the inner surface of the hollow
sphere and is given by

Ti − T1
Qci = hi Ai [Ti – T1] = (3.25)
Rci

71
Qr Qco
Ti Qci To
Rci R Rco
Figure 3.8: Thermal circuit for a hollow sphere with convective boundary conditions

where T1 = the inside surface temperature of the sphere and

Rci = 1 / (hiAi) = the thermal resistance for convection for the inside surface
1
Or: Rci = 4 π r2 h (3.25b)
1 1

Qr = Rate of heat transfer by conduction through the hollow sphere

T1 − T2 𝒓 −𝒓
Or: Qr = with R = 4 π𝟐k r 𝟏r
R 1 2

and Qco = Rate of heat transfer by convection from the outer surface of the sphere to the outer
fluid and is given by

T2 − To
Qco = ho Ao [T2 – To] = (3.26a)
Rco

Where T2 = outside surface temperature of the sphere and

Ao = outside surface area of the sphere = 4 π r22 so that


1
Rco = 4 π r2 h (3.26b)
2 o

Now Equation (3.24) can be written as:

Ti − T1 T1 − T2 T2 − To
Qr = = =
Rci R Rco

Ti − To
Or: Qr = R (3.27)
ci + R+ Rco

3.2.4. Steady State conduction in composite solids:

There are many engineering applications in which heat transfer takes place through a medium
composed of several different layers, each having different thermal conductivity. These layers
may be arranged in series or in parallel or they may be arranged with combined series-parallel

72
arrangements. Such problems can be conveniently solved using electrical analogy as illustrated
in the following sections.

Composite Plane wall:- (i) Layers in series: Consider a plane wall consisting of three layers
in series with perfect thermal contact as shown in Figure 3.10.The equivalent thermal
resistance network is also shown. If Q is the rate of heat transfer through an area A of the
composite wall then we can write the expression for Q as follows:

L1 L2 L3

Surface in
contact Surface in contact with a fluid
with fluid at T0 and surface heat
at Ti and k1 k2 k3 transfer coefficient ho
surface
heat
transfer
coefficient
hi
T1 T2 T3 T4

Rci R1 R2 R3 Rco

Q Q

Ti T1 T2 T3 T4 To

Figure 3.10: A composite plane wall with three layers in series and the equivalent
thermal resistance network
Ti − T1 T1 − T2 T2 − T3 T3 − T4 T4 − Tco
Q= = = = =
Rci R1 R2 R3 Rco

Ti − To
Or: Q=R (3.28)
ci + R1 + R2 + R3 + Rc0

Overall heat transfer coefficient for a composite wall:- It is sometimes convenient to


express the rate of heat transfer through a medium in a manner which is analogous to the
Newton’s law of cooling as follows:

If U is the overall heat transfer coefficient for the composite wall shown in Figure (3.10) then

Q = U A (Ti – To) (3.29)

Comparing Equation (3.28) with Equation (3.29) we have the expression for U as

73
1
U = A(R (3.30)
ci + R1 + R2 + R3 + Rc0 )

Substituting the expressions for the resistances we get,

1
U= 1 𝐿 𝐿 𝐿 1
𝐴[ + 1+ 2+ 3+ ]
ℎ𝑖𝐴 𝐴𝑘1 𝐴𝑘2 𝐴𝑘3 ℎ𝑜𝐴

1
Or: U= 1 𝐿 𝐿2 𝐿 1
(3.31)
[ + 1 + + 3+ ]
ℎ𝑖 𝑘1 𝑘2 𝑘3 ℎ𝑜

(ii) Layers in Parallel:- Figure3.11 shows a composite plane wall in which three layers are
arranged in parallel. Let ‘b’ be the dimension of these layers measured normal to the plane of
the paper. Let one surface of the composite wall be in contact with a fluid at temperature Ti
and surface heat transfer coefficient hi and the other surface of the wall be in contact with
another fluid at temperature To with surface heat transfer coefficient ho. The equivalent thermal
circuit for the composite wall is also shown in Figure 3.11. The rate of heat transfer through
the composite wall is given by

Q = Q1 + Q2 + Q3 (3.32)

where Q1 = Rate of heat transfer through layer 1,

Q2 = Rate of heat transfer through layer 2, and

Q3 = Rate of heat transfer through layer 3.

T1 − T2
Q1 = (3.33a)
R1

L
Where: R1 = bH .
1 k1

T1 − T2
Similarly: Q2 = (3.33b)
R2

L
Where: R2 = ,
bH2 k2

T1 − T2
And: Q3 = (3.33c)
R3

L
Where: R3 = bH .
3 k3

74
L H1

Surface in
contact with k1 H2
fluid at Ti
with heat
transfer
coefficient hi
k2
H3

k3
b Suface in contact
with fluid at To and
surface heat transfer
Q1 coefficient ho

R1
Ti
T1 Q2 T2 To
Rci R2 Rco
Q
Q3
R3

FFigure 3.11: Schematic and equivalent thermal circuit for a composite wall with
layers in parallel

Substituting these expressions in Equation (3.32) and simplifying we get

(Ti − T1 ) (T1 − T2 ) (T1 − T2 ) (T1 − T2 ) (T2 − To )


Q= = + + =
Rci R1 R2 R3 Rco

(Ti − T1 ) (T1 − T2 ) (T2 − To ) (Ti − To )


Or: Q= = = =R (3.34)
Rci Re Rco ci + Re + Rco

1 1 1 1
Where: = +R +R (3.35)
Re R1 2 3

Composite Coaxial Cylinders : Figure 3.12 shows a composite cylinder having two layers
in series. The equivalent thermal circuit is also shown in the figure.The rate of heat

75
transfer through the composite layer is given by
k2

k1
r2 r3
r1

Surface in contact with


Surface in contact with fluid at Ti and surface
fluid at To and surface heat transfer coefficient
heat transfer coefficient h0 hi

Ti T1 T2 T3 To

Q Rci R1 R2 Rco

Figure 3.12: Schematic and thermal circuit diagrams for a composite


cylinder
(Ti − T1 ) (T1 − T2 ) (T2 − T3 ) (T3 − To )
Q= = + +
Rci R1 R2 Rco

(Ti − To )
Or: Q=R (3.36a)
ci + R1 + R2 + Rco

r r
1 1 ln( 2 ) ln( 3 ) 1 1
r1 r
Where: Rci = h A = 2πr ; R1 = 2πLk ; R2 = 2πLk2 ; Rco = h = 2πr . (3.36b)
i i 1 Lhi 1 2 o Ao 3 Lho

The above expression for Q can be extended to any number of layers.

Overall Heat Transfer Coefficient for a Composite Cylinder:- For a cylinder the area of
heat flow in radial direction depends on the radius r we can define the overall heat transfer
coefficient either based on inside surface area or based on outside surface area of the
composite cylinder. Thus if Ui is the overall heat transfer coefficient based on inside surface
area Ai and Uo is the overall heat transfer coefficient based on outside surface area Ao then

Q = UiAi (Ti – To) (3.37)


From equations (3.36a) and (3.37) we have,

76
(Ti − To )
UiAi(Ti – To) = R
ci + R1 + R2 + Rco

Substituting the expressions for Ai, Rci,R1,R2 and Rco in the above equation we have
1
2 π r1L Ui = r r
ln( 2 ) ln( 3 )
1 r1 r2 1
[ + + + ]
2 π r1 L hi 2πLk1 2πLk2 2 π r3 L ho

1
Or: Ui = 1 r r r r r 1
(3.38)
[ + 1 ln( 2 ) + 1 ln( 3 ) + 1 ]
hi k1 r1 k2 r2 r3 ho

Similarly it can be shown that


1
Uo = r 1 r r r r 1
(3.39)
[ 3 + 3 ln( 2 ) + 3 ln( 3 ) + ]
r2 hi k1 r1 k2 r2 ho

Composite Concentric Spheres:- Figure3.13 shows a composite sphere having two layers
with the inner surface of the composite sphere in contact with fluid at a uniform temperature
Ti and surface heat transfer coefficient hi and the outer surface in contact with another fluid at
a uniform temperature To and surface heat transfer coefficient ho. The corresponding thermal
circuit diagram is also shown in the figure.

k2

k1
r1 r2 r1

r3

Surface in contact with


fluid at Ti and surface
Surface in contact with heat transfer coefficient
fluid at To and surface hi
heat transfer coefficient h0

Ti T1 T2 T3 To

Q Rci R1 R2 Rco

Figure 3.13: Schematic and thermal circuit diagrams for a composite


sphere

Equation (3.36) is also applicable for the composite sphere of Figure 3.13 except that the
expression for individual resistance will be different. Thus

77
(Ti − To )
Q=R (3.36a)
ci + R1 + R2 + Rco

1 1 (r − r1 ) (r − r2 ) 1 1
Where: Rci = h A = 4πr2 h ; R1 = 4πk2 ; R3 = 4πk3 ; Rco = h = 4πr2 h . (3.40)
i i 1 i 1 r1 r2 2 r2 r3 o Ao 3 o

Example 3.1: Consider a plane wall 100 mm thick and of thermal conductivity 100
W/(m-K). Steady state conditions are known to exist with T1 = 400 K and T2 = 600 K.
Determine the heat flux (magnitude and direction) and the temperature gradient dT/dx
for the coordinate system shown in Figure P3.1.

T(x) T(x)
T(x)

T2 T2
T2
T1 T1
T1
L L L
x
x x
(a) (b) (c)

Figure P3.1.Schematic for problem 3.1


Schematic for example 3.1(a):

T(x), Solution:
(a) It can be seen from the figure that the temperature is
L increasing with increase in x: i.e., dT/dx is +ve.

dT (T2 − T1 ) (600−400)
Therefore dx = = = 2000 ℃ ∙m−1.
L 0.1
T2
T1 dT
Heat flux = qx = ─ k dx = ─ 100 × 2000 = −200,000 W∙m −2
− ve sign for qx indicates that heat transfer is taking place in
x the − ve direction of x.

= ─ 2 x 10 5 W / m2.

78
(b) Schematic for example 3.1(b):
T(x) Solution:It can be seen from the figure
shown that temperature is decreasing
with increase in x or in other words
dT/dx is – ve.
dT T − T 400 − 600
Therefore dx = 1 L 2 = 0.1

= ─ 2000 ℃∙m−1.

T2 dT
T1 qx = ─ k dx = ─ 100 × (─ 2000)
L = + 200,000 W∙ m −2.
+ ve sign for qx indicates that heat
x transfer is taking place in the + ve
direction of x.

(c) Schematic for example 3.1(c):


Solution: It can be seen from the figure
T(x) shown that the temperature is increasing with
dT
increase in x: i.e., dx is + ve.

dT (600−400)
Therefore dx = = 2000 ℃∙ m−1.
0.1

dT
Heat flux = qx = ─ k dx
T2
T1 = ─100 × 2000 = ─ 200,000 W∙m−2.
x
Negative sign in qx indicates that heat
transfer takes place in a direction opposite to
the + ve direction of x.

Example 3.2.Figure P3.2 shows a frustum of a cone (k = 3.46 W∙m−1∙K−1). It is of circular


cross section with the diameter at any x is given by D = ax, where a = 0.25. The smaller cross
section is at x1 = 50 mm and the larger cross section is at x2 = 250 mm. The corresponding
surface temperatures are T1 = 400 K and T2 = 600 K. The lateral surface of the cone is
completely insulated so that conduction can be assumed to take place in x-direction only.
(i) Derive an expression for steady state temperature distribution, T(x) in the solid and
(ii) calculate the rate of heat transfer through the solid.

79
Schematic:
T2 T1
.Known: X1 = 50 mm;X2 =
250 mm; D= ax with a =
0.25;T1 = 400 K; T2 = 600 K;
k = 3.46 W∙m−1∙K−1

D2 D1 Find: (i) T(x) ; (ii) Q


D
Assumptions: (i) Steady state
X1 conduction (ii) One
dimensional conduction
x through the cone; (iii) thermal
X2 conductivity is constant.

Figure P3.2.Figure for problem 3.2.

Solution: By Fourier’s law, the rate of heat transfer in x-direction across any plane at a
distance x from the origin ‘o’ is given by

dT
Qx = ─ k Ax ( dx )

For steady state conduction without heat generation Qx will be a constant. Also at any x,
D = ax. Therefore

πD2 dT
Qx = ─ k ( ) (dx )
4

π(ax)2 dT
Or: Qx = ─ k ( ) ( dx )
4
Separating the variables we get,

4 dx
dT = ─ (πka2 )Qx ( x2 )

Integrating the above equation we have,


T 4Q x dx
∫T dT = ─ (πkax2 ) ∫X ( x2 )
1 1

4Q 1 1
T – T1 = ─ (πkax2 ) [𝑥 − ] (1)
𝑋1

At x = X2, T = T2. Substituting this condition in Equation (1) and solving for Qx we get,

80
πa2 k (T2 − T1 )
Qx = ( ) 1 1 (2)
4 ( − )
X2 X1
Substituting this expression for Qx in Equation (1) we get the temperature distribution in the
cone as follows:
1 1
(T2 − T1 )( − )
x X1
T(x) = T1 + 1 1 (3)
( − )
X2 X1

Substituting the given numerical values for X1, X2, T1 and T2 in Equation(3) we get the
temperature distribution as follows:
1 1
(600−400)× ( − )
𝑥 0.05
T(x) = 400 + 1 1
( − )
0.25 0.05

Or: T(x) = 400 + 12.5 [ 20 – 1/x]

π ×(0.25)2 × 3.46 × [ 600 – 400


And: Qx = 1 1 = − 2.123 W
4×[ − ]
0.25 0.05

Example 3.3.A plane composite wall consists of three different layers in perfect thermal
contact. The first layer is 5 cm thick with k = 20 W∙m−1∙K−1,the second layer is10 cm thick
with k = 50 W∙m−1∙K−1and the third layer is 15 cm thick with k = 100 W∙m−1∙K−1.The outer
surface of the first layer is in contact with a fluid at 400 ℃ with a surface heat transfer
coefficient of 25 W∙m−2∙K−1while the outer surface of the third layer is exposed to an
ambient at 30 ℃ with a surface heat transfer coefficient of 15 W∙m−2∙K−1.Draw the
equivalent thermal circuit indicating the numerical values of the thermal resistances and
calculate the heat flux through the composite wall. Also calculate the overall heat transfer
coefficient for the composite wall.

Schematic:

L1 L2 L3

hi h0

k1 k2 k3

Ti
To
Q

Rci R1 R2 R3 Rc0

81
Known: L1 = 0.05 m ; L2 = 0.10 m ; L3 = 0.15 m ; k1 = 20 W∙m−1∙K−1;

k2 = 50 W∙m−1∙K−1; k3 = 100 W∙m−1∙K−1; hi = 25 W∙m−2∙K−1; h0 = 15 W∙m−2∙K−1;

Ti = 400 ℃; T0 = 30 ℃.

Find: (i) heat flux q; (ii) U

Assumptions: (i) Steady state conduction (ii) One dimensional conduction through the
composite wall; (iii) thermal conducvity of each layer is constant.
1 1
Solution: Rci = h A = 25 ×1 = 0.04 m2∙K∙W−1
i 1

L1 0.05
R1 = k = 20 ×1 = 0.0025 m2∙K∙W−1
1 A1

L2 0.10
R2 = k = 50 ×1 = 0.002 m2∙K∙W−1
2 A2

L3 0.15
R3 = k = 100 ×1 = 0.0015 m2∙K∙W−1
3 A3

1 1
Rco = h = 15 ×1 = 0.067 m2∙K∙W−1
o Ao

∑R = Rci + R1 + R2 + R3 + Rco = 0.04 + 0.0025 + 0.002 + 0.0015 + 0.067 = 0.113 m2∙K∙W−1.

(Ti − To ) (400−30)
Heat Flux through the composite slab = q = ∑R
= = 3274.34 W∙m −2.
0.113

If ‘U’ is the overall heat transfer coefficient for the given system then,

qA q 1 1
U = A(T − T ) = (T − T ) = ∑ 𝑅 = 0.113
i o i o

= 8.85 W∙m −2∙K−1.

Example 3.4. A composite wall consisting of four different materials is shown in Fig
P3.10. Using the thermal resistance concept determine the heat transfer rate per m2 of
the exposed surface for a temperature difference of 300 ℃between the two outer
surfaces. Also draw the thermal circuit for the composite wall.

82
Schematic:

T1 T4

k1 = 100 W∙m−1∙K−1;
k1 kk22 k4
k2 = 0.04 W∙m−1∙K−1;
2m
k3 = 20 W∙m−1∙K−1;

k3 k4 = 70 W∙m−1∙K−1
k3 1 m 1m

4cm 10 cm 5 cm

Known: k1 = 100 W∙m−1∙K−1; L1 =0.04 m; k2 = 0.04 W∙m−1∙K−1; L2 = 0.1 m;

k3 = 20 W∙m−1∙K−1; L3 = 0.1 m ; k4 = 70 W∙m−1∙K−1; L4 = 0.05 m;

W = Width of the wall perpendicular to the plane of paper = 1 m (assumed).

T1 – T4 = 300 ℃; A1 = A4 = 1 × 2 = 2 m2; A2 = A3 = 1 × 1 = 1 m2.

Find: (i) q; (ii) Thermal circuit

Assumptions: (i) Steady state conduction (ii) One dimensional conduction through the
composite wall; (iii) thermal conducvity of each layer is constant.

L1 0.04
Solution: R1 = k = 100 ×2 = 0.0002 ℃ ∙W−1
1 A1

L2 0.10
R2 = k = 0.04 ×1 = 2.5 ℃ ∙W−1
2 A2

L3 0.10
R3 = k = 20 ×1 = 0.005 ℃ ∙W−1
3 A3

L4 0.05
R4 = k = 70 ×2 = 0.00036 ℃ ∙W−1
4 A4

83
Q R2
R1 R4
R3
T4
T1

Figure: Thermal Circuit for the example 3.4.

R2 and R3 are resistances in parallel and they can be replaced by a single equivalent
resistance Re, where,
1 1 1 1 1
=R + = 2.5 +
Re 2 R3 0.005

Or: Re = 0.00499 ℃ ∙W−1

R1, Re and R4 are resistances in series so that,

(T1 − T4 ) 300
Q=R (0.002+0.00499+0.00036)
= 40.82 × 103 W = 40.82 kW.
1 + R e + R 4

Heat transfer per unit area of the exposed surface is given by

Q 40.82
q=A = = 20.41 kW.
1 2.0

Example 3.5. A composite cylindrical wall is composed of two materials of thermal


conductivity kA and kB. A thin electric resistance heater for which the interfacial contact
resistances are negligible separates the two materials. Liquid pumped through the inner
tube is at temperature Ti with the inside surface heat transfer coefficient hi. The outer
surface of the Composite wall is exposed to an ambient at a uniform temperature of To
with the outside surface heat transfer coefficient ho. Under steady state conditions a
uniform heat flux of qh is dissipated by the heater.
(a) Sketch the equivalent thermal circuit for the composite wall and express all thermal
resistances in terms of the relevant variables
(b)Obtain an expression that may be used to determine the temperature of the heater,Th.
(c)Obtain an expression for the ratio of heat flows to the outer and inner fluid, qo/qi.

84
Schematic:

ro heater
r1
ri
hi,Ti
Qi kA

Q0 ho,To

kB
Ri Rci Qi
Qtotal
Ro Rco Qo

Find: (b) Expression to determine Temperature of heater Th; (ii) q0 / qi

Assumptions: (i) Steady state conduction (ii) One dimensional radial conduction through the
composite wall; (iii) thermal conductivity of each layer is constant; (iii)No contact resistance
between the surfaces of the two layers

Solution: Refer the thermal circuit shown above.

ln (r1 /ri ) 1 1
Ri = ; Rci = h = 2πr Lh
2 π L kA i Ai i i

ln (ro /r1 ) 1 1
Ro = ; Rco = h = 2πr
2 π L kB o Ao o L ho

T −T T −T
Qi = (R h+ R i ) ; Qo = (R h+ R o )
i ci o co

Th − Ti Th − To
Hence: Qtotal = Qi + Qo = (Ri + Rci )
+ (Ro + Rco )

Th − Ti Th − To
Or: 2π r1L q h = ln (r1 /ri ) 1
+ ln (ro /r1 ) 1
( + ) ( + )
2 π L kA 2 π ri L hi 2 π L kB 2 π ro L ho

Th − Ti Th − To
Hence: qh = ln (r1 /ri ) r
+ ln (ro /r1 ) r
(r1 + 1 ) ( r1 + 1 )
kA ri hi kB ro ho

The temperature Th of the heater can be obtained from the above equation.

85
Qo (Th − To )/(Ro + Rco ) (T − T ) × (R + R )
= (Th − Ti )/(Ri + Rci )
= (T h− T o) × (R i+ Rci ) .
Qi h i o co

Substituting the expressions for various resistances and simplifying we get,


ln (r1 /ri ) 1
(Th − To ) × ( + )
Qo kA ri hi
Or: = ln (ro /r1 ) 1
Qi (Th − Ti ) × ( + )
kB ro ho

Example 3.6. A hollow aluminum sphere with an electrical heater in the centre is used to
determine the thermal conductivity of insulating materials. The inner and outer radii of the
sphere are 15 cm and 18 cm respectively and testing is done under steady state conditions with
the inner surface of the aluminum maintained at 250 ℃. In a particular test, a spherical shell
of insulation is cast on the outer surface of the aluminum sphere to a thikness of 12 cm. The
system is in a room where the air temperature is 20 ℃ and the convection coefficient is 30 W∙m
−2 −1
∙K . If 80 W are dissipated by the heater under steady state conditions, what is the thermal
conductivity of the insulating material?

Sketch for example 3.6:

ho,To
r3 Known: r1 = 0.15 m ; r2 =
r2 0.18 m ; r3 = 0.18 + 0.12 =
r1 0.3 m ;
k1 = 204 W∙m −1∙K−1 (from
tables); ho = 30 W∙m −2∙K−1;
T1
Q = 60 W; T1 = 250 ℃ ;
k1 To = 20 ℃.
Find: k2
k2

Assumptions: (i) Steady state conduction (ii) One dimensional radial conduction through
the composite wall; (iii) thermal conductivity of each layer is constant; (iii)No contact
resistance between the surfaces of the two layers

Solution:

(r − r1 ) 0.18−0.15
R1 = 4 π k2 = 4 × π ×204 ×0.15 ×0.18 = 4.335 × 10 – 4 ℃∙W−1.
1 r1 r2

1 1 1
Rco = = 4 π r2 h = 4 × π × (0.3)2 ×30 = 0.0295 ℃∙W−1.
Ao h0 3 o

(T1 − To )
Q=R .
1 + R2 + Rco

86
(T1 − To )
Or: R2 = − (R1 + R co )
Q

250−20
=( ) − (4.335 × 10 ─ 4 + 0.0295) = 2.412
80

(𝑟3 − 𝑟2 )
Therefore = 2.412 .
4 𝜋 𝑘2 𝑟3 𝑟2

(0.3− 0.18)
Or: = 2.412.
4 𝜋 𝑘2 ×0.18 × 0.3

Solving for k2 we get: k2 = 0.0734 W∙m −1∙K−1

Example 3.7. In a hollow sphere of inner radius 10 cm and outer radius 20, the inner surface
is subjected to a uniform heat flux of 1.6 × 10 5 W∙m− 2 and the outer surface is maintained at
a uniform temperature of 0 ℃.The thermal conductivity of the material of the sphere is 40
W∙m −1∙K−1 .Assuming one-dimensional radial steady state conduction determine the
temperature of the inner surface of the hollow sphere.

Schematic:

T0

R2
q0
R1

Find:[𝑇]𝑟=𝑅1

Assumption: (i)Steady state one dimensional radial conduction without heat generation; (ii)
thermal conductivity of the sphere is constant.

Solution: The governing equation for one-dimensional steady-state radial conduction in a


sphere without heat generation is given by

d dT
(r 2 dr ) = 0 (1)
dr

𝑑𝑇
The boundary conditions are: (i) at r = R1, ─ k [ 𝑑𝑟 ] = q0
𝑟=𝑅1

(ii) at r = R2 T(r) = 0.

87
Integrating Equation (1) w.r.t. r once, we get

dT
r2 ( dr ) = C1

dT C1
Or: = (2)
dr r2

Integrating once again w.r.t. r we get

C1
T(r) = ─ + C2 (3)
r

dT C
From Equation (2): [ dr ]r = R1 = R12
1

Hence condition (i) gives

C
─ k (R12 ) = q0
1

qo R21
Or: C1 = ─ k

C
Condition (ii) in Equation (3) gives, ─ R1 + C2 = 0
2

Or C2 = C1 / R2 = ─ (q0R12) / (kR2)

Substituting the expressions for C1 and C2 in Equation (3) we have,

qo R21 qo R21
T(r) = −
kr k R2

Substituting the numerical values for q0, k, R1 and R2 we have,

1.6 × 105 ×(0.1)2 1 1.6 × 105 ×(0.1)2 1


T(r) = × − ×
40 𝑟 40 0.2

40
Or: T(r) = − 200
𝑟
40
Therefore: T(r) |r = R1 = 0.1 − 200 = 200 ℃.

3.2.5. Thermal Contact Resistance: In the analysis of heat transfer problems for composite
medium it was assumed that there is “perfect thermal contact” at the interface of two layers.
This assumption is valid only the two surfaces are smooth and they produce a perfect contact
at each point.But in reality, even flat surfaces that appear smooth to the naked eye would be
rather rough when examined under a microscope as shown in Figure 3.14 with numerous .
peaks and valleys.

88
T2
T1
Rcont

LA LB

Gap between solids

T1 Enlarged view of the contact surface


Tc1

Tc2 T2
Figure3.14: Temperature drop
across a contact resistance

The physical significance of thermal contact resistance is that the peaks will form good thermal
contact, but the valleys will form voids filled with air.As a result the air gaps act as insulation
because of poor thermal conductivity of air.Thus the interface offers some resistance to heat
conduction and this resistance is called the “thermal contact resistance,Rcont”. The value of
Rcont is determined experimentally and is taken into account while analyzing the heat
conduction problems involving multi-layer medium.The procedure is illustrated by means of
a few examples below.

Example 3.8. A composite wall consists of two different materials A [k = 0.1 W∙m −1∙K−1] of
thickness 2 cm and B[ k = 0.05 W∙m −1∙K−1] of the thickness 4 cm. The outer surface of layer
A is in contact with a fluid at 2000C with a surface heat transfer coefficient of 15
W∙m −2∙K−1and the outer surface of layer B is in contact with another fluid at 50 0 C with a
surface heat transfer coefficient of 25 W∙m −2∙K−1 .The contact resistance between layer A and
layer B is 0.33 m2∙K∙W−1. Determine the heat transfer rate through the composite wall per unit
area of the surface. Also calculate the interfacial temperatures and the inner and outer surface
temperature.

89
Schematic and equivalent thermal circuit:

h0,T0 Known: Ti = 200 ℃; T0 = 50 ℃ ;


hi,Ti
Rcont
hi = 15 W∙m −2∙K−1; h0 = 25 W∙m −2∙K−1
kA kB
kA = 0.1 W∙m −1∙K−1;
LA LB
kB = 0.05 W∙m −1∙K−1;

Rcont = 0.33 m2∙K∙W−1.

T1 Find: (i) q; (ii) Tc1, Tc2, T1, T2


Tc1
Assumptions: (i) Steady state
Tc2 T2 conduction (ii) One dimensional
conduction through the composite wall;
(iii) thermal conductivity of each layer
is constant;

T1 Tc1 Tc2 T2
R1 Rcont Q
R1 R1
Solution: The equivalent thermal circuit is also shown in the figure above.
1 1
Rci = h A = 15 ×1 = 0.067 m2∙K∙W−1.
i A

LA 0.02
R1 = k = = 0.2 m2∙K∙W−1.
A AA 0.1 ×1

LB 0.04
R2 = k = 0.05 ×1 = 0.8 m2∙K∙W−1.
B AB

1 1
Rco = h = 25 ×1 = 0.04 m2∙K∙W−1.
o AB

∑R = Rci + R1 + Rcont + R2 + Rco = 0.067 + 0.2 + 0.33 + 0.8 + 0.04

= 1.437 m2∙K∙W−1.

(Ti − To ) (200−50)
Heat flux: q= ∑R
= = 104.4 W∙m −2
1.437

(Ti − T1 )
Also: q= Rci

Or: T1 = Ti – q Rci = 200 – (104.4 ×0.067) = 193℃.

90
Similarly: Tc1 = T1 – q R1 = 193 – (104.4 × 0.2) = 172.12℃.

Tc2 = Tc1 – q Rcont = 172.12 – (104.4 × 0.33) = 137.67℃.

T2 = Tc2 – q R2 = 137.67 – (104.4 × 0.8) = 54.15 ℃.

Check : To = T2 – q Rco = 54.15 – (104.4 × 0.04) = 49.97 ℃

Example 3.9. A very thin electric heater is wrapped around the outer surface of a long
cylindrical tube whose inner surface is maintained at 5 ℃. The tube wall has inner and
outer radii of 25 mm and 75 mm respectively and a thermal conductivity of 10 W∙m −1∙K−1.
The thermal contact resistance between the heater and the outer surface of the tube
per unit length is 0.01 m∙K∙ W−1. The outer surface of the heater is exposed to a fluid
with a temperature of – 10 0C and a convection coefficient of 100 W∙m −2∙K−1. Determine
the heater power required per metre length of the tube to maintain a heater
temperature of 25 ℃.

Schematic and equivalent thermal circuit:

h,T∞
Heater
r2
r1
Qtube T1 T2

Qamb
Qamb
Rco
Qtotal
Qtube Rcont R

Known: r1 = 0.025 m ; r2 = 0.075 m ; k = 10 W∙m −1∙K−1; T∞ = ─ 10℃;

Rcont =0.01 m∙K∙ W−1. h = 100 W∙m −2∙K−1T1 = 5 ℃ ; T2 = 25 ℃.

Find: Qtotal per metre length

91
Assumptions: (i) Steady state conduction (ii) One dimensional conduction through the
composite wall; (ii) thermal conductivity of each layer is constant.

1 1
Solution: Rco = hA = 100 ×2 × 𝜋 ×0.075 ×1 = 0.0212 m∙K∙ W−1.
0

r 0.075
ln( 2 ) ln( )
R = 2 π L k = 2 × 𝜋 ×1 ×10 = 0.0175 m∙K∙ W−1.
r1 0.025

(T2 − T∞ ) [25−(−10)]]
Qamb = = = 1651 W∙m−1.
Rco 0.0212

(T2 − T1 ) (25−5)
Qtube = R = (0.01+0.0175) = 727.3 W∙m−1.
cont + R

Power required = Qtotal = Qamb + Qtube = 1650 + 727.3 = 2378.3 W∙m−1.

3.3. One Dimensional Steady State Conduction With Heat Generation:The governing
equations for one-dimensional steady state conduction in solids of constant thermal
conductivity, which are generating heat is given as follows.

d2 T 𝑞′′′
(i) Plane wall : + =0 (3.41)
dx2 𝑘

1 d dT 𝑞′′′
(ii) Radial conduction in cylinder: r dr (r )+ =0 (3.42)
dr 𝑘

1 d dT 𝑞′′′
(iii) Radial conduction in spheres: r2 dr (r 2 )+ =0 (3.43)
dr 𝑘

The following examples illustrate the method of analysis of steady state heat conduction
in solids generating heat.

Example 3.10. A plane wall of thickness L and thermal conductivity k has one of its
surfaces insulated and the other surface is kept at a uniform temperature T1. Heat is
generated in the wall at a rate q’’’(x) where q’’’(x) = q0 cos{(πx) / (2L)} W∙m-3,where q0 is a
constant.
(a) Develop an expression for one-dimensional steady state temperature distribution in the
solid and (b) develop an expression for the temperature of the insulated surface.

92
Schematic:

L
Insulated T1

T = T(x)
q’’’= q0 cos (πx/2L)
x
𝜋𝑥
Known: (i) Expression for heat generation q’’’ = q0 cos ( ) W∙m-3;(ii) the surface at x = 0
2𝐿
𝑑𝑇
is insulated i.e. [𝑑𝑥 ] = 0; (iii) T = T1 at x = L
𝑥=0

Find: (a) expression for temperature distribution in the solid T(x); (ii) [𝑇]𝑥=0

Assumptions: (i) One dimensional steady state conduction ; (ii) solid is of constant thermal
conductivity

Solution: The governing equation to determine T(x) is given by

d2 T 𝑞′′′
+ =0
dx2 𝑘

Substituting the given expression for q’’’ the above equation reduces to
πx
d2 T qo cos ( )
2L
+ =0 (1)
dx2 k

dT
The boundary conditions are ; (i) at x = 0 = 0 (Insulated)
dx

(ii) at x = L, T = T1

Integrating Equation(1) once w.r.t. x we get,

dT π qo πx
= sin (2L)+ C1 (2)
dx 2L k

Integrating once again w.r.t. x we get

π 2q πx
o
T(x) = (2 L) cos (2L) + C1x + C2 (3)
k

Condition (i) in Equation (2) gives 0 = 0 + C1 or C1 = 0.

Condition (ii) in Equation (3) gives T1 = 0 + 0 + C2 or C2 = T1.

93
Substituting the values of C1 and C2 in Equation (3) we get the temperature distribution as

π 2q πx
o
T(x) = (2 L) cos (2L) + T1 (4)
k

At the insulated surface (x = 0) the temperature is given by

π 2q
o
T(x) |x=0 = (2 L) + T1.
k

Example 3.11. A long cylindrical rod of radius 5 cm and k = 10 W∙m −1∙K−1contain


radioactive material which generates heat uniformly within the cylinder at a rate of
3×10 5W∙m-3. The rod is cooled by convection from its cylindrical surface by ambient air
at 50 ℃ with a heat transfer coefficient of 60 W∙m-2∙K−1. Assuming one-dimensional
radial conduction determine the temperature at the centre of the rod as well as at the
outer surface of the rod.

Schematic:

T = T(r)
q’’’
h, T∞

Known: Outer radius of cylinder = R = 5 cm; k = 10 W∙m −1∙K−1; q’’’ = 3×10 5 W∙m-3;

T∞ = 50 ℃; h = 60 W∙m-2∙K−1.

Find: (i) T/r = 0 ; (ii) T/r = R

Assumptions: (i) one dimensional radial conduction; (ii) steady state conduction; (iii)
thermal conductivity of the rosd is constant

Solution: The governing differential equation to determine one-dimensional steady state


radial conduction with heat generation is given by

1 d dT 𝑞′′′
(r )+ =0 (1)
r dr dr 𝑘

dT
The boundary conditions are : (i) at r = 0, = 0 (axis of symmetry);
dr

94
dT
(ii) at r= R, ─ k [ dr ] = h [T |r= ─ T∞]
r=R

d dT 𝑞 ′′′ 𝑟
Equation (1) can be written as: (r )+ =0
dr dr 𝑘

Integrating once w.r.t. r we get,


dT 𝑞 ′′′ 𝑟 2
r + = C1
dr 2𝑘

dT q′′′ r C1
Or: + = (2)
dr 2k r

Integrating once again w.r.t. r we have:

q′′′ r
T(r) = ─ + C1 ln r + C2 (3)
4k

Condition (i) in Equation (2) gives: C1 = 0.

𝑞 ′′′ 𝑅 2
From Equation (3) we have: T(r)|r=R = ─ + C2
4𝑘

dT q′′′ R
and from Equation (2) we have [ dr ] = ─
r=R 2k

Therefore condition (ii) gives:

q′′′ R 𝑞 ′′′ 𝑅 2
─ k [− ] = h [─ + C2 ─ T∞]
2k 4𝑘

q′′′ R q′′′ R2
Or: C2 = + + T∞
2h 4k

Substituting the expressions for C1 and C2 in Equation (3) we get the temperature distribution
in the cylinder as

q′′′ R2 r 2 q′′′ R
T(r) = T∞ + [1 – (R) ] + (4)
4k 2h

q′′′ R2 (3×105 )× (0.052 )


Now: = = 18.75 ℃
4k 4 ×10

q′′′ R (3×105 )×0.05


= = 125 ℃
2h 2 ×60

(3×105 )× (0.052 ) r 2 (3×105 )×0.05


T(r) = 50 + × [1 – (R) ] +
4 ×10 2 ×60

r 2
Or: T(r) = 175 + 18.75 [1 – (R) ]

95
At the centre T(r) |r=0 = 175 + 18.75 = 193.75 ℃.

At the surface T(r) |r=R = 175 + 18.75 [ 1 – 1] = 175 ℃.

Example 3.12. In a cylindrical fuel element for a gas-cooled nuclear reactor, the heat
generation rate within the fuel element due to fission can be approximated by the
equation
q’’’ = qo [1 – (r/R)2] W∙m−3,
where R is the outer radius of the fuel element and qo is a constant. The outer surface of the
cylinder is maintained at a uniform temperature To. Assuming one-dimensional radial
conduction obtain an expression for the temperature distribution in the element. If R = 2 cm,
k = 10 W∙m−1∙K−1, and qo = 1.16 × 10 5 W∙m-3, what would be the temperature difference
between centre temperature and the outer surface temperature.

Schematic:

T0

q’’’ = q0 [ 1 – (r/R)2]

Known: R = 2 cm; k = 10 W∙m−1∙K−;1qo = 1.16 × 10 5 W∙m-3.

Find: (T/r=0 – T/r=R)

Assumptions: (i) one dimensional steady state conduction; (ii) Thermal conductivity of the
cylinder is constant

Solution:The governing equation to determine the one-dimensional steady-state radial


temperature distribution in a cylinder with heat generation is given by

1 d dT 𝑞′′′
(r dr )+ 𝑘 = 0
r dr
Multiplying by r and substituting for q’’’ in the given expression we have

96
r 2
d dT qo r [1− ( ) ]
R
(r )+ =0
dr dr k
(1)

dT
Boundary conditions are: (i) at r = 0, = 0 (axis of symmetry);
dr

(ii) at r = R, T = T0.

Integrating Equation (1) w.r.t. r once we get

dT 𝑞𝑜 𝑟 2 𝑟4
r + [2 − ] = C1
dr 𝑘 4𝑅 2

dT 𝑞𝑜 r r3 C1
Or: + [2 − ]= (2)
dr 𝑘 4R2 r

Integrating once again w.r.t. r we have,

qo r2 r4
T(r) = − [4 − ] + C1 ln r + C2 (3)
k 16R2
Condition (i) in Equation(2) gives:

C1 = 0.

Condition (ii) in Equation (3) gives:


qo r2 R2
T0 = ─ [4 − ] + C2
k 16

3 q o R2
Therefore: C2 = T0 + 16 k

Substituting the expressions for C1 and C2 in Equation (3) we have,

qo r2 r4 3 q o R2
T(r) = − [4 − ] + T0 + 16
k 16R2 k

qo r2 r4 3 q o R2
Or: T(r) ─ T0 = − [4 − ] + T0 + 16
k 16R2 k

3 qo R2 3 ×(1.16 × 105 )× (0.022 )


Therefore: T(r) |r=0 ─ T0 = 16 = = 0.87 ℃
k 16 ×10

Example3.13. Develop an expression for one-dimensional radial steady state


temperature distribution in a solid sphere of radius R in which heat is generated at a
rate given by

97
q’’’ = qo [ 1 – (r/R)] W∙m-3.
Assume that the outer surface is maintained at a uniform temperature To.

Schematic for example 3.13:


Known:Outer surface temperature = T0;
Solid is generating heat according to the law
q’’’ = q0(1 – r/R)

T0 Find: One dimensional radial temperature


q’’’ = q0(1 – r/R) distribution T(r)

Assumptions: (i) steady state one


dimensional conduction; (ii) thermal
conductivity of the cylinder is constant

Solution: The governing differential equation to find the one-dimensional radial steady state
temperature distribution in a sphere with heat generation is given by

1 d dT 𝑞′′′
(r 2 )+ =0
r2 dr dr 𝑘

Multiplying throughout by r2 and substituting the given expression for q′′′ we have
r
d dT r2 q0 [1− ]
(r 2 )+ R
=0 (1)
dr dr k

dT
Boundary conditions are : (i) at r = 0, = 0 (axis of symmetry)
dr

(ii) at r = R, T = T0.

Integrating Equation (1) w.r.t. r once we get,

dT q0 r3 r4
r2 + [3 − ] = C1
dr k 4R

dT q0 r r2 C1
Or: =− [3 − ]+ (2)
dr k 4R r2

Integrating once again w.r.t. r we get,

q0 r2 r3 C1
T(r) = − [6 − ]− + C2 (3)
k 12R r

98
Condition (i) in Equation (2) gives: 0 = 0 + C1 / 0 or C1 = 0.

q0 R2 R2
Condition (ii) in Equation (3) gives: T0 = − [6 − ] + C2
k 12

q 0 R2
Or: C2 = T0 + 12 k

Substituting the expressions for C1 and C2 in Equation (3) we have,

q0 r2 r3 q 0 R2
T(r) = − [6 − ] + T0 +
k 12R 12 k

q 0 R2 r 2 r 3
Or: T(r) = T0 + [1 − 2 (R) + (R) ]
12 k

Example 3.14. A plane wall of thickness 2L is generating heat according to the law

q’’’ = q0 [1 – β(T – Tw)]

where qo, β, and Tw are constants and T is the temperature at any section x from the mid-
plane of the wall. The two outer surfaces of the wall are maintained at a uniform temperature
Tw. Determine the one-dimensional steady state temperature distribution, T(x) for the wall.

Schematic for example 3.14:


q’’’ = q0 [1 – β(T – Tw)]

Tw Tw

2L

Known: The two outer surfaces of the wall maintained at Tw; Solid is generating heat at a
rate given by q’’’ = q0 [1 – β(T – Tw)].

Find: One dimensional steady state temperature T(x)

Assumptions: (i) One dimensional steady state conduction; (ii) Solid is of constant thermal
conductivity

99
Solution: Governing differential equation for one-dimensional steady state conduction in a
plane wall with constant thermal conductivity and which is generating heat is given by
d2 T q′′′
+ =0
dx2 k

d2 T q0 [1− β(T− Tw )]
Substituting for q′′′ we have, + =0
dx2 k

Defining a new variable θ = T – Tw, the above equation can be written as:

d2 θ q0 [1− βθ]
+ =0
dx2 k

d2 θ q0
Or: – m2 θ = − (1a)
dx2 k

q0 β
where: m2 = (1b)
k

Equation(1a) is a second order linear non-homogeneous differential equation whose solution


is given by

θ (x) = θh(x) + θp(x) (2)

where θh(x) satisfies the differential equation,

d2 θh
- m2θh = 0 (3)
dx2

θh(x) = A1 e mx + A2 e – mx (4)

θp(x) satisfies the differential equation,

d2 θp q0
dx2
- m2θp = − k
(5)

The term qo/k makes the governing differential equation non-homogeneous. Since this is a
constant θp(x) is also assumed to be constant. Thus let.

θp(x) = C, where C is a constant. (6)

Substituting this solution in Equation (5) we get


q0
m 2C = k

q
Or: C = k m0 2

Substituting for m2 we get: C = 1 / β.

100
1
Hence: θp(x) = (7)
𝛽

The complete solution θ(x) is therefore given by


1
θ(x) = A1 e mx + A2 e − mx + 𝛽 (8)

The constants A1 and A2 in Equation(8) can be determined by using the two boundary
conditions, which are:

dT dθ
(i) at x = 0, = 0 (axis of symmetry) i.e., =0
dx dx

(ii) at x = L, T = Tw ; i.e., θ = 0


From Equation(8), = m[A1e mx – A2 e − mx ]
dx

Substituting condition (i) we get m[A1 – A2] = 0

Or: A1 = A2.
1
Substituting condition (ii) in Equation (8) we get A1[e mL + e − mL] = − 𝛽

(1/β)
Or: A1 = − [emL + e−mL ]

Substituting the expressions for A1 and A2 in Equation (8) we get the temperature distribution
in the plane wall as:
(1/β) 1
θ(x) = T(x) – Tw = − [emL + e−mL ] [emx + e−mx ] + 𝛽

1 (emx + e−mx )
Or: T(x) – Tw = 𝛽 [1 − (emL + e−mL )
]

3.4. Critical Radius of Insulation:- For a plane wall adding more insulation will result in a
decrease in heat transfer as the area of heat flow remains constant .But adding insulation to a
cylindrical pipe or a conducting wire or a spherical shell will result in an increase in thermal
resistance for conduction and at the same time will result in a decrease in the convection
resistance of the outer surface because of increase in surface area for convection. Therefore
the heat transfer may either increase or decrease depending on the relative magnitude of these
two resistances.

3.4.1.Critical Radius of Insulation for Cylinder:- Let us consider a cylindrical pipe of outer
radius rs maintained at a constant temperature of Ts. Let the pipe now be insulated with a
material of thermal conductivity k and outer radius r. Let the outer surface of the insulation be

101
in contact with a fluid at a uniform temperature T∞ with a surface heat transfer coefficient h.
Then the thermal circuit for this arrangement will be as shown in Figure3.15.

r Surface in contact with a fluid at T∞ and


surface heat transfer coefficient h
Ts
rs
T∞
Ts
Rins Rco Q

Figure3.15: Schematic of a cylindrical pipe covered with an insulation


and exposed to an ambient and the corresponding thermal circuit

The rate of heat transfer from the pipe to the ambient is given by,

(Ts − T∞ ) (T − T∞ )
Q= = ln( r s) (3.44)
Rins + Rco rs 1
+
2πLk 2πrLh

It can be seen from Equation (3.44) that if Ts and h are assumed not to vary with ‘r’ then Q
depends only on r and the nature of variation of Q with r will be as shown in Figure3.16.
The value of r at which Q reaches a maximum can be determined as follows:

(Ts − T∞ )
Equation (3.44) can be written as: Q=
F(r)

r
ln( ) 1
rs
Where: F(r) = + (3.45)
2πLk 2πrLh

dF
Hence for Q to be maximum, F(r) has to be minimum: i.e., dr = 0

Diffrentiating Equation (3.45) w.r.t. ‘r’ and equating it to zero we get,

dF 1 1 1 1
= 2πLk = 2πLh =0
dr r r2

k
Solving for r we get, r=h

102
Q

Qmax

Qbare

r
rs rcr = k / h

Figure3.16: Variation of Q with outer radius of insulation

This value of r is called “critical radius of insulation, rcr”.

k
Therefore rcr = h (3.46)

The above expression for critical radius of insulation has been arrived with the assumption
that the change in outer radius of insulation r will have no effect on the inside surface
temperature of insulation or on the outer surface heat transfer coefficient h. Sparrow
considered a heat transfer coefficient that varies as

h – r –m │Ts − T∞│n (3.47a)

For this case, it is found that the heat transfer is maximized when

(1 −m)k
r = rcr = (3.47b)
(1+n)h

(1−m)
The quantity is a correction factor (≤ 1) accounting for the r and ΔT dependencies of
(1+n)
h. As an example, consider forced convection flow across a cylinder. For Reynolds number
based on diameter between 4000 and 40,000, m = 0.382 and n=0. Correspondingly
Equation(3.47b) gives the correction factor

103
(1−m)
(1+n)
= 0.618
As a second example for free convection about a horizontal cylinder for which m = n = 0.25,
the correction factor will be 0.6.
It can be seen from Figure(3.16) that if the outer radius of the bare tube or bare wire is
greater than the critical radius then, any addition of insulation on the tube surface
decreases the heat loss to the ambient. But if the outer radius of the tube is less than the
critical radius , the heat loss will increase continuously with the addition of insulation
until the outer radius of insulation equals the critical radius. The heat loss becomes
maximum at the critical radius and begins to decrease with addition of insulation beyond
the critical radius.
The value of critical radius rcr will be the largest when k is large and h is
small. The lowest value of h encountered in practice is about 5 W∙m-2∙ K−1 for free
convection in a gaseous medium and the thermal conductivity of common insulating
materials is about 0.05 W∙m−1K−1. Hence the largest value of rcr that we may likely to
encounter is given by,
0.05
rcr = = 0.01 m = 1 cm.
5

The critical radius would be much less in forced convection (it may be as low as 1mm)
because of large values of h associated with forced convection. Hence we can insulate hot
water or steam pipes freely without worrying about the possibility of increasing the heat
loss to the surroundings by insulating the pipes.
The radius of electric wires may be smaller than the critical radius. Therefore, the
plastic electrical insulation may enhance the heat transfer from electric wires, there by
keeping their steady operating temperatures at lower and safer levels.

3.4.2.Critical Radius Insulation for a Sphere:- The analysis described above for cylindrical
pipes can be repeated for a sphere and it can be shown that for a sphere the critical radius
of insulation is given by

2k
rcr = (3.48)
h

A calculation for a sphere to account for variation of h and Ts with radius r similar to that of
a cylinder yields an expression for the critical radius as:
m
1− 2k
2
rcr = [ 1+n ] (3.48a)
h
m
1−
2
Thus the quantity [ 1+n ] can be treated as a correction factor similar to that obtained for the
cylinder.

Example 3.15. A conductor with 8 mm diameter carrying an electric current passes


through an ambient at 30 ℃ with a convection coefficient of 120 W∙m-2∙K−1. The
temperature of the conductor is to be maintained at 130℃. Calculate the rate of heat

104
loss per metre length of the conductor when (a) the conductor is bare and (b) conductor
is covered with bakelite insulation [k = 1.2 W∙m−1∙K−1 ] with radius corresponding to the
critical radius of insulation.

Schematic:
Ts
Dc ,k
D
Ts

h,T∞ h, T∞
(a) Conductor without Insulation. (b) Conductor with critical thickness of
Insulation
Known: D = 0.008 m; Ts = 130℃ ; for insulation k = 1.2 W∙m−1∙K−1; h = 120 W∙m-2∙K−;
T∞ = 30 ℃

Find: (a) Qbare ;(b) Qwith insulation

Assumptions: (i) one dimensional steady radial conduction (ii) the thermal conductivity of
insulation is constant (iii) by adding the insulation the outer surface temperature of the wire
will not change; (iv) the surface heat transfer coefficient for wire with and without insulation
remain same

Solution:(a) When the conductor is bare the rate of heat loss to the ambient is given by

Qbare = h πD L (Ts - T∞) = 120 × π ×0.008 × 1 × (130 – 30) = 301.6 W∙m−1.

(b) When the conductor is covered with critical thickness of insulation,

k 1.2
Dcr = 2 rcr = 2(h) = 2 × (120)= 0.02 m.

ln(Dcr /D) ln(0.02/0.008)


R insulation = = 2 × 𝜋 ×1.0 ×1.2 = 0.1215 m∙℃∙W−1.
2πLk

1 1 1
Rco = hA = π D = 𝜋 ×0.02 ×1 ×120 = 0.133 m∙℃∙W−1.
cr cr Lh

(Ts − T∞ ) (130−30)
Qwith insulation = R = 0.1215+0.133 = 392.93 W∙m−1.
insulation + Rco

Example 3.16. An electrical current of 700 A flows through a stainless steel cable
having a diameter of 5 mm and an electrical resistance of 6 ×10 ─ 4 ohms per metre
length of the cable. The cable is in an environment at a uniform temperature of 30 ℃
and the surface heat transfer coefficient of 25 W∙m-2∙ K−1.

105
(a) What is the surface temperature of the cable when it is bare?
(b)What thickness of insulation of k = 0.5 W∙m-1∙K−1will yield the lowest value of the maximum
insulation temperature? What is this temperature when this thickness is used?

Schematic:
Ts
Dc ,k
D
Ts

h,T∞ h, T∞

Known: Current flow through the cable = I = 700 A; diameter of cable = D = 5 mm;
Electrical resistance of cable / metre length = ρ = 6× 10−4 Ω/m; h= 25 W∙m-2∙K−1;
T∞ = 30℃; for insulation k =0.5 W∙m-1∙K−1
Find: (a) Ts when the wire is bare; (ii) thickness of insulation for lowest value of maximum
insulation temperature; (iii) lowest value of maximum insulation temperature

Assumptions: i) one dimensional steady radial conduction (ii) the thermal conductivity of
insulation is constant (iii) by adding the insulation the outer surface temperature of the wire
will not change; (iv) the surface heat transfer coefficient for wire with and without insulation
remain same

Solution: (a) When the cable is bare:

Power dissipated: Q = I2 Re = (700)2 × 6 × 10─ 4 = 294 W∙m−1.

Also: Q = hA(Ts - T∞) = (πD L) h (Ts - T∞)

Q 294
Or: Ts = T∞ + = 30 + 𝜋 ×0.005 ×1 ×25 = 779 ℃
(πD L)h

(b) When the cable is covered with insulation: Lowest value of maximum insulatiuon
temperature occurs when the outer radius of insulation is equal to critical radius.

k 0.5
Hence critical radius is: rcr = h = = 0.02 m∙K∙W−1.
25

ln(rcr /ro ) ln(0.02/0.0025)


Rinsulation = = = 0.662 m∙K∙W−1.
2πLk 2 × 𝜋 ×1 ×0.5

1 1 1
Rco = hA = = = 0.318 m∙K∙W−1.
o 2πrcr Lh 2 × 𝜋 ×0.02 ×1 ×25

106
(Ts − T∞ )
Q=R
Insulation + Rco

Or: Ts = T∞ + Q (Rinsulation + Rco)

= 30 + 294 × (0.662 + 0.318) = 318.12 ℃

Example 3.17. A 2 mm-diameter and 10 m-long electric wire is tightly wrapped


With a 1 mm-thick plastic cover whose thermal conductivity is 0.15 W∙m-1∙K−1.
Electrical measurements indicate that a current of 10 A passes through the wire and
there is a voltage drop of 8 V along the wire. If the insulated wire is exposed to a
medium at 30 ℃ with a heat transfer coefficient of 24 W∙m-2∙K−1, determine the
temperature at the interface of the wire and the plastic cover in steady operation.
Also determine if doubling the thickness of the plastic cover will increase or decrease
this interface temperature.

Schematic:

r Surface in contact with a fluid at T∞ and


surface heat transfer coefficient h
Ts
rs
T∞
Ts
Rins Rco
Q

Known: Outer radius of the bare wire = rs = 1 mm = 0.001 m ; Length of the wire = L = 10
m ; outer radius of plastic insulation = r = 1 + 1 = 2 mm = 0.002 m ;
Current through the wire = I = 10 A ; Voltage drop in the wire = V = 8 V ; Ambient temperature
= T∞ = 30 ℃ ; Thermal conductivity of the plastic cover = k = 0.15 W∙m-1∙K−1; Surface heat
transfer coefficient = h = 24 W∙m-2∙K−1.

Find: (i) Interface temperature = Ts ; (ii) Whether Ts increases or decreases when the
thickness of insulation is doubled.

Assumptions: i) one dimensional steady radial conduction (ii) the thermal conductivity of
insulation is constant (iii) by adding the insulation the outer surface temperature of the wire
will not change; (iv) the surface heat transfer coefficient for wire with and without insulation
remain same

Solution: (i) Q = VI = 8 × 10 = 80 W.

107
𝑟 0.002
ln( ) ln( )
Rinsulation = 2 π L k = 2 × 𝜋 ×10 ×0.15 = 0.0735 K∙W−1.
𝑟𝑠 0.001

1 1
Rco = 2 π rL h = 2 × 𝜋 ×0.002 ×10 ×24 = 0.3316 K∙W−1.

(Ts − T∞ )
Q= Rinsulation + Rco

Or: Ts = T∞ + Q(R insulation + R co )

= 30 + 80 × (0.0735 + 0.3316) = 62.41 ℃

k 0.15
(ii) Critical radius of insulation = rcr = h= = 0.00625 m
24

Since rcr > r, increasing the thickness of plastic insulation will increase the heat transfer rate
if Ts is held constant or for a given heat transfer rate the interface temperature Ts will
decrease till the critical radius is reached. Now when the thickness is doubled then
r = 3 mm = 0.003 m . Therefore,
0.003
ln( )
Rinsulation = 2 × 𝜋 ×10 ×0.15 = 0.1166 K∙W−1.
0.001

1
Rco = = 0.221 K∙W−1
2 × 𝜋 ×0.003 ×10 ×24

Therefore: Ts = 30 + 80 × (0.1166 + 0.221) = 57 ℃

3.5. Extended Surfaces (Fins):

3.5.1 .Introduction: Convection: Heat transfer between a solid surface and a moving fluid is
governed by the Newton’s cooling law: Q = hA(Ts-T∞), where Ts is the surface temperature and
T∞ is the fluid temperature. Therefore, to increase the convective heat transfer, one can
• Increase the temperature difference (Ts-T∞) between the surface and the fluid.
• Increase the convection coefficient h. This can be accomplished by increasing the fluid flow
over the surface since h is a function of the flow velocity and the higher the velocity, the
higher the h. Example: a cooling fan.
• Increase the contact surface area A. Example: a heat sink with fins.
Many times, when the first option is not in our control and the second option (i.e. increasing
h) is already stretched to its limit, we are left with the only alternative of increasing the effective
surface area by using fins or extended surfaces. Fins are protrusions from the base surface into
the cooling fluid, so that the extra surface of the protrusions is also in contact with the fluid.
Most of you have encountered cooling fins on air-cooled engines (motorcycles, portable
generators, etc.), electronic equipment (CPUs), automobile radiators, air conditioning
equipment (condensers) and elsewhere.

108
Fins are generally classified as (i) straight fins, (ii) annular fins and (iii) spines.The
term straight fin is applied to the extended surface attached to a wall which is otherwise
plane.The fin cross sectional area may or maynot vary along the length of the fin.An annular
fin is one which is attached, circumferentially, to a cylindrical surface.A spine or pin fin is an
extended surface of cyclindrical or conical shape. These definitions are illustrated in Figures
3.17 (straight fins),3.18(annular fins) and 3.19(spines).

3.5.2. Extended surface analysis:


In this section consideration will be limited to steady state analysis of rectangular or pin fins
of constant cross sectional area. Annular fins or fins involving a tapered cross section may be
analyzed by similar methods, but will involve solution of more complicated equations.
Numerical methods of integration or computer programs can be used to advantage in such
cases.
One dimensional approximation for extended surface analysis:We can examine when
temperature variation across the fin can be neglected in comparison to the temperature
variation along the length of the fin.Consider the heat transfer in y direction for the fin of length
L, width W and uniform thickness t (t << L and t << W) as shown in Figure 3.20. At any x ,if
the rate of heat transfer in x direction is of the same order of magnitude as the rate in y direction
we have

𝜕𝑇 𝜕𝑇
−k Ax (𝜕𝑥 ) ≈ −k Ay (𝜕𝑦)

Using first order linear approximation, the above equation can be written as

(𝛥𝑇)𝑥 (𝛥𝑇)𝑦
−k t W ≈ −k L W 𝑡
𝐿
2

(𝛥𝑇)𝑦 𝑡 2
Or (𝛥𝑇)𝑥
≈ (𝐿 ) (3.49)

For fins t is very much smaller than L; therefore, for the heat transfers to be of the same order
of magnitude ,it follows that the temperature variation in y direction must be much smaller
𝜕𝑇 𝜕𝑇
than the temperature variation in x direction: i.e. (𝜕𝑦) ≪ (𝜕𝑥 ).Hence conduction problems
in fins is treated as one-dimensional where temperature varies along the length of the fin.

3.5.2.1. Governing differential equation for 1-D analysis:


The one dimensional steady state Conduction Equation in solids with heat generation and
constant thermal conductivity is givn by[Equation (2.16):
d2 T 𝑞′′′
dx2
+ 𝑘
=0 (2.16)

This is a second order, ordinary differential equation and will require 2 boundary conditions
to evaluate the two constants of integration that will arise.

109
Consider the cooling fin shown in Figure3.20.The fin is situated on the surface of a hot surface
at Ts and surrounded by a coolant at temperature T∞, which cools with convective coefficient,
h. The fin has a uniform cross sectional area, Ac, (This is the area through with heat is
conducted.) and an overall length, L.Note that as energy is conducted down the length of the
fin, some portion is lost, by convection, from the sides. Thus the heat flow varies along the
length of the fin.We further note that the arrows indicating the direction of heat flow point in
both the x and y directions. This is an indication that this is truly a two- or three-dimensional
heat flow, depending on the geometry of the fin. However, quite often, it is convenient to
analyse a fin by examining an equivalent one–dimensional system. The equivalent system
y

x Fin attached to the primary surface


and exposed to surroundings at T∞
with a heat transfer coefficient h

Qfin
t

Primary surface at T0

Qc = hPΔx (T - T∞)

Δx

Figure3.20. Heat dissipation from the fin to


surroundings

will involve the introduction of heat sinks (negative heat sources), which remove an amount
of energy equivalent to what would be lost through the sides by convection.

Consider a differential length of the fin. Across this segment the heat loss will be h(PΔx)(T-
T∞), where P is the perimeter around the fin. The equivalent heat sink would be ─ q’’’Ac Δx
Equating the heat source to the convective loss we get

─ q’’’Ac Δx = hPΔx (T - T∞)

110
hP
Or: q’’’ = ─ (kA ) (T - T∞) (3.50)
c
Heat and Mass Transfer Pro
Indian Institute of Science Bangalore
Substituting this expression for q’’’ into Equation(2.16.) we get

d2 T hP
−( ) (T - T∞) = 0.
dx2 kAc

d2 T
Or: dx2
– m2 (T - T∞) = 0. (3.51a)

hP
Where m2 = (3.51b)
kAc

which is the equation for a fin with a constant cross sectional area. This is a Second Order
linear differential Equation.

Alternative method of obtaining equation Equation(3.50a): Equation (3.50a) can also be


obtained from basic principles by writing the energy balance equation for an elemental length
of the fin as follows:
Qconv

Qx Qx+dx

Under steady state conditions, the energy balance equation for the fin element shown above
can be written as

Qx = Qx+dx + Qconv

dQ
= Qx + ( dxx )dx + hP dx (T - T∞)

dQ
Or ( dxx ) + hP (T - T∞) = 0

Writing the expression for Qx using Fourier’s law we get

d dT
[− kAx ( dx )] + hP (T - T∞) = 0
dx

For a fin of uniform cross sectional area Ax = Ac = constant. Also if the thermal conductivity
of the fin material is constant then the above equation reduces to

d2 T
– m2 (T - T∞) = 0 (3.51a)
dx2

111
3.5.2.2.Solution to the fin equation: We notice that equation (3.50) is non-homogeneous
(due to the T∞ term). Recall that non-homogeneous differential equations require both a
homogeneous and a particular solution. We can make this equation homogeneous by
introducing the fin temperature relative to the surroundings:

Let θ ≡ T - T∞ (3.52)

Then Equation (3.51a) reduces to,

d2 θ
─ m2θ = 0 (3.53)
dx2

Equation (3.53) is called the fin equation.

Equation (3.53) is a second order linear homogeneous differential equation the general
solution for which is given by

θ = Aemx + Be─mx (3.54)

The solution for Equation (3.53) is also given by

θ = C cosh mx + D sinh mx (3.55)

Generally the exponential solution is used for very long fins, the hyperbolic solutions for
other cases to facilitate easy evaluation of constants of integration.

Boundary Conditions:

Since the solution results in 2 constants of integration we require 2 boundary conditions. The
first one is obvious, as one end of the fin will be attached to a hot surface and will come into
thermal equilibrium with that surface.
Hence, at the fin base (x = 0),

θ =T0 - T∞ = θ0 (3.56a) e
of Science
The second boundary condition depends on the condition imposed at the other end of the
fin. There are various possibilities, as described below.

(i)Very long fins:

For very long fins, the end located a long distance from the heat source will approach the
temperature of the surroundings. Hence,

θ(∞) = 0 (3.56b)
Substitute the second condition into the exponential solution of the fin equation:

112
θ(∞)= 0 = Ae m∞ + Be ─m∞

The first exponential term is infinite and the second is equal to zero. The only way that this
equation can be satisfied is if A = 0. Now apply the second boundary condition.

θ (0) = θ 0 =B

The general temperature profile for a very long fin is then given by:

θ(x) = θ0 e – mx (3.57)

If we wish to find the heat flow through the fin, we may apply Fourier Law:
At any x, the rate of heat conduction is given by

dT dθ
Q = ─ kAc(dx ) = ─ kAc(dx )

All the heat dissipated by the fin to the surroundings is coming from the fin base. Hence the
rate of heat transfer from the fin to the surroundings is given by


Qfin = Q|x=0 = ─ kAc (dx ) (3.58)
𝑥=0


From Equation (3.57) we have, (dx ) = ─ mθ0
𝑥=0

Substituting this expression in Equation (3.58) we get

Qfin = kmAcθ0= √(hpkAc θ0 ) (3.59)

(ii)The fin with insulated tip:

Assume that the tip is insulated and hence there is no heat transfer at the fin tip.
Hence the boundary condition at the tip is given by


At x = L, ( ) =0
dx 𝑥=𝐿

Substituting this condition and the boundary condition at x = 0 and simplifying we get the
temperature profile for this case as

cosh m(L−x)
θ(x) = θ0 (3.60)
cosh mL

We may find the heat flow at any value of x by differentiating the temperature profile and
substituting it into the Fourier Law:

113

Qfin = Q|x=0 = ─kAc (dx ) .
𝑥=0

dθ sinh mL
From Equation (3.60), ( dx ) = ─ θ0 cosh mL
𝑥=0

Therefore: Qfin = kmActanh mL (3.61)

If we compare this result with that for the very long fin, we see that the primary difference in
form is in the hyperbolic tangent term. That term, which always results in a number equal to
or less than one, represents the reduced heat loss due to the shortening of the fin
Indian Institute of Science Bangalore
Other tip conditions:
We have already seen two tip conditions, one being the long fin and the other being the
insulated tip. Two other possibilities are usually considered for fin analysis: (i) a tip subjected
to convective heat transfer, and (ii) a tip with a prescribed temperature. The expressions for
temperature distribution and fin heat transfer for all the four cases are summarized in the table
below.

Table 3.1: Expressions for Temperature distribution and rate of heat dissipation from a
fin for different tip conditions

Tip Expression for


Condition Expression for Temperature Profile Rate of Heat Transfer

Long Fin θ(x) = θ0 e − mx Qfin = kmAcθ0

cosh m(L−x)
Tip Insulated θ(x) = θ0 Qfin = √hPkAc θ0tanh mL
cosh mL

Tip with prescribed


θL 𝜃
sinh mx+sinh m(L−x) cosh 𝑚𝐿 − 𝐿
θ0 𝜃0
Temp,TL θ(x) = Qfin = √hPkAc θ0
sinh mL sinh 𝑚𝐿

h
e ) sinh m(L−x) he ) cosh mL
cosh m(L−x)+ (km sinh mL+ (km
Tip dissipating heat θ(x) = h Qfin = θ0 √hPkA c h
cosh mL+ ( e ) sinh mL cosh mL+ ( e ) sinh mL
km km
by convection with
surface heat transfer
coefficient,he

114
3.5.2.3.Performance of a fin:

Fin Efficiency: The performance of a fin is assessed by using a parameter called “efficiency
of a fin, η” given by,

Actual rate of heat transfer from the fin Qfin


η = Maximum Possible heat transfer rate from fin = (Q (3.62)
fin )max

The rate of heat transfer from the fin will be maximum when the entire fin surface is at the fin
base temperature (This means that the fin is having an infinite thermal conductivity): i.e.,

(Qfin)max = hPL(To - T∞) = hPLθ0 (3.63)

Fin Effectiveness: Fins are used to enhance the heat transfer, and the use of fins on a surface
cannot be recommended unless the enhancement in heat transfer justifies the added cost and
complexity associated with the fins. In fact, there is no assurance that by adding fins on a
surface, the heat transfer rate will enhance.The performance of the fin is judged on the basis of
enhancement in heat transfer relative to the no-fin case.To know how effective an extended
surface as compared to a bare primary surface is, another parameter called effectiveness is
used. It is given by,

Heat Transfer with fin Q Qfin


ε = Heat Transfer without fin = Q fin = (3.64)
bare hAb θ0

In equation (3.64), Ab is the cross sectional area of the fin at its base.
It should be noted that both the fin efficiency and the fin effectiveness are related
to the performance of the fin, but they are different quantities.However they are related to
each other as shown below.

Q η h Af θ0 A
ε = hAfinθ = = A f ηθ0 (3.65)
b 0 hAb θ0 b

Hence, the fin effectiveness can be determined easily when the efficiency of the fin is known,
or vice-versa.

Considerations in the design and selection of fins:

(i) Long fin:-The rate of heat transfer from a very long fin of uniform cross section under
steady state conditions is given by Equation(3.59).Substituting this expression in
Equation(3.65) we get,

Qfin kmAb θ0 𝑘 ℎ𝑃
εlong= = = × √( )
hAb θ0 hAb θ0 ℎ 𝑘𝐴

𝑘𝑃
Or: εlong = √( ) (3.66)
ℎ𝐴

115
Several important conclusions can be drawn from Equation (3.66) which helps in design and
selection of long fins for various applications.

(i) The therml conductivity k of the fin material should be as high as possible.Hence
fins are made of materials like copper.aluminium are iron. The most widely used
material is aluminium because of its low cost and weight and its resistance to
corrosion.
(ii) The ratio of the perimeter to the cross-sectional area of the fin P/A should be as
high as possible. This criterion is satisfied by employing thin plate fins and slender
pin fins.
(iii) The use of fins is most efficient in applications involving low convection heat
transfer coefficients. Thus the use of fins is easily justified when the medium is gas
instead of a liquid and the heat transfer mechanism is by free convection
rather than by forced convection.Therfore in a liquid to gas heat exchangers like
automobile radiators, fins are placed on the gas side of the heat exchangers.

(ii) Straight Fin of finite length:Consider a straight fin of uniform thickness ‘w’ and length
L. For unit width of the fin, the parameter m defined in Equation (3.51b) is given by,

2h
m=√ ,
kw

The heat dissipation for a fin of finite length is

h
sinh mL+ ( e ) cosh mL
km
Qfin = θ0 √hPkAc h
cosh mL+ ( e ) sinh mL
km

Per unit width, assuming he = h, theabove equation reduces to

2h hw 2h
sinh √ L+√ Lcosh √
kw 2k kw
Qfin× = θ0 √2hkw .
2h hw 2h
cosh √ L+ √ sinh √ L
kw 2k kw

The above equation can be written as:

2h hw
tanh √ + √
kw 2k
Qfin× = θ0 √2hkw (3.67)
hw 2h
1+ √ tanh √ L
2k kw

Equation will indicate the effect of varying the length L of the fin on heat dissipation , if
other things are being held fixed. Hence this equation is useful in finding out the condition
under which fins must be employed to increase the heat dissipation.The condition for

116
optimum value of Qfin× can be obtained by differenting Qfin× [Equation (3.67)]with rrespect
to L and equating it to zero.

hw 2h hw hw 2h
(1+√ tanh √ L)−√ (√ +tanh √ L)
dQ∗fin 2k kw 2k 2k kw 2h 2h
= θ0 √2hkw [ 2 ] × [√kw sech2 √kw L] = 0
dL hw 2h
(1+ √ tanh √ L)
2k kw

hw
Or: =1 (3.68)
2k

hw dQ∗
Further regardless of the value of L, if 2k > 1, dLfin is negative.These statements are valid
for all values of L > 0 and hence they may be applied to determine whether or not a fin
hw
should be used at all.If 2k > 1, fins produce insulating effect. From cost considerations, fins
are to be provided only if,
hw
<< 1 (3.69)
2k

Example 3.18. A steel rod of diameter 2 cm and thermal conductivity 50 W∙m−1∙K−1 is


exposed to ambient air at 20 ℃ with a heat transfer coefficient 64 W∙m−2∙K−1. One end of the
rod is maintained at a uniform temperature of 120 ℃. Determine the rate of heat from the
rod to the ambient and the temperature of the tip of the rod exposed to ambient if (i) the rod
is very long, (ii) rod is of length 10 cm with negligible heat loss from its tip, (ii) rod is of
length 25 cm with heat loss from its tip.

Schematic: k h,T∞
x

Known: D = 2 cm.; k = 50 W∙m−1∙K−1;T∞ = 20 ℃; h = 64 W∙m−2∙K−1;T0 = 120 ℃

Find: (i) Qfin if the rod is long; (ii) Qfin if L = 10 cm and negligible heat loss from the tip;
(iii)Qfin if L = 25 cm and with heat loss from the tip

Assumptions: (i) one dimensional steady state conduction along the length of the fin; (ii)
thermal conductivity of the rod is constant

Solution: (i) Long rod:


ℎ𝑃 𝜋𝐷ℎ 4ℎ 4 ×64
m = √(𝑘𝐴) = √( 𝐷2
) = √(𝑘𝑑) = √(50 ×0.02) = 16 m−1
𝜋 𝑘
4

117
For a very long fin the rate of heat transfer is given by,

Qfin = kmA(T0 - T∞) = 50 × 16 × (π / 4) × 0.022 × [ 120 – 20] = 25.13 W

(ii) Rod of finite length with negligible heat loss from the tip:

L = 0.10 m. Hence mL = 16 × 0.1 = 1.6

For rod of finite length with negligible heat loss from the tip we have

Q fin = kmA(T0 - T∞) tanh mL = 50×16×(π / 4)×0.022×(120 – 20)× tanh 1.6

= 23.16 W

(iii) Rod of finite length with heat loss from the tip:

When the heat loss from the rod tip is not negligible with he = h , then we can use the same
formula as in case (ii) with modified length Le given by

A πD2 /4 𝐷 0.02
Le = L + P= L + = L + 4 = 0.1 + = 0.105 m
πD 4

mLe = 16 × 0.105 = 1.68 and tanh mLe = tanh 1.68 = 0.9329

Hence: Q fin = 25.13 × 0.933 = 23.44 W

Example 3.19. A thin rod of uniform cross section A, length L and thermal conductivity
k is thermally attached from its ends to two walls which are maintained at temperatures
T1 and T2. The rod is dissipating heat from its lateral surface to an ambient at
temperature T∞ with a surface heat transfer coefficient h.
(a) Obtain an expression for the temperature distribution along the length of the rod
(b) Also obtain an expression for the heat dissipation from the rod to the ambient

Scheme:

T1 k h,T∞
x T2

118
Known: Temperatures at the two ends of the rod, rod cross sectional area A,thermal
conductivity of the rod k, ambient temperature T∞ and surface heat transfer coefficient h.

Find: (a) Temperature distribution T(x) along the length of the rod; (b)Rate of heat dissipation
from the rod to ambient

Assumptions: (i) one dimensional steady state conduction along the length of the fin; (ii)
thermal conductivity of the rod is constant

Solution: The general solution for the one-dimensional steady-state temperature distribution
along the length of a rod dissipating heat by convection from its lateral surface is given by

θ(x) = C1 cosh mx + C2 sinh mx (1)

hP
where: θ(x) = T(x) - T∞ and m = √(kA)

π D2
P = perimeter of the rod = π D and A = Area of cross section of the rod = .
4

The boundary conditions are: (i) at x = 0, T = T1 or θ = T1 - T∞ = θo (say).

(ii) at x = L, T = T2 or θ = T2 - T∞ = θL (say).

Condition(i) in Equation (1) gives: θo = C1.

Condition (ii) in Equation (1) gives: θL = θ0 cosh mL + C2 sinh mL.

(θL − θ0 cosh mL)


Or: C2 = sinh mL

Substituting for C1 and C2 in Equation (1) we have,

(θL − θ0 cosh mL)


θ(x) = θ0 cosh mx + sinh mx
sinh mL

θ0 cosh mx sinh mL+ θL sinh mx − θ0 cosh mL sinh mx


Or: θ(x) = sinh mL
θ0 sinh m(L –x )+ θL sinh mx
= (2)
sinh mL
Expression for the rate of heat dissipation from the rod:

119
Qamb
x

Q|x=0
Q |x=L

Energy balance for the rod is given by

dθ dθ
Q amb = Q |x = 0 ─ Q |x = L = ─ kA ( ) + kA ( ) (3)
dx x=0 dx x=L

dθ − m [θL cosh mx+ θL cosh m(L−x)) ]


From Equation (2) we have dx =
sinh mL

dθ − m [θL + θ0 cosh mL ]
Therefore: (dx) =
x=0 sinh mL

dθ − m [θL cosh mL+ θ0 ]


And: (dx) =
x=L sinh mL

kmA [θL +θ0 cosh mL − θL cosh mL ─ θ0]


Hence: Qamb =
sinh mL

kmA [(θL ─ θ0 ) (1 −cosh mL) ]


=
sinh mL

Example 3.20. Heat is generated at a constant rate of q’’’ W∙m-3 in a thin circular rod of
length L and diameter D by the passage of electric current. The two ends of the rod are
maintained at uniform temperatures with one end at temperature T0 and the other end
at 0 ℃, while heat is being dissipated from the lateral surface of the rod to an ambient
at 0℃with a surface heat transfer coefficient h.(a) Derive the one-dimensional steady state
energy equation todetermine the temperature distribution along the length of the rod and(b)
Solve the above equation and obtain the temperature distribution.

Schematic:

120
0℃ h,T∞
q’’’ W∙m-3

T = T0

x
L
Qamb

Qx Qx+dx

dx Qg
x

Known: Thin rod of diameter D and length L is generating heat at a constant rate of q’’’ W∙m-
3
;One end of the rod is maintained at 0℃and the other end at T0; Lateral surface of the rod is
exposed to ambient at T∞ with a surface heat transfer coefficient h.

Find: (a) One dimensional steady state energy equation;


(b)Temperature distribution T(x) along the length of the rod

Assumptions: (i) one dimensional steady state conduction along the length of the rod; (ii)
thermal conductivity of the rod is constant

Solution: Since the rod is generating heat and dissipating heat to the ambient, the governing
differential equation to determine the one-dimensional steady state temperature distribution
has to be obtained from first principles as illustrated below.

Consider an elemental length ‘dx’of the rod as shown in the figure above. The various
energies crossing the boundaries of the rod as well as the energy generated are also shown in
the figure. For steady state condition the energy balance equation for the rod element can be
written as

Qx + Qg = Qx+dx + Qamb

Or Qx + Qg = Qx + (dQx/dx) dx + Qamb

dQ
Or ( dxx )dx + Qamb = Qg

d dT
(─ kA dx ) dx + hPdx (T - T∞) = Adx q’’’
dx

121
d2 T hP q′′′
Or: – kA (T - T∞) = ─
dx2 k

hP
Let T - T∞ = θ and kA = m2. Then the above equation reduces to

d2 θ q′′′
─ m2 θ = ─ (1)
dx2 k

Equation(1) is a non-homogeneous linear second order ordinary differential equation whose


solution can be written as

θ(x) = θh(x) + θp(x) (2)

where θh(x) satisfies the homogeneous part of the differential equation namely

d2 θh
─ m 2 θh = 0 (3)
dx2

and θp(x) is the particular integral which satisfies Equation (1). Solution to Equation (3) is
given by,

θ(x) = C1 e mx + C2 e ─ mx (4)

To find θp(x) : Since the RHS of Equation(1) is a constant let us assume θp(x) = B, where B
is a constant. Substituting this solution in Equation(1) we have
q′′′
0 – m2 B = ─ k

q′′′
Or: B= k
Therefore the complete solution for Equation (1) can be written as

q′′′
θ(x) = C1 e mx + C2 e ─ mx + k m2

q′′′
Or: T(x) = T∞ + C1 e mx + C2 e ─ mx + k m2 (5)

Boundary conditions are: (i) at x = 0, T = 0

(ii) at x = L, T = T0

Condition (i) in Equation (5) gives


q′′′
0 = T∞ + C1 + C2 + k m2

122
q′′′
Or: C1 + C2 = ─ T∞ ─ k m2 (a)

q′′′
Condition(ii) in Equation(5) gives T0 = T∞ + C1 e mL + C2 e ─ mL + k m2

q′′′
Or C1 e mL + C2 e ─ mL = T0 ─ T∞ ─ k m2 (b)

q′′′
From Equation(a): C2 = ─ C1 ─ T∞ ─ k m2 .

Substituting this expression in Equation(b) we have,

q′′′ q′′′
C1 e mL ─ [C1 + 𝑇∞ + ] e ─ mL = T0 ─ T∞ ─
k m2 k m2

q′′′
T0 − {T∞ + }{1 ─ e−mL}
k m2
Solving for C1 we get: C1 = and,
{emL ─ e−mL }

q′′′
q′′′ T0 − {T∞ + }{1 ─ e−mL }
k m2
C2 = − {T∞ + }−
k m2 {emL ─ e−mL}

q′′′
{T∞ + }{1 ─ emL } − T0
k m2
Or: C2 =
{emL ─ e−mL }

Substituting the expressions for C1 and C2 in Equation (5) and simplifying we get
q′′′
q′′′ [T0 − {T∞ + }{1 ─ e−mL }] emx
k m2
T(x) = T∞ + +
k m2 {emL ─ e−mL }

q′′′
[ {T∞ + }{1 ─ emL} − T0 ] e−mx
k m2
+
{emL ─ e−mL }

Example 3.21. Two very long slender rods of the same diameter are given. One rod is
of aluminum (k = 200 W∙m−1K−1). The thermal conductivity of the other
rod is not known. To determine this, one end of each rod is thermally attached to a metal
surface maintained at a uniform temperature T0. Both rods are losing heat to the ambient air
at T∞ by convection with a surface heat transfer coefficient h. The surface temperature of each
rod is measured at various distances from hot base surface. The temperature of the aluminum
rod at 40 cm from the base is same as that of the rod of unknown thermal conductivity at 20
cm from the base. Determine the unknown thermal conductivity.

Schematic:

123
Ta
xa
ka

Tb = Ta
xb
kb = ?

Known: Two slender rods A and B are of same diameter;Base temperature T0 is same for
both the rods;Conductivity of rod A = ka = 200 W∙m−1K−;Ambient temperature T∞ same for
both the rods. Surface heat transfer coefficient h same for both the rods; Ta = Tb with xa =
0.4 m and xb = 0.2m

Find: Thermal conductivity of rod B , kb

Assumptions: (i) one dimensional steady state conduction along the length of the rods; (ii)
thermal conductivity of the two rods is constant (iii)rods are assumed to be of infinite length

Solution:For very long slender rods the steady-state one-dimensional temperature


distribution along the length of the rod is given by

θ (x) = θ0 e ─ mx (1)

where θ(x) = T(x) - T∞ and θ0 = T0 - T∞.

For rod A Equation(1) can be written as: θa(x) = θ0 e− ma xa (2)

And for rod B it can be written as: θb(x) = θ0 e− mb b (3)

It is given that when xa = 0.4 m and xb = 0.2 m, θa(xa) = θb(xb)

Therefore we have: θ0 e− 0.4 ma = θ0 e− 0.2 mb

Or: mb = 2 m a
hPb hPa
Hence: √k = 2 × √k
b Ab a Aa

Since Pa = Pb and Aa = Ab, we have, ka = 4 kb


200
Therefore: kb = 4 = 50 W∙m−1K−1-.
Comments:This problem was solved assuming that the rods are of infinite length and perhaps
give inaccurate results.For better results the governing equations used should be based on
finite length of the rods as illautrated in example 3.22.

Example 3.22. Two rods of the same diameter and same length are given.

124
One rod is of aluminum whose thermal conductivity is 200 W∙m−1∙K−1.The thermal
conductivity of the other rod is not known. To determine this, one end of each rod is
thermally attached to a metal surface maintained at a uniform temperature 120 ℃. Both rods
are losing heat to the ambient air at 20 ℃ by convection. The surface temperatures of each
rod is measured at two identical locations one at 5 cm and the other 10 cm from hot base
surface. These temperatures for the aluminum rod are 66 ℃ 𝑎𝑛𝑑 55 ℃ respectively. For the
other rod the corresponding temperatures are 49℃ and 28 ℃ respetively. Determine the
unknown thermal conductivity

Schematic:
[𝑇𝑅 ]𝑟𝑜𝑑 𝐴
2R [𝑇2𝑅 ]𝑟𝑜𝑑 𝑎

R [𝑇𝑅 ]𝑟𝑜𝑑 𝑏
kb = ?

[𝑇2𝑅 ]𝑟𝑜𝑑 𝑏

Known: T0 = 120 ℃ ; T∞ = 20℃ ; ka = 200 W∙m−1∙K−1; R = 0.05 m; 2R = 0.1m; [𝑇𝑅 ]𝑟𝑜𝑑 𝑎 =

66℃; [𝑇2𝑅 ]𝑟𝑜𝑑 𝑎 = 55℃; [𝑇𝑅 ]𝑟𝑜𝑑 𝑏 = 49℃; [𝑇2𝑅 ]𝑟𝑜𝑑 𝑏 = 28℃.

Find:kb

Assumptions: (i)One dimensional steady state conduction along the length of both the
rods;(ii)Thermal conductivity of both the rods are constant

Solution: The genral solution for temperature distribution for a fin of finite length is given
by,
θ(x) = Bemx + De−mx (1)

At x = 0, T = T0 or θ = T0 - T∞ = θ0. Hence,
B + D = θ0 = 120 – 20 =100 ℃ (2)

At x = R, T = TR.Or θ = TR - T∞= θR. Hence,


θR = BemR + De−mR (3)

And at x = 2R, T = T2R. Or θ = T2R - T∞= θ2R. Hence,

θ2R = Be 2mR + De−2mR (4)

θ0 + θ2R
Let, r= θR
(5)

125
Then using Equations (1), (2) and (3) we have,

B + D+ Be2mR + De−2mR
r= BemR + De−mR

= emR + e−mR = 2cosh mR

𝑟
Or: mR = cosh – 1 (2) (6)

𝑟
𝑚𝑎 𝑅𝑎 𝑚 cosh – 1 ( 𝑎 )
Hence: = 𝑚𝑎 = 𝑟
2
(7)
𝑚𝑏 𝑅𝑏 𝑏 cosh – 1 ( 𝑏 )
2

𝑟
𝑚𝑎 𝑘𝑏 cosh – 1 ( 𝑎 )
2
Or = √𝑘 = 𝑟 (8).
𝑚𝑏 𝑎 cosh – 1 ( 𝑏 )
2

θ0 + θ2R 100+ (55−20)


Now: ra = ( ) = (66 − 20)
= 2.935.
θR 𝑟𝑜𝑑 𝑎

θ0 + θ2R 100+(28−20)
And: rb = ( ) = = 3.724
θR 𝑟𝑜𝑑 𝑏 49−20

2.935
𝑘𝑏 cosh – 1 ( )
2
Therefore : √𝑘 = 3.724 = 0.756
𝑎 cosh – 1 ( )
2

And kb = (0.653)2 ka = (0.756)2 × 200

= 114.5 W∙m−1∙K−1.
Comments: The reason for measuring the temperature T2R, at twice the distance from the
location at which TR is measured, is that the dimensionless parameter ‘r’ as defined will
enable us to eliminate the constants of integration B and D from the general solution. This
will hep in expressing r as a function of m and R only.

Example 3.24.A 1.25 cm diameter rod of iron ( k = 45 W∙m−1∙K−1) has its one end
maintained at a uniform temperature of 260 ℃ and is epxposed to an environment at 38
℃.The surface heat transfer coefficient is 8.5 W∙m−2∙K−1.What should be the length of the rod
in order that its end temperature exposed to the environment may be computed with less than
5 % error by use of equation for rod of infinite length?What error in the heat dissipation is
made with this length by the use of infinite rod equation?

Schematic:

126
T0 h,T∞

T = T|x=L

x
L= ∞

Known: d = 0.0125m;k = 45 W∙m−1∙K−1; h = 8.5 W∙m−2∙K−1; T0 = 260 ℃; T∞ = 38 ℃;

Find: (i) L if the formula of infinite rod is used with an error of ≤ 5 %; (ii) The error in Q if
this L is used in infinite rod equation.

Assumptions: (i)One dimensional steady state conduction along the length of the
rods;(ii)Thermal conductivity of both the rods are constant;(iii) rod is of infinite length.

hP 𝜋𝑑ℎ 4ℎ 4 ×8.5
Solution: m = √kA = √ 𝑑2
= √𝑘𝑑 = √45 ×0.0125 = 7.775 m−1.
𝑘𝜋 ( )
4

With 5 % error, T|x = L = 1.05 T∞.

Or: θ|x = L = T|x = L − T∞ = 1.05 T∞ − T∞

= 0.05 T∞ = 0.05 × 38 = 1.9 ℃.

θ|x=L
For a rod of infinite length, = e –mL
θ0

1 θ 1 260−38
Or: L = 𝑚 ln [θ| 0 ] = 7.775 ln [ ]
x=L 1.9

= 0.612 m

𝜋
(ii) For rod of infinite length, Q∞ = k m A θ0 = 45×7.775× ( 4 × 0.01252 ) × (260 −38)

= 9.532 W
The actual heat dissipated by the rod of finite length, neglecting tip loss is,

Qf = k m A θ0 tanh mL = 9.532 × tanh (7.775 × 0.612)

= 9.530 W

Percent Error in heat dissipation by assuming the rod to be of infinite length is

127
9.532−9.53
= × 100 = 0.02 %
9.53

Example 3.25. Show that for a finned surface the total heat transfer rate is given by

Qtotal = [η β + (1 – β)] a h θ0 = ή a h θ0

Where η = fin efficiency ; β= af / a : af = surface area of the fin, a = total heat transfer area
(i.e. finned surface + unfinned surface) ; θ0 = T0 - T∞, with T0 = fin base temperature and T∞
= ambient temperature, and ή = area – weighted fin efficiency.

Solution:
Qtotal = Qfin + Qbare

Where Qtotal = Total heat transfer rate, Qfin = Heat transfer rate from the finned surface

and Qbare = Heat transfer rate from the bare surface.

Therefore Qtotal = η h af θ0 + h(a – af) θ0

𝜂𝑎𝑓 𝜂𝑎𝑓
= h a θ0 [( ) + (1 − )]
𝑎 𝑎

= ha θ0 [ηβ + (1 – β)]

= ή ha θ0, where ή = [ηβ + (1 – β)]

Example 3.26. The handle of a ladle used for pouring molten lead at 327 ℃ is 30 cm long
and is made of 2.5 cm × 1.5 cm mild steel bar stock (k = 43 W∙m−1∙ K−1). In order to reduce
the grip temperature, it is proposed to make a hollow handle of mild steel plate 1.5 mm thick
to the same rectangular shape. If the surface heat transfer coefficient is 14.5 W∙m−2∙ K−1and
the ambient temperature is 27℃, estimate the reduction in the temperature of the grip. Neglect
the heat transfer from the inner surface of the hollow shape.

Schematic: (a) When the handle is made of solid steel bar:

2.5 cm
Known: h = 14.5 W∙m−1∙ K−1 ;

1.5 cm k = 43 W∙m−2∙ K−1;

θ0 = 327 – 27 = 300 ℃
Cross section of the handle

Area of cross section of the bar = A = 2.5 × 1.5 × 10 ─ 4 m2 = 3.75 × 10 ─ 4 m2

128
Perimeter of the bar = P = 2 [ 2.5 + 1.5] × 10 ─ 2 m = 8 ×10 ─ 2 m

hP 14.5 ×(8 × 10−2 )


Therefore: m = √kA = √43 ×(3.75 × 10−4) = 8.48 m−1.

And: mL = 8.48 × 0.3 = 2.54

When the heat loss from the tip of the handle is neglected the temperature at any point along
the length of the handle is given by,

cosh m(L−x)
θ(x) = θ0 [ ]
cosh mL

cosh m(L – x)
θ(x) = θ0 ----------------------
cosh mL

θ 300
Therefore: θ(x)|x=L = cosh0mL = cosh 2.54 = 47 ℃.

Or: T(x)|x=L = 47 + 27 = 74 0 C.

(b) When the handle is hollow made out of a sheet:


Area of the cross section of the fin is
2.5 cm
A = [(2.5 × 1.5) – (2.5 – 0.3) ×(1.5 – 0.3)]

= 1.11 cm 2 = 1.11 × 10 − 4 m2
1.5 cm
P = 2 × [ 2.5 + 1.5 ] = 8 cm = 8 × 10 − 2 m

1.5 mm thick

hP 14.5 × (8 × 10−2 )
m = √kA = √43 ×(1.11 × 10−4) = 15.59 m−1

Therefore: mL = 15.59 × 0.3 = 4.68

θ (327−27)
θ(x)|x=L = cosh0mL = cosh 4.68 = 5.57 ℃
and: T(x)|x=L = 5.57 + 27 = 32.57 ℃

Hence reduction in grip temperature = 74 – 32.57 = 41.43 ℃

129
Example 3.27. Derive an expression for the overall heat transfer coefficient across a plane
wall of thickness ‘b’ and thermal conductivity ‘k’ having rectangular fins on both sides. Given
that over an overall area A of the wall, the bare area on both sides, not covered by the fins are
Au1 and Au2, the fin efficiencies are η1 and η2, and the heat transfer coefficients h1 and h2.

Schematic:

Au1, η1,h1

Au2, η2,h2
Solution:Let Ti be the temperature of the fluid in contact with the surface 1, T0 be the
temperature of the fluid in contact with surface 2, T1 be the temperature of primary surface 1
and T2 be the temperature of primary surface 2.Let Ti >T0. Then the rate of heat transfer from
Ti to T0 is given by,
Q = Qbare + Q fin

= hiAu1 (Ti – T1) + hi η1Af1(Ti – T1)


(Ti − T1 ) (Ti − T1 ) (Ti − T1 )
Or: Q= 1
− = (1)
( ) 1 1 1
h1 Au1 ( ) ( )+ ( )
h1 η1 Af h1 Au1 h1 η1 Af
1 1

(T2 − T0 )
Similarly: Q= (2)
1 1
( )+ ( )
h2 Au2 h2 η2 Af
2
(𝑇1 − 𝑇2 )
Also: Q= 𝑏 (3)
( )
𝐴𝐾

It follows from Equations (1), (2) and (3) that,

(Ti − T1 )+ (T1 − T2 )+ (T2 − T0 )


Q= 1 1 𝑏 1 1
( )+ ( )+ ( )+ ( )+ ( )
h1 Au1 h1 η 1 A f 𝐴𝐾 h2 Au2 h2 η2 Af
1 2

130
(Ti − T0 )
Or: Q= (4)
1 1 𝑏 1 1
( )+ ( )+ ( )+ ( )+ ( )
h1 Au1 h1 η 1 A f 𝐴𝐾 h2 Au2 h2 η2Af
1 2

If U = overall heat transfer coefficient for the plane wall then


Q = UA(Ti – T0) (5)
From Equations (4) and (5) we have:
1
U= 1 1 𝑏 1 1
𝐴 [( )+ ( )+ ( )+ ( )+ ( )]
h1 Au1 h1 η1 Af 𝐴𝐾 h2 Au2 h2 η2 Af
1 2

Example 3.28. Calculate the effectiveness of the composite pin fin shown in Figure below.
Assume k1 = 15 W/(m-K), k2 = 50 W∙m−1∙K−1 and h = 12 W∙m−2∙K−1.

Schematic:

k1 3 mm = d1
d2 = 10 mm

x k2

Qc

Qx Qx+dx

dx

Known: K1=15 W∙m−1∙K−1, K2=50W∙m−1∙K−1, h=12 W∙m−2∙K−1.

Find: ε for the composite fin

Assumptions: (i) One dimensional steady state conduction aloing the length of the fin; (ii)
Therml conductivity of the two materials are constants.

Solution: Energy balance equation for the fin element shown in the figure is given by,

dQ
Qx = Qx+dx + Qc = Qx + ( dxx )dx + Qc

dQx
Or: + Qc = 0 (1)
dx

Qx consists of two components namely the heat transfer Qx1 through the material of thermal
conductivity k1 and the rate of heat transfer Qx2 through the material of conductivity k2.

131
dT dT
Therefore: Qx = Qx1 + Qx2 = − k1A1 ( dx ) − k2A2 (dx )

dT
= − (k1A1 + k2A2) ( dx )

And: Qc = (hP2 dx) (T - T∞).

Substituting these expressions for Qx and Qc in equation (1) we get

d2 T ℎ𝑃2
− (𝑘 × (𝑇 − 𝑇∞ ) = 0
dx2 1 𝐴1 + 𝑘2 𝐴2 )

d2 θ
Or: − m2 θ = 0 (2)
dx2

ℎ𝑃2
Where: θ = T – T∞ and m = √(𝑘
1 𝐴1 + 𝑘2 𝐴2 )

When the heat loss from the fin tip is negligible, the solution to equation (2) is given by

cosh m(L−x)
θ(x) = θ0 [ ] (3)
cosh mL

The rate of heat transfer from the fin base is given by

𝑑𝜃
Qx|x=0 = − (k1A1 + k2A2) [𝑑𝑥 ]
𝑥=0

(k1 A1 + k2 A2 ) × [sinh m(L−x)]x=0 ×(−m)θ0


=− cosh mL

= mθ0 (k1A1 + k2A2) tanh mL

(Qx )x=0 𝑚𝜃0 (k1 A1 + k2 A2 ) tanh mL


η= =
Qmax ℎ𝑃2 𝐿𝜃0

hP2
Noting that (k = m2, the above expression for η simplifies to
1 A1 + k2 A2 )

tanh mL
η= mL

𝜋
In the given problem A1 = ( 4 ) × (0.003)2 = 7.1 × 10 − 6 m2.

𝜋
A2 =(4 ) × [ (0.01)2 – (0.003)2] = 7.15 × 10 – 5 m2.

132
P2 = π × 0.01= 0.0314 m.

ℎ𝑃2 12 × 0.0314
m = √(𝑘 )
= √(15 ×7.1 × 10−6 + 50 × 7.15 × 10−5 ) = 10.12 m−1.
1 𝐴1 + 𝑘2 𝐴2

tanh mL tanh(10.12 ×0.1)


Therefore: η= = = 0.757
mL 10.12 ×0.1

3.6. One dimensional steady state Conduction in Solids with Variable Thermal
Conductivity: The analysis of conduction in solids with variable thermal conductivity is
illustrated by the following examples.

Example 3.29. A plane wall 4 cm thick has one of its surfaces in contact with a
fluid at 130 ℃with a surface heat transfer coefficient of 250 W∙m−2∙K−1and the
other surface is in contact with another fluid at 30 ℃ with a surface heat transfer
coefficient of 500 W∙m−2∙K−1. The thermal conductivity of the wall varies with
temperature according to the law
k = 20 [ 1 + 0.001 T]
where T is the temperature. Determine the rate of heat transfer through the wall and
the surface temperatures of the wall.

Given:- L = 0.04 m; Ti = 130 ℃; hi = 250 W∙m−2∙K−1 ;To = 30 ℃ ;ho = 500

W∙m−2∙K−1; k = 20 [ 1 + 0.001 T].

To find:- (i) Qx (ii) T1 and T2

Schematic:

k = k0 [ 1 + βT ]

hi,Ti

h0,T0

Solution:

Rci = Thermal resistance for convection at the surface in contact with fluid at Ti

133
1 1
= h A = 250 ×1 = 0.004 m2∙K ∙W−1
i

Rco = Thermal resistance for convection at the surface in contact with fluid at To
1 1
= = 500 ×1 = 0.004 m2∙K ∙W−1
h0 A

(Ti − T1 )
Q= , where T1 = Surface temperature in contact with fluid at Ti.
Rci

Hence: T1 = Ti – QRci = 130 – 0.004 Q (1)

Similarly: T2 = To + QRco = 30 + 0.002Q (2)

From equations (1) and (2) we have,

T1 – T2 = 100 – 0.006Q (3)

T1 + T2
And: Tm = = 80 – 0.001Q (4)
2

Hence: km = ko [ 1 + βTm] = 20 × [1 + 0.001× {80 – 0.001Q}]

= 21.6 – 2 × 10 − 5Q
L 0.04
Hence thermal resistance offered by the wall = R = Ak = [21.6−2 × 10−5 Q]
m

[T1 − T2 ] 100 – 0.006Q


And: Q= = 0.04
R ( )
[21.6−2 × 10−5 Q]
Cross multiplying and simplifying we have,

Q2 – 1.41 × 10 6 Q + 1.8 × 10 10 = 0.
Hence: Q = (1.41 × 10 6 ±1.39 × 10 6) / 2

For physically meaningful solution T1 should lie between Ti and To. This is possible only
if,
Q = (1.41 × 10 6 − 1.39 × 10 6) / 2 = 10000 W.

Hence: T1 = Ti – QRci = 130 – 10000 × 0.004 = 90 ℃


And: T2 = T0 + QRco = 30 + 10000 × 0.002 = 50 ℃.

Example 3.30. The thermal conductivity of a plane wall varies with temperature
according to the equation
k(T) = k0 [ 1 + β T2 ]
where k0 and β are constants.(a)Develop an expression for the heat transfer through the wall
per unit area of the wall if the two surfaces are maintained at temperatures T1 and T2 and the

134
thickness of the wall is L.(b)Develop a relation for the thermal resistance of the wall if the
heat transfer area is A.
Schematic:

K = k0 [ 1 + βT2]

T1

L
L

x
dT
For steady state conduction we have, Qx = − kA( dx ) = constant.

dT
Or: Qx = − k0[1 +βT2]A( dx )

Separating the variables we have, Qxdx = − k0[1 +βT2]A dT

Integrating the above equation between x = 0 and x = L we have,

L T
∫0 Qx dx = − k0A ∫T (1 + βT 2 )dT
2
1

Or: QxL = − k0A [(T2 – T1) + (β/3)(T23 – T13)]

k0 A β
Hence: Qx = ( )(T1 – T2) [1 + (3) (T12 + T1 T2 + T22 )]
L

(𝑇1 − 𝑇2 )
= 1
k A β
( 0 )[1 + ( ) (T 2 +T T + T2 )]
1 1 2 2
L 3

Therefore thermal resistance of the wall is given by,


1
R= k A β
( 0 )[1 + ( ) (T21 +T1 T2 + T22 )]
L 3

Example 3.31. Find the steady-state heat flux through the composite slab made up of two
materials as shown in Figure P 3.29. Also find the interfacial temperature. The thermal
conductivities of the two materials vary linearly with the temperature in the following manner:
k1 = 0.05 [1 + 0.008 T]W∙m−1∙K−1 and k2 = 0.04 [1 + 0.0075 T] W∙m−1∙K−1.

135
600 0 C 30 0 C
k1 k2

5 cm 10 cm

Figure P 3.31: Schematic for problem 3.26

Schematic:

T3
Known: T1 = 600 0 C ; T2 = 30 0 C ;
T2
L1 = 0.05 m ; L2 = 0.10 m ;
T1
k1 k2
k1 = 0.05 [ 1 + 0.008T ] W∙m−1∙K−1 ;

k2 = 0.04 [ 1 + 0.0075T ] W∙m−1∙K−1;

Find : T3 and q

L1 L2

Assumptions: (i)One dimensional steady state conduction;(ii) No contact resistance between


the two layers

Solution: Mean thermal conductivity for the first layer is given by,

𝑇1 + 𝑇2
km1 = 0.05[1 + 0.008 ( )] = 0.05[1 + 0.004×(600 +T3)]
2

= 0.17 + 2 × 10 − 4 T3 W∙m−1∙K−1

136
𝑇2 + 𝑇3
Similarly : km2 = 0.04 [ 1 + 0.0075( )] = 0.04[1 + 0.00375×(30 +T3)]
2

= 0.0445 + 1.5 × 10 − 4 T3 W∙m−1∙K−1.

Rate of heat transfer through the first layer is given by,

km1 A(T1 − T3 ) (0.17 + 2 × 10−4 T3 )×1 × (600− T3 )


Q= =
L1 0.05

Similarly for the second layer we have,

km2 A(T3 − T2 ) (0.0445 + 1.5 × 10−4 T3 )×1 × (T3 − 30)


Q= =
L2 0.10

Equating the two expressions for Q we have


(0.17 + 2 × 10−4 T3 )×1 × (600− T3 ) (0.0445 + 1.5 × 10−4 T3 )×1 × (T3 − 30)
( ) =
0.05 0.10

Cross multiplying and simplifying we get,

T32 + 254.55 T3 – 373337 = 0

− 254.55 ± √[(254.55)2 −4 × 373337]


Therefore: T3 = 2

Since T3 has to lie between T1 and T2 it follows that,

− 254.55+ √[(254.55)2 −4 × 373337]


T3 = = 496.9 ℃
2

Therefore: km1 = 0.17 + 2 × 10 − 4 × 496.9 = 0.269 W∙m−1∙K−1.

km1 A(T1 − T3 ) 0.269 × (600−496.8)


Heat flux: q=Q/A= = = 555.22 W ∙m-2.
L1 0.05

Example 3.32. Consider a slab of thickness L. The two boundary surfaces are maintained at
temperatures T1 and T2. The thermal conductivity of the slab varies with temperature
according to the equation

k(T) = k0 [ 1 + βT ]

where k0 and β are constants. Develop an expression for the heat flux q(x) in the slab.
Measurements show that steady-state conduction through a plane wall without heat generation
produced a convex temperature distribution such that the mid-point temperature was ΔT0
higher than expected for a linear temperature distribution. Assuming that the thermal

137
conductivity has a linear dependence on temperature [k = k0(1 + βT)], where k0 and β are
constants,develop a relationship to evaluate β in terms of ΔT0, T1 and T2.

Schematic:
L

k = k0 [ 1 + βT ]

ΔT0 k = k0

T1
T2

For constant thermal conductivity k0, the temperature distribution in the wall is linear and is
given by,
x
T(x) = T1 – (T1 − T2 ) L

(T1 + T2 )
Therefore: T(x)|x=L/2 = (1)
2

When the thermal conductivity varies with temperature the temperature distribution in the
wall is determined as follows.

dT dT
Qx = − kA ( dx )= − k0 (1 + βT)A ( dx )

Therefore: Qx dx = − k0A(1 + βT) dT (2)

Integrating the above equation between x = 0 and any x at which the temperature is given by
T×(x) we have,

x T∗
∫0 Qx dx = ∫T − k 0 A(1 + βT)dT
1

β
Or: Qx x = − k0A [(T ∗ − T1 ) + (T ∗2 − T12 )] (3)
2

If equation is integrated between x = 0 and x = L we get

β
Qx L = − k0A [T2 − T1 ) + (T22 − T12 )]
2

138
T +T
k0 A [1+ β ( 1 2 )]
2
Or: Qx = (T1 − T2 )
L

km A
= (T1 − T2 ) (4)
L

T1 + T2
Where: km = k 0 [1 + β ( )]
2

Substituting this expression for Qx in Equation (3) we get,

x β
km (T1 – T2) (L) = − k0 [(T ∗ − T1 ) + (T ∗2 − T12 ) ]
2

The above equation simplifies to :

2 2 k x
T ∗2 + (β) T ∗ + [(β) {( km ) (T1 − T2 ) (L) − T1 } − T12 ] = 0
0

2 2 2 2 k x
− ( ) ± √{( ) − 4 [( ){( m )(T1 − T2 )( ) − T1 } − T21 ] }
β β β k0 L
Therefore: T× = 2

1 1 2 2 k x 2
= − 𝛽 ± √{(β) − [(β) {( km ) (T1 − T2 ) (L) + T12 + (β) T1 } ] }
0

1 1 2 k 2
T×|x=L/2 = − (𝛽) ± √{(β) − (βkm ) (T1 − T2 ) + T12 + (β) T1 }
0

But T×|x=L/2 = T(x)|x=L/2 + ΔT0 = (T1 + T2) / 2 + ΔT0.

Hence we have
______________________________________________
(T1 + T2)/2 +ΔT0 = − (1/β) ± √ (1/β2) – [{1 + β(T1 + T2)/2}/β ](T1 – T2) + T12 + (2 / β)T1

Example 3.31. A slab of thickness ‘L’ has its two surfaces at x=0 and x = L maintained at
uniform temperatures of T0 and TL respectively. The thermal conductivity of the slab has spatial
variation according to the law k = k0 [1 + α x], where ko and α are constants. Obtain
expressions for (i) temperature distribution in the slab, and (ii) rate of heat transfer through
the slab assuming one dimensional steady state conduction.

Solution:

139
L

k = k0 [1 + α x]

T1 T2

Figure E3.30 : Figure for Example


3.30

The governing differential equation for one dimensional steady state conduction without
heat generation is given by

d dT
dx
(k dx
) =0

substituting for k we have

d dT
dx
(k 0 [1 + αx] dx
) =0 (1)

Let 1 + α x = y ; Then α dx = dy or dy/dx = α. Then Equation(1) can be written


in terms of the variable ‘y’ as follows.

d dT
{dy [k 0 y (dy ) α]} α = 0

d dT
Or: [k 0 y (dy ) α]= 0
dy

Integrating w.r.t ‘y’ once we get,

dT
[k 0 y (dy ) α] = C1

140
C1 dy
Or: dT = ( )
α k0 y

Integrating once again we have,

C1
T=( ) ln y + C2
α k0

Substituting for ‘y’ in terms of ‘x’ we have

C1
T=( ) ln (1 + α x) + C2 (2)
α k0

Equation (2) is the general solution of Equation (1). The values of C1 and C2 can be obtained
from the two boundary conditions at x = 0 and at x = L as follows.

(i) at x = 0, T = T1; and (ii) at x = L, T = T2 ;

Condition (i) in Equation (2) gives: T1 = C2;

C1
Condition (ii) in Equation (2) gives: T2 = ( ) ln (1 + α L) + T1
α k0

α k0
Or: C1 = (T2 – T1) .
ln(1+ 𝛼𝐿)

Substituing the values of C1and C2 in Equation(2) we get the temperature distribution as:

ln (1+ αx)
T(x) = (T2 – T1) + T1
ln (1+ αL)

Expression for rate of heat transfer:


dT
At any ‘x’, Qx = - k A ( dx )

dT
Or: Qx = − [k0 (1 + α x)] A ( dx )

d ln (1+ αx)
= − [k0 (1 + α x)] A dx [(T2 – T1 ) ln (1+ αL) + T1 ]

2A(T − T )
1 α
= − [k0 (1 + α x)] ln (1+ ×
αL) ln (1+ αx)

k αA(T − T )
Or: Qx = 0ln (1+2 αL)1

Example 3.32. If in the above problem the thermal conductivity varies with distance
according to the law

141
K = k0 [1 + α x2]

Obtain expressions for (i) the temperature distribution T(x) and (ii) the rate of heat transfer.

Solution:
L

k = k0 [1 + α x 2]

T1 T2

Figure E3.31 : Figure for Example


3.31

The governing differential equation for one dimensional steady state conduction without heat
generation is given by

d dT
[k ]=0
dx dx

d dT
Or: [k 0 (1 + α x 2 ) ]=0 (1)
dx dx

Equation (1) can be solved by using the following substitution.

Let √α x = tan y.

dy
Differnatiating both sides w.r.t.’x’ we have, √𝛼 = sec 2 y (dx)

dy √𝛼 √𝛼
Or: (dx) = sec2 y = (1+ α x2 ) ,

dT dT dy √𝛼 dT
And: = × =( 2
dx dy dx 1+ α x ) dy

d dT √𝛼
Hence Equation reducues to: [k 0 √α ] × sec2 y = 0
dy dy

d dT
Or: [k 0 √α ]=0
dy dy

dT
Integrating w.r.t.’y’ we have, k 0 √α = C1
dy

142
C1
Or: dT = (k ) dy.
0 √α

C1
Integrating once again we get: T = (k ) y + C2
0 √α

C1
Or: T(x) = (k ) tan −1(√α x) + C2 (2)
0 √α

Boundary conditions are: (i) at x = 0 T (x) = T1 and (ii) at x = L ,T = T2.

Condition (i) in Equation (2) gives: C2 = T1.

C1
Condition (ii) in Equation (2) gives: T2 = (k ) tan −1(√α L) + T1
0 √α

(T2 − T1 ) k0 √α
Or: C1 = tan −1(√α L)

Substituting the values of C1 and C2 n Equation (2) we have,

tan−1 (√α x)
T(x) = (T2 − T1 ) tan−1 (
√α L)

Expression for Rate of Heat Transfer:

dT
Qx = − k0 [1 + α x2] A ( dx )

√ α
= − k0 [1 + α x2] A (T2 − T1 ) (1+ α x2 ) tan−1 (
√α L)

k0 A √α (T2 − T1 )
Or: Qx = tan−1 (√α L)

Exercise Problems:

3.1.A plane wall of 1 mm thick and conductivity 100 W∙m−2∙K−1 separates two fluids. One
surface of this wall is in contact with fluid A which is at a uniform temperature of 100℃ and
the surface heat transfer coefficient is 10 W∙m−2∙K−1. The other surface of the wall is in contact
with fluid B which is at 0℃ and the surface heat transfer coefficient is 100 W∙m−2∙K−1.(a) Draw
a resistance network for one dimensional steady state conduction across the thickness of the
wall and calculate the value of each resitance assuming unit area of the wall.(b)If the heat
transfer from fluid A to fluid B is to be calculated very accurately,
which parameters have to be measured very carefully and which are not important?Justify
your answer.

143
3.2. A composite wall which consists of two materials. Material A has a conductivity of 1.0
W∙m−1∙K−1 and material B has a conductivity of 5 W∙m−1∙K−1. The thickness of both the
material is 10 mm.A very thin electric heater is firmly attached to the outer surface of material
A and the other surface of the heater is completely insulated.The heater provides a heat flux of
5000 W∙m−2.The outer surface of the wall B is in contact with a fluid at 27℃ with a surface
heat transfer coefficient of 100 W∙m−2∙K−1.Neglecting the radiation and contact resistances,
(a) draw the equivalent thermal network and calculate all the resistances, (b)determine the
temperatures of the heating element,the interface temperature and the outer surface
temperature of wall B.(c) Sketch the temperature distribution for thecomposite wall.

3.3.If in problem 3.2,the contact resistance between layer A and layer B is 0.01K∙m2∙W−1, (a)
draw the equivalent thermal net work and calculate the outer surface tempearature of wall A
and the outer surface temperature of wall B.

3.4. Determine the rate of heat transfer through the walls of a closed rectangular enclosure of
dimensions 1 × 3 × 3 m made from a 5 mm thick steel plate. The enclosure contains hot gas
at 300 ℃ and the air surrounding the enclosure is at 30 ℃. The inside surface heat transfer
coefficient is 10 W∙m−2∙K−1 and the outside heat transfer coefficient is 20 W∙m−2∙K−1 .What is
the reduction in heat transfer rate if fibre insulation 2 cm thick is put all round the
enclosure?Assume that the thermal conductivity of the insulation varies with temperature in
the manner shown in the following table:

Temperature (℃): 0 100 200 300

Conductivity,k (W∙m−1∙K−1): 0.027 0.037 0.050 0.070

3.5.Determine the overall heat transfer coefficient based on the outer surface of an insuilated
steam pipe and the rate of heat transfer. The insulated steam pipe has the following dimensions
and specifications:
Innerdiameter of the pipe = 3 cm; Thickeness of pipe = 0.2 cm; Thickness of insulation = 1
cm; Heat transfer coefficient on inside surface = 10 W∙m−2∙K−1;Hat transfer coefficient on
outside surface = 10 W∙m−2∙K−1;Temperature of steam = 100℃;Temperature of surroundings
= 25℃;Thermal conductivity of pipe material = 15 W∙m−1∙K−1; Thermal conductivity of
insulation = 0.05 W∙m−1∙K−1.Also draw the equivalent thermal network indicating the
magnitudes of all resistances.

3.6.A fluid at a temperature of 400℃ is contained in a spherical container made of steel. The
container has 2 m inside diameter and 50 mm thick.In order to reduce the heat transfer from
the fluid to the ambient, the container is covered with mineral insulation thermal conductivity
0.06 W∙m−1∙K−1.The ambient temperature is 30 ℃.Calculate the rate of heat transfer without
and with insulation assuming that the surface heat transfer coefficient for the inner surface of
the container and on the outer surface of the insulation are both equal to 15 W∙m−2∙K−1.

3.7.A spherical metal dewar(k =15 W∙m−1∙K−1) of inside radius 10 cm and a thickness of 2.5
mm contains saturated liquid oxygen at 1.7 bar (saturation temperature = 95.4 K).The dewar

144
is covered by polystyrene foam insulation (k =0.033 W∙m−1∙K−1) 1 cm thick, followed by
another metal liner(k =15 W∙m−1∙K−1)of 2.5 mm.The outer surface of the outer metal liner has
an emissivity of 0.7 and is exposed to atmospheric air at 20 ℃ with a surface heat transfer
coefficient of 6 W∙m−2∙K−1.The heat transfer coefficient between oxgen and the inner surface
of thedewar is 150 W∙m−2∙K−1.The are specific contact resistance that charecterises the
interfaces between the insulation and the adjacent metal liners is 3.0 × 10−3 K∙m2∙W−1.(a)Draw
the equivalent thermal circuit showing all the resistances.(b) Fine out all the resistances and
the rate of heat transfer from the ambient to the oxygen in the dewar.

3.8.A nuclear fuel element in the form of a long cylindrical rod is 14 mm diameter and has a
thermal conductivity of 0.8 W∙m−1∙K−1.It is generating thermal energy at a uniform rate of
0.5× 10 8 W∙m−3.The elment is covered by a cladding material(k = 9 W∙m−1∙K−1) of 0.5 mm
thick.The heat generated is disspated to pressurized water flowing on the outside of the
cladding material with a bulk temperature of 300 ℃.The convective heat transfer coefficient
is 15,000 W∙m−2∙K−1.Determine the maximum temperature in the rod.

3.9.Radioactive waste (k = 2.5 W∙m−1∙K−1)is stored in a thick spherical steel container of inside
radius 45 cm and outside rdius of 50 cm with k = 8.5 W∙m−1∙K−1.Heat is generated in the waste
at a uniform rate of 10,000 W∙m−3.Heat is lost to the surroundings from the outer surface of
the container by natural convection with a heat transfer coefficient of 12 W∙m−2∙K−1. If the
surroundings temperature is 30 ℃,find the steady state temperature of the waste at the centre
of the sphere.

3.10.Two rods of identical size and shape are both supported between two heat sources at 100
℃ and are surrounded by air at 27 ℃. One rod is known to have a therml conductivity of 30
W∙m−1∙K−1and its mid point temperature is found to be 49 ℃. If the mid point temperature of
the other rod is measured to be 75 ℃ , find its thermal conductivity.

3.11.Two circular rods, both of diameter D and lemgth L are joined at one end and both are
heated to the same temperature T0 at their free ends.The film coefficient h is same for all
surfaces. If T∞ is thetempearture of the surrounding fluid and if the thermal conductivities of
the two rids are ka and kb, show that the temperature of the jun ction,Tj is generally:

k
Tj − T∞ √(k a ) sinh mb L+sinh maL
b
=
T0 − T∞ ka
√(k ) [sinh mb L cosh maL ]+ [sinh ma L cosh mbL ]
b

3.12.A hollow cylinder has its internal surface at radius r1 maintained at a uniform
temperature T1 and external surface at radius r2 maintained at a uniform temperature T2. The
thermal conductivity of the material of the cylinder varies with radius according to the law k
= k0 [1 + α r], where k0 and α are constants. Derive expressions for (i) radial temperature
distribution in the cylinder and (ii) rate of heat transfer through the cylinder. Assume one-
dimensional radial steady state conduction in the cylinder.

3.13. A hollow cylinder has its internal surface at radius r1 maintained at a uniform
temperature T1 and external surface at radius r2 maintained at a uniform temperature T2. The

145
thermal conductivity of the material of the cylinder varies with radius according to the law k
= k0 [1 + α r2], where k0 and α are constants. Derive expressions for (i) radial temperature
distribution in the cylinder and (ii) rate of heat transfer through the cylinder. Assume one-
dimensional radial steady state conduction in the cylinder.

3.14. A hollow sphere has its internal surface at radius r1 maintained at a uniform temperature
T1 and external surface at radius r2 maintained at a uniform temperature T2. The thermal
conductivity of the material of the cylinder varies with radius according to the law k = k0 [1 +
α r], where k0 and α are constants. Derive expressions for (i) radial temperature distribution
in the cylinder and (ii) rate of heat transfer through the cylinder. Assume one-dimensional
radial steady state conduction in the cylinder.

3.15. A hollow sphere has its internal surface at radius r1 maintained at a uniform temperature
T1 and external surface at radius r2 maintained at a uniform temperature T2. The thermal
conductivity of the material of the cylinder varies with radius according to the law k = k0 [1 +
α r2], where k0 and α are constants. Derive expressions for (i) radial temperature distribution
in the cylinder and (ii) rate of heat transfer through the cylinder. Assume one-dimensional
radial steady state conduction in the cylinder.

146
Chapter 4

Transient Conduction
4.1.Introduction: In general, the temperature of a body varies with time as well as position.In
chapter 3 we have discussed conduction in solids under steady state conditions for which the
temperature at any location in the body do not vary with time. But there are many practical
situations where in the surface temperature of the body is suddenly altered or the surface may
be subjected to a prescribed heat flux all of a sudden. Under such circumstances the
temperature at any location within the body varies with time until steady state conditions are
reached. In this chapter, we take into account the variation of temperature with time as well as
with position.However there are many practical applications where in the temperature variation
with respect to the location in the body at any instant of time is negligible. The analysis of such
heat transfer problems is called the “lumped system analysis”. Therefore in lumped system
analysis we assume that the temperature of the body is a function of time only.

4.2. Lumped system analysis: Consider a solid of volume V, surface area A, density ρ,
Specific heat Cp and thermal conductivity k be initially at a uniform temperature Ti.Suddenly
let the body be immersed in a fluid which is maintained at a uniform temperature T∞, which
is different from Ti.The problem is illustrated in Figure4.1.Now if

Surface in contact with fluid at


T∞ with surface heat transfer
Coefficient h

V = volume
A=surface area
ρ = density
Cp = specific heat
k = conductivity

Figure4.1: Nomenclature for lumped system analysis of transient


Conduction heat transfer

147
T(t) is the temperature of the solid at any time t, then the energy balance equation for the
solid at time t can be written as

Rate of increase of energy of the solid = Rate of heat transfer from the fluid to the solid

dT
i.e., ρVCp ( ) = hA[T∞ - T(t)]
dt

𝑑𝑇 hA
Or: ( 𝑑𝑡 ) = ρV C [T∞ − T(t)]
p

hA
For convenience, a new temperature θ(t) = T(t) - T∞ is defined and denoting m =
ρV Cp
the above equation can be written as


( dt ) = − m θ (4.1)

Equation(4.1) is a first order linear differential equation and can be solved by separating the
variables. Thus


= − m dt
θ

Integrating we get ln θ = − mt + ln C, where ln C is a constant.

Or: θ = C e – mt (4.2)

At time t = 0 , T(t) = Ti or θ = Ti − T∞ = θi (say). Substituting this condition in Equation (4.2)


we get,
C = θi.
Substituting this value of C in Equation (4.2) we get the temperature θ(t) as follows.

θ(t) = θi e − mt

θ(t)
Or: = e – mt (4.3)
θi

Since LHS of Equation (4.3) is dimensionless, it follows that 1/m has the dimension of time
and is called the time constant.Figure 4.2 shows the plot of Equation(4.3) for different values
of m. Two observations can be made from this figure and Equation (4.3).

1. Equation (4.3) can be used to determine the temperature T(t) of the solid at any time t
or to determine the time required by the solid to reach a specified temperature.

148
2.The plot shows that as the value of m increases the solid approaches the surroundings
temperature in a shorter time.That is any increase in m will cause the solid to
respond more quickly to approach the surroundings temperature.

1.0
θ(t)
θi

t
Figure4.2: Dimensionless temperature as a function of time for
a
solid with negligible internal temperature gradients

The definition of m reveals that increasing the surface area for a given volume and the heat
transfer coefficient will increase m. Increasing the density, specific heat or volume decreases
m.

4.2.2.Criteria for Lumped System Analysis:To establish a criterion to neglect internal


temperature gradient of the solid so that lumped system analysis becomes applicable, a
Characteristic length Ls is defined as

Ls = V /A (4.4)

and the Biot.number Bi as


hLs
Bi = (4.5)
k

For solids like slabs, infinite cylinder, and sphere, it has been found that the error by
neglecting internal temperature gradients is less than 5 %, if

Bi < 0.1 (4.6)


The physical significance of Biot number can be understood better by writing the expression
for Biot number as follows:

hLs (Ls /Ak) Thermal resistance for conduction


Bi = = (1/hA)
= Thermal resistance for convection
k

149
Hence a very low value of Biot number indicates that resistance for heat transfer by
conduction within the solid is much less than that for heat transfer by convection and
therefore a small temperature gradient within the body could be neglected.

Example 4.1. A copper cylinder 10 cm diameter and 15 cm long is removed from a liquid
nitrogen bath at ─ 196 ℃ and exposed to room temperature at 30 ℃. Neglecting internal
temperature gradients find the time taken by the cylinder to attain a temperature of 0 ℃,
with the following assumptions: Surface heat transfer coefficient = 30 W∙m-2∙K−1.
Density of the copper cylinder = 8800 kg∙m-3.Specific heat of the cylinder = 0.38 kJ∙kg−1∙K−1.
Thermal conductivity of the cylinder = 350 W∙m−1∙K−1.

Schematic:

Ti Known: Ti = − 196 ℃; T∞ = 30 ℃;

D = 10 cm or R = 0.05 m; L = 0.15 m
T∞ D h = 30 W∙m-2∙K−1; k = 350 W∙m−1∙K−1;

ρ = 8800 kg∙m-3;Cp = 0.38 kJ∙kg−1∙K−1;

h T(t) = 0

Find: Time taken t for the solid to reach 0 ℃.

Assumptions: (i) Internal temperature gradients are negligible; (ii) solid properties are
constant

hR 30 ×0.05
Solution: Biot Number = = = 0.0043 which is << 0.1. Hence internal
k 350

temperature gradients can be neglected. Let θ(t) = T(t) – T∞

hA 2(πR2 + πRL)h 2(R+L)h 2 ×(0.05+0.15)×30


We have,m = = = = = 4.785 ×10 − 4 s−1.
ρVCp πR2 L ρCp RL ρCp 0.05 ×0.15 ×8800 ×0.38

And: θ(t) = T(t) – T∞ = θi e –mt.

Hence 0 – 30 = ( - 196 – 30) × exp {− 4.785 × 10 – 4 × t}

Solving for t we get: t = 4226 s = 1 hr 10.43 mins.

150
Example 4.2.A person is found dead at 5 PM in a room where the temperature is 20 ℃. The
temperature of the body is measured to be 25 ℃ when the body was found and the surface
heat transfer coefficient is estimated to be 8 W∙m-2∙K−1.Modeling the body as a cylinder of 30
cm diameter and 1.75 m long. Neglecting internal temperature gradients (lumped system
analysis)and assuming the body temperature when the person is alive to be 37 ℃, determine
the time of death.Assume the following properties for the body: density = 985 kg∙m-3and
specific heat at constant pressure = 3470 J∙kg−1∙K−1

Schematic::

d = 0.3 m

L = 1.75 m

Known: T∞ = 20 ℃ ; Ti = 37 ℃ ; T = 25 ℃ ; h = 8 W∙m-2∙K−1; ρ = 985 kg∙m-3;


Cp = 3475 J∙kg−1∙K−1.

Find : Time of death

Assumptions: (i) Internal Temperature gradients are negligible, i.e T = T(t)


(ii)Thermo physical properties of the body and the surface heat transfer coefficient are
constant

𝜋𝑑2 𝑑
Solution: Total Surface area of the body = A = 2 × + π d L = πd ( 2 + 𝐿)
4
0.3
= π × 0.3 × ( 2 + 1.75) = 1.79 m2
𝜋𝑑2 𝜋×0.32
Volume of the body = V = ×L= ×1.75 = 0.124 m 3
4 4

hA 8.0× 1.79
m = ρVC = = 3.374 × 10 ─ 5 s−1.
p 985×0.124× 3475

θ(t)
= e – mt.
θi

25−20
= exp {− mt}
37−20

Solving for mt we get , mt = 1.224.


1.224
Therefore: t = 3.374∗ 10─ 5 ∗3600 = 10 hours

Therefore time of death = (12 +5) ─ 10 = 7.00 AM.

Example 4.3: A thin copper wire having a diameter D and length L (insulated at

151
the ends) is initially at a uniform temperature of T0. Suddenly it is exposed to a gas
stream, the temperature of which changes with time according to the equation

T∞ = Tf (1 ─ e─ ct) + T0

where Tf, T0 and c are constants. The surface heat transfer coefficient is h. Obtain an
expression for the temperature of the wire as a function of time t.

Schematic:
h,T∞

T(0) = T0

L
Known: Rod of diameter D and length L, initially at T0, is suddenly exposed to en
environment whose temperature varies with time according to the law T∞ = Tf (1 ─ e─ ct) +
T0, where Tf, T0 and c are constant

Find: Temperature of rod as a function of time

Assumptions: (i) Internal temperature gradients for the rod is negligible; (ii) Thermal
properties of the rod are constant ;(iii)Heat transfer from the circular faces of the cylinder is
negligible.

Solution: Let T(t) be the temperature of the cylinder at any time t. Energy balance for the
cylinder for a time interval dt is given by

hA [T∞(t) - T(t)] dt = ρVCp dT


where dT is the increase in temperature of the cylinder in time dt.

dT hA
Or: = ρ V C [T∞(t) - T(t)]
dt p

hA
Putting m = ρ V C the above equation reduces to:
p

dT
+ m T(t) = m T∞(t)
dt

Substituting the given expression for T∞ we have,

dT
+ m T(t) = m [T0 + Tf (1 – e - ct)]
dt

152
dT
Or: + m [T(t) – T0] = mTf (1 – e – ct )
dt

Let θ (t) = T(t) – T0. Then the above equation reduces to:


+ m θ (t) = mTf (1 – e – ct ) (1).
dt

dy
This equation is of the form + Py = Q, which is solved by multiplying throughout by an
dx
integrating factor and then integrating. For Equation (1) the integrating factor is

e ∫mdt = e mt. Therefore multiplying Equation (1) by e mt we get,


[e mt ( dt ) + m e mt θ (t)] = mTf [e mt – e (m – c) t]

d
Or: (emtθ) = mTf [e mt – e (m – c)t]
dt

Integrating with respect to t we get,

𝑒 𝑚𝑡 𝑒 (𝑚−𝑐)𝑡
emt θ(t) = mTf [( ) − (𝑚−𝑐)
] + C1
𝑚

m
Or: θ(t) = Tf − Tf e − ct + C1e − mt (2)
(m −c)

When t = 0 , T(0) = T0 i.e., θ(0) = 0. Substituting this condition in Equation (2) we get
m
0 = Tf − Tf + C 1
(m −c)

c
Or: C1 = ((m −c)) Tf.

Substituting this expression for C1 in Equation (2) we get the temperature of the cylinder as:
m c
θ(t) = Tf − Tf e – c t + Tf e − mt
(m −c) (m −c)

m c
Or: T(t) – T0 = Tf [1 − (m −c) e−ct + (m −c) e−mt ],

hA πDLh 4h
Where: m = ρVC = π = ρDC .
p ρ( D2 )Cp p
4

153
Example 4.4: A solid sphere of radius R is initially at a uniform temperature T0. At a certain
instant of time (t >0), the sphere is suddenly exposed to the surroundings at a temperature Tf
and the surface heat transfer coefficient, ‘h’. In addition from the same instant of time, heat is
generated within the sphere at a uniform rate of q’’’ units per unit volume. Neglecting internal
temperature gradients, derive an expression for the temperature of sphere as a function of time

Schematic:

T(0) = T0

q′′′ for t > 0


R

h,Tf

Known: Solid sphere of radius R is initiallt at temperature T0; Suddenly at time t = o the
sphere is generating heat at q’’’ W∙m-3 and simultaneously the sphere is losing heat to the
ambient at Tf with a surface heat transfer coefficient h.

Find: Temperature of sphere as a function of time T(t)

Assumptions: (i) Internal temperature gradients for the rod is negligible; (ii) Thermal
properties of the rod are constant.

Solution:Energy balance equation for the sphere at any time t can be written as:

Rate of energy generated + Rate of heat transfer to the sphere = Rate of increase of energy of
the sphere; i.e.,
4 4 dT
(3 × πR3 ) q′′′ + 4πR2 h [Tf − T(t)] = (3 × πR3 ) ρCp ( dt )

dT 3h q′′′
Or: ( dt )+ (ρRC ) [T(t) − Tf ] =
p ρCp

Let θ(t) = T(t) – Tf. Then the above equation reduces to:


( dt ) + mθ = q0 (1)

154
3h 𝑞′′′
Where: m = (ρRC ) and q0 = 𝜌𝐶 .
p 𝑝

Multiplying Equation (1) by the integrating factor e mt we have,


emt ( dt )+ emt mθ = q0 emt

d
Or: (θemt) = q0 emt
dt

Integrating throughout w.r.t. t we get,

q
θ emt = ( m0 ) emt + C1

q
Or: θ = ( m0 ) + C1e – mt (2)

At t = 0 , T = T0 or θ = T0 – Tf = θ0 (say). Substituting this condition in Equation (2) we

q
get C1 = (T0 – Tf) – ( m0 ). Therefore the temperature in the sphere as a function of time is
given by:

q q
θ(t) = [[(T0 – Tf ) – ( m0 )] e − mt + ( m0 )

q
Or: θ(t) = ( m0 ) [1 – e−mt ] + (T0 – Tf) e − mt

q0 q′′′ ρ R Cp q′′′ R
Where: = ρC × = ( )
m p 3h 3h

Example 4.5: A solid steel ball (ρ =8000 kg∙m-3; cp = 0.42 kJ∙kg−1∙K−1) 5 cm in diameter is
at a uniform temperature of 450 ℃. It is quenched in a controlled environment which is
initially at 90 ℃ and whose temperature increases linearly with time at the rate of 10 ℃ per
minute.If the surface heat transfer coefficient is 58 W∙m-2∙K−1, determine the variation of the
temperature of the ball with time neglecting internal temperature gradients. Find the value of
the minimum temperature to which the ball cools and the time taken to reach this minimum
temperature.

155
Schematic:

T(0) = Ti

h,Tf

Known: T(0) = Ti = 450 ℃;h = 58 W∙m-2∙ K−1 ; Cp = 0.42 kJ∙kg−1K−1; ρ = 8000 kg∙m-3;

𝑑𝑇𝑓
at t = 0, Tf = 90 ℃; (
𝑑𝑡
) = 10 ℃ per minute = (1/6) ℃∙ s−1.

Find: (i) Tminimum ; (ii) time taken to reach Tminimum

Assumptions: (i) Internal temperature gradients for the rod is negligible,i.e.Temperture of


the solid is a function of time only; (ii) Thermal properties of the ball are constant

Solution: Tf = a + bt, where a and b are constants;

dTf 1
at t = 0, Tf = 90 0C and = 6 ℃∙ s−1.
dt

dTf 1
Therefore a = 90 0 C and b = ( )=6
dt

𝑡
Or: Tf = 90 + 6 (t in seconds) (1)

Energy balance equation for the sphere at any time t can be written as

dT
ρVCp ( ) = hA [Tf(t) – T(t)]
dt

dT hA
Or: ( dt ) = (ρVC ) [Tf(t) – T(t)]
p

hA
Letting m = ( ) the above equation can be written as
ρVCp
dT
( dt ) + mT(t) = m Tf (t)

156
Substituting for Tf(t) from Equation (1) we have

dT 𝑡
( dt ) + mT(t) = m [90 + 6]

Multiplying the above equation with the integrating factor e mt we get,

dT 𝑡
e mt ( ) + mT(t) emt = m [90 + 6] e mt
dt

d 𝑡
Or: (T e mt) = m [90 + 6] e mt
dt

Integrating throughout w.r.t. t we have,

𝑡
T e mt = m ∫ [90 + 6] emt dt + C1

t
Or: T = m e−mt ∫ [90 + 6] emt dt + C1 e−mt

90 emt temt emt


= m e − mt [( ) + ( 6m ) − (6m2 )]+ C1 e − mt
m

t 1
Or: T(t) = [90 + − ]+ C1 e – mt (2)
6 6𝑚

When t = 0 , T(t) = Ti. Substituting this condition in the above equation and solving for C1
we get,
1
C1 = Ti – 90 + 6𝑚

Therefore the temperature of sphere as a function of time is given by,

𝑡 1 1
T(t) = [90 + − ] + [Ti – 90 + ] e − mt (3)
6 6𝑚 6𝑚

dT
ForT(t) to be extremum ( dt ) = 0.

dT 1 1
Therefore we have: ( dt ) = 6 + [Ti – 90 + ] e − mt (− m) = 0
6𝑚

Substituting Ti = 450 0 C and simplifying we get

1 1
(360 m + 6) e − mt = 6

Or: e mt = (2160 m + 1) (4)

157
hA 4π R2 h 3h 3×58
m=( )=( 4 )= = = 2.07 × 10 – 3
ρVCp ρGp (3)πR3 ρCpR 8000×0.025×0.42× 103
(5)

Using this value of m in Equation (4) and solving for t we get the time t× at which T(t) will
be optimum. Thus,

ln(2160 𝑚+1) ln(2160 ×2.07 × 10 −3 + 1)


t× = = = 821 s.
𝑚 2.07 × 10 −3

Substituting this value of t in Equation (3) we get the optimum value of T(t).Hence,

821 1
[T(t)]opt = [90 + − ]
6 6× 2.07 × 10−3

1
+ [450 – 90 + ] exp(−2.07 × 10−3 × 821)
6× 2.07 × 10−3

= 327 ℃

d2 T
It can be seen form Equation (3a) that >0
dt2

Hence [T(t)]opt obtained will be Tminimum.

Example 4.6: A house hold electric iron has a steel base [ρ =7840 kg∙m-3 ; cp = 450
J∙kg−1∙K−1 ;k = 70 W∙m−1∙K−1] which weighs 1 kg. The base has an ironing surface area of
0.025 m2 and is heated from the other surface with a 250 W heating element. Initially the
iron is at a uniform temperature of 20 ℃ with a heat transfer coefficient of 50 W∙m−2∙K−1. (a)
Determine the temperature of the plate 300 seconds after the heater switch is on .(b) What
would be the equilibrium temperature of the iron if the control of the iron box did not switch
of the current?

Schematic:

Qc h ,T∞
A

158
Known: Q = 250 W; A = 0.025 m2 ; T∞=20 ℃ h = 50 W∙m−2∙K−1; ρ = 7840 kg∙m-3;

Cp = 450 J∙kg−1∙K−1; k = 70 W∙m−1∙K−1; m = 1 kg ; t = 5 minutes = 300 s

Find: (a)T/t = 300 s. (b)T after steady state is reached

Assumptions: (i) Internal temperature gradients for the rod is negligible; (ii) Thermal
properties of the iron are constant
𝑚 1
Solution: (a) V = = 7840 = 0.0001275 m3 = 1.275 ×10 − 4 m3.
𝜌

𝑉 1.275 × 10−4
Charecteristic length: L=𝐴= = 0.0051 m
0.025

hL 50 ×0.0051
Bi = = 0.00364
k 70

Since Bi < 0.1, it can be assumed that temperature gradients within the plate are negligible.
Hence the temperature of the plate depends only on time till steady state condition is reached.
Energy balance at any time t for the plate can be written as

dT
Q − Qc = ρVCp ( )
dt

dT
Or: Q – hA(T - T∞) = ρVCp ( )
dt

dT Q
Or: ( dt ) + m(T - T∞) = (ρVC ) (1)
p

hA
where m = ρVC .Letting θ = T - T∞, Equation (1) can be written as
p

dθ Q
( dt ) + m θ = ρVC .
p

Multiplying the above equation by the integrating factor e mt, ( e∫mdt=emt) we get,

dθ Q
( dt ) e mt + m θe mt = (ρVC ) emt
p

d Q
Or: (θe mt) = (ρVC ) emt
dt p

Q 1
Or: (θe mt) = (ρVC ) emt (𝑚) + C1
p

159
Q
Or: θ = (ρVC ) + C1e − mt (2)
pm

When t = 0, T = Ti.Hence θ = Ti - T∞ = 20 – 20 = 0 ℃

Substituting this condition in Equation (2) we get

Q
0 = (ρVC ) + C1
pm

Q
Or: C1 = − (ρVC )
pm
Therefore the temperature in the plate as a function of time is given by

Q
θ = (ρVC ) [ 1 − e − mt ]
pm

Q
Or: θ = (hA)[ 1 − e − mt ] (3)

Q 250 50×0.025
= = 200 and m = = 2.8 × 10 – 3
hA 50×0.025 1×450

Therefore: θ = 200 [ 1 – e −0.028t]

When t = 300 s, θ = T - T∞ = 200 ×[ 1 – exp (− 0.028 × 300)] = 113.7 ℃

Or: T = 113.7 + 20 = 133.7℃.

(b) When the control switch is not switched off and the iron is left in the ambient, steady
state condition will be attained as t tends to ∞ so that the heat transferred to the base plate
will be convected to the ambient. i.e., Q = Qconvection

Therefore 250 = 50 ×0.025 ×[T – 20 ]

Or: T = 220 0 C.

This answer can also be obtained by putting t = ∞ in Equation (3) and solving for T.

4.3 One-dimensional Transient Conduction (Use of Heissler’s Charts): There are many
situations where we cannot neglect internal temperature gradients in a solid while analyzing
transient conduction problems. Then we have to determine the temperature distribution within
the solid as a function of position and time and the analysis becomes more complex. However
the problem of one-dimensional transient conduction in solids without heat generation can be
solved readily using the method of separation of variables.The analysis is illustrated for solids
subjected to convective boundary conditions and the solutions were presented in the form of

160
transient – temperature charts by Heissler. These charts are now familiarly known as
“Heissler’s charts”.

4.3.1.One-dimensional transient conduction in a slab:- Let us consider a slab of thickness


2L, which is initially at a uniform temperature Ti. Suudenly let the solid be exposed to an
environment which is maintained at a uniform temperature of T∞ with a surface heat transfer
coefficient of h for time t > 0.Figure4.3 shows the geometry , the coordinates and the boundary
conditions for the problem. Because of symmetry in the problem with respect to the centre of
the slab the ‘x’ coordinate is measured from the centre line of the slab as shown in the figure.

2L
Surfaces
exposed to a T = Ti at t = 0
fluid at T∞
with heat
transfer T = T(x,t)
coefficient h
for time t ≥ 0

x
Figure4.3: Geometry, coordinates and boundary conditions for one
dimensional transient conduction in a slab
conduction in a slab

The mathematical formulation of this transient conduction problem is given as follows:

∂2 T 1 ∂T
Governing differential equation: = (𝛼) (4.7a)
∂ x2 ∂t

Initial condition: at t = 0, T = Ti in 0 < x < L (4.7b)

Boundary conditions are:

𝜕𝑇
(i) at x = 0, 𝜕𝑥 = 0 (axis of symmetry) for all t > 0 (4.7c)

∂T
(ii) at x = L, − k ⌈ ∂x ⌉ = h(T|x = L − T∞) for all t > 0 (4.7d)
𝑥=𝐿

It is more convenient to analyze the problem by using the variable θ(x,t), where

161
θ(x,t) = T(x,t) - T∞. Then equations (4.7a) to (4.7d) reduce to the following forms:

∂2 θ 1 ∂θ
= (𝛼) ∂t (4.8a)
∂ x2

Initial condition : at t = 0, θ = Ti − T∞ = θi in 0 < x < L (4.8b)

Boundary conditions reduce to :

∂θ
(i) at x = 0, = 0 for all t > 0 (axis of symmetry) (4.8c)
∂x

∂θ
(ii) at x = L, − k ⌈∂x⌉ = hθ|x = L for all t > 0 (4.8d)
𝑥=𝐿

Equation(4.8a) is a homogeneous seocd order linear partil differential equation which can be
solved by the method of separation of variables as shown below:

Let θ(x,t) = X(x) Y(t) (4.9)

Substituting this in Equation (4.8a) we get

d2 X X dY
Y( ) = α ( dt )
dx2

1 d2 X 1 dY
Or :
X
( dx2 ) = Yα ( dt ) (4.10)

LHS of Equation (4.10) is a function of x only and the RHS of Equation (4.10) is a function
of t only.They can be equal only to a constant say − λ2.(The reason to choose the negative
sign is to get a physically meaningful solution as explained later in this section).Hence we
have two equations namely,

1 d2 X 1 dY
𝑋
( dx2 ) = − λ2 and Yα
( dt ) = − λ2

d2 X
Or: + λ2X = 0 (4.11)
dx2

dY
And: = −αλ2 Y (4.12)
dt

Solution to Equation (4.11) is X(x) = C1 cos (λx) + C2 sin (λx) (4.13)

and solution to Equation (4.12) is: Y(t) = D exp (− αλ2t) (4.14)

162
with C1, C2 and D are constants of integration. Substituting these solutions in Equation(4.9)
we have
θ(x,t) = D exp (− αλ2t) [C1 cos (λx) + C2 sin (λx)]

Or: θ(x,t) = exp (− αλ2t) [A1 cos (λx) + A2 sin (λx)] (4.15)

Equation (4.15) is the general solution involving the constants A1, A2 and λ which can be
determined using the two boundary conditions and the initial condition as illustrated below.

∂θ
Now from Equation (4.15), = λ exp (− αλ2t) [ −A1 sin (λx) + A2 cos (λx)]
∂x

Substituting boundary condition (i) we have 0 = λ exp (− αλ2t) [0 + A2] for all t.

Hence A2 = 0. Therefore Equation (4.15) reduce to

θ(x,t) = A1 exp (− αλ2t) cos (λx) (4.16)

Now, θ(L,t) = A1 exp (− αλ2t) cos (λL)

∂θ
And: = λ exp (− αλ2t) [ −A1 sin (λx) ]
∂x

∂θ
Hence: [∂x ] = −λ A1 exp (− αλ2t) sin (λL)
x=L

Therefore boundary condition (ii) can be written as

k λ A1 exp (− αλ2t) sin (λL) = h A1 exp (− αλ2t) cos (λL)

h
Or; tan (λL) = kλ

i.e., λL tan (λL) = Bi (4.17)

where Bi = hL / k.
Equation (4.17) is called the “characteristic equation” and has infinite number of roots
namely λ1, λ2, λ3, ..............Corresponding to each value of λ we have one solution and hence
there are infinite number of solutions. Sum of all these solutions will also be a solution as the
differential equation is linear. Therefore the solution θ(x,t) can be written as follows:

θ(x,t) = ∑∞
𝑛=1 An exp (− αλn 2t) cos (λnx) (4.18)

To find An: The constants An in Equation (4.18) can be found using the orthogonal property of
trigonometric functions as shown below.Substituting the initial condition we have

θi = ∑∞
𝑛=1 An cos (λnx) (4.19)

163
Multiplying both sides of Equation(4.19) by cos λmx and integrating w.r.t ‘x’ between the
limits 0 and L we have,
𝐿 𝐿
∫0 θi cos (λm x) dx = ∫0 ∑∞
𝑛=1 An cos (λm x) cos (λn x) dx

Using the orthogonal property ,


∫ An cos (λmx) cos (λnx) dx = 0 for λn ≠ λm

The above equation reduce to


𝐿 𝐿
∫0 θi cos (λn x) dx = ∫0 An cos 2 (λn x) dx

L
θi ∫0 cos (λn x) dx
Or: An = L
∫0 cos 2 (λn x) dx

It is very convenient to express Equation (4.18) in dimension less form as follows:

θ(x,t) λ∗n x
= ∑ (An* exp (− λn* 2 Fo) cos ( ) (4.20)
θi L

An αt
where An* = ; λn* = λnL ; Fo = Fourier Number = L2 ;
θi

4.3.2.Heissler’s Charts for transient conduction:- For values of Fo > 0.2 the above series
solution converges rapidly and the solution will be accurate within 5 % if only the first term in
the series is used to determine the temperature. In that case the solution reduces to:

θ(x,t) λ∗1 x
= (A1* exp (− λ1* 2 Fo) cos ( ) (4.21)
θi L

From the above equation the dimensionless temperature at the centre of the slab (x =0) can be
written as:

θ(0,t)
= (A1* exp (− λ1* 2 Fo) (4.22)
θi

The values of A1* and λ1* for different values of Bi are presented in the form of a table (See
Table 4.1). These values are evaluated using one term approximation of the series solution.
θ(x,t)
It can also be concluded from Equation (4.21) at any time‘t’the ratio θ(0,t) will be independent
of temperature and is given by,

θ(x,t) λ∗1 x
= cos ( ) (4.23)
θ(0,t) L

164
Heissler has represented Equation (4.22) and (4.23) in the form of charts and these charts are
normally referred to as Heissler’s charts. Equation (4.22) is plotted as Fourier number Fo
θ(o,t)
versus dimensionless centre temperature θ using Biot number,Bi as the parameter
i
θ(x,t)
[Figure4.4(a)], where as Equation (4.23) is plotted as Biot number [Figure 4.4(b)]using
θ(0,t)
𝑥
the dimensionless distance 𝐿 as the parameter.In Figure[4.4(a)], the curve for Bi=∞
corresponds to the case h → ∞, or the outer surfaces of the slab are maintained at the ambient
temperature T∞. For very small values of Biot number, ie. Bi < 0.1 implies that the internal
conductance is verylarge in comparison with the surface heat transfer coefficient and this in
turn, implies that the temperature distribution within the solid is sufficiently uniform and hence
lumped system analysis becomes applicable.Figure 4.4(c) shows the plot of
Table 4.1: Coefficients used in one term approximate solution of transient one dimensional heat conduction
in plane walls, cylinders, and spheres (Bi = hL/k for a plane wall of thickness 2L, and Bi = hr o/k for a
cylinder or sphere of outer radius r0)

Bi Plane Wall Infinite Cylinder Sphere


λ1* A1* λ1* A1* λ1* A1*
0.01 0.0998 1.0017 0.1412 1.0025 0.1730 1.0030
0.02 0.1410 1.0033 0.1995 1.0050 0.2445 1.0060
0.04 0.1987 1.0066 0.2814 1.0099 0.3450 1.0120
0.06 0.2425 1.0098 0.3438 1.0148 0.4217 1.0179
0.08 0.2791 1.0130 0.3960 1.0197 0.4860 1.0239
0.10 0.3111 1.0161 0.4417 1.0246 0.5423 1.0298
0.20 0.4328 1.0311 0.6170 1.0483 0.7593 1.0592
0.30 0.5218 1.0450 0.7465 1.0712 0.9208 1.0880
0.40 0.5932 1.0580 0.8516 1.0931 1.0528 1.1164
0.50 0.6533 1.0701 0.9408 1.1143 1.1656 1.1441
0.60 0.7051 1.0814 1.0184 1.1345 1.2644 1.1713
0.70 0.7506 1.0918 1.0873 1.1539 1.3525 1.1978
0.80 0.7010 1.1016 1.1490 1.1724 1.4320 1.2236
0.90 0.8274 1.1107 1.2048 1.1902 1.5044 1.2488
1.00 0.8603 1.1191 1.2558 1.2071 1.5708 1.2732
2.00 1.0769 1.1785 1.5995 1.3384 2.0288 1.4793
3.00 1.1925 1.2102 1.7887 1.4191 2.2889 1.6227
4.00 1.2646 1.2287 1.9081 1.4698 2.4556 1.7202
5.00 1.3138 1.2403 1.9898 1.5029 2.5704 1.7870
6.00 1.3496 1.2479 2.0490 1.5253 2.6537 1.8338
7.00 1.3766 1.2532 2.0937 1.5411 2.7165 1.8673
8.00 1.3978 1.2570 2.1286 1.5526 2.7654 1.8920
9.00 1.4149 1.2598 2.1566 1.5611 2.8044 1.9106
10.0 1.4289 1.2620 2.1795 1.5677 2.8363 1.9249
20.0 1.4961 1.2699 2.2880 1.5919 2.9857 1.9781
30.0 1.5202 1.2717 2.3261 1.5973 3.0372 1.9898
40.0 1.5325 1.2723 2.3455 1.5993 3.0632 1.9942
50.0 1.5400 1.2727 2.3572 1.6002 3.0788 1.9962
100.0 1.5552 1.2731 2.3809 1.6015 3.1102 1.9990
∞ 1.5708 1.2732 2.4048 1.6021 3.1416 2.0000

165
Q
dimensionless heat transferred as a function of dimensionless time for different values of
Q0
the Biot number for a slab of thickness 2L. Here Q represents the total amount of thermal
energy which is lost by the slab up to any time t during the transient conduction heat transfer.
The quantity Q0, defined as,
Q0 = ρ V Cp[Ti - T∞] (4.23)
represents the initial thermal energy of the slab relative to the ambient temperature.

4.3.3.Transient-Temperature charts for Long cylinder and sphere: The dimensionless


transient-temperature distribution and the heat transfer results for infinite cylinder and sphere
can also be represented in the form of charts as in the case of slab. For infinite cylinder and
sphere, the radius of the outer surface R is used as the characteristic length so that the Biot
hR 𝑟
number is defined as Bi = k and the dimensionless distance from the centre is 𝑅where r is any
radius (0 ≤ r ≤ R).These charts are illustrated in Figs. (4.5a) to (4.6c).

4.3.4.Illustrative examples on the use of Transient Temperature Charts:Use of the transient


temperature charts for slabs, infinite cylinders and spheres is illustrated in the following
examples.

Example 4.6. A brick wall ( α = 0.5 ×10 ─ 6 m2∙s−1, k = 0.69 W∙m−1∙K−1 and ρ = 2300 kg∙m-3)
of 10 cm thick is initially at a uniform temperature of 230 ℃. The wall is suddenly exposd to a
convective environment at 30 ℃ with a surface heat transfer coefficient of 60 W∙m−2∙K−1. Using
the transient-temperature charts, determine (a)the centre temperature at ½ hour and 2 hours
after the exposure to the cooler ambient,(b)temperature at the outer surface of the wall ½ hour
and 2 hours after the exposure to the cooler ambient,(c)energy removed from the wall per m2
during ½ hour and during 2 hours.(d) What would be the time taken for the surface of the wall
to reach a temperature of 40 ℃.

Schematic:

x Known: α = 0.5 ×10 − 6 m2∙s−1; ρ = 2300 kg∙m-3;

k = 0.69 W∙m−1∙K−1; 2L = 0.1 m ; Ti = 230 ℃ ;

Ti T∞ = 30 ℃.; h = 60 W / (m2 – K) ;Ti = 230 ℃.

Find: (a) T/x=0 at t = ½ hour and at t = 2 hours


(b) T/x=L at t = ½ hour and at t = 2 hours
(c) Qremoved per m2 during ½ hour and 2 hours

(d) t for T/x=L = 40 ℃

2L

Assumptions: (i) one dimensional conduction; (ii) thermal conductivity of the solid is
constant

166
Solution: (a) (i) t = 0.5 h = 0.5 ×3600 = 1800 s.

hL 60 ×0.05
Bi = = = 4.35
k 0.69

Since Bi > 0.1, internal temperature gradients cannot be neglected. i.e. T = T(x,t)

Hence Heissler’s transient temperature charts are to be used.

αt (0.5 × 10−6 ) × 1800


Fourier nuber, Fo = L2 = (0.05)2
= 0.36

From the Heissler’s chart for a slab of thickness 2L,

T0 − T∞
θ0 = = 0.8 (T0 is the centre temperature of the slab)
Ti − T∞

Therefore: T0 = T∞ + 0.8 (Ti - T∞) = 30 + 0.8 × (230 – 30) =190 ℃

(ii) when t = 2 hours = 7200 s we have:

(0.5 × 10−6 ) × 7200


Fo = (0.05)2
= 1.44

θ(0,t) T0 − T∞
From chart: = = 0.125
θi Ti − T∞

Therefore: T0 = T∞ + 0.125 (T0 – T∞) = 30 + 0.125 ×(230 – 30) = 55℃.


x
(b) (i) t = 1800 s. At the surface L= 1.0 and Bi = 4.35

(T|x=L − T∞ )
Hence from the chart: (T0 − T∞ )
= 0.275

Or: T|x=L = T∞ + 0.275 (T0 – T∞) = 30 + 0.275 ×(190 – 30)

= 74 ℃.

(ii) t = 7200 s. Hence: T|x=L = T∞ + 0.275 (T0 – T∞) = 30 + 0.275 × (55 – 30)

= 36.9 ℃.

(c) (i) Bi2 Fo = 4.35 2 × 0.36 = 6.81; From chart:


QRemoved
= 0.50 .
QMax

167
k 0.69 ×1 ×0.1
Qmax = ρCpV (Ti – T∞) = (α)V(Ti – T∞) = (0.5 × 10−6 ) × (230 – 30)

= 27.6 × 10 6 = 27.6 MJ

Therefore: Qremoved = 0.5 × 27.6 = 13.8 MJ

(ii) when t = 7200 s: Bi2 Fo = 4.35 2 × 1.44 = 27.25

QRemoved
From chart: = 0.9.
QMax

Therefore: Q = 0.9×27.6 = 24.8 MJ.

(d) It is given that T(L,t) = 30 0C.


θ(L,t) 40−30
Hence: = 230−30 = 0.05.
θi

Now for x / L = 1.0 and Bi = 4.35, from the chart the ratio of surface temperature difference
to the centre temperature difference can be read as:

θ(L,t)
= 0.225
θ(0,t)

θ(0,t) 0.05
Hence: = 0.225 = 0.2222
θi

θ(0,t)
From the chart corresponding to this value of and Bi = 4.35, the Fourier number can
θi
be read as:
αt
Fo = L2 = 1.0.

L2 (0.05)2
Therefore: t= = 0.5 × 10−6 = 5000 s = 1.39 hours
α

Example 4.7: A long solid cylinder [α = 0.05 m2 per hour, k = 50 W∙m−1∙K−1] of 5 cm


diameter is initially at 200 ℃. Suddenly it is immersed in water at a temperature of 20 ℃.
Assuming the heat transfer coefficient to be 200 W∙m−2∙K−1, determine (a) the centre and the
Surface temperatures after 10 minutes have elapsed, and (b) the energy removed from the
cylinder during this 10 minute period.

Schematic :

168
T∞,h

R T(t)|t=0 =Ti

Known: α = 0.05 m2 per hour = 0.05 / 3600 = 1.39 ×10 − 5 m2∙s−1 ; k = 50 W∙m−1∙K−1;

Ti = 200 ℃; T∞ = 20 ℃; R = 0.025 m ; h = 200 W∙m−2∙K−1;t = 10 × 60 = 600 s

Find: (a) T/r=0 and T/r = R after t = 10 min; (b) Qremoved during 10 minute period.

Assumptions: (i) one dimensional conduction; (ii) thrmal conductivity of the solid is constant

Solution: (a) (i) To find centre temperature T0 :

hR 200 ×0.025
Bi = = = 0.1.
k 50

Since Bi = 0.1, internal temperature gradients cannot be neglected.

αt (1.39 × 10−5 ) ×600


Fo = R2 = (0.025)2
= 13.34

From chart for transient conduction in an infinite cylinder we have:

θ(0,t) T0 − T∞
= = 0.08
θi Ti − T∞

Hence : T0 = T∞ + 0.08 (Ti - T∞) 20 + 0.08 ×(200 – 20)

Or: = 34.4℃.

𝑟
(ii) To find the surface temperature, T|r=R: 𝑅 = 1.0

(T|r=R − T∞ )
Therefore from chart : (T0 − T∞ )
= 0.13

Or: T|r=R = T∞ + 0.13 (T0 – T∞) = 20 + 0.13 × (34.4 – 20) = 21.9 ℃.

(b) Bi2 Fo = (0.12) ×13.34 = 0.133

169
QRemoved
From energy chart for the infinite cylinder, = 0.875
QMax

50
Qmax = ρCp V (Ti – T∞) = (1.35 × 10−6 )
× (π ×0.0252 × 1) × (200 – 30)

= 1.27 x 10 6 kJ / m = 1.27 MJ∙m−1.

Therefore: QRemoved = 0.875 ×1.27 = 1.11 MJ∙m−1.

Example 4.8:An orange of 10 cm diameter is initially at a uniform temperature of 30 ℃.


Suddenly it is placed in a refrigerator in which the air temperature is 2 ℃. If the surface
heat transfer coefficient is 50 W∙m-2∙K−1, determine the time required for the centre of the
orange to reach 10 ℃. Assume for the orange α = 1.4 × 10 ─ 7 m2∙s−1 and
k = 0.59 W∙m-1∙K−1.

Schematic:

h,T∞
T(t)|t=0 = Ti
R

Known: R = 0.05 m ; Ti = 30 ℃ ; T∞ = 2 ℃ ; T0 = 10 ℃ ;

h = 50 W∙m-2∙K−1;k = 0.59 W∙m-1∙K−1;


α =1.4 ×10−7 m2∙s−1.

Find: Time t required for the centre temperature, T/r=0 to reach 10 ℃.

Assumpttions: (i) Orange is assumed to be in the shape of a sphere.(ii) one dimensional


radial conduction; (iii) Orange properties are constant.

Solution:

θ(0,t) Tr=0 − T∞ 10−2


= = 30−2 = 0.286
θi Ti − T∞

hR 50 ×0.05
Bi = = = 4.24
k 0.59

From chart for transient conduction in sphere we have FO = 0.3

170
αt
Therefore: = 0.3.
R2
0.3 × 𝑅2 0.3 × (0.05)2
Or: t= = = 5357 s = 1.5 hours
𝛼 1.4 × 10−7

Solution using Tables: For the given problem we have

Bi = 4.24. Therefore from Table 4.1,by interpolation λ1× = 2.4831 and A1× = 1.7362.

Tr=0 − T∞
= A1× exp[− λ1× 2Fo]
Ti − T∞

10−2
Therefore: = 1.7362 × exp (− 2.48312 Fo)
30−2

Solving for Fo we get, Fo = 0.2925.


αt
Therefore: = 0.2925
R2

0.2925×R2 0.2925× 0.052


Or: t= = = 5223 s =1.45 hours
α 1.45×10− 7

4.3.5. Transient conduction in semi-infinite solids:A semi-infinite solid is an idealized


body that has a single plane surface and extends to infinity in all directions.The transient
conduction problems in semi-infinite solids have numerous practical applications in
engineering. Consider, for example, temperature transients in a slab of finite but large
thickness, initiated by a sudden change in the thermal condition at the boundary surface. In the
initial stages, the temperature transients near the boundary surface behave similar to those of
semi-infinite medium, because some time is required for the heat to penetrate the slab before
the other boundary condition begins to influence the transients.The earth for example, can be
considered as a semi-infinite solid in determining the variation of its temperature near its
surface
Consider a semi-infinite solid with constant thermo physical properties, without any
internal heat generation and uniform thermal conditions on its exposed surface.Let the solid
initially be at a uniform temperature Ti. Heat transfer in this case occurs only in the direction
normal to the surface (x-direction) and thus it is one dimensional. The depth of the solid is very
large (x → ∞) compared to the depth upto which heat can penetrate and these phenomena can
be expressed mathematically as a boundary condition as T(x→∞,t) = Ti. Heat conduction in a
semi-infinite solid is governed by the thermal condition imposed on the exposed surface , and
thus the solution depends strongly on the boundary condition at x = 0
We come across basically three types of boundary conditions while
analyzing the problem of one-dimensional transient conduction in semi-infinite solids.These
three problems are as follows:

Problem 1:The solid is initially at a uniform temperature Ti and suddenly at time t>0
the boundary-surface temperature of the solid is changed to and maintained at a uniform
temperature T0 which may be greater or less than the initial temperature Ti.

171
Problem 2:The solid is initially at a uniform temperature Ti and suddenly at time t>0 the
boundary surface of the solid is subjected to a uniform heat flux of q0 W∙m-2.

Problem 3:The solid is initially at a uniform temperature Ti. Suddenly at time t>0 the boundary
surface is exposed to an ambience at a uniform temperature T∞ with the surface heat transfer
coefficient h. T∞ may be higher or lower than Ti.

Solution to Problem 1: The schematic for problem 1 is shown in Figure 4.10. The
mathematical formulation of the problem to determine the unsteady temperature distribution
in an infinite solid T(x,t) is as follows:
The governing differential equation [Equation 4.7(a)] is

∂2 T 1 ∂T
= (α ) (4.7a)
∂ x2 ∂t

The initial condition is at time t = 0, T(x,0) = Ti (4.7b)

and the boundary condition is at x = 0, T(0,t) = T0. (4.24)

It is convenient to solve the above problem in terms of the variable θ(x,t), where θ(x,t) is
defined as
T(x,t)− T0
θ(x,t) = (4.25)
Ti − T 0
The governing differential equation in terms of θ(x,t) will be

∂2 θ 1 ∂θ
= (α) ( ∂t ) (4.26a)
∂x2

For t > 0, the surface at T0

Initially (t=0) solid at Ti

0 x

Figure 4.10. A semi-infinite solid with specified surface temperature T0 for t > 0

Ti − T0
The initial condition will be: at time t = 0, θ(x,0) = =1 (4.26b)
Ti − T0

172
T0 − T0
And the boundary condition will be at x = 0, θ(0,t) = =0 (4.26c)
Ti − T0

This problem has been solved analytically and the solution θ(x,t) is represented graphically as
x
θ(x,t) as a function of the dimensionless variable x× = as shown in Figure 4.12.
2 √(αt)
In engineering applications, the heat flux at the boundary surface x = 0 is also
of interest. The analytical expression for heat flux at the surface is given by,

k(T0 − Ti)
qs(t) = (4.27)
√(παt)

Solution to problem 2: The schematic for this problem is shown in Figure 4.11.

T(x,t) = Ti at t = 0

q0 W∙m-2

for t > 0 x

Figure 4.11: A semi- infinite solid subjected to a constant heat flux at x = 0 for t > 0

Governing differential equation in terms of T(x,t) and the initial condition are same as that for
problem 1[i.e. equations (4.26a) and (4.26b)].

∂T ∂θ
The boundary condition is : at x = 0, − k [ ∂x ] = q0 = − k [∂x]
x=0 x=0

The temperature distribution within the solid T(x,t) is given by,

2q0 1
T(x,t) = Ti + (√αt) [( π) exp(− ξ2 ) + ξ erf(ξ) − ξ] (4.28 a)
k √

x 2 𝜉
Where: ξ= and erf(ξ) = ∫ exp (−y 2 )dy (4.28b)
2 √(αt) √𝜋 0

Here erf (ξ) is called the “error function” of argument ξ and its values for different values of
ξ are tabulated as shown in Table 4.2.

Solution to Problem 3 : The solid is initially at a uniform temperature Ti and suddenly for t >0
the surface at x = 0 is brought in contact with a fluid at a uniform temperature T∞ with a surface
heat transfer coefficient h [see Figure 4.12]. For this problem the solution is represented in the

173
form of a plot where the dimensionless temperature [1 − θ(x,t)] is plotted against dimensionless
x h√(αt)
distance , using as the parameter. It can be noted that the case h → ∞ is equivalent
√(αt) k
to the boundary surface ay x = 0 maintained at a constant temperature T∞.
𝟐 𝝃 𝟐
Table 4.2: Error function erf (ξ) for different values of ξ [erf(ξ) = ∫𝟎 𝒆− 𝒚 𝒅𝒚 ]
√𝝅

z erf(z) z erf(z) z erf(z) z erf(z) z erf(z)


0 0 0.5 0.5204999 1 0.8427007 1.5 0.9661051 2 0.9953223
0.01 0.0112834 0.51 0.5292436 1.01 0.8468104 1.51 0.9672767 2.01 0.9955248
0.02 0.0225646 0.52 0.5378986 1.02 0.850838 1.52 0.9684135 2.02 0.9957195
0.03 0.0338412 0.53 0.5464641 1.03 0.8547842 1.53 0.9695162 2.03 0.9959063
0.04 0.0451111 0.54 0.5549392 1.04 0.8586499 1.54 0.9705857 2.04 0.9960858
0.05 0.056372 0.55 0.5633234 1.05 0.8624361 1.55 0.9716227 2.05 0.9962581
0.06 0.0676216 0.56 0.5716158 1.06 0.8661435 1.56 0.9726281 2.06 0.9964235
0.07 0.0788577 0.57 0.5798158 1.07 0.8697733 1.57 0.9736026 2.07 0.9965822
0.08 0.0900781 0.58 0.5879229 1.08 0.8733261 1.58 0.974547 2.08 0.9967344
0.09 0.1012806 0.59 0.5959365 1.09 0.8768031 1.59 0.975462 2.09 0.9968805
0.1 0.1124629 0.6 0.6038561 1.1 0.880205 1.6 0.9763484 2.1 0.9970205
0.11 0.1236229 0.61 0.6116812 1.11 0.883533 1.61 0.9772068 2.11 0.9971548
0.12 0.1347584 0.62 0.6194115 1.12 0.8867879 1.62 0.9780381 2.12 0.9972836
0.13 0.1458671 0.63 0.6270464 1.13 0.8899706 1.63 0.9788428 2.13 0.997407
0.14 0.156947 0.64 0.6345858 1.14 0.8930823 1.64 0.9796218 2.14 0.9975253
0.15 0.167996 0.65 0.6420293 1.15 0.8961238 1.65 0.9803756 2.15 0.9976386
0.16 0.1790118 0.66 0.6493767 1.16 0.8990962 1.66 0.9811049 2.16 0.9977472
0.17 0.1899925 0.67 0.6566277 1.17 0.9020004 1.67 0.9818104 2.17 0.9978511
0.18 0.2009358 0.68 0.6637822 1.18 0.9048374 1.68 0.9824928 2.18 0.9979506
0.19 0.2118399 0.69 0.6708401 1.19 0.9076083 1.69 0.9831526 2.19 0.9980459
0.2 0.2227026 0.7 0.6778012 1.2 0.910314 1.7 0.9837905 2.2 0.9981372
0.21 0.2335219 0.71 0.6846653 1.21 0.9129555 1.71 0.984407 2.21 0.9982244
0.22 0.2442959 0.72 0.6914328 1.22 0.9155339 1.72 0.9850028 2.22 0.9983079
0.23 0.2550226 0.73 0.6981037 1.23 0.9180501 1.73 0.9855785 2.23 0.9983878
0.24 0.2657001 0.74 0.7046778 1.24 0.9205052 1.74 0.9861346 2.24 0.9984642
0.25 0.2763264 0.75 0.7111554 1.25 0.9229001 1.75 0.9866717 2.25 0.9985373
0.26 0.2868997 0.76 0.7175365 1.26 0.9252359 1.76 0.9871903 2.26 0.9986071
0.27 0.2974182 0.77 0.7238214 1.27 0.9275136 1.77 0.9876909 2.27 0.9986739
0.28 0.3078801 0.78 0.7300102 1.28 0.9297342 1.78 0.9881742 2.28 0.9987377
0.29 0.3182835 0.79 0.7361032 1.29 0.9318986 1.79 0.9886405 2.29 0.9987986
0.3 0.3286268 0.8 0.7421008 1.3 0.9340079 1.8 0.9890905 2.3 0.9988568
0.31 0.3389082 0.81 0.7480031 1.31 0.9360631 1.81 0.9895245 2.31 0.9989124
0.32 0.349126 0.82 0.7538106 1.32 0.9380651 1.82 0.9899432 2.32 0.9989655
0.33 0.3592787 0.83 0.7595236 1.33 0.940015 1.83 0.9903468 2.33 0.9990162
0.34 0.3693645 0.84 0.7651426 1.34 0.9419137 1.84 0.9907359 2.34 0.9990646
0.35 0.3793821 0.85 0.7706679 1.35 0.9437622 1.85 0.991111 2.35 0.9991107
0.36 0.3893297 0.86 0.7761001 1.36 0.9455614 1.86 0.9914725 2.36 0.9991548
0.37 0.399206 0.87 0.7814397 1.37 0.9473124 1.87 0.9918207 2.37 0.9991968
0.38 0.4090095 0.88 0.7866872 1.38 0.949016 1.88 0.9921562 2.38 0.9992369
0.39 0.4187387 0.89 0.7918431 1.39 0.9506733 1.89 0.9924793 2.39 0.9992751
0.4 0.4283924 0.9 0.7969081 1.4 0.9522851 1.9 0.9927904 2.4 0.9993115
0.41 0.4379691 0.91 0.8018827 1.41 0.9538524 1.91 0.9930899 2.41 0.9993462
0.42 0.4474676 0.92 0.8067676 1.42 0.9553762 1.92 0.9933782 2.42 0.9993793
0.43 0.4568867 0.93 0.8115635 1.43 0.9568572 1.93 0.9936556 2.43 0.9994108
0.44 0.4662251 0.94 0.8162709 1.44 0.9582966 1.94 0.9939226 2.44 0.9994408
0.45 0.4754817 0.95 0.8208907 1.45 0.959695 1.95 0.9941793 2.45 0.9994694
0.46 0.4846554 0.96 0.8254236 1.46 0.9610535 1.96 0.9944263 2.46 0.9994966
0.47 0.4937451 0.97 0.8298702 1.47 0.9623729 1.97 0.9946637 2.47 0.9995226
0.48 0.5027497 0.98 0.8342314 1.48 0.9636541 1.98 0.994892 2.48 0.9995472
0.49 0.5116683 0.99 0.838508 1.49 0.9648979 1.99 0.9951114 2.49 0.9995707

174
T(x,t) = Ti at t = 0

h,T∞
for t > 0 x

Figure 4.12: A semi- infinite solid subjected to convective heat transfer at x = 0 for
t>0

Example 4.9:A thick stainless steel slab [α = 1.6 ×10 ─ 5 m2∙s−1 and k = 61 W∙m−1∙K−1] is
initially at a uniform temperature of 150℃. Its surface temperature is suddenly lowered to 20
℃. By treating this as a one-dimensional transient conduction problem in a semi-infinite
medium, determine the temperature at a depth 2 cm from the surface and the heat flux 1 minute
after the surface temperature is lowered

Schematic:

For t > 0, the surface at T0

Initially (t=0) solid at Ti

0 x

Known: Ti = 150 ℃; T0 = T|x=0 = 20 ℃; α = 1.6 × 10 − 5 m2∙s−1; k = 61 W∙m−1∙K−1; x = 0.02


m ;t = 1 minute.

Find: (i) T/x= 0.02 m ; (ii) qs

Assumptions: (i) one dimensional conduction; (ii) thermal properties of the solid is constant
x 0.02
Solution (i) : ξ= = = 0.323
2√(αt) 2 × √(1.6 × 10−5 ) ×60

175
T(x,t)− T0
From chart: = 0.35
Ti − T0

Therefore: T(x,t) = T0 + 0.35 (Ti – T0) = 20 + 0.35 × (150 – 20)

= 65.5℃.

k(T0 − Ti ) 61 × (20−150)
(ii) qs(t) = = = − 435.5 W / m2
√(παt) √(𝜋 ×1.6 × 10−5 ×60)

Example 4.10: A semi-infinite slab of copper (α = 1.1×10 ─ 4 m2∙s−1 and k = 380 W∙m−1∙K−1
is initially at a uniform temperature of 10 0 C. Suddenly the surface at x = 0 is raised to 100
℃.Calculate the heat flux at the surface 5 minutes after rising of the surface temperature.
How long will it take for the temperature at a depth of 5 cm from the surface to reach 90℃?

Schematic:
For t > 0, the surface at T0

Initially (t=0) solid at Ti

0 x

Known: Ti = 10 ℃ ; T0 = 100 ℃ ; k = 380 W∙m−1∙K−1; α = 1.1 × 10 ─ 4 m2∙s−1; t = 300 s ;

Find: Time t required for T/x = 5 cm = 90 ℃

Assumptions: (i) one dimensional conduction; (ii) thermal conductivity of the solid is
constant.

k(T0 − Ti ) 380 ×(100−10)


Solution: qs(t) = = = 11012 W∙m-2.
√(παt) √(𝜋 ×1.1 × 10−4 ×300)

T(x,t)− T0 90−100
θ(x,t) = = 10−100 = 0.11
Ti − T0

From chart: ξ = 0.1


x
ξ= .
2√(αt)

176
x2 0.052
Or: t = 4 α ξ2 = 4 × 1.1 × 10−4 × (0.1)2 = 586 s

Example 4.11:A thick bronze [α = 0.86 × 10 ─ 5 m2∙s−1and k = 26 W∙m−1∙K−1] is initially at


250 ℃. Suddenly the surface is exposed to a coolant at 25 0 C. If the surface heat transfer
coefficient is 150 W∙m−2∙K−1, determine the temperature 5 cm from the surface 10 minutes
after the exposure.

Schematic:

T(x,t) = Ti at t = 0

h,T∞
for t > 0 x

A semi- infinite solid subjected to convective heat transfer at x = 0 for t > 0

Known: Ti = 250 ℃; T∞ = 25 ℃; h = 150 W∙m−2∙K−1; k = 26 W∙m−1∙K−1 t = 600 s;

α=0.86 × 10 ─ 5 m2∙s− ; x = 0.05 m ;

Find: T/x = 5 cm when t = 10 minutes=600 s.

Assumptions: (i) one dimensional conduction; (ii) thermal conductivity of the solid is
constant.

x 0.05
Solution: ξ= = = 0.35
2√(αt) 2 × √(0.86 × 10−5 )×600

h (√αt) 150 × √((0.86 × 10−5 )×600)


= = 0.414
k 26

T(x,t)− T∞
Therefore from chart: 1- = 0.15
Ti − T∞

Solving for T(x,t) we have T(x,t) = T∞ + (1 – 0.15)(Ti – T∞)

= 25 + 0.85 × (250 – 25 ) = 216.25 ℃.

Example 4.12:- A thick wood [α = 0.82 ×10 ─ 7 m2∙s−1 and k = 0.15 W∙m−1∙K−1] is initially at

177
20 ℃.The wood may ignite at 400 0 C. Suddenly the surface of the wood is exposed to gases
at 500 ℃. If the surface heat transfer coefficient is 45 W∙m−2∙K−1, how long will it take forthe
surface of the wood to reach 400 ℃?

Schematic:

T(x,t) = Ti at t = 0

h,T∞
for t > 0 x

A semi- infinite solid subjected to convective heat transfer at x = 0 for t > 0

Known: Ti = 20 ℃ ;T∞ = 500 ℃ ;h = 45 W∙m−2∙K−1;k = 0.15 W∙m−1∙K−1; α = 0.82 × 10 − 7


m2∙s−1.

Find: Time t for x = 0 to reach 400 ℃.

Assumptions: (i) one dimensional conduction; (ii) thermal conductivity of the solid is
constant.

h (√αt) 45 × √((0.82 × 10−7 )×𝑡)


Solution: = = 0.086 √𝑡
k 0.15

x T(0,t)− T∞ 400−500
ξ= =1- = 1 – ( 20−500 ) = 0.9799
2√(αt) Ti − T∞

h (√αt)
Hence from chart: = 2.75 = 0.086 √𝑡
k

Solving for we get: t = 1023 s = 17 minutes.

4.3.6. Use of Heissler’s charts for Solving Multi-dimensional Transient Conduction


Problems (Product Solution):

The solution to the problem of multi-dimensional transient conduction in solids without heat
generation subjected to convective boundary conditions can be obtained using the Heissler’s
charts developed for one dimensional transient conduction. The approach adopted is called
the “Product Solution”.

178
Product Solution for Transient Conduction in a Rectangular Bar:
Consider a rectangular bar of sides 2L1 and 2L2 confined to the region – L1 ≤ x ≤ L1 and – L2
≤ y ≤ L2 as shown in Figure4.13.Initaially the bar is at a uniform temperature Ti. Suddenly at

y
h1, T∞
L2

h2, T∞
−L1 o L1
x
Solid initially at Ti
h2,T∞
−L2

−L2

h1,T∞

Figure 4.13: Product solution for transient conduction in a rectangular bar

t = 0 all boundary surfaces are subjected to convection to an ambient at a constant


temperature T∞.The mathematical formulation of this conduction problem is given by

∂2 T ∂2 T 1 𝜕𝑇
+ = in – L1 ≤ x ≤ L1 and – L2 ≤ y ≤ L2 (4.29a)
∂x2 ∂y2 𝛼 𝜕𝑡
∂T
The boundary conditions are: at x = 0, = 0 (axis of symmetry) (4.29b)
∂x
∂T
at x= L1, − k = h1 [T −T∞] (4.29c)
∂x
∂T
at y = 0, = 0 (axis of symmetry) (4.29d)
∂y
∂T
at y= L2, − k = h2 [T −T∞] (4.29e)
∂y
The initial condition is : at t = 0, T = Ti (4.29f)

A dimensionless temperature θ(x,y,t) is defined as follows:

T(x,y,t)− T∞
θ(x,y,t) = (4.30)
Ti − T∞

179
Equations (4.29a) to (4.29f) in terms of θ can be written as follows:

∂2 θ ∂2 θ 1 𝜕𝜃
+ = in – L1 ≤ x ≤ L1 and – L2 ≤ y ≤ L2 (4.31a)
∂x2 ∂y2 𝛼 𝜕𝑡
∂θ
The boundary conditions are: at x = 0, = 0 (axis of symmetry) (4.31b)
∂x
∂θ
at x= L1, − k = h1 θ (4.31c)
∂x
∂θ
at y = 0, = 0 (axis of symmetry) (4.31d)
∂y
∂θ
at y= L2, − k = h2 θ (4.31e)
∂y
The initial condition is : at t = 0, θ = 1 (4.31f)

It can be shown that the solution for the problem given by Equation 4.31a to 4.31f can be
expressed as a product of solutions of two one − dimensional problems θ1(x,t) and θ2(y,t) as
illustrated below:

Let θ(x,y,t) = θ1 (x,t) × θ2(y,t) (4.32)

Substituting this in Equation 4.31a to 4.31f and rearranging we have θ1 (x,t)to be the solution
of the following one-dimensional problem:

∂2 θ1 1 ∂θ1
= in – L1 ≤ x ≤ L1 (4.33a)
∂x2 α ∂t
∂θ1
The boundary conditions are: at x = 0, = 0 (axis of symmetry) (4.33b)
∂x
∂θ1
at x= L1, − k = h1 θ1 (4.33c)
∂x
The initial condition is : at t = 0, θ1 = 1 (4.33d)

and θ2 (y,t) to be the solution of the following one-dimensional problem:

∂2 θ2 1 𝜕𝜃2
= in – L2 ≤ y ≤ L2 (4.34a)
∂y2 𝛼 𝜕𝑡
∂θ2
The boundary conditions are: at y = 0, = 0 (axis of symmetry) (4.34b)
∂x
∂θ2
at x= L2, − k = h1 θ1 (4.34c)
∂x
The initial condition is : at t = 0, θ2 = 1 (4.34d)

The above one − dimensional problems for θ1(x,t) and θ2(y,t) are exactly the same as that whose
solution is given by the transient temperature charts for infinite slabs. Thus the solution for

180
two− dimensional transient conduction problem for a rectangular region – L1 ≤ x ≤ L1 and –
L2 ≤ y ≤ L2 can be constructed as the product of two one-dimensional transient conduction
problems for infinite slabs.This concept of product solution is also illustrated in Figure4.14(a).
y

θ(x,y,t)

2L2 x

2L1

Figure4.14(a): Product Solution for Transient conduction in a Rectangular Bar


The basic idea developed here can be extended to other configurations. For example the
product solution ,θ(r,x,t)for two dimensional transient conduction in a finite cylinder of radius
R and height 2L will be the product of one dimensional conduction solution for an infinite slab
of thickness 2L, θ1 (x,t) and one dimensional solution for an infinite cylinder of radius R,
θ2(r,t).This is illustrated in Figure4.14(b).

Infinite cylinder of radius R

Infinite slab of thickness 2L

r
2L

Figure4.14(b): Product Solution for Transient conduction in a Finite Cylinder

181
Example 4.13: A rectangular iron bar (α = 1.6 × 10 −5 m2∙s−1, k= 60 W∙m−1∙K−1), 5 cm × 4
cm is initially at a uniform temperature of 225 ℃. Suddenly the bar is immersed in a fluid
which is maintained at a uniform temperature of 25℃.. The surface heat transfer coefficient
for all the outside surfaces of the bar is 500 W∙m−2∙K−1. Determine (i) the temperature at the
centre of the bar 2 minutes after the immersion of the bar into the fluid, (ii) the temperature
at any one corner of the bar 2 minutes after immersion.

Schematic:

h2,T∞

θ(x,y,t)
h1,T∞

2L2 x

2L1

Known: 2L1 = 5 cm; 2L2 = 4 cm; α = 1.6 × 10− m2∙s−1; h1= h2= 500 W∙m−2∙K−1; T∞ =25℃;
Ti = 225℃; t=2×60 = 120 s; k = 60 W∙m−1∙K−1.

Find: (i)T(0,0,2min);(ii) T(L1,L2,2 min)

Assumptions:(i) Surface heat transfer coefficient is constant and is same for all surfaces,i.e.
h1= h2; (ii)Thermal properties of the solid are constant

Solution:
(i) θ(0,0,t) = θ1 (0,t) ×θ2(0,t)

T(x,y,t)− T∞ T1 (x,t)− T∞ T2 (y,t)− T∞


i.e. = ×
Ti − T∞ Ti − T∞ Ti − T∞

h1 L1 500×0.025 αt 1.6× 10−5 ×120


To find θ1 (0,t): Bi = = = 0.21 ; Fo = = = 3.072
k 60 L21 0.0252

From Heisller chart we have θ1 (0,t) = 0.58

182
h2 L2 500×0.02 αt 1.6×10−5 ×120
To find θ2 (0,t): Bi = = = 0.167 ; Fo = = = 4.8
k 60 L22 0.022

From Heisller chart we have θ2 (0,t) = 0.50

T(0,0,2 min)− T∞
Hence θ(0,0,2 min) = = 0.58 × 0.50 = 0.29
Ti − T∞

Therefore T(0,0,2min) = 25 + 0.29× (225 – 25) = 83 ℃

T(L1 ,L2 ,2 min)− T∞


(ii) = θ1 (L1,t) × θ2(L2,t)
Ti − T∞

θ1 (L1 ,t)
θ1 (L1,t) = θ1 (0,t)
× θ1 (0,t)

x
For the corner: = 1 and Bi = 0,21 as calculated before.
L1
Hence from Heisller’s chart:

θ1 (L1 ,t)
= 0.9. Hence θ1 (L1,t) = 0.9 × 0.58 = 0.522
θ1 (0,t)

Similarly: θ2 (L1,t) = 0.92 × 0.5 = 0.46

T(L1 ,L2 ,2 min)− T∞


Therefore: = 0.522 × 0.46 = 0.24
Ti − T∞

And: T(L1,L2, 2 min) = 0.24 × (225 – 25) + 25 = 73 ℃

Example 4.14: A short cylinder made of Iron (k = 60 W∙m−1∙K−1; α = 1.6 × 10−5m2∙s−1) is 5


cm in diameterand 4 cm in height. It is initially at a uniform temperature of 225℃and is
suddenly exposed to an environment at 25℃ with a surface heat transfer coefficient of 300
W∙m−2∙K−1. Determine (i) the centre temperature and (ii) the temperature at a location 1 cm
from the outer surface of the cylinder and at a height of 2 cm from one of its ends 2 minutes
after exposure to the environment.

183
Schematic:

Infinite cylinder of radius R

Infinite slab of thickness 2L

r
2L

Known: Short cylinder with R = 2.5 cm and 2L = 4 cm; α = 1.6 × 10−5 5m2∙s−1; k = 60
W∙m−1∙K−1; h =300 W∙m−2∙K−1;t = 2 × 60 = 120 s;Ti = 225℃ ;T∞ = 25℃

Find: (i) T(0,0,2 min) ; (ii) T(1.5cm,2cm,2min)

Assumption: (i) Two-demensional conduction; (ii) Thermal properties of the cylinder are
constant; (iii) the surface heat transfer coefficient is constant and is same for all boundary
surfaces; (iv) The solid is not generating any heat

Solution: (i) θ(r,x,t) = θ1 (r,t) × θ2(x,t)

θ(0,0,120s) = θ1 (0,120 s) × θ2(0,120 s)

hR 500×0.025
To find θ1 (0,2 min) : Bi = = = 0.208
k 60

αt 1.6× 10−5 ×120


Fo = = = 3.1
R2 0.0252

From Heisller’s chart for infinite cylinder we have θ1 (0,120 s) = 0.31.

184
hL 500× 0.02
Similarly for infinite slab, Bi = = = 0.167
k 60

αt 1.6× 10−5 ×120


Fo = = = 4.8
L2 0.022

Hence from chart, θ2(0,120 s) = 0.45

T(0,0,120 s)− T∞
And: θ(0,0,120 s) = = 0.31 × 0.45 = 0.14
Ti − T ∞

Therefore T(0,0,120 s) = 0.14 × (225 – 25) + 25 = 53 ℃

(ii) θ(1.5 cm,2 cm,120 s) = θ1 (1.5 cm,120 s) × θ2(2 cm,120 s)

r 1.5
To find θ1 (1.5 cm,120 s): for r = 1.5 cm, R= 2.5 = 0.6.

θ1 (1.5 cm,120 s)
Hence from chart: = 0.95 .
θ1 (0,120 s)

Or: θ1(1.5 cm, 120 s) = 0.95 × 0.31 = 0.2945.

x 2 hL 300 ×0.02
To find θ2 (2 cm,120 s): For infinite slab, we have = = 0.5 ; Bi = = = 0.1.
L 4 k 60

𝜃2 (2 𝑐𝑚,120 𝑠)
Therefore from chart we have: = 0.94
𝜃2 (0,120 𝑠)

Or: θ2(2 cm, 120 s) = 0.94 × 0.45 = 0.423

Therefore : θ (1.5 cm,2 cm, 120 s) = θ1 (1.5 cm, 120 s) × θ2(2 cm, 120 s)

= 0.2945 × 0.423 = 0.1246

𝑇(1.5𝑐𝑚,2𝑐𝑚,120 s) − 𝑇∞
Therefore: = 0.1246
𝑇𝑖 − 𝑇∞

Or: T(1.5cm,2cm, 120 s) = 0.124 × (225 – 25) + 25 = 48.8 ℃.

Example 4.15:An aluminium ingot (k = 200 W∙m−1∙K−1;ρ= 2700 kg∙m-3; Cp= 890 J∙kg−1∙K−1)
of dimensions 6 cm × 5 cm × 3 cm is initially at a uniform temperature of 175℃. Suddenly
the ingot is immersed in a fluid which is maintained at a uniform temperature of 25 with the
surface heat transfer coefficient of 250 W∙m−2∙K−1. Determine (i) The temperature at the centre

185
of the ingot 2 minutes after immersion and (ii)temperature at one of the corners after the same
time interval.

Schematic:

2L2
2L3

2L1

Known: 2L1 = 6 cm ; 2L2 = 5 cm ; 2L3 = 3 cm ; k = 200 W∙m−1∙K−1;ρ= 2700 kg∙m-3;

Cp = 890 J∙kg−1∙K−1;Ti = 175℃; T∞ = 25℃; h = 250 W∙m−2∙K−1; t = 2 × 60 = 120 s.

Find: (i) T(0,0,0,2 min) ; (ii) T (L1,L2,L3, 120 s)

Assumptions: (i)Properties of the ingot are constant; (ii) the surface heat transfer
coefficient is constant and is same for all boundary surfaces; (iii) The solid is not generating
any heat

Solution:

T(0,0,0,120 𝑠)− T∞
(i) θ(0,0,0,2 min) = = θ1 (0, 120 s) × θ2(0, 120 s) × θ2(0, 120 s)
Ti − T∞

k 200
To find θ1 (0, 120 s): α= = = 8.323 × 10 −5 m2∙s-1
ρ Cp 2700×890

h L1 250×0.03 αt 8.323× 10−5 ×120


Bi = = = 0.0375 ; Fo = = = 11.1
k 200 L21 0.032

From chart for infinite slab, θ1 (0, 120 s) = 0.6

Similarly it can be found that θ2 (0, 120 s) = 0.4

186
And: θ3 (0, 120 s) = 0.35

T(0,0,0,120 s)− T∞
Therefore: = 0.6 × 0.4 × 0.35 = 0.084
Ti − T∞

And: T(0,0,0, 120 s) = 0.084 × (175 – 25) + 25 = 37.6 ℃

T(L1 ,L2 ,L3 ,120 s)− T∞


Hence: = θ1 (L1, 120 s) × θ2(L2, 120 s) × θ2(L3, 120 s)
Ti − T∞

x
(ii)To find θ1 (L1,2 min): = 1 and Bi = 0.0375.
L1

θ1 (L1 ,120 s)
From chat: = 0.975
θ1 (0,120 s)

Therefore: θ1 (L1, 120 s) = 0.975 × 0.6 = 0.585

Similarly: θ2 (L2, 120 s) = 0.975 × 0.4 = 0.39

And: θ3 (L3, 120 s) = 0.985 × 0.35 = 0.34475

T(L1 ,L2 ,L3 ,120 s)− T∞


Therefore: = 0.585 × 0.39 × 0.34475 = 0.07865
Ti − T∞

Or: T(L1 , L2 , L3 , 120 s) = 0.07865 × (175 – 25) + 25 = 36.8 ℃

Exercise Problems:

4.1.A hollow sphere of thermal conductivity, density ρ and specific heat Cp has inner radius r1
and outer radius r2.It is initially at a uniform temperature Ti. Suddenly the sphere is immersed
in an oil bath which is maintained at a uniform temperature T∞. The inner andthe outer surface
heat transfer coefficients are h. Neglecting internal temperature gradient obtain an expression
for the temperature of the sphere as a function of time.
If r1= 5 cm, r2 = 10 cm, k = 50W∙m−1∙K−1,ρ = 8000 kg∙m−3,Cp = 420 J∙kg−1∙K−1, Ti =500 ℃
and T∞ = 30℃, determine the temperature of the sphere 2 minutes after immersion in the oil
bath.

4.2.An infinite plate of thickness L is initially at a uniform temperature Ti.Suddenly heat is


supplied to the plate from one of its boundary surfaces (x = 0)at a constant rate of q0 W∙m−2,
while the other boundary surface dissipates heat to the surroundings which is at a uniform
temperature T∞, with a heat transfer coefficient h.Assuming the surface area of the plate as A,
density ρ, specific heat Cp and thermal conductivity k obtain an expression for the temperature
of the slab as a function oif time. Neglect internal temperature gradients for the plate.

187
4.3. Consider an infinite plate of thickness L. Initilly the plate is at a uniform temperature of
T0.Suddenly one of its surfaces is exposed to a fluid at a uniform temperature T1 with a surface
heat transfer coefficient h.The other boundary surface exchanges heat by radiation with an
ambient temperature at T2 which can be regarded as a black body.Applying lumped system
analysis derive an expression for the temperature T(t)of theplate as a function of time.

4.4.Consider a solid of surface area A and volume V. The solid and the environment are
initially at a uniform emperatur of Ti.Suddenly the environment temperature is stepped up to
T∞ and maintained at that value. Assuming the surface heat transfer coefficient to be h, the
density of the solid to be ρ and the heat cpscity to be Cp,, derive an expresiion for the
temperature of the solid as a function of time. Neglect temperature gradients within the solid.

4.5.Solve problem 4.4 assuming that the environment temperature oscillates sinusoidally with
time such that 𝑇̅ is the average ambient temperature, ΔT is the amplitude of temperature
oscillation, ω is the frequency of oscillation of temperature in rad/s.

4.6.Solve problem 4.4 assuming that the environment temperature vaies with time linearly with
a slope of β.

4.7.An infinite plate of thickness L is suddenly immersed in a cold fluid. If the boundary
conditions and the initial temperature distribution are given by:
T(0,t) = 0; T(L,t) =0; T(x,0) = 400 sin (πx/L)
Determine the unsteady temperature distribution of the plate.

4.8. An infinite slab of thickness L has its one surface at x=0 maintained at a uniuform
temperature T0 and the surface at x = L, it is subjected to a uniform heat flux of qL per unit
area of the surface. If the initial temperature of the slab is T0, determine the one dimensional
unsteady state temperature distribution in the solid.

4.9.A long solid cylinder [Thermal diffusivity = 0.05 m2 per hour;k = 50 W∙m−1∙K−1]of 5 cm
diameter is initially at a uniform temperature of 200℃.Suddenly the cylinder is immersed in
water which is at a uniform temperature of 20℃.The surface heat transfer coefficient is 200
W∙m−2∙K−1.Determine (i) the centre temperature and (ii) the surface temperature of the
cylinder 10 minutes after immersion.

4.10.A solid sphere of 5 cm diameter is initially at a uniform temperature of 900℃.It is


quenched in a liquid which is maintained at a uniform temperature of 100℃.If the Biot number
is 0.1 and the thermal diffusivity of the sphere is 2.8× 10−6 m2∙s−1,find the time taken for the
surface of the sphere to reach a temperature of 500℃.

4.11.A very long steel bar [k = 40 W∙m−1∙K−1;α = 11.0 ×10−6 m2∙s−1]of rectangular cross
section of 6 cm × 3 cm is initially at a uniform temperature of 30℃. Suddenly it is placed in a
furnace where the temperature is 1000 ℃ and the surface heat transfer coefficient is 100
W∙m−2∙K−1.Determine the temperature at the centre of the bar after 15 minutes have elapsed.

188
4.12.Stainless steel cylinders at a uniform temperature of 300℃ are quenched in an oilbath
which is maintained at a uniform temperature of 30℃.Each cylinder is 6 cm in diameter and
6 cm in length.Find the temperature at the centre of the circular faces of the cylinders after
180 s have elapsed.Also find the temperature on the curved surface of the cyclinder at midplane
(z =0) after the same time has elapsed. Assume the surface heat transfer coefficient to be 400
W∙m−2∙K−1.

4.13.Atemperature sensor is installed in a chemical reactor which operates in a cyclic


fashion.The temperature of the fluid in the reactor varies in an approximately sinusoidal
manner with a mean temperature of 𝑇̅∞ = 320 ℃ , an amplitude ΔT∞ = 50 ℃ , and a frequency
f = 0.5 Hz.The sensor can be modeled as a sphere of 1.0 mm diameter,themal conductivity of
50 W∙m−1∙K−1,a density of 16000 kg∙m−3and a specific heat capacity of 150 J∙kg−1∙K−1.In order
to provide corrosion resistance, the sensor has been coated with a thin layer (100 μm thick)of
plastic (k = 0.2 W∙m−1∙K−1)and has negligible heat capacity relative to the sensor.The surface
heat transfer coefficient is 500 W∙m−2∙K−1.The initial temperature of the sensor is 260 ℃
.(i)Calculate the Biot number and ascertain whether internal temperature gradients of the
sensor is negligible.(ii)Find the time constant associated with the sensor, and (iii)derive the
equation to determine the temperature response of the sensor.

189
CHAPTER 5
NUMERICAL METHODS IN CONDUCTION
5.1. Introduction: Analytical solutions that allow for the determination of the exact
temperature distribution in a solid are only available for limited ideal cases.Graphical solutions
have been used to gain an insight into complex conduction heat transfer problems, where
analytical solutions are not available, but they have limited accuracy and are primarily used
for two-dimensional problems. For solids having complicated geometries, boundary
conditions, and temperature − dependent thermal properties, numerical method of solution has
to be used.Advances in numerical computing now allow for complex heat transfer problems to
be solved rapidly on computers using numerical techniques..The current numerical techniques
include: finite-difference analysis; finite element analysis (FEA); and finite-volume analysis.
In general, these techniques are routinely used to solve problems in heat transfer, fluid
dynamics, stress analysis, electrostatics and magnetics, etc.In this chapter the use of finite-
difference analysis to solve conduction heat transfer problems is illustrated. In the finite
difference method the governing partial differential equation of heat conduction is
approximated by a set of algebraic equations for temperature at a number of nodal points over
the region. Hence the first step in the analysis is the finite difference representation of the
governing partial differential equation into a set of algebraic equations, either by using Taylor’s
series expansion and neglecting higher order terms or by writing the energy balance equation
for each node in the region. Then the set of algebraic equations thus obtained are solved for
unknown temperatures.

5.2. Finite Difference Formulation for One-dimensional Steady state Conduction in a


Plane wall:

5.2.1. Finite difference formulation from differential equation using Taylor’s Series
Expansion:

The governing differential equation for one dimensional steady-state conduction in a plane
wall of thickness L with heat generation is given by

d2 T q′′′
i
+ =0 in 0 < x ≤ L (5.1)
dx2 k

The region 0 < x ≤ L is divided into M equal sub-regions each of size

L
Δx = (5.2)
M

190
There are M + 1 nodes for i = 0 to i = M as illustrated in Figure5.1. Node i corresponds to a
location whose coordinate is iΔx. Let Ti be the temperature at node i;Then the region
contains M + 1 nodal temperatures for i = 0, 1,2,3,………………..,M nodes.

Δx Δx
L

0 1 2 i−1 i i+1 M−1 M


x

iΔx

i−½ i+½

Figure 5.1 Nomenclature for finite difference representation of derivatives

d2 T(x)
The second derivative of temperature at a node i can be represented in finite
dx2
dT(x)
differences as shown below.The first derivative of temperature at locations i + ½ and
dx
i − ½ can be approximated using Taylor’s series expansion as

dT(x) Ti+1 − Ti
│i + ½) ≈ (5.3a)
dx Δx

dT(x) Ti − Ti−1
│(i − ½) ≈ (5.3b)
dx Δx

d2 T(x)
Now the second derivative of temperature at node i can be approximated as
dx2

dT(x) dT(x)
d2 T(x) │(i + ½) − │(i − ½)
│i ≈ dx dx
dx2 𝛥𝑥

Substituting the expressions from Equations (5.3a) and (5.3b) and simplifying we get

Ti−1 −2 Ti + Ti+1
= (5.4)
𝛥𝑥 2

When Equation 5.4 is introduced into Equation (5.1) we get

191
Ti−1 −2 Ti + Ti+1 (q′′′
g ) i
+ =0
𝛥𝑥 2 k

Δx2
Or Ti-1 − 2Ti + Ti+1 = −(q′′′
g ) (5.5)
i k

Where (q′′′
g ) represents the heat generation per unit volume at node i.Equation (5.5) is
i
known as the finite difference form of the heat conduction Equation (5.1) and is valid for all
interior nodes, i.e. i= 1, 2, 3,…………………….M – 1, of the region.In this equation the heat
generation q′′′
i , thermal conductivity k and mesh size Δx are known quantities. Then
Equation (5.5) gives M – 1 algebraic equations for the M + 1 unknown node temperatures Ti,
i= 0,1,2,3……M+1. Two additional relations are needed to make the number of equations
equal to the number of unknown node temperatures Ti. These two relations are obtained from
the finite difference representation of the two boundary conditions at nodes i=0 and i=M.

5.2.2. Finite difference formulation using Energy Balance Approach:

Equation (5.5) can also be obtained by writing the energy balance equation for the control
volume surrounding the node i (shown by hatched portion in Figure5.1) as follows. For
steady state conduction the energy balance equation can be written as

Rate of conduction into the control volume from the left + Rate of heat generation within the
control volume – Rate of heat conduction out of the control volume to the right = 0.

i.e. Q(i – 1), i + Qg – Qi,(i + 1) = 0 (5.6)

Assuming linear variation of temperature between the adjacent nodes and using Fourier’s of
conduction, Equation (5.6) can be written as

Ti+1 − Ti Ti − Ti+1
(kA)│(i-1),i + Ai Δx(q′′′
g ) − (kA)│i,(i + 1) =0 (5.7)
Δx i Δx

Ti+1 − Ti Ti − Ti+1
Or + + ΔV (q′′′
g ) =0 (5.8)
R(i−1),i R(i+1),i i

𝛥𝑥
Where R (i−1),i = ( ) = Thermal Resistance between nodes i-1 and i (5.9a)
𝑘𝐴 (i−1),i

𝛥𝑥
R (i+1),i = ( ) = Thermal Resistance between nodes i+1 and i (5.9b)
𝑘𝐴 (i+1),i

ΔV = AiΔx = Volume of the element surrounding the node i (5.9c)

192
Equation (5.8) is the finite difference formulation of the one-dimensional steady state
conduction problem with heat generation. This equation is more general than Equation (5.5),
as it allows for the variation of thermal conductivity and cross sectional area with position
along the x-axis.If k and A are constants, Equation (5.7) reduces to Equation (5.5)

5.2.3.Finite Difference formulation for Boundary Nodes:

(i)Boundaries with prescribed temperatures: Let the temperatures at the boundaries be


specified as
(i) at x = 0, T(x) = T0 (5.10a)

and (ii) at x = L, T(x) = TM. (5.10b)

where T0 and TM are known temperatures. Then the temperatures at the nodes i = 0 and
i = M in Figure5.1 are taken as
T│i=0 = T0 (5.11a)

And: T│i = M = TM (5.11b)

Equations (5.11a) and (5.11b) provide the two additional relations needed to make the
number of equations equal the number of unknown node temperatures.

(ii)Boundaries with prescribed heat flux: Let the boundary surface at x = 0 is subjected to
a prescribed heat flux of q0 and the boundary surface at x = L is subjected to a prescribed
heat flux of qM as shown in Figure 5.2.

q0
0 1 2 M–2 M−1 M x
qM

𝚫𝐱
Δx Δx
𝟐
𝚫𝐱
𝟐

Figure 5.2 Nomenclature for finite difference formulation for prescribed surface heat
flux hboundary conditions

The energy balance equation for a differential volume element surrounding the grid point ‘o’
can be written as follows:
q0 A0 + Qg – Q 0,1 = 0 (5.12)

193
(T0 − T1 )
q0 A0 + A0(Δx/2)q0’’’ – kA0 =0
(Δx)

q′′′0 Δx2 2q0 Δx


Or: 2T1 – 2T0 + ( )+( ) =0 for i = 0 (5.13)
k k

Similarly for the volume element surrounding the grid point M, the energy balance equation
after simplification reduces to

q′′′M Δx2 2qM Δx


2TM – 1 – 2TM + ( )+( ) = 0 for i = M (5.14)
k k

For the insulated boundary at x = 0 we have q0 = 0 and for the insulated boundary at x = L
we have qM = 0.Substituting these conditions in Equations (5.13) and (5.14) we get the
finite difference equations for the boundary nodes as:

q′′′0 Δx2
2T1 – 2T0 + ( ) =0 (5.15)
k

q′′′M Δx2
And: 2TM – 1 – 2TM + ( ) =0 (5.16)
k

(iii)Boundary surfaces subjects to convection:


Let the two boundary surfaces are exposed to ambient at a uniform temperature T∞ with
surface heat transfer coefficient h as shown in Figure5.3.

T∞,h
0 1 2 M–2 M−1 M
T∞, h

𝚫𝐱 𝚫𝐱
Δx Δx
𝟐 𝟐

Figure 5.3 Nomenclature for finite difference formulation for convection boundary
conditions

The energy balance equation for the element surrounding node 0 can be written as

𝛥𝑥 (T0 − T1 )
hA0[T∞ − T0] + A0 ( 2 ) q0’’’ – kA0 =0
(Δx)

2 Δx h Δx2 q′′′ 2 Δx h
Or: 2T1 – (2 + ) T0 + 0
+ T∞ = 0 for i = 0 (5.17)
k k k

194
2 Δx h Δx2 q′′′ 2 Δx h
And: 2TM −1 – (2 + ) TM + M
+ T∞ = 0 for i = M
k k k

(5.18)

Example 5.1: A slab of thickness 1 cm is generating heat at a uniform rate of 7.2 ×107
W∙m-3.The boundary surface at x = 0 is maintained at a uniform temperature of 50℃and the
boundary surface at x = L is in contact with a fluid at a temperature of 100 ℃ with a surface
heat transfer coefficient of 200 W∙m-2∙K-1. The thermal conductivity of the slab is 18
W∙m-1∙K-1.Dividing the region into five equal sub regions write the finite difference
formulation of this heat conduction problem and express the equations fro determination of
unknown temperatures in matrix form.

Schematic:

L
0 1 2 3 4 5
T∞, h

Δx
𝚫𝐱 𝚫𝐱
Δx
𝟐 𝟐

Known: L = 0.01m; k = 18 W∙m-1∙K-1;T0=500C; T∞ = 1000C; h = 200 W∙m-2∙K-1;Δx = 0.01/5


= 2× 10−3m; 𝑞𝑖′′′ = 7.2×107 W∙m-3.

Find: Finite difference equations to determine the unknown temperatures; (ii) Express these
equations in matrix form.

Assumptions: (i) One dimensional steady state conduction: (ii) Thermal conductivity of the
slab is constant

Solution: For intermediate nodes 1 to 5 the finite difference equations are given by Equation
(5.5):

q′′′
i Δx
2
Ti-1 − 2Ti + Ti+1 = − i = 1,2,…….,5
k

For 1 = 1 we therefore have:

q′′′
1 Δx
2
T0 – 2T1 + T2 = −
k

195
7.2× 107 × 0.0022
Or: 50 – 2T1 + T2 = −
18

Or: −2T1 + T2 = −66 (1)

For i = 2 Equation (5.5) gives:

7.2×107 × 0.0022
T1 – 2T2 + T3 = −
18

Or: T1 – 2T2 + T3 = − 16 (2)

Similarly for i = 3 and i = 4 we have:

T2 – 2T3 + T4 = − 16 (3)

and : T3 – 2T4 + T5 = − 16 (4)

For the boundary nod 5 the finite difference equation is given by Equation (5.18):

2 Δx h Δx2 q′′′
5 2 Δx h
2T4 – (2 + ) 𝑇5 + + T∞ = 0
k k k

2×0.002 × 200 0.0022 × 7.2×107 2×0.002 × 200


2T4 − (2 + ) 𝑇5 + + × 100 = 0
18 18 18

Or: 2T4 – 2.044 T5 = −20.44 (5)

Equations(1) to (5) can be written in matrix form as:

−2 1 0 0 0𝑇1 −66
1 −2 1 0 0𝑇2 −16
0 1 −2 1 0 𝑇3 = −16 (6)
0 0 1 −2 1 𝑇4 −16
[0 0 0 ]
−2 −2.044 {𝑇5 } {−20.44}

The above system of Equations are solved using ONLINE solutions using MATLAB and the
unknown temperatures are found to be as shown in the Table 5.1 below. In the same table the
exact solution is also given for comparison. The exact solution for this example is given by
x x x
T(x) = 50 + 5× + 200 × (19 − ) × ( ) (7)
L L L

196
Table 5.1: Comparison betweenfinite difference solution and exact solution for one
dimensional steady state conduction in a slab
𝒙 0.2 0.4 0.6 0.8 1.0
𝑳
Temperature in ℃ T1 T2 T3 T4 T5
Finite Difference Solution 119.05 172.09 209.13 230.18 235.22
Exact Solution 119.00 172.00 209.00 230.00 235.00

5.3. Finite Difference Formulation for One-dimensional Steady state Conduction in a


Fin of Uniform Cross Section:
Consider a fin of uniform cross section whose base is maintained at a uniform temperature T0
and is exposed to an ambient at a uniform temperature T∞..Let L be the length of the fin and
A be the area of cross section of the fin.Let h be the surface heat transfer coefficient for the
lateral surface of the fin and he be the surface heat transfer coefficient for the fin tip. The fin
is subdivided into M sub regions as shown in Figure5.4(a).

T∞, h
Fin base
iΔx he

0 1 2 i-1 i i+1 M-1 M

𝚫𝐱 𝚫𝐱
Δx
𝟐 𝟐

Figure 5.4(a) Nomenclature for finite difference formulation for a fin of uniform cross
section

Finite difference equations for intermediate nodes 1 to M -1: For the sub region surrounding
an intermediate node i the difference equation is obtained by writing the energy balance
equation for the sub region shown in Figure5.4(b)

Qconvection

Node i-1 Node i+1

iii

Node i

Q i-1,i Qi,i+1

197
Fig5.4(b) Thermal Energy crossing the boundaries of the volume element surrounding
the node i
Rate of conduction into the sub region from the node i-1 to node i

– Rate of conduction out of the region from node i to node i+1 – Rate of convective heat

transfer from the boundary surface of the region into the ambient = 0

i.e. Q i-1,i − Qi,i+1 − Qconvection = 0

Ti−1 − Ti Ti − Ti+1
or kA [ ] – kA [ ]– hPΔx [Ti − T∞] = 0
Δx Δx

Substituting θ = T - T∞ in the above equations and simplifying we get

θ i - 1 − [ 2 + N2Δx2] θ i + θ i + 1 = 0 i = 1 ,2,…4 (5.19)

Where N2 = hP/(kA) (5.19a)

Finite difference equations for the outer boundary node M: The sub region surrounding the
outer boundary node M is shown Figure5.4(c).

Qc1

Node M - 1 Node M
Qc2

he, T∞
QM-1,M

Fig5.4(c) Thermal Energy crossing the boundaries of the volume element surrounding
the boundary node M

Energy balance equation for the sub region surrounding the node 5 can be written as

Q4-5 – Qconvection = 0

i.e.
TM−1 − TM Δx
kA [ ] − hP [TM − T∞] − heA [TM − T∞ ] = 0
Δx 2

In terms of θ the above equation reduces to

he Δx
2 θM-1 – [2 + (NΔx)2 + 2 ( )] θM = 0 (5.20)
k

198
If the heat loss from the tip is negligible or if the tip is insulated then he = 0. In that case
Equation(5.20) reduces to

2 θM-1 – [2 + (NΔx)2 ] θM = 0 (5.21)

Example 5.2:An iron rod (k = 50 W∙m−1∙K−1 ) of length 5 cm and diameter 2 cm protrudes


from a wall and is exposed to an ambient at 20℃ with a surface heat transfer coefficient of
100 W∙m−2∙K−1.The base of the rod is at 320℃and the heat transfer from the tip of the rod is
negligible. Determine the temperature distribution along the length of the rod using finite
difference method and compare the solution with the exact solution.

Sketch for example 5.2:

k h,T∞
x Insulated

Known: D = 2 cm.; k = 50 W∙m−1∙K−1 T∞ = 200C; h = 100 W∙m−2∙K−1T0 = 320℃

Find: Temperature distribution along the length using finite difference method and compare
this with the exact solution

Assumptions: (i) One dimensional steady state conduction along the length of the rod
(ii)Thermal conductivity of the rod is constant

Solution: The rod is divided into 5 equal subdivisions as shown below. Then Δx = L/5 = 5/5
= 1 cm = 0.01 m

T∞, h

L
Insulated
0 1 2 3 4 5

Δx
𝚫𝐱 𝚫𝐱
Δx
𝟐 𝟐

For 5 sub regions, the difference equation for intermediate nodes are given by Equation5.20

θi-1 – [2 + (NΔx)2] θi + θi+1 = 0 i = 1 ,2,…4 (1)

199
Finite difference equations for the boundary node 5: Since the heat loss from the tip is
negligible the difference equation for node 5 is giv en by Equation(5.21). i.e.

2 θ4 – [2 + (NΔx)2] θ5 = 0 (2)

4×100× 0.01 2
Now: (NΔx)2 = ( ) = 0.04
50×0.02

Substituting this numerical value in Equation (1) and (2) we get the finite difference
equations for temperatures of the nodes as

θi-1 – 2.04 θi + θi+1 = 0 i= 1.2.3.4 (3)

and 2 θ4 – 2.04 θ5 = 0 (4)

Therefore we have

For i = 1, θi-1 = θ0 = 320 – 20 = 300

– 2.04 θ1 + θ2 = − 300 (5a)

for i = 2, θ1 – 2.04 θ2 + θ3 = 0 (5b)

for i = 3, θ2 – 2.04 θ3 + θ4 = 0 (5c)

for i = 4, θ3 – 2.04 θ4 + θ5 = 0 (5d)

and for i = 5 2 θ4 – 2.04 θ5 = 0 (5e)

Eqs. (5a) to (5e) can be arranged in matrix form as

The above system of equations is solved using ONLINE Linear Algebra Tool Kit and the
results are compared with exact solution in the table 5.2 given below

Table 5.2: Comparison between finite difference solution and exact solution for one
dimensional steady state conduction in a fin of uniform cross section

x/L 0.2 0.4 0.6 0.8 1.0


Temperature in 0C θ1 θ2 θ3 θ4 θ5
Finite Difference Solution 260.10 230.6 210.4 198.6 194.7
Exact Solution 260.00 230.5 210.2 198.3 194.4

200
5.4. Finite Difference Formulation for One-dimensional Steady state Radial Conduction
in cylinders and spheres:

5.4.1. Cylindrical Geometry: The finite difference formulation for the problem of one
dimensional radial steady state conduction in a solid cylinder is illustrated in this section.Let
the solid cylinder is of radius R and is generating thermal energy at rate of q’’’(r) W/m3.The
R
region 0 ≤ r ≤ R is divided into M cylindrical subregions, each of thickness Δr = as shown
M
in Figure5.5.

TM
TM-1

Ti+1
Ti
Ti-1

T1

T0
0 1 i−1 i i+1 M-1 M
𝚫𝐫
𝟐

Δr
iᴧr

Figure5.5 Nomenclature for finite difference formulation for one dimensional radial
conduction in a solid cylinder

5.4.1.1Finite difference equation for an intermediate node i: Energy balance equation for
the sub region surrounding the grid point i can be written per unit length of the cylinder as

𝛥𝑟 Ti−1 − Ti
k [2𝜋 (𝑖𝛥𝑟 − ) × 1] [ ] + (2𝜋𝑖𝛥𝑟 × 𝛥𝑟 × 1)𝑞𝑖′′′
2 Δr

𝛥𝑟 Ti − Ti+1
− k [2𝜋 (𝑖𝛥𝑟 + ) × 1] [ ] =0 i = 1,2,3……M-1
2 Δr

The above equation simplifies to:

1 1 Δr2 q′′′
i
[1 − ] Ti-1 – 2Ti + [1 + ] Ti+1 = − i= 1, 2,3,…. M – 1 (5.22a)
2𝑖 2𝑖 k

201
5.4.1.2.Finite difference equation for the innermost node 0: The energy balance equation
for the sub region surrounding the node 0 can be written as

Δr 2 Δr T0 − T1
q′′′
0 π ( 2 ) ×1 – k (2𝜋 × 1) [ ]=0
2 Δr

q′′′
0 Δr
2
Or − T0 + T1 = − (5.22b)
4k

5.4.1.3. Finite difference equation for the outermost node M:

(i)Boundary surface is subjected to convection boundary condition: Let the boundary


surface is exposed to an environment at a uniform temperature T∞ with a surface heat transfer
coefficient h.The various energies crossing the boundary surface of the volume element
surrounding node M is shown in Figure5.6

Qconv

QM-1.M

o M–1 M
𝚫𝐫
𝟐

Figure 5.6 Energy balance for outer most node M

Energy balance equation for the volume element surrounding the node M can be written as

QM-1, M + Qgen − Qconv = 0

𝛥𝑟 TM−1 –Tm Δr
Or k × 2π [(𝑀 − 1)𝛥𝑟 + ]×1× + q′′′
M × 2πMΔr × ×1
2 Δr 2

− h × 2πMΔr [TM - T∞] = 0

After simplifying we get

1 1 hΔr q′′′
M Δr
2 hΔr
[1 − ] TM-1 – [(1 − )+ ] TM = − − T∞ (5.22c)
2M 2M k 2k K

202
Eqs.(5.22a) to (5.22c) form a set of M+1 simultaneous equations which can be solved for
unknown temperatures T0 to TM+1.

(ii)Boundary surface is maintained at a specified temperature,TM: For this case , since the
temperature at node M is specified, the number of unknown temperatures will be M and
Eqs.(5.22a) and (5.22b) will form a set of M equations which can be solved for unknown
temperatures T0,T1, …………………,TM-1.

5.4.2. Spherical Geometry: Following the procedure illustrated in section 5.4.1for radial
steady state conduction in a solid cylinder, the finite difference formulation for radial
conduction in a solid sphere can be written as follows:

1 2 1 2 Δr2 q′′′
(1 − ) (Ti−1 − Ti ) + (1 + ) (Ti+1 − Ti ) + i
=0 i= 1,2,………M-1
2𝑖 2i k

(5.23)
1 2 1 1 2 1
Making the approximation that (1 − ) ≈ [1 − ] and (1 + ) ≈ [1 + ]
2𝑖 2𝑖 2𝑖 2𝑖

Equation (5.23) reduces to

1 1 Δr2 q′′′
i
[1 − 𝑖 ] Ti-1 – 2Ti + [1 + 𝑖 ] Ti+1 = − i= 1, 2,3,…. M – 1 (5.24a)
k

Equation (5.24a) is similar to Equation(5.23a) for a solid cylinder except that 1/2i is replaced
by 1/i.

The difference equation for the innermost node ‘i’ is given by

Δr2 q′′′
0
6T1 – 6T0 = − (5.24b)
k

The difference equation for the outermost node M is given by

1 2 1 2 hΔr hΔr q′′′


M Δr
2
(1 − ) TM-1 − [(1 − ) + ] TM = − 𝑇∞ −
2𝑀 2𝑖 K K 2k

1 2 1 1 2 1
Making the approximation that (1 − ) ≈ [1 − ] and (1 + ) ≈ [1 + ]
2𝑖 2𝑖 2𝑖 2𝑖

We get

1 1 hΔr hΔr q′′′


M Δr
2
[1 − ] TM-1 − [(1 − )+ ]TM = − 𝑇∞ − (5.24c)
𝑀 M k K 2k

203
Eqs. (5.24a) to (5.24c) form a set of M+1 simultaneous equations which can be solved for
unknown temperaturatures T0 to TM.

Example 5.3: A long cylindrical fuel element of radius 1 cm and thermal conductivity of 25
W∙m−1∙K−1generates energy at a constant rate of 5×10 8 W∙m -3. The boundary surface is
maintained at a uniform temperature of 100℃.Assuming one-dimensional radial conduction,
determine the radial temperature distribution in the fuel element using finite difference method
and compare the values with exact solution for the problem.

Sketch for Example 5.3:Refer Figure5.5 and Figure 5.6

Known: R = 0.01m; k = 25 W∙m−1∙K−1;𝑞𝑖′′′ = 5×10 8 W∙m -3for all the nodes;TM = 100℃.

Find: Radial steady state temperature distribution in the cylinder by finite difference method
and compare with exact solution

Assumptions: (i)One dimensional radial steady state conduction; (ii)Thermal conductivity is


constant

Solution: Divide the region 0 ≤ r ≤ R into 5 cylindrical sub regions so that


0.01
Δr = = 0.002 m
5

The finite difference equation for the innermost node ‘0’ is obtained from Equation(5.22b) as

q′′′
0 Δr
2 5×108 ×0.0022
T0 – T1 = − =−
4k 4×25

Or −4T0 + 4T1 = − 80 ………………………………(1)

The finite difference equations for intermediate nodes i = 1 to 4 is given by Equation(5.22a)


as

1 1 Δr2 q′′′
i
[1 − ] Ti-1 – 2Ti + [1 + ] Ti+1 = − i= 1, 2,3,…. M – 1
2𝑖 2𝑖 k

1 1 Δr2 q′′′
i
Hence for i = 1 we have [1 − ] Ti-1 – 2Ti + [1 + ] Ti+1 = − = − 80
2𝑖 2𝑖 k

0.5T0 – 2T1 + 1.5T2 = − 80 (2)

For i = 2: 0.75T1 −2T2 + 1.25T3 = − 80 (3)

For i = 3; 0.83T2 – 2T3 + 1.17T4 = − 80 (4)

For i = 4 0.88T3 – 2T4 + 1.13 × 100 = − 80

204
Or 0.88T3 – 2T4 = − 193 (5)

Eqs. (1) to (5) are solved using Online Linear Algebra Tool Kit and tabulated as shown in
the Table below. This table also gives the exact solution at the 5 nodes for comparison. The
exact solution is given by
𝑟 2
T(r) = 100 + 500× [1 − (𝑅) ] (6)

Table 5.3:: Comparison between finite difference solution and exact solution for one
dimensional steady state radial conduction in a cylinder

r/R 0.0 0.2 0.4 0.6 0.8


Temperature in ℃. T0 T1 T2 T3 T4
Finite Difference Solution 600.3 580.3 520.3 420.3 280.4
with 5 sub regions
Finite Difference Solution 600.27 580.0 520.0 420.0 280.0
with 10 sub regions
Exact Solution 600.0 580.0 520.0 420.0 280.0

The numerical solution agrees very well with the exact solution (the maximum deviation being
0.143 %) and hence there is no need to divide the region 0 ≤ r ≤ R into more number of sub
regions to improve the accuracy. It can also be seen from the above table that by dividing the
given region into 10 sub regions the numerical predictions exactly matches with the exact
solution

Example 5.4. A spherical fuel element of radius 1 cm and thermal conductivity of 25


W∙m−1∙K−1 generates energy at a constant rate of 7.5× 10 8 W∙m -3. The boundary surface is
maintained at a uniform temperature of 100℃..Assuming one-dimensional radial conduction,
determine the radial temperature distribution in the fuel element using finite difference method
and compare the values with exact solution for the problem.

Sketch for example 5.4: Refer Figure5.5 and 5.6

Known: R = 0.01m; k = 25 W∙m−1∙K−1; 𝑞𝑖′′′ = 7.5×10 8 W∙m -3for all the nodes; TM = 100℃

Find: Radial steady state temperature distribution in the sphere by finite difference method
and compare with exact solution

Assumptions: (i)One dimensional radial steady state conduction; (ii)Thermal conductivity is


constant

Solution: Divide the region 0 ≤ r ≤ R into 5 cylindrical sub regions so that


0.01
Δr = = 0.002 m
5

205
Finite difference equation for the inner most node ‘0’ is obtained from Equation(24b) as

Δr2 q′′′
0 (0.002)2 ×7.5× 108
6T1 – 6T0 = − =−
k 25

Or − 6T0 + 6T1 = −120 (1)

For intermediate nodes form i = 1 to 4 Equation(5.24a) is applicable:

1 1 Δr2 q′′′
i
[1 − 𝑖 ] Ti-1 – 2Ti + [1 + 𝑖 ] Ti+1 = − i= 1, 2,3,4
k

For i =1 we have: −2T1 + 2T2 = − 120 (2)


(2)

For i = 2: 0.5T1 – 2T2 + 1.5 T3 = − 120 (3)

For i = 3: 0.667T2 – 2T3 + 1.333T4 = − 120 (4)

For i = 4 0.75T3 – 2T4 + 1.25 × 100 = − 120

Or 0.75T3 – 2T4 = − 245 (5)

Eqs. (1) to (5) are solved using online Linear Algebra Tool Kit and the solution is shown in
Table below.The table also gives the exact solution for comparison. The exact solution is
given by

𝑟 2
T(r) = 100 + 500 [1 + (𝑅) ]

Table 5.3:: Comparison between finite difference solution and exact solution for one
dimensional steady state radial conduction in a sphere

r/R 0.0 0.2 0.4 0.6 0.8


Temperature in ℃ T0 T1 T2 T3 T4
Finite Difference Solution 600 580 520 420 280
with 5 sub regions
Exact Solution 600 580 520 420 280

Example 5.5: A long triangular fin attached to a surface is made of Aluminium (k = 180
W∙m−1∙K−1The fin is 5 cm long having a base thickness of 1 cm.The base is maintained at a
uniform temperature of 200 ℃. The fin is exposed to an ambient at 25℃and the convective
heat transfer coefficient of 15 W∙m−2∙K−1.Using finite difference method with six equally

206
spaced nodes determine(a) the temperature at the nodes, (b) the rate of heat dissipation from
the fin to the ambient and (c) the fin efficiency

Sketch for example 5.5 h,T∞


θ 1 2 3 4

T0 θ
0 5
b 11

Known: L = 0.05 cm; b = 0.01 cm; k = 180 W∙m−1∙K−1; T0= 200 ℃; T∞ = 250C; h = 15
W∙m−2∙K−1; Δx = L/5 = 0.05/5 = 0.01m ;
Find: (a) Temperatures at nodes 1 to 5; (b) Qfin; (c) fin efficiency η

Assumptions: (i) one dimensional steady state conduction; (ii) thermal conductivity of the fin
is constant; (iii) heat dissipation to the ambient is by convection only

Solution: Volume element surrounding an intermediate node i is shown below with adjacent
nodes.

𝚫𝐱
A i- 1/2 slant Length =
𝐜𝐨𝐬 𝛉
Fin Base Node i
Ai+1/2

Node (i – 1) Node (i + 1)
Δx

iΔx

Energy balance equation for the volume element surrounding node i can be written as

Q(i-1) -i – Qi – (i+1) − Qconvection = 0

Ti−1 − Ti Ti−1 − Ti 2wΔx


k A i- ½ ( ) − k A i+ ½ ( ) − h ( cos θ ) (Ti,j − T∞ ) = 0
Δx Δx

i = 1,2,3 and 4 (1)

where A i- ½ = 2w[L – (i – ½)Δx] tan θ ; A i+ ½ = 2w[L – (i + ½)Δx] tan θ

207
Substituting these expressions in Equation(1) we get

1 Ti−1 − Ti 1 Ti − Ti−1
2kw[L – (i − 2)Δx]tan θ − 2kw[L – (i + 2)Δx]tan θ
Δx Δx

2wΔx
− h( cos θ ) (Ti,j − T∞ ) = 0

1 Δx Δx hΔx2 1 Δx
Or [1 – (i − 2) ]Ti – 1 − [2 – 2i − kL sin θ] Ti + [1 – (i + 2) ]Ti + 1
L L L

hΔx2
=− T∞ i = 1,2,3,and 4 (2)
kL sin θ

0.01
Now tan θ = b/2L = 2∗0.05 = 0.1 Hence sin θ = 0.0995

For i =1 Equation(2) gives 0.9 × 200 + 0.7T2 – 1.6 T1 = − 0.0017 ×25

Or − 1.6 T1 + 0.7 T2 = − 180.042 (a)

For i = 2 ; 0.7T1 – 1.198 T2 + 0.5T3 = − 0.042 (b)

For i = 3; 0.5T2 – 0.802 T3 + 0.3T4 = − 0.042 (c)

For i = 4; 0.3T3 – 0.402 T4 + 0.1T5 = − 0.042 (d)

The finite difference equation for node i = 5 is obtained by writing the energy balance equation
for the volume element having a length of Δx / 2 surrounding node 5 as shown in Figure below

Δx/(2cos θ)
4 5

Δx/2 Δx/2

Energy balance equation for node 5 can be written as

𝛥𝑥 T4 − T5 Δx/2
k ×2w 2
tan θ (
Δx
) – h ×2w × cos θ (T5 - T∞) = 0
hΔx hΔx
or T4 – [1 + ]T5=− T∞
k sin θ k sin θ

208
Hence for node 5 we have T4 – 1.008 T5 = − 0.2 (e)
Solving Eqs. (a) to (e) using ONLINE Linear Algebra Tool Kit we get the unknown
temperatures as follows:

Node 1 2 3 4 5
Temp (℃) 199.76 198.99 197.31 195.70 194.30

(b) Qfin = Sum of heat dissipated from all the volume elements to the surroundings

Δx/2 Δx
= h× 2w cos θ (T0 - T∞) + h × 2W cos θ [(T1 − T∞) + (T2 − T∞) + (T3 − T∞)
Δx/2
+ (T4 − T∞)] + h × 2w (cos θ) (T5 − T∞)

𝑤𝛥𝑥
= h (cos 𝜃) [T0 + 2(T1 + T2 + T3 + T4) + T5 −10 T∞]

15×1×0.01
= × [200 + 2× (199.76 + 198.99 + 197.31 + 195.7) + 194.3 – 10 ×25]= 260.46 W
cos 5.71

2𝑤𝐿 15×2×1×0.05
(c) Qmax = h Afin,total (T0 − T∞) = h(cos 𝜃) (T0 − T∞) = × (200 – 25) = 263.8 W
cos 5.71

260.46
Fin efficiency = η = = 0.987 = 98.7%
263.8

Example 5.6: A pin fin of variable circular cross section has a diameter of 2 cm at its base
and 1 cm at its tip and is 12 cm long. It is made of a material of thermal conductivity of 1 W∙m-
∙K . The fin base temperature is 125℃ and the fin is exposed to an ambient at 25℃ with a
1 -1

surface heat transfer coefficient of 10 W∙m-2∙K-1.Determine the steady state temperature


distribution along the length of the fin using finite difference method.

Schematic for example 5.6 showing the nomenclature adopted

Δx/2

Δx/2
θ D2

0 1 2 3 4 5 6

D1

Δx
L

209
Known: D1 = 0.02 m; D2 = 0.01 m; L = 0.12 m; k = 1 W∙m−1∙K−1; h = 10 W∙m−2∙K−;1

T0 = 125℃; T∞ = 25℃

Find: (a) Steady state temperature distribution along the length using finite difference method;
(b)Qfin

Assumptions: (i) one dimensional steady state conduction; (ii) thermal conductivity of the fin
is constant; (iii) heat dissipation to the ambient is by convection only

Solution: The fin is divided equally into 6 subregions each of length Δx so that

(𝐷1 − 𝐷2 ) (0.02 −0.01)


Δx = L/6 = 0.12/6 = 0.02 m. Also tan θ = = = 0.0417 or θ = 2.388 0
2𝐿 2×0.12

Difference equations for intermediate nodes from 1 to 5: The volume element surrounding an
intermediate node ‘i’ along with corresponding nomenclature is shown in Figure E5.6(a).

Δx / cos θ
X

i–1 i i +1

i Δx

Δx

Ai–½ Ai +1/2

Figure E5.6(a): Nomenclature for the volume element surrounding an intermediate


node ‘i’

Energy balance equation for the volume element shown can be written as

Ti−1 − Ti Ti − Ti+1
kAi – ½ ( ) − kAi +½ ( ) – h Asi [Ti −T∞] = 0 i = 1 to 5 (1)
Δx Δx

hΔx
Let φ = T − T∞. and Hi = Asi
k

Then Equation(1) can be written as

Ai – ½ φi-1 – [Ai – ½ + Ai +½ + H i ] φi + Ai +½ φi+1 = 0 i = 1 to 5 (2)

𝛥𝑥 𝛥𝑥
where Ai – ½ = π [D2/2 + (L – iΔx + ) tan θ]2 ; Ai +½ = π [D2/2 + (L – iΔx − ) tan θ]2
2 2

210
Δx
Asi = (cos θ) 2π [(D2/2 + (L – iΔx) tan θ]

Now Ai – ½ = π [0.005 + (0.12 – 0.02i +0.01) 0.0417]2 = π (0.0104 – 8.34×10−4 i) 2

Ai +½ = π [0.005 + (0.12 – 0.02i − 0.01) 0.0417]2 = π (9.587 × 10−3 – 8.34×10−4 i)2

0.02 0.01
Asi = ×2×π× [ + (0.12 − 0.02𝑖) × 0.0417]
cos 2.388 2

= 1.258× 10 – 3 – 1.049 ×10 – 6× i

Numerical values of Ai – ½ , Ai +½ and Hi found for different values of i from 1 to 5 are tabulated
as shown below.

Node Ai – ½ Ai +½ = π (9.587 × 10−3 – 8.34×10−4 i)2 Hi


= π (0.0104 – 8.34×10−4 i) 2 ℎ𝛥𝑥
= [1.258× 10 – 3 – 1.049 × 10 – 6 i]
𝑘

1 2.875 × 10 −4 2.407× 10 −4 3.771 × 10 − 4


2 2.395× 10 −4 1.970× 10 −4 3.768× 10 − 4
3 1.960×10 −4 1.577× 10 −4 3.765× 10 − 4
4 1.568×10 −4 1.228× 10 −4 3.761× 10 − 4
5 1.219×10 −4 0.922× 10 −4 3.758× 10 − 4

Substituting the calculated values of Ai – ½ , Ai +½ and Hi for different nodes the final form of
Eqs.(3) for nodes 1 to 5 c an be written as

For i = 1; 2.875 × 10 −4 (125 − 25) – 9.053×10 −4 φ1 + 2.407 × 10 −4φ2 = 0

Or −9.053 φ1 + 2.407 φ2 = − 287.5 (a)

Similarly for i =2: 2.395φ1 – 8.133 φ2 + 1.970φ3 = 0 (b)

i = 3: 1.960φ2 – 7.302 φ3 + 1.577φ4 = 0 (c)

i = 4; 1.568φ3 – 6.557 φ4 + 1.228 φ5 = 0 (d)

i = 5; 1.219φ4 – 5.899φ5 + 0.922 φ6 = 0 (e)

Difference equations for node 6: The volume element surrounding node 6 is shown in Figure
E5.6(b).

211
A5+1/2
𝛥𝑥
2 cos 𝜃

5 6
D2

Δx/2 Δx/2

Figure E5.6(b): Nomenclature for volume element surrounding node 6

Energy balance equation for the volume element shown can be written as follows:

Q5-6 – Qconvection = 0

T 5 − T6
i.e. k A5+1/2 (
Δx
) – h As6 [T6 − T∞] = 0

𝐷 𝛥𝑥 2 πD22 Δx D Δx
Where A5+1/2 = π ( 22 + tan 𝜃) and As6 = [ + (2 cos θ) × 2π × ( 22 + tan θ)]
2 4 2

Sustituting φ = T - T∞ and simplifying we get

ℎ𝛥𝑥
A5+1/2 φ5 – [A5+1/2 + 𝐴𝑠6 ] φ6 = 0
𝑘

0.01 0.02 2
Now A5+1/2 = π ( + × 0.0417) = 9.219 × 10 – 5
2 2

π∗ 0.012 0.02 0.01 0.02


and As6 = + (2 cos 2.388) × 2π × ( + ∗ 0.0417) = 4.192 × 10 −4
4 2 2

15∗0.02
Or 9.219 × 10 – 5 φ5 – [9.219 × 10 – 5 + × 4.192 × 10 −4] φ6 = 0
1

Or φ5 – 2.364 φ6 = 0 (f)

Eqs. (a) to (f) can be solved using any standard technique of solving a system of linear
equations.

5.5. Finite Difference Formulation for Two-dimensional Steady State conduction with
Energy Generation

5.5.1. Difference Equation in terms of Cartesian Coordinates: The two dimensional steady-
state heat conduction equation with heat generation in a solid with constant thermal
conductivity is given by

212
∂2 T ∂2 T q′′′(x,y)
2
+ + =0 in region R (5.25)
∂x ∂x2 k

The region R is replaced by a number of small rectangular meshes each of Δx by Δ.y as


shown in Figure5.7.Symbols i,j are used to denote the location of a nodal point whose
coordinates are x = iΔx, y = jΔy. Then the temperature at a node (i,j) is denoted by Ti,j.

y
(i – 1, j) (i, j+1) (i, j) (i+1, j)
Δx
(i, j-1)
(j +1)Δy

jΔy
Δy
(j – 1)Δy

x
(i −1)Δx iΔx (i +1)Δx

Figure5.7: Rectangular mesh of size Δx, Δy and intermediate node (i,j) surrounded by 4
neighboring nodes

Difference equation for all intermediate nodes: Control volume surrounding the nodal point
i,j is shown in Figure5.7.The energy balance equation for this control volume under steady
state conditions can be written as follows:

Q(i-1,j) – (i,j) + Q(i,j+1) – (i,j) + Q(i+1,j) – (i,j) + Q(i,j+1) – (i,j) + Qg = 0 (5.26)

Using Fourier’s law Equation(5.26) can be written as

Ti−1,j −Ti,j Ti,j+1 −Ti,j Ti+1,j −Ti,j


k (Δy ×1) + k (Δx ×1) + k (Δy ×1) +
Δx Δy Δx

Ti,j−1 −Ti,j
k (Δx ×1) + (Δx ×Δy×1) q′′′
i,j = 0
Δy

Choosing Δx = Δy, the above equation reduces to

213
𝐪′′′
𝐢𝐣
𝟐
Ti – 1,j + Ti ,j+1 + Ti + 1,j + Ti ,j – 1 − 4 Ti ,j + 𝜟𝒙 ( )=0 (5.27)
𝐤
Equation(5.27) is the finite difference form of the heat conduction equation for any interior
nodal point (i,j).

Difference equations for boundary nodal points:

(i) Boundary subjected to prescribed heat flux:

(i , j+1)

(i +1, j)
q0 ( i , j)

Δy Figure5.9 Nomenclature for a boundary node


subjected to prescribed heat flux
(i, j – 1)
Δx

Energy ba;ance equation for the control volume surrounding the nodal point (i,j) can be
written as:

q0×(Δy×1) + Q(i , j+1) , (i,j) + Q(i , j+1) , (i,j) + Q(i , j+1) , (i,j) + Qg = 0 (5.28)

Using Fourier’s law for heat conduction the above equation reduces to

(Ti ,j+1 − Ti ,j ) (Ti+1,j − Ti ,j )


q0×(Δy×1) + k (Δx/2 ×1) + + k (Δy ×1)
Δy Δx

(Ti ,j−1 − Ti ,j ) Δx
+ k (Δx/2 ×1) + (Δy ∗ ∗ 1) q′′′
i ,j = 0
Δy 2

Substituting Δx = Δy and simplifying, the above equation reduces to

𝟐𝐪𝟎 ∗ 𝚫𝐲 𝐪′′′
𝐢 ,𝐣 ∗ 𝚫𝐲
𝟐
Ti , j+1 + 2Ti+1 , j + Ti, j−1 −4Ti , j = − − (5.29)
𝐤 𝐤

If the boundary containing the node (i,j) then q0 = 0. In that case Equation (5.29) reduces to

𝐪′′′
𝐢 ,𝐣 ∗ 𝚫𝐲
𝟐
Ti , j+1 + 2Ti+1 , j + Ti, j−1 −4Ti , j = − (5.29a)
𝐤

214
(ii) Boundary subjected to convection:

(i , j+1)

(i +1, j)
( i , j)

h,T∞ Δy Figure5.9 Nomenclature for a boundary node


subjected to convection
(i, j – 1)
Δx

Energy balance equation for the control volume surrounding the nodal point (i,j) can be
written as:

Qconvection + Q(i , j+1) , (i,j) + Q(i , j+1) , (i,j) + Q(i , j+1) , (i,j) + Qg = 0

(Ti ,j+1 − Ti ,j ) (Ti+1,j − Ti ,j )


h(Δy×1)[T∞ − Ti j] + k (Δx/2 ×1) + + k (Δy ×1)
Δy Δx

(Ti ,j−1 − Ti ,j ) Δx
+ k (Δx/2 ×1) + (Δy ∗ ∗ 1) q′′′
i ,j = 0
Δy 2
Substituting Δx = Δy and simplifying, the above equation reduces to

𝟐𝐡 𝚫𝐲 𝟐𝐡𝚫𝐲 𝐪′′′
𝐢 ,𝐣 ∗ 𝚫𝐲
𝟐
Ti , j+1 + 2Ti+1 , j + Ti, j−1 – [𝟒 + ]Ti , j = − 𝐓∞ − (5.30)
𝐤 𝐤 𝐤

If the boundary containing the node (i,j) is insulated then h = 0 and there fore Equation(5.30)
reduces to Equation (29a).

Difference equations for corner nodal point:

(i) Difference equation for exterior corner nodal point subjected to convection with ambient
temperature at T∞ and surface heat transfer coefficient h.

The nomenclature for an external nodal point is shown in Figure5.10

Q2-convection

(i , j) (i +1, j)

215
Q1-convection
Δy Figure5.10 Nomenclature for an exterior nodal point

(i, j – 1) Δx

Q1-convection + Q2-convection + Q(i +1, j)- (i,j) + Q(i , j −1)- (i,j) + Qg = 0

Δy 𝛥𝑥 Δy (Ti+1,j − Ti,j )
h( × 1) [𝑇∞ − 𝑇𝑖 ,𝑗 ] + h( × 1) [𝑇∞ − 𝑇𝑖 ,𝑗 ] + k ( ∗ 1)
2 2 2 Δx

𝛥𝑥 (Ti,j −1 − Ti,j ) Δx Δy
+k( × 1) +[ × × 1] q′′′
i,j = 0
2 Δy 2 2

Substituting Δx = Δy and simplifying we get

𝟒𝐡𝚫𝐱 𝟒𝐡𝚫𝐱 𝜟𝒙𝟐 𝐪′′′


𝐢,𝐣
2Ti+1, j + 2Ti, j−1 – [𝟒 + ] 𝐓𝐢,𝐣 = − T∞ − (5.31)
𝐤 𝐤 𝒌

(ii) Difference equation for nodal point at the intersection of two convection boundaries
with ambient temperature at T∞ and surface heat transfer coefficient h.

The nomenclature for the nodal point (i,j) at the intersection of two convection boundaries is
shown in Figure 5.11.

(i, j+1)

(i-1 j) (i, j) (i+1,j)

h, T∞ Figure5.11 Nomenclature for a node at


the
intersection of two convection boundaries
(i,j-1)

216
Energy balance equation for the control volume surrounding the node (i,j) can be written as
follows:

Q(i-1,j) – (i,j) + Q(i,j+1) – (i,j) + Q(i+1,j) – (i,j) + Q(i ,j-1) – (i,j) + Qconvection + Qg = 0

Ti−1,j − Ti,j Ti,j+1 − Ti,j Ti+1,j − Ti,j


k (Δy ×1) ( ) + k (Δx ×1) ( ) + k (Δy/2×1) ( )
Δx Δy Δx

Ti,j−1 − Ti,j 𝛥𝑥 𝛥𝑦 𝛥𝑥 𝛥𝑦 ′′′


+ k (Δx/2 ×1) ( )+ h[2 + ] [T∞ − T,j] + 3[ 2 × × 1] 𝑞𝑖,𝑗 =0
Δx 2 2

Substituting Δx = Δy and simplifying we get

𝟐𝒉𝜟𝒙 𝟐𝒉𝜟𝒙 𝟑 𝜟𝒙𝟐


Ti,j-1 + 2Ti-1,j + 2Ti,j+1 + Ti+1,j – (𝟔 + ) Ti,j = − T∞ − 𝟐 𝒒′′′
𝒊,𝒋 = 0 (5.32)
𝒌 𝒌 𝒌

Difference equation for a node next to an irregular boundary

Consider a node (i,j) located next to a curved surface as shown in Figure5.12.Let the two
stirngs from the node (i,j) intersect the curved boundary at say points A and B as shown in
Figure5.12.

Node (i-1,J)
A
Node (i,j)
ξΔy
B
Δy Figure 5.12 Nomenclature for a node adjacent
to a
Δx ηΔx curved boundary

Node (i,j-1)

Assuming that the temperatures at A and Bi.e. TA and TB are known the energy balance
equation for the control volume surrounding the node (i,j) can be written as follows

Q(i-1,j) – (i,j) + QA – (i,j) + QB – (i,j) + Q(i, j-1) – (i,j) + Qg = 0

Δy ξΔy (Ti−1,j − Ti,j ) Δx ηΔx (TA − Ti,j ) Δy ξΔy (TB − Ti,j )


k[( 2 + ) × 1] + k[( 2 + ) × 1] + k[( 2 + ) × 1]
2 Δx 2 ξΔy 2 ηΔx

Δx ηΔx (Ti,j−1 − Ti,j ) Δx ηΔx Δy ξΔy


+ k[( 2 + ) × 1] +(2 + )(2 + ) q′′′
i,j = 0
2 Δy 2 2

217
Assuming Δx = Δy and simplifying we get

𝐓𝐢−𝟏,𝐣 𝐓𝐢,𝐣−𝟏 𝟏 𝟏 𝐓𝐁 𝐓𝐀 𝚫𝐱 𝟐 𝐪′′′


𝐢,𝐣
(𝟏+ 𝛈)
+ (𝟏+ 𝛏) − (𝛈 + 𝛏 ) Ti,j = − 𝛈(𝟏+ 𝛈) − 𝛏(𝟏+ 𝛏) − (5.33)
𝟐𝐤

Example 5.7: A solid body of square cross section of side 2 cm. is generating heat at a uniform
rate of 2×10 6W∙m- 3.The thermal conductivity of the body is 15 W∙m-1∙K-1.The left surface of
the body is insulated and the bottom surface is maintained at a uniform temperature of
1000C.The right surface is subjected to a uniform heat flux of 5000 W/m2 and the top surface
is in contact with a fluid at a temperature of 250C with a surface heat transfer coefficient of
75 W∙m−2∙K−1.Assuming Δx = Δy = 1 cm obtain the finite difference equations for the unknown
temperatures assuming steady state conduction at the six nodes and solve for these
temperatures.

Sketch for example 5.7


Node 2
Δx h = 75 W/(m2-K), T∞ = 25 0C

1 2 3
ΔΔΔ

Insulated 4 6 qs = 5000 W/m2


Node 5
Δy

7 8 9

T = 100 0C

Known: Δx = Δy = 0.01 m; k = 15 W∙m−1∙K−1;𝑞𝑖′′′ =2×10 6 W∙m-3 for all ‘i’;qs = 5000 W∙m-2;

h = 75 W∙m−2∙K−1; T7 = T8 = T9 = 1000C; T∞ = 250C

Find: (i) Finite difference equations to find unknown temperatures T1 to T6 (ii) Numerical
values of T1 to T6 by solving the finite difference equations

Assumptions: (i) steady state two dimensional conduction heat transfer; (ii) thermal
conductivity and heat generation in the solid are constant

Solution: Finite difference equation for node 1:

h, T∞
1 2

Insulated Δy/2

218
Δx/2 FigureE5.7(1) : Volume element
Δy/2 surrounding node 1
4

Energy balance equation for the volume element surrounding node 1 per unit width measured
normal to the paper can be written as:

Q4-1 + Q2-1 + Qconvection + Qg = 0

Δx T4 − T1 Δy T2 − T1 Δx 𝛥𝑥 𝛥𝑦
k ×1( )+k ( )+h ×1(T∞ − T1 ) + ( 2 × × 1) q′′′
1 =0
2 Δy 2 Δx 2 2

Substituting Δy = Δx in the above equation and simplifying we get

hΔx hΔx 𝛥𝑥 2
T4 + T2 – [2 + ]T1 = − −( ) q′′′1
k k 2𝑘

Substituting the given data we get

75∗0.01 75∗0.01 0.012


T4 + T2 – [2 + ]T1 = − –( ) × 2 × 106
15 15 2∗15

Or – 2.05T1 + T2 + T4 = −6.717 (a)

Finite difference equation for node 2: Volume element surrounding node 2 with the
nomenclature is shown in Figure E5.7(2).

Δx h, T∞

1 2 3
Δy/2

Δx/2 Δy/2
5

FigureE5.7(2): Volume element surrounding node 2

Energy balance equation for the volume element surrounding node 2 per unit width measured
normal to the paper can be written as:

Q1-2 + Q3-2 + Q5-2 + Qconvection + Qg = 0

Δy T1 − T2 Δy T3 − T2 T5 − T2
k( × 1) ( ) + k ( 2 × 1) ( ) + k (Δx ×1)( ) + h (Δx ×1) (T∞ - T2)
2 Δx Δx Δy

219
Δy
+ (Δx × × 1) q′′′
2 = 0
2
Substituting Δy = Δx and simplifying we get

2hΔx 2hΔx Δx2


T1 – [4 + ]T2 + T3 +2T5 = − T∞ − q′′′
2 =0
k k k

Substituting the given data in the above equation we have

2×75×0.01 2×75×0.01 0.012


T1 – [4 + ]T2 + T3 +2T5 = − ×25 − × 2 × 10 6 = 0
15 15 15

Or T1 – 4.1 T2 + T3 +2T5 = − 15.83 (b)

Finite difference equation for node 3: Volume element surrounding node 3 with the
nomenclature is shown in Figure E5.7(3).

h, T∞
2 3

Δy/2 qs

Δy Δx/2

Δx 6

Figure E5.6(3) Volume element surrounding node 3

Energy balance equation for the volume element surrounding node 3 per unit width measured
normal to the paper can be written as:

Q2-3 + Q6-3 + Qconvection + Qs + Qg = 0

Δy T2 − T3 Δx T6 − T3 𝛥𝑥
Or k ( × 1) ( ) + k ( 2 × 1) ( ) + h( 2 × 1)(T∞ − T3)
2 Δx Δy

𝛥𝑦 𝛥𝑥 𝛥𝑦
+( × 1)qs + ( × × 1) q′′′
3 =0
2 2 2

Substituting Δy = Δx and simplifying we get

hΔx hΔx Δx Δx2


T2 – [2 + ] T3 + T6 = − T∞ − qs − q′′′
3 =0
k k k 2k

Substituting the given data in the above equation we have

220
75∗0.01 75∗0.01 0.01 0.012
T2 – [ 2 + ] T3 + T6 = − ×25 – × 5000 – × 2 × 10 6
15 15 15 2∗15

Or T2 – 2.05T3 + T6 = − 11.25 (c)

Finite difference equation for node 4: Volume element surrounding node 4 with the
nomenclature is shown in Figure E5.7(4).

h, T∞

1
Insulated

4 5

Δy FigureE5.6(4) Volume element surrounding


node 4

7
Δx

Energy balance equation for the volume element surrounding node 4 per unit width measured
normal to the paper can be written as:

Q1-4 + Q5-4 + Q7-4 + Qg = 0

Δx T 1 − T4 T5 − T 4 Δx T7 − T 4
Or k( × 1) ( ) + k (Δy×1) ( ) + k ( 2 × 1) ( )
2 Δy Δx Δy

Δx
+ (Δy × × 1) q′′′
4 =0
2

Substituting Δy = Δx and simplifying we get

𝛥𝑥 2
T1 – 4T4 + 2T5 = − T7 − q′′′
3
𝑘

Substituting the given data in the above equation we have

0.012
T1 – 4T4 + 2T5 = − 100 − × 2 × 10 6
15

Or T1 – 4T4 + 2T5 = −113.33 (d)

221
Finite difference equation for node 5: Volume element surrounding node 5 with the
nomenclature is shown in Figure E5.7(5).

2
5

4 5 6

Δx Δy
8

Figure E5.6(5) Volume element surrounding node 5

Energy balance equation for the volume element surrounding node 3 per unit width measured
normal to the paper can be written as:

Q4-5 + Q2-5 + Q6-5 + Q8-5 + Qg = 0

T −T T −T T −T T −T
Or k Δy ( 4Δx 5) + k Δx ( 2Δy 5) + k Δy ( 6Δx 5 ) + k Δx ( 8Δy 5 ) + (Δx× Δy×1) q′′′
5 =0

Substituting Δy = Δx and simplifying we get


𝛥𝑥 2
T2 + T4 + T6 – 4 T5 = − T8 − 𝑘 q′′′
3
Substituting the given data in the above equation we have
0.012
T2 + T4 + T6 – 4 T5 = − 100 − × 2 × 10 6
15

Or T2 + T4 + T6 – 4 T5 = − 113.33 (e)

Finite difference equation for node 6: Volume element surrounding node 6 with the
nomenclature is shown in Figure E5.7(6).

Δx
Δy

222
5 6
qs

Figure E5.6(6) Volume element surrounding node 6

Energy balance equation for the volume element surrounding node 6 per unit width measured
normal to the paper can be written as:

Q5-6 + Q3-6 + Q9-6 + Qs + Qg = 0

T −T Δx T3 − T6 Δx T9 − T6
Or k Δy ( 5Δx 6) + k ( × 1) ( ) + k ( 2 × 1) ( Δy ) + (Δy×1)qs
2 Δy

Δx
+ (Δy × × 1) q′′′
6 =0
2
Substituting Δy = Δx and simplifying we get

2𝛥𝑥 𝛥𝑥 2
T3 + 2T5 – 4T6 = − T9 – 𝑞𝑠 – q′′′
6
𝑘 𝑘

Substituting the given data in the above equation we have

2∗0.01∗5000 0.012
T3 + 2T5 – 4T6 = − – × 2× 10 6
15 15

Or T3 + 2T5 – 4T6 = − 120 (f)

Eqs. (a) to (f) are solved using Online Linear Algebra Tool Kit and the nodal temperatures
are found to be as follows:
T1 = 118.47 0C ; T2 = 119.630C; T3 = 121.930C; T4 = 116.510C; T5 = 117.130C ;
T6 = 119.030C

Example 5.8:Hot combustion gases of a furnace are flowing through a concrete chimney
[(k = 1.4 W∙m−1∙K−1] of rectangular cross section as shown in Figure E5.8.The flow section
of the chimney is 20 cm X 40 cm, and the thickness of the wall of the chimney is 10 cm. The
average temperature of the hot gases in the chimney is Ti = 280 0C, and the average convective
heat transfer coefficient inside the chimney is hi = 75 W∙m−2∙K−1. The chimney is losing heat
from its outer surface to the ambient air at 15 0C by convection with a surface heat transfer
coefficient of ho = 18 W/(m2-K). Using the finite difference method with Δx = Δy = 10 cm and
taking full advantage of the symmetry, determine (a) the finite difference formulation of the

223
problem to find steady two-dimensional temperatures at the nodal points, (b) the temperatures
at these nodal points, and (c) the rate of heat loss for a 1-m-long section of the chimney.
h0, T0
k

10 cm

20 cm

10 cm
H

10 cm 40 cm 10 cm

hi , Ti

FigureE5.8: Figure for example 5.

Sketch for example 5.8 with nodes:Because of symmetry only one fourth of the cross section
of the chimney is considered for analysis as shown below.

Axis of symmetry
(Insulated boundary)
1 2

224
hi,Ti

Δx
3 4 5 6
4
Δy/2

Δy/2

7 8 9 10
Δx/2 h0,T0
Known: k = 1.4 W∙m−1∙K−1; Δx = Δy = 0.1 m ; hi =75 W∙m−2∙K−1; Ti = 280 0C;

h0 = 18 W∙m−2∙K−1; T0 = 15 0C

Find: (i) finite difference equations to determine steady two dimensional temperatures at
different nodes in the chimney T1 to T10 (ii) temperatures at the nodes and (iii) rate of heat
transfer for 1 m height of the chimney

Assumptions: (i) Two dimensional steady state conduction; (ii) Thermal conductivity of the
chimney is constant; (iii) No radiation losses from the outer surface of the chimney to the
surroundings

Solution: Finite difference equation for node 1: The volume element surrounding node 1 is
shown in Figure E5.8(1). Energy balance equation for the volume element can be written as

Axis of Symmetry (Insulated)


1 2

Δy/2
hi,Ti

Δy
Δx/2

Figure E5.8(1): Volume element surrounding node 1

Q5-1 + Q2-1 + Qconvection = 0

Δx T 5 − T1 Δy T 2 − T1 Δy
Or k ( × 1) ( ) + k ( 2 × 1) ( ) + hi ( 2 × 1)(Ti – T1) = 0
2 Δy Δx

225
Substituting Δy = Δx and simplifying we get

ℎ𝑖 𝛥𝑥 ℎ𝑖 𝛥𝑥
−[2 + ] T1 + T2 + T5 = − [ ] Ti
𝑘 𝑘

75∗0.1 75∗0.1
−[2+ ]× T1 + T2 + T5 = − [ ]×280
1.4 1.4

Or −7.357T1 + T2 + T5 = − 1500 (a)

Finite difference equation for node 1: The volume element surrounding node 2 is shown in
Figure E5.8(2). Energy balance equation for the volume element can be written as

Δx Axis of Symmetry (Insulated)


1 2

Δy/2
ho,To

Δy
Δx/2

Figure E5.8(1): Volume element surrounding node 2

Q1-2 + Q6- 2 + Qconvection = 0

Δy T 1 − T2 Δx T6 − T2 Δy
Or k( × 1) ( ) + k ( 2 × 1) ( ) + ho ( 2 × 1)(To – T2) = 0
2 Δx Δy

Substituting Δy = Δx and simplifying we get


ho Δx ho Δx
T1 – [2 + ] T2 + T6 = − To
k k
Substituting for the given quantities we have

75×0.1 75×0.1
T1 − [ 2 + 1.4
]× T2 + T6 = − [ 1.4
]×15

Or T1 – 7.357T2 + T6 = − 110.355 (b)

Finite difference equation for node 3: The volume element surrounding node 3 is shown in
Figure E5.8(3). Energy balance equation for the volume element can be written as follows.

hi,Ti

226
3 4

Δy/2

Δy
Δx/2

7
Axis of Symmetry (Insulated)

Figure E5.8(3): Volume element surrounding node 3

Q7-3 + Q4-3 + Qconvection = 0

Δx T7 − T3 Δy T 4 − T3 𝛥𝑥 T i − T3
Or k( × 1) ( ) + k ( 2 × 1) ( ) + hi( 2 × 1) ( )=0
2 Δy Δx Δy

Substituting Δy = Δx and simplifying we get

hi Δx hi Δx
T7 – [2 + ]T3 + T4 = − Ti
k k

Substituting for the given quantities we have

75×0.1 75×0.1
T7 − [ 2 + 1.4
]× T3 + T4 = − [ 1.4
]×15

Or – 7.357T3 + T4 + T7 = − 110.355 (c)

Finite difference equation for node 4: The volume element surrounding node 4 is shown in
Figure E5.8(4). Energy balance equation for the volume element can be written as follows.

Δx hi , Ti
4
3 5
Δy/2

Δx/2 Δy/2
8

FigureE5.8(4): Volume element surrounding node 4

Q3-4 + Q8-4 + Q5-4 + Qconvection = 0

227
Δy T3 − T4 T8 − T 4 Δy T5 − T4
Or k ( × 1) ( ) + k (Δx ×1) ( ) + k ( 2 × 1) ( )
2 Δx Δy Δx

+ hi(Δx×1)(Ti – T4) = 0

Substituting Δy = Δx and simplifying we get

𝛥𝑥 𝛥𝑥
T3 – [4 + 2hi ( 𝑘 )] T4 + 2T8 = − 2hi ( 𝑘 )Ti

Substituting for the given quantities we have


2∗75∗0.1 2∗75∗0.1
T3 − [ 4 + ]× T4 + 2T8 = − × 280
1.4 1.4

Or T3 – 14.714T4 + 2T8 = − 3000 (d)

Finite difference equation for node 5: The volume element surrounding node 5 is shown in
Figure E5.8(5). Energy balance equation for the volume element can be written as follows.

hi,Ti

Δy/2
4 5 6

Δy/2

Δx/2 Δx/2

Figure5.8(5): Volume element surrounding node 5

Q4-5 + Q1-5 + Q6-5 + Q9-5 + Qconvection = 0

𝛥𝑦 𝑇4 − 𝑇5 𝛥𝑥 𝑇1 − 𝑇5 𝑇6 − 𝑇5
Or k ( 2 × 1) ( ) + k ( 2 × 1) ( ) + k (𝛥𝑦 × 1) ( )
𝛥𝑥 𝛥𝑦 𝛥𝑥

𝑇9 − 𝑇5 𝛥𝑥 𝛥𝑦
+ k (𝛥𝑥 × 1) ( ) + hi[( 2 + ) × 1] (𝑇𝑖 − 𝑇5 ) = 0
𝛥𝑦 2
Substituting Δy = Δx and simplifying we get

T4 – [6 + 2hi (Δx/k)]T5 + T1 +2T6 + 2T9 = − 2hi (Δx/k)Ti

Substituting for the given quantities we have

228
T4 – [ 6 + 2×75× 0.1/1.4]T5 + T1 + 2T6 + 2T9 = − 2×75× (0.1/1.4)× 280

Or T1 + T4 – 10.714T5 + 2T6 + 2T9 = − 3000 (e)

Finite difference equation for node 6: Volume element surrounding node 6 with the
nomenclature is shown in Figure E5.8(6). Energy balance equation for the volume element
can be written as follows:

Δx
Δy ho, To

5 6

10

Figure E5.8(6) Volume element surrounding node 6

Q5-6 + Q3-6 + Q10-6 + Qconvection = 0

T5 − T6 Δx T3 − T6 Δx T10 − T6
Or k (Δy×1) ( ) + k ( 2 ∗ 1)) ( ) + k ( 2 × 1)) ( )
Δx Δy Δy

+ ho(Δy ×1) (To – T6) = 0

Substituting Δy = Δx and simplifying we get

𝛥𝑥
T3 + 2T5 – [4 + 2ho(Δx/k)]T6 + T10 = − 2 ho( 𝑘 )To

Substituting for the given quantities we have

T3 + 2T5 – [4 + 2× 18×(0.1/1.4)]T6 + T10 = − 2× 18×(0.1/1.4)× 15

Or T3 + 2T5 – 6.571T6 + T10 = − 38.571 (f)

Finite difference equation for node 7: Volume element surrounding node 7 with the
nomenclature is shown in Figure E5.8(7). Energy balance equation for the volume element
can be written as follows:

229
Q3-7 + Q8-7 + Qconvection = 0

Δx T3 − T7 Δy T8 − T7 𝛥𝑥
Or k( × 1) ( ) + k ( 2 × 1) ( ) + ho( 2 × 1)(To – T7) = 0
2 Δy Δx

Axis of symmetry
3
Δx
Δy

7 8

ho, To

Figure E5.8(7) Volume element surrounding node 7

Substituting Δy = Δx and simplifying we get

T3 – [2 + hoΔx / k] T7 + T8 = − ho( Δx / k) To

Substituting for the given quantities we have

T3 – [2 + 18× 0.1 / 1.4] T7 + T8 = − (18× 0.1 / 1.4) × 15

Or T3 – 3.286T7 + T8 = − 19.286 (g)

Finite difference equation for node 8: Volume element surrounding node 8 with the
nomenclature is shown in Figure E5.8(8). Energy balance equation for the volume element
can be written as follows:

Q7-8 + Q4-8 + Q9-8 + Qconvection = 0

Δy T7 − T8 T4 − T 8 Δy T9 − T8
Or, k ( ∗ 1) ( ) + k (Δx ×1) ( ) + k ( 2 ∗ 1) ( )
2 Δx Δy Δx

+ ho (Δx ×1)(To – T8) = 0

230
4
Δx

Δy

7 8 9
ho,To

Figure E5.8(8) Volume element surrounding node 8

Substituting Δy = Δx and simplifying we get

T7 – [4+ 2ho (Δx/k)]T8 + 2T4 = − 2 ho (Δx/k)To

Substituting for the given quantities we have

T7 – [4 + 2×18× 0.1 / 1.4] T8 + 2T4 = − 2×(18×0.1 / 1.4) × 15

Or T7 – 6.571T8 + 2T4 = − 38.571 (h)

Finite difference equation for node 9: Volume element surrounding node 9 with the
nomenclature is shown in Figure E5.8(9). Energy balance equation for the volume element
can be written as follows:

5
Δx

Δy

8 9 10

ho, To

Figure E5.8(9) Volume element surrounding node 9

231
Q8-9 + Q5-9 + Q10- 9 + Qconvection = 0

Δy T8 − T9 T5 − T9 Δy T10 − T9
Or k( × 1) ( ) + k (Δx ×1) ( ) + k ( 2 × 1) ( )
2 Δx Δy Δx

+ ho (Δx ×1)(To – T9) = 0

Substituting Δy = Δx and simplifying we get

2T5 + T8 – [4 + 2ho (Δx/k)]T9 + T10 = − 2ho (Δx/k)To

Substituting for the given quantities we have

2T5 + T8 −[4 + 2×18× 0.1 / 1.4] T9 + T10 = − 2×(18×0.1 / 1.4) × 15

Or 2T5 + T8 – 6.571T9 + T10 = − 38.571 (i)

Finite difference equation for node 10: Volume element surrounding node 10 with the
nomenclature is shown in Figure E5.8(10). Energy balance equation for the volume element
can be written as follows:

Δx

Δy

9 10

ho, To

Figure E5.8(10) Volume element surrounding node 10

Q9-10 + Q6-10 + Qconvection = 0

Δy T9 − T10 Δx T6 − T10 Δx Δy
Or k( × 1) ( ) + k ( 2 × 1) ( ) + ho ( 2 + ) × 1[To –T10] = 0
2 Δx Δy 2

Substituting Δy = Δx and simplifying we get

232
T9 – [2 +2 hoΔx /k] T10 + T6 = − 2hoΔx /k T0

Substituting for the given quantities we have

T6 + T8 −[2 + 2×18× 0.1 / 1.4] T10 + T9 = − 2×(18×0.1 / 1.4) × 15

Or T6 – 4.571T10 + T9 = − 38.571 (j)

Equations (a) to (j) form a system of 10 linear equations and are listed below:

−7.357T1 + T2 + T5 = − 1500 (a)


T1 – 7.357T2 + T6 = − 110.355 (b)
– 7.357T3 + T4 + T7 = − 110.355 (c)
T3 – 14.714T4 + 2T8 = − 3000 (d)
T1 + T4 – 10.714T5 + 2T6 + 2T9 = − 3000 (e)
T3 + 2T5 – 6.571T6 + T10 = − 38.571 (f)
T3 – 3.286T7 + T8 = − 19.286 (g)
T7 – 6.571T8 + 2T4 = − 38.571 (h)
2T5 + T8 – 6.571T9 + T10 = − 38.571 (i)
T6 – 4.571T10 + T9 = − 38.571 (j)

The above system of equations are written in matrix form as follows:

[ai,j]{Ti} = {ci}

5.6. Finite Difference Formulation For Two-dimensional Steady State Conduction in r


and z directions in Cylinders with Energy Generation

5.6.1.Finite difference equations for intermediate nodes: The cylinder is divided into small
sub regions as shown in Figure 5.13 (a).The volume element surrounding the node i,j is
shown below:

separately in Figure5.13(b). The volume element is in the form of a circular ring of thickness
Δz in the z-direction and of radial thickness Δr in the radial direction as shown in the
figure.Energy balance equation for this volume element can be written as follows:

Δz iΔr

233
Δr

jΔz

i,j+1
i+1,j
i,j
i-1,j

i, j-1

Figure5.13(a): Nomenclature for finite difference formulation for a


cylinder

Q(i,j-1)-(i,j) + Q(i+1,j)-(i,j) + Q(i,j+1)-(i,j) + Q(i-1,j)-(i,j) + Qg = 0

Ti,j−1 − Ti,j Ti+1,j − Ti,j


Or k×2π iΔr×Δr ( ) + k×2π (iΔr + Δr/2)Δz ( )
Δz Δr

Ti,j+1 − Ti,j Ti−1,j − Ti,j


+ k×2π iΔr×Δr ( ) + k×2π (iΔr − Δr/2)Δz ( ) + 2πiΔr×Δr×Δz q′′′
i,j = 0
Δz Δr

for I = 1,2,3 ……….M-1 and j = 1,2,3……N-1

Assuming Δz = Δr and simplifying we get

𝟏 𝟏 𝜟𝒓𝟐 ′′′
Ti,j-1 + Ti,j+1 + (𝟏 − ) Ti-1,j + (𝟏 + ) Ti+1,j − 4Ti,j = − 𝑞𝑖,𝑗 (5.34)
𝟐𝒊 𝟐𝒊 𝒌

for i = 1,2,3 ……….M-1 and j = 1,2,3……N-1

234
(j+1)Δz

jΔz
(j-1)Δz (i,j)
(i+1,j)

(i,j-1) (i,j+1) Δr

(i+1)Δr Δz

(i-1,j)
iΔr

Axis of the cylinder


(i-1)Δr

Figure5.13(b): Nomenclature for the volume element surrounding node i,j

5.6.2.Finite difference equations for intermediate nodes on the axis of the


cylinder:Figure5.13(c) shows a volume element surrounding the node (0,j) on the axis of the
cylinder .

(1,j)

Δz
Δr
Axis of cylinder

(0, j-1) (0,j) (0,j+1)


Δz

Figure5.13(c): Nomenclature for the volume element surrounding an


intermediate node (0,j) on the axis of the cylinder
Energy balance equation for the volume element can be written as follows:

Q(0,j-1) – (0,j) + Q(1,j) – (0,j) + Q(0,j+1) – (0,j)+ Qg = 0 for j = 1,2,3,………,N-1

T0,j−1 – T0,j T1,j – T0,j T0,j+1 – T0,j


k π (Δr/2)2 ( ) + k× 2π (Δr/2) Δz ( ) + k π (Δr/2)2 ( )
Δz Δr Δz

235
′′′
+ π (Δr/2)2Δz 𝑞0,𝑗 =0

Assuming Δz = Δr and simplifying we get

𝜟𝒓𝟐
𝐓𝟎,𝐣−𝟏 + 𝐓𝟎,𝐣+𝟏 + 4𝐓𝟏,𝐣 − 6𝐓𝟎,𝐣 = − ( ) 𝒒′′′𝟎,𝒋 j = 1,2,3……N-1 (5.35)
𝒌

5.6.3.Finite difference equations for intermediate nodes on the outer curved surface of the
cylinder subjected to convection boundary condition: Figure5.13(d) shows a volume
element surrounding the node (M,j) on the outer surface of the cylinder subjected to the
convection boundary condition.Energy balance equation for this volume element can be
written as follows:

hM, T∞
jΔz

M,j-1 M,j M,j+1

Δr/2

Δz Δr

M-1,j

MΔr Outer surface of the cylinder

Axis of the cylinder

Figure5.13(d): Nomenclature for the volume element surrounding node (M,j)

Q(M,j-1) – (M,j) + Q(M,j+ 1) – (M,j) + Q(M-1,j) – (M,j) + Qg + Qconvection = 0 for j = 1,2,3,…N-1

𝛥𝑟 2 𝑇𝑀,𝑗−1 − 𝑇𝑀,𝑗 𝛥𝑟 2 𝑇𝑀,𝑗+1 − 𝑇𝑀,𝑗


kπ [(𝑀𝛥𝑟)2 – (𝑀𝛥𝑟 − ) ]( ) + kπ [(𝑀𝛥𝑟)2 – (𝑀𝛥𝑟 − ) ]( )
2 𝛥𝑧 2 𝛥𝑧

236
𝛥𝑟 𝑇𝑀−1,𝑗 − 𝑇𝑀,𝑗 𝛥𝑟 2 ′′′
+ k×2π (𝑀𝛥𝑟 − )Δz ( ) + [(𝑀𝛥𝑟)2 – (𝑀𝛥𝑟 − ) ] Δz𝑞𝑀,𝑗
2 𝛥𝑟 2

+ 2π M Δr Δz hM (T∞ − TM,j) = 0

Assuming Δz = Δr and simplifying we get

𝟑𝑴−𝟏 𝟐𝒉𝑴 𝜟𝒓 𝜟𝒓𝟐 𝟐𝒉𝑴 𝜟𝒓 𝑻∞


TM,j-1 + TM,j+1 – [4(𝟒𝑴−𝟏) + ] TM,j = − 𝐪′′′
𝐌,𝐣 −
𝒌 𝒌 𝒌

j = 1,2,3,…N-1 (5.36)

5.6.4.Finite difference equations for intermediate nodes on the outer curved surface of the
cylinder subjected to prescribed heat flux condition: Figure5.13(e) shows a volume element
surrounding the node (M,j) on the outer surface of the cylinder subjected to the convection
boundary condition.Energy balance equation for this volume element can be written as
follows:
(qs)M,j

jΔz

M,j-1 M,j M,j+1

Δr/2

Δz Δr

M-1,j

MΔr Outer surface of the cylinder

Axis of the cylinder

Figure5.13(e): Nomenclature for the volume element surrounding node (M,j)

Q(M,j-1) – (M,j) + Q(M,j+ 1) – (M,j) + Q(M-1,j) – (M,j) + Qg + Qs = 0 for j = 1,2,3,…N-1

𝛥𝑟 2 𝑇𝑀,𝑗−1 − 𝑇𝑀,𝑗 𝛥𝑟 2 𝑇𝑀,𝑗+1 − 𝑇𝑀,𝑗


kπ [(𝑀𝛥𝑟)2 – (𝑀𝛥𝑟 − ) ]( ) + kπ [(𝑀𝛥𝑟)2 – (𝑀𝛥𝑟 − ) ]( )
2 𝛥𝑧 2 𝛥𝑧

237
𝛥𝑟 𝑇𝑀−1,𝑗 − 𝑇𝑀,𝑗 𝛥𝑟 2 ′′′
+ k×2π (𝑀𝛥𝑟 − )Δz ( ) + [(𝑀𝛥𝑟)2 – (𝑀𝛥𝑟 − ) ] Δz𝑞𝑀,𝑗
2 𝛥𝑟 2

+ 2π M Δr Δz (qs)M,j = 0

Assuming Δz = Δr and simplifying we get

𝟑𝑴−𝟏 𝜟𝒓𝟐 𝟐(qs )M,j Δr


TM,j-1 + TM,j+1 – 4(𝟒𝑴−𝟏) TM,j = − 𝐪′′′
𝐌,𝐣 −
𝒌 𝒌

j = 1,2,3,…N-1 (5.37)

5.6.5.Finite difference equations for intermediate nodes on the outer flat surface of the
cylinder subjected to convection boundary condition: Figure5.13(f) shows a volume
element surrounding the node (i,0) on the flat surface at z = 0 of the cylinder subjected to the
convection boundary condition. Energy balance equation for this volume element can be
written as follows

i+1,0

h0,T∞ Δr

i,0 i,1

i-1,0 iΔr

Δz

Figure 5.13(f): Volume element surrounding the node i-1,0

Q(i+1, 0) – (i,0) + Q(i,1) – (i,0) + Q(i-1,0) - (i,0) + Qconvection + Qg = 0 for i = 1,2,3,……,M-1

Δr Δz Ti+1,0 − Ti,0) Δr 2 Δr 2 Ti,1 − Ti,0


k×2π (iΔr + )(2)( ) + k π [(iΔr + ) − (iΔr − ) ]( )
2 Δr 2 2 Δz

238
Δr Δz Ti−1,0 − Ti,0) Δr 2 Δr 2
+ k×2π (iΔr − )(2)( ) + h0 π [(iΔr + ) − (iΔr − ) ] (T∞ − Ti,0)
2 Δr 2 2

Δr 2 Δr 2 Δz
+ π [(iΔr + ) − (iΔr − ) ] ( 2 ) q′′′
i,0 = 0
2 2

Assuming Δz = Δr and simplifying we get

𝟏 𝟏 𝟐𝐡𝟎 𝚫𝐫 𝟐𝐡𝟎 𝚫𝐫 𝚫𝐫 𝟐
(𝟏 + ) Ti+1,0 + (𝟏 − ) Ti−1,0 + 2 Ti,1 – [2 + ] Ti,0 = − T∞ − q′′′
i,0
𝟐𝐢 𝟐𝐢 𝐤 𝐤 𝟐𝐤

i = 1,2,3,……………M-1 (5.38)

Similarly the energy balance equation for the volume element surrounding the node i,N [See
Figure5.13(g)] can be written as follows:

𝟏 𝟏 𝟐𝐡𝟎 𝚫𝐫 𝟐𝐡𝐍 𝚫𝐫 𝚫𝐫 𝟐
(𝟏 + ) Ti+1,N + (𝟏 − ) Ti−1,N + 2 Ti,N-1 – [2 + ] Ti,N = − T∞ − q′′′
i,N
𝟐𝐢 𝟐𝐢 𝐤 𝐤 𝟐𝐤

i = 1,2,3,……………M-1 (5.39)

i+1,N

Δr

hN,T∞
i,N-1 i,N

i-1,N iΔr

Δz

Figure 5.13(g): Volume element surrounding the node i-1,N

5.6.6.Finite difference equations for volume elements surrounding the nodes (0,0) and
(0,N): The nomenclature for the volume elements surrounding the nodes (0,0) and (0,N) are
shown in Figure 5.13(h). The energy balance equation for the volume element surrounding
the node (0,0) can be written as:

239
Δz h0,T∞ hN,T∞ Δz
(1,0) (1,N)

Δr Δr

(0,0) (0,1) (0,N-1) (0,N)

Axis of symmetry

Figure5.13(k): Nomenclature for volume elements surrounding nodes (0,0) and (0,N)

Q(1,0) – (0,0) + Q(0,1) – (0,0) + Qconvection + Qg = 0

𝑇1,0 − 𝑇0,0 𝑇0,1 − 𝑇0,0


k×2π (Δr/2)(Δz/2) ( ) + k× π(Δr/2)2( ) + h0 π(Δr/2)2[T∞ − T0,0]
𝛥𝑟 𝛥𝑧

′′′
+ π(Δr/2)2(Δz/2)𝑞0,0 =0

Assuming Δz = Δr and simplifying we get

2T1,0 + T0,1 – [4 + (h0Δr/k)] T0,0 = − (h0Δr/k)] T∞ − (Δr2/2k) 𝒒′′′


𝟎,𝟎 (5.40)

Similarly it can be shown that the energy balance equation for the volume element
surrounding the node (0,N) is given by

2T1,N + T0,N-1 – [4 + (hNΔr/k)]T0,N = − (hNΔr/k)] T∞ − (Δr2/2k) 𝒒′′′


𝟎,𝑵 (5.41)

5.6.7.Finite difference equations for volume elements surrounding the nodes (M,0) and
(M,N):
The nomenclature for the volume elements surrounding the nodes (M,0) and (M,N) are
shown in Figure 5.13(k). The energy balance equation for the volume element surrounding
the node (M,0) can be written as:

hM,T∞

240
(M,0) (M,1) (M,N-1) (M,N)

Δr Δr

Δz Δz

(M-1,0) MΔr (M-1,N)

Figure5.13(k): Nomenclature for volume elements surrounding nodes (M,0) and (M,N)

Q(M,1) – (M,0) + Q(M-1,0) – (M,0) + Qconvection + Qg = 0

𝑇𝑀,1 − 𝑇𝑀,0 𝑇𝑀−1,0 − 𝑇𝑀,0


kπ [(MΔr)2 – (MΔr – Δr/2)2] ( ) + k 2π (MΔr – Δr/2)(Δz/2)( )
𝛥𝑧 𝛥𝑟

+ hM {π[(MΔr)2 – (MΔr – Δr/2)2] + 2πMΔr(Δz/2)}(T∞− TM,0)


′′′
+ π[(MΔr)2 – (MΔr – Δr/2)2] (Δz/2)𝑞𝑀,0 =0

Assuming Δz = Δr and simplifying we get

1 1
(2 − ) M(Δr/2) [TM,1 – TM,0] + (1 − ) MΔr [TM-1,0 – TM,0]
2𝑀 2𝑀

1 1 𝛥𝑟 2
′′′
+ hMMΔr (Δr/k) [2 − 4𝑀 ] [T∞ − TM,0] + (2 − )MΔr ( 4𝑘 ) 𝑞𝑀,0 =0
2𝑀

𝟏 𝟏 𝟏 𝒉𝑴 𝜟𝒓 𝟏
Or (𝟏 −
𝟒𝑴
)TM,1 + (𝟏 − 𝟐𝑴) TM-1,0 – {(𝟏 − 𝟐𝑴) + 𝒌
(𝟐 − 𝟒𝑴
)}TM,0

𝟏 𝒉𝑴 𝜟𝒓 𝟏 𝟏 𝜟𝒓𝟐
= {(𝟏 − )+ (𝟐 − )}T∞ − (𝟐 − ) 𝒒′′′
𝑴,𝟎 (5.42)
𝟐𝑴 𝒌 𝟒𝑴 𝟐𝑴 𝟒𝒌

Similarly for the volume element surrounding the node (M,N) the energy balance equation
reduces to

𝟏 𝟏 𝟏 𝒉𝑴 𝜟𝒓 𝟏
(𝟏 − 𝟒𝑴)TM,N-1 + (𝟏 − 𝟐𝑴) TM-1,N – {(𝟏 − 𝟐𝑴) + 𝒌
(𝟐 − 𝟒𝑴
)}TM,N

241
𝟏 𝒉𝑴 𝜟𝒓 𝟏 𝟏 𝜟𝒓𝟐
= {(𝟏 − )+ (𝟐 − )}T∞ − (𝟐 − ) 𝒒′′′
𝑴,𝟎 (5.43)
𝟐𝑴 𝒌 𝟒𝑴 𝟐𝑴 𝟒𝒌

5.7. Finite Difference Formulation For Two-dimensional Steady State Conduction in r


and θ directions in Cylinders with Energy Generation

5.7.1.Finite difference equations for intermediate nodes: The cylinder is divided into small
sub regions and the volume element surrounding the node i,j will be as shown in Figure5.14.

(i,j+1)

(i+1,j)

(i,j)
(i-1,j)

(i, j-1)
Δθ
Δθ

iΔr Δr

Figure5.14: Nomenclature for the volume element surrounding an intermediate node


(i,j)

Energy balance equation for the volume element surrounding the node (i,j) can be written as
follows:

Q(i-1,j) – (i,j) + Q(i+1,j) – (i,j) + Q(i,j+1) – (i,j) + Q(i,j-1) – (i,j) + (𝑄𝑔′′′ ) =0


(𝑖,𝑗)

for i = 1,2,3…. M and j = 1,2,3,…….N

𝛥𝑟 𝑇𝑖−1,𝑗 − 𝑇𝑖,𝑗 𝛥𝑟 𝑇𝑖+1,𝑗 − 𝑇𝑖,𝑗 𝑇𝑖,𝑗−1 − 𝑇𝑖,𝑗


Or k [(iΔr − 2
)Δθ×1] ( 𝛥𝑟
) + k (iΔr + 2
)Δθ ( 𝛥𝑟
) + k [Δr×1]( 𝑖𝛥𝑟 𝛥𝜃
)

242
𝑇𝑖,𝑗+1 − 𝑇𝑖,𝑗
+ k [Δr×1]( ) + (iΔrΔθ×Δr×1) (𝑞𝑔′′′ ) =0
𝑖𝛥𝑟 𝛥𝜃 (𝑖,𝑗)

𝟏 𝟏 𝟏 𝟏
Or [1 − 𝟐𝒊 ] Ti−1,j + [1 + 𝟐𝒊 ] Ti+1,j + (𝒊𝜟𝜽)𝟐 Ti,j-1 + (𝒊𝜟𝜽)𝟐 Ti,j+1

𝟐 𝜟𝒓𝟐
− [2 + (𝒊𝜟𝜽)𝟐 ] Ti,j = − (𝒒′′′
𝒈 )
𝒌 (𝒊,𝒋)

for i = 1,2,3…. M and j = 1,2,3,…….N (5.44)

5.7.2 Finite difference equations for innermost node (0,0): The volume element
surrounding the innermost node (0,0) will be as shown in Figure5.15.Energy balance
equation

Δθ
(1,j)

(0,0)

Δr

Figure5.15: Nomenclature for volume element surrounding the node (0,0)

for the volume element surrounding the node (0,0) can be written as follows:

Q(1,j) -(1,j) + (Q′′′


g ) =0 for j = 1,2,3, …..N-1
(0,0)

𝛥𝑟 𝑇1,𝑗 − 𝑇0,0 𝛥𝑟 𝛥𝑟
Or k ( 2 𝛥𝜃 ∗ 1)) ( )+ ( 2 𝛥𝜃 ∗ ∗ 1) (q′′′
g ) = 0 for j = 1,2,3, …..N-1
𝛥𝑟 2 (0,0)

𝜟𝒓𝟐
Or T1,j – T0,0 = − (𝐪′′′
𝐠 ) for j = 1,2,3, …..N-1 (5.45)
𝟐𝒌 (𝟎,𝟎)

243
5.7.3.Finite difference equations for nodes on the outer surface with convective boundary
condition: The volume element surrounding a node (M,j) on the outer surface of the cylinder
will be as shown in Figure5.16.Energy balance equation for this element can be written as
follows:

(M,j+1)
h,T∞

(M,j)
(M-1,j)

Δθ
Δθ

(M,j-1)
MΔr

Figure5.16: Nomenclature for volume element surrounding the node (M,j)

Q(M-1,j) – (M,j) + Q(M,j+1) – (M,j) + Q(M,j−1) – (M,j) + Qconvection + (Qg)M,j = 0 for j = 1,2,3,….N

𝛥𝑟 𝑇𝑀−1,𝑗 − 𝑇𝑀,𝑗 𝛥𝑟 𝑇𝑀,𝑗−1 − 𝑇𝑀,𝑗 𝛥𝑟


Or k { (MΔr − )Δθ×1}( ) + k ( 2 × 1) ( ) + k ( 2 × 1)
2 𝛥𝑟 𝑀𝛥𝑟𝛥𝜃
𝑇𝑀,𝑗+1 − 𝑇𝑀,𝑗
( )
𝑀𝛥𝑟𝛥𝜃

𝛥𝑟 𝛥𝑟
+ h (MΔrΔθ×1)(T∞ − TM,j) + (MΔr − )Δθ× ×1 (q′′′
g ) = 0 for j
4 2 (M,j)
=1,2,3,….N

Simplifying we get

𝟏 𝐓𝐌,𝐣−𝟏 𝐓𝐌,𝐣+ 𝟏 𝟏 𝟏 𝐡𝚫𝐫


(𝟏 − )TM-1,j + (𝟐(𝐌𝚫𝛉)𝟐 ) + (𝟐(𝐌𝚫𝛉)𝟐 ) − [(𝟏 − )+ (𝐌𝚫𝛉)𝟐
+( ) ] TM,j
𝟐𝐌 𝟐𝐌 𝐤

𝐡𝚫𝐫 𝟏 𝚫𝐫 𝟐
=−( ) T∞ − (𝟏 − )( ) (𝐪′′′
𝐠 ) for j=1,2,3,..N (5.46)
𝐤 𝟒𝐌 𝟐 (𝐌,𝐣)

Example 5.9.A a heating element in the form of a short solid cylinder of radius 2 cm and
length 6 cm is generating heat at a uniform rate of 2×106 W/m3. The thermal conductivity of
the heating element may be taken as 50 W/(m-K). The element is exposed to the surroundings
at a uniform temperature of 300C with a surface heat transfer coefficient of 200 W/(m2-

244
k).Assuming two dimensional steady state conduction in r and z direction (i) obtain the finite
difference equations to determine the steady state temperature in the cylinder assuming Δr =
Δz = 1 cm (ii) solve the system of equations thus obtained for temperature distribution in the
cylinder.Assume that the circular faces of the cylinder are insulated.

Sketch for example5.9.


r
Symmetry lines
8 9 10 11
h,T∞

4 5 6 7 R Insulated
Δr
0 1 2 3 z

Δz

Known: L = 6 cm; R = 2 cm; k = 50 W∙m-1∙K-1; h = 200 W∙m-2∙K-1; Δr = ΔL = 1 cm


′′′
𝑞𝑖,𝑗 = 2×106 W∙m-3; T∞ = 30 0C.

Find: (i) Finite difference equations to determine the temperatures at discrete points in the
cylinder;(ii Temperatures from the finite difference equations thus formed

Assumptions: (i) two dimensional steady state conduction in r and z directions only
(ii)Thermal conductivity and the heat generation are constants.(iii) Heat transfer coefficients
from all the outer surfaces (the curved surface and the flat surface)are same.

Solution: Since there are two symmetry lines as shown in the figure only one fourth of the
cylinder need be considered for finite difference formulation .Since Δr = Δz = 1 cm there will
be 12 nodes as shown in the figure above.

Finite difference Equations for node ‘0’:

Lines of symmetry (Insulated surfaces)


4

245
Δr

0 1
Δz

The nomenclature for the volume element surrounding node 0 is shown in the figure
above.Energy balance equation for this volume element can be written as

Q4-0 + Q1-0 + Qg = 0

𝑇4 − 𝑇0 𝑇1 − 𝑇0 ′′′
Or k 2π (Δr/2)(Δz/2) ( ) + k π (Δr/2)2( ) + π (Δr/2)2)(Δz/2) (𝑞𝑔 ) = 0
𝛥𝑟 𝛥𝑟 0

Putting Δz = Δr and simplifying we get

𝛥𝑟 2 ′′′
4(T4 – T0) + 2(T1 – T0) = − (𝑞𝑔 )
𝑘 0
′′′
Substituting for Δr,k and (𝑞𝑔 ) we get
0

0.012
− 6T0 + 2T1 + 4T4 = − ×2×10 6
50

Or − 3T0 + T1 + 2T4 = − 2 (1)

Finite difference Equations for node ‘3’: The volume element surrounding the node 3 is
shown in figure below.Energy balance equation for the volume element surrounding node 3
can be written as

7 Insulated
Axis of symmetry
Δr

2 3

Δz

Q7-3 + Q2-3 + Qg = 0

𝑇7 − 𝑇3 𝑇2 − 𝑇3 ′′′
Or k 2π (Δr/2)(Δz/2) ( ) + k π (Δr/2)2( ) + π (Δr/2)2)(Δz/2) (𝑞𝑔 ) = 0
𝛥𝑟 𝛥𝑟 3

Putting Δz = Δr and simplifying we get

T2 – 3T3 + 2T7 = − 2 (2)

246
Finite Difference Equations for nodes 1 and 2: The volume elements surrounding nodes 1
and 3 are shown below.

5 6
Δz
Δr

0 2 1 1 2 3
Axis of symmetry
Energy balance equation surrounding the node 1 can be written as

Q0-1 + Q5-1 + Q2-1 + Qg = 0

𝑇0 − 𝑇1 𝑇5 − 𝑇1 𝑇2 − 𝑇1 ′′′
k π(Δr/2)2( ) + k 2π (Δr/2)Δz ( ) + k π(Δr/2)2( ) + π (Δr/2)2Δz (𝑞𝑔 ) =0
𝛥𝑧 𝛥𝑟 𝛥𝑧 1

Substituting Δz = Δr and simplifying we have

𝛥𝑟 2 ′′′
T0 – 6T1 + T2 + 4T5 = − (𝑞𝑔 )
𝑘 1
′′′
Substituting the values for Δr,k and (𝑞𝑔 ) we get
0

0.012
T0 – 6T1 + T2 + 4T5 = − × 2× 106
50

Or T0 – 6T1 + T2 + 4T5 = − 4 (3)

Energy equation for the element surrounding node 2 is

Q1-2 + Q6-2 + Q3-2 + Qg = 0

𝑇1 − 𝑇2 𝑇6 − 𝑇2 𝑇3 − 𝑇2
k π(Δr/2)2( ) + + k 2π (Δr/2)Δz ( ) + k π(Δr/2)2( )
𝛥𝑧 𝛥𝑟 𝛥𝑧

′′′
+ π (Δr/2)2Δz (𝑞𝑔 ) =0
2

Substituting Δz = Δr and simplifying we have

𝛥𝑟 2 ′′′
T1 + 4T6 + T3 – 6 T2 = − (𝑞𝑔 )
𝑘 2
′′′
Substituting the values for Δr,k, and (𝑞𝑔 ) we get
0

0.012
T1 + 4T6 + T3 – 6T2 = − × 2× 106
50

Or T1 – 6T2 + T3 + 4T6 = −4 (4)

247
Finite Difference Equations for node 4: The volume element surrounding the node 4 is
shown in the figure below.Energy balance equation for the element is

Q0-4 + Q8-4 + Q5-4 + Qg = 0

𝑇0 − 𝑇4 𝑇8 − 𝑇4
Or k 2π (Δr/2) (Δz/2) ( ) + k 2π (Δr +Δr/2) (Δz/2) ( ) +
𝛥𝑟 𝛥𝑟

𝛥𝑟 2 𝛥𝑟 2 𝑻𝟓 − 𝑻𝟒
k π [(𝛥𝑟 + ) − (𝛥𝑟 − ) ]( )
2 2 𝜟𝒛

𝛥𝑟 2 𝛥𝑟 2 ′′′
+ π [(𝛥𝑟 + ) − (𝛥𝑟 − ) ](Δz/2) (𝑞𝑔 ) =0
2 2 4

Substituting Δz = Δr and simplifying we have

Symmetry line (Insulated boundary)

4 5

Δr

Δz
0

2 𝛥𝑟 2 ′′′
T0 – 8T4 + 3T8 = − (𝑞𝑔 )
𝑘 4

′′′
Substituting the values for Δr,k, and (𝑞𝑔 ) we get
0

T0 – 8T4 + 3T8 = − 8 (5)

Finite Difference Equations for nodes 5 and 6: Both these nodes are internal nodes and
hence the heat transfer across the boundary surfaces of these nodes are only by
conduction.The volume elements surrounding node 5 is shown below.

248
5

4 6

Δr
Δz
1
Figure: Volume element surrounding node 5

Energy balance equation for the volume element surrounding node 5 can be written as

Q4-5 + Q9-5 + Q6-5 + Q1-5 = 0

𝛥𝑟 2 𝛥𝑟 2 𝑇4 −𝑇5 𝛥𝑟 𝑇9 −𝑇5
Or k π[(𝛥𝑟 + ) − (2) ]( ) + k 2π (𝛥𝑟 + )Δz( )
2 𝛥𝑧 2 𝛥𝑟

𝛥𝑟 2 𝛥𝑟 2 𝑇6 −𝑇5 𝛥𝑟 𝑇1 −𝑇5
+ k π[(𝛥𝑟 + ) − (2) ]( ) + k 2π ( )Δz( )
2 𝛥𝑧 2 𝛥𝑟

𝛥𝑟 2 𝛥𝑟 2 ′′′
+ π[(𝛥𝑟 + ) − ( 2 ) ] 𝛥𝑧(𝑞𝑔 ) =0
2 5

Substituting Δz = Δr and simplifying we get

2 𝛥𝑟 2 ′′′
T1 + 2T4 − 8T5 + 2T6 + 3T9 = − (𝑞𝑔 )
𝑘 5

′′′
Substituting the values for Δr,k, and (𝑞𝑔 ) and simplifying we get
0

T1 + 2T4 − 8T5 + 2T6 + 3T9 = − 8 (6)

Similarly the energy balance equation for node 6 can be obtained as

T2 + 2T5 − 8T6 + 2T7 + 3T10 = − 8 (7)

Finite Difference Equations for node 7: The volume element surrounding node 7 is shown
below.The energy balance equation is

11

Δr Insulated

249
6 7

𝜟𝒛
3

Figure Volume element surrounding node 7

Q6-7 + Q11-7 + Q3-7 + (Qg)7 = 0

𝛥𝑟 2 𝛥𝑟 2 𝑇6 −𝑇7 𝑇11 − 𝑇7
Or k π[(𝛥𝑟 + ) − (2) ]( ) + k 2π (Δr +Δr/2) (Δz/2) ( ) +
2 𝛥𝑧 𝛥𝑟

𝑇3 − 𝑇7
k 2π (Δr/2) (Δz/2) ( )+
𝛥𝑟
𝛥𝑟 2 𝛥𝑟 2 ′′′
+ π[(𝛥𝑟 + ) − ( 2 ) ](Δz/2) (𝑞𝑔 ) =0
2 7

Substituting Δz = Δr and simplifying we get

2 𝛥𝑟 2 ′′′
T3 + 4T6 – [8] T7 + 3T11 = − (𝑞𝑔 )
𝑘 7

′′′
Substituting the values for Δr,k,h,T∞ and (𝑞𝑔 ) and simplifying we get
7

T3 + 4T6 – 8T7 + 3T11 = − 8 (8)

Finite Difference Equations for node 8: The volume element surrounding the node 8 is
shown in the figure below. Energy balance equation for the element is given by

Q4-8 + Q9-8 + Qconvection + (Qg)8 = 0

Or
𝑇4 − 𝑇8 3𝛥𝑟 2 𝑇9 − 𝑇8
k 2π (Δr+ Δr/2)Δz/2)( ) + k π ((2𝛥𝑟) 2 − { } )( )+
𝛥𝑟 2 𝛥𝑧

3𝛥𝑟 2 ′′′
h{2π 2Δr (Δz/2)}(T∞ −T8) + π ((2𝛥𝑟)2 − { } ) (Δz/2) (𝑞𝑔 ) =0
2 8

8 h,T∞ 9

250
Δr

Δz
Symmetry line

Δr

Figure Volume element surrounding node 8

Substituting Δz = Δr and simplifying we get

7 𝛥𝑟 2 ′′′
6 T4 – [13 + 8 (hΔr/k)]T8 + 7T9 = − 8 (hΔr/k)T∞ − (𝑞𝑔 )
𝑘 8

′′′
Substituting the values for Δr,k,h,T∞ and (𝑞𝑔 ) and simplifying we get
8

6T4 – 13.32 T8 + 7 T9 = − 37.6 (9)

Finite Difference Equations for nodes 9 and 10 : The volume element surrounding node 9
is shown in figure below.Energy balance equation for the volume element shown is

h,T∞

8 9 10

Δr
Δz
5
Axis of cylinder

Δr

Q8-9 + Q5-9 + Q10-9 + Qconvection + (Qg)9 = 0

Or
𝑇 −𝑇 𝑇 −𝑇
k π{(2Δr)2 – (Δr + Δr/2)2}( 8𝛥𝑧 9) + k 2π(Δr + Δr/2)Δz( 5 𝛥𝑟 9 ) +

251
𝑇10 − 𝑇9
k π{(2Δr)2 – (Δr + Δr/2)2}( ) + h2π(Δr + Δr/2)Δz (T∞ − T9)
𝛥𝑧

′′′
+ π{(2Δr)2 – (Δr + Δr/2)2}Δz (𝑞𝑔 ) =0
9

Substituting Δz = Δr and simplifying we get


′′′
12 T5 + 7T8 – [26 + (12hΔr/k)]T9 + 7T10 = − (12hΔr/k)T∞ − 7 (Δr2/k) (𝑞𝑔 )
9
′′′
Substituting the values for Δr,k,h,T∞ and (𝑞𝑔 ) and simplifying we get
8

12 T5 + 7T8 – 26.48 T9 + 7T10 = − 42.4 (10)

The volume element surrounding node 10 is shown in the figure below.

h,T∞

9 10 11

Δr
Δz
6
Axis of cylinder

Δr

It can be shown that the energy balance equation for this volume element will be

12 T6 + 7T9 – 26.48 T10 + 7T11 = − 42.4 (11)

Finite Difference Equations for node 11: The volume element surrounding node 11 is
shown in the figure below. The energy balance equation for this element is as follows :

Q10-11 + Q7-11 + Qconvection + (Qg)11 = 0

Or

𝑇10 − 𝑇11 𝑇7 − 𝑇11


k π[(Δr + Δr/2)2 – (Δr/2)2] ( ) + k 2π (Δr + Δr/2)Δz ( )
𝛥𝑧 𝛥𝑟

+ h 2π(2Δr)(Δz/2)(T∞ − T11)
′′′
+ π[(Δr + Δr/2)2 – (Δr/2)2](Δz/2) (𝑞𝑔 ) =
11

252
h,T∞
10 11 Insulated

Δr

Δz

7
Figure: Volume element surrounding node 11

Δr

Substituting Δz = Δr and simplifying we get


′′′
3T7 + 2T10 – [5 + 2hΔr/k]T11 = − 2(hΔr/k)T∞ − (Δr2/k) (𝑞𝑔 )
11

′′′
Substituting the values for Δr,k,h,T∞ and (𝑞𝑔 ) and simplifying we get
8

3T7 + 2T10 – 5.24T11 = − 6.4 (12)

Equations (1) to (12) form a system of linear equations which can be solved by using any of
the standard techniques.

5.8. Finite Difference Formulation Transient Conduction

5.8.1. Finite Difference Formulation for Transient Conduction in Solids with Negligible
Temperature Gradients:

In this section, the simplest transient conduction (lumped capacitance problem) is solved
numerically using different numerical schemes.The different schemes are illustrated and
compared for the problem where the solid and the surroundings areinitially at a uniform
temperature and suddenly there is a step change in the surroundings temperature to T∞.
If T is the temperature of the solid at any time ‘t’ , V is the volume , A is the surface area , ρ
is the density, Cp is the heat capacity of the solid and h is the surface heat transfer coefficient,
then the energy balance equation at any time t can be written as:

The rate of increase of energyof the solid = Rate of heat transfer by convection
from the surroundings to the solid. That is,

253
dT
ρVCp ( dt ) = hA[T∞ − T(t)]

dT hA
= − ρVC [T(t) − T∞ ]
dt p

dT [T(t) − T∞ ]
Or: =− (5.47a)
dt 𝜏𝑙𝑢𝑚𝑝𝑒𝑑

ρVCp
Where: τlumped = (5.47b)
hA

The initial condition is when t = 0, T = Ti (5.47c)

The analytical solution of Eq.(5.47(a)) subject to the initial condition (Eq.(5.47(c)) is given
by,
𝑡
T(t) = T∞ + [Ti - T∞] exp ( − 𝜏 ) (5.48)
𝑙𝑢𝑚𝑝𝑒𝑑

There are a number of numerical methods available to solve Eq.(5.47(a)) subject to the
initial condition Eq.(5.47c)) and some of them are illustrated as shown below.

Euler’s method:

Each numerical scheme needs that the total simulation time (tsim) is broken into small equal
time steps Δt, where
t
Δt = sim (5.49)
M
Where M is the number of time steps at which the temperatures of the solid will be
evaluated. The temperature after j th time step, Tj is computed by the numerical model of
Eq.(5.47a). After the first time step Δt can be aritten in finite difference form as follows:

T1 − Ti T − T∞
For t = 1 × Δt: = − [τ i ]
Δt lumped

Δt Δt
Or: T1 = Ti [1 − ] + (τ ) T∞ (5.50)
τlumped lumped

Δt Δt
Similarly for t = 2 Δt: T2 = T1 [1 − ] + (τ ) T∞ (5.51)
τlumped lumped

Δt Δt
And for any t = j Δt: Tj = Tj-1 [1 − ] + (τ ) T∞ ; j = 1,2,3 …M
τlumped lumped

(5.52)

Example 5.10.An aluminum sphere (ρ = 2770 kg∙m-3 and Cp = 875 J∙kg−1∙K−1) of 3 cm


diameter is initially in thermal equilibrium with the surroundings at 300 K .Suddenly the

254
temperature of the surroundings is raised and maintained at a temperature of 400 K. The
surface heat transfer coefficient is 121 W∙m−2∙K−1. Neglecting internal temperature gradients
determine using Euler’s numerical method the temperature of the sphere as a function of time
up to using a time step of (1) 250s, (ii)200 s, (iii) 145 s, (iv) 50 s and (v) 5 s. Compare the
results for different values of time step and comment on the results.

Known:d = 3 cm; ρ = 2770 kg∙m-3; Cp = 875 J∙kg−1∙K−1;Ti =300 K ;T∞ = 400 K ;

Tsim = 500 s. h = 121 W∙m−2∙K−1.

Find: T(t) at different time steps using Euler’s method

Soution: (i) when Δt = 250 s:

ρVCp ρCp 𝑑 2770 ×875 × 0.03


𝜏𝑙𝑢𝑚𝑝𝑒𝑑 = = ×6= = 100 s
hA h 121 ×6

Δt Δt
Now: T1 = Ti [1 − ] + (τ ) T∞
τlumped lumped

250 250
= 300 × [1 − ] + (100) × 400 = 550 K
100

Δt Δt
And: T2 = T1 [1 − ] + (τ ) T∞
τlumped lumped

250 250
= 550 × [1 − ] + (100) × 400 = 175 K
100

Δt Δt
T3 = T2 [1 − ] + (τ ) T∞
τlumped lumped

250 250
= 175 × [1 − ] + (100) × 400 = 737.5 K
100

Similarly we get T4 = − 106.25 K

𝑡
The exact solution is: T(t) = T∞ + [Ti - T∞] exp ( − 𝜏 )
𝑙𝑢𝑚𝑝𝑒𝑑
𝑡 100
= 400 + (300 – 400) × exp (− ) = 400 - 𝑡 (1)
100 exp( )
100

It can be seen from the results that the temperature of the solid fluctuates with time;that is the
solution is numerically unstable whenever ∆t is greater than 𝜏𝑙𝑢𝑚𝑝𝑒𝑑 .
Following this procedure the temperature of the solid as a function of time is calculated
using different time time steps and the results are tabulated as shown.The exact solution is
also indicated in the table.

255
# Δt Ti(K) T1(K) T2(K) T3(K) T4(K) T5(K) T6(K) T7(K) T8(K) T9(K) T10(K)
(s)
1 250 300 550 175 737.5 −106.25 Solution is unstable
2 200 300 500 300 500 300 Solution is unstable
3 150 300 450 375 412.5 −18.75 Solution is unstable
4 100 300 400 400 400 400 400 400 400 400 400 400
5 50 300 350 375 387.5 393.75 396.75 398.375 399.2 399.6 399.8 399.9
6 10 300 310 319 327.1 334.39 340.95 346.86 352.17 356.95 361.26 365.13
Exact t 20 s 40 s 60 s 80 s 100 s 150 s 200 s 250 s 300 s 400 s
solution
[Eq.(1) 300 318.13 332.97 345.12 355.07 363.21

256
257
5.8.2. Finite Difference Formulation for One-dimensionalTransient Conduction.

In steady state conduction problems the solution obtained is valid for any time since under
steady state conditions, the temperatures in the solid do not change with time. But in transient
conduction problems the temperatures change with time as well as with position. Therefore the
finite difference solution of transient problems requires discretization in time in addition to
discretization in space as shown in Figure5.17 for one dimensional transient conduction
problem.The discretization in time domain is done by selecting a suitable time step Δt and
solving for unknown nodal temperatures for each time step Δt until the solution at the desired
time is obtained.In a transient conduction problem a superscript (say ‘k’) is used to indicate
the index or counter of time steps with k = 0 corresponding to the specified initial condition.
Thus Tik represents the temperature at the node ‘i’ after time steps kΔt have elapsed.The energy
balance equation for transient conduction involves one additional term representing the change
in the energy content of the solid with time, Hence this equation for a volume element for time
duration of Δt can be written as follows:

{Net amount of heat transfer into the volume element across all its surfaces during time interval
Δt} + {Heat generated within the element during time interval Δt}

= {The amount of increase of the energy content of the element during the time
interval Δt}.

Or Δt × ∑Q + Δt ×[Qg]element = ΔEelement (5.44)


All sides

Where the rate of heat transfer Q normally consists of conduction terms for interior nodes,
but may involve convection, surface heat flux and radiation for boundary nodes.
Noting that ΔEelement = ρ ΔVCp ΔT, where ρ is the density,ΔV is the volume and Cp is the
specific heat of the element and dividing Equation (5.44) by Δt we have
𝛥𝑇
∑Q + [Qg]element = ρΔV Cp (5.45)
𝛥𝑡
All sides

Equation(5.45) is applicable to volume element surrounding any node ‘i’ in the domain.ΔT
represents the change in the temperature of the volume element surrounding the node i for a
time step of Δt and this can be written as [Tik+1 – Tik]. Hence Equation (5.45) reduces to

(Tk+1
i − Tk
i)
∑Q + [Qg]element = ρΔV Cp (5.45)
Δt
All sides

258
k +1 k+1 k+1
T Ti-1 Ti Ti+1

k +1
Δt
k

k–1 k
Ti+1
k
Ti k
Ti-1

Δt Δx Δx
x
0 1 2 i–1 i i +1

Figure 5.17: Finite difference formulation of transient one dimensional conduction


problem

The term ∑Q involves temperatures at adjacent nodes i – 1 and i +1 and question arises as
All sides

whether to use these temperatures at previous time step ‘k’ or the new time step ‘k+1’. Both
these approaches are used in practice. The approach using the previous time step is called the
explicit method where as the method using the new time step is called the implicit method
Therefore Equation(5.45) takes the following forms for the two methods:

(Tk+1
i − Tk
i)
ExplicitMethod: ∑Qi + [Qg]ielement = ρΔV Cp (5.46)
Δt
All sides

(Tk+1
i − Tk
i)
Implicit Method: ∑Qi+1 + [Qg]i+1element = ρΔV Cp (5.47)
Δt
All sides

The explicit and implicit formulations given in Eqs.(5.46) and (5.47) are quite general and can
be used in any coordinate system and is applicable to multi dimensional transient conduction
problems also except that the volume element in multi dimensional problems will have more
surfaces and thus involve more terms in the summation.
The explicit method is easy to implement but imposes a limit on the allowable time step
in order to avoid instabilities in the solution and the implicit method requires the nodal
temperatures to be solved simultaneously for each time step but imposes no limit on the
magnitude of the .time step

259
5.8.3.Explicit Method for Transient conduction:

(a) Explicit Method for One-dimensional Transient conduction in a Plane Wall:


Consider unsteady one dimensional conduction in a plane wall of thickness L with heat
generation per unit volume equal to 𝑞𝑔′′′ (x,t). In generalthe heat generation may wary with
time and position. Let the conductivity of the wall be a constant equal to k. The wall is
subdivided into M small regions so that the thickness of each such small region is Δx = L/M.For
any internal small regions shown in Figure5 18 the energy balance equation per unit time can
be written as follows:

(Q′′′
g ) i

Qi-1,I Qi+1,i

x
0 1 2 3 i-1 i i+1 M-1 M

Figure 5.18: Nomenclature for the volume element surrounding an internal


node i

ΔE
Qi-1,i+ Qi+1,i + (Qg) i = Δt

Tk k
i−1 − Ti Tk k
i+1 − Ti Tk+1 − Tk
Or k AΔx ( ) + k AΔx ( ) + AΔx q′′′
g = ρAΔxCp (
i i
)
Δx Δx Δt

Tk+1 − Tk Tk k k
i−1 + Ti+1 −2Ti
( i
Δt
i
)=α( (Δx)2
)

The above equation simplifies to

𝐪′′′
𝐠 𝚫𝐱
𝟐
𝐓𝐢𝐤+𝟏 = τ (𝐓𝐢−𝟏
𝐤 𝐤
+ 𝐓𝐢+𝟏 ) + (𝟏 − 𝟐𝛕)𝐓𝐢𝐤 + τ ; i = 1,2,3,…..M-1 (5.48)
𝐤

𝛂𝚫𝐭
Where τ = 𝚫𝐱𝟐 (5.49)

260
τ is the Fourier number for the element under consideration with characteristic length Δx.

Equation (5.48) is called the explicit scheme because the temperature at any node i at the
present time step (k+1) is explicitly expressed in terms of the temperatures at the previous
time step k.

Restriction on τ (Stability Criterion): The explicit method, though easy to use has an
undesirable feature which restricts its utility. The method is not unconditionally stable and
there is a restriction on the value of the parameter τ if the scheme has to be stable.The
restriction is that in Equation(5.48) (𝟏 − 𝟐𝝉) ≥ 0 i.e.

0<τ≤ ½ (5.50)

This restriction implies that for given values of α and Δx, the time step Δt cannot exceed the
limit imposed on it by Equation (5.50). This is called the stability criterion. It can be shown
mathematically or by invoking second law of thermodynamics that the stability criterion is
satisfied if the coefficients of all 𝑇𝑖𝑘 in the expressions for 𝑇𝑖𝑘+1 are greater than or equal to
zero for all nodes.If this stability criterion is violated then the solution becomes unstable as
shown in Figure5.19.It is instructive to determine this stability criterion by the following
physical argument.
𝑘 𝑘
Let at any time step k the temperatures 𝑇𝑖−1 and 𝑇𝑖+1 at nodes i-1 and i+1 are equal but
𝑘
less than 𝑇𝑖 at node i between them. Then if the value of τ exceeds ½, the coefficient (1 − 2𝜏)
becomes negative. Then, according to Equation(5.48),for negative value of (1 − 2𝜏), the
temperature 𝑇𝑖𝑘+1 at node i at the next time step will be less than that at the neighbouring two
nodes. This is not possible according to the second law of thermodynamics, since we had
assumed that 𝑇𝑖𝑘 higher than that at neighboring nodes. Hence to obtain meaningful solution it
is necessary that (𝟏 − 𝟐𝝉) should not be negative.

Finite difference solution with τ > ½

Finite difference solution with τ ≤ ½

Exact solution

Figure 5.19: Effects of the parameter τ on stability


of
finite difference solution

Finite Difference Equations for Nodes on Boundaries: Equation (5.48) can be used for all
internal nodes (i = 1,2,3,……………M-1).If the boundary temperatures are specified then we

261
have M-1 equations to solve for the temperatures at M-1 nodes.If the boundaries are subjected
to convection or prescribed surface heat flux then we have to develop the finite difference
equations for the boundary nodes also to determine the boundary temperatures.The time
dependent energy balance equation for the volume element surrounding the node 0 (see
Figure5.20) subjected to convection can be written as follows:

k
(Q′′′
g ) 0

h0 , T∞
0 1
Δx/2

Δx

Figure5.20: Nomenclature for the volume element surrounding boundary node 0

k 𝛥𝐸
Q1-0 + Qconvection + (Q′′′
g ) =0 𝛥𝑡

Tk K
1 − T0 k Tk+1 − Tk
Or kA ( ) + h0 A (T∞ − T0k ) + A(Δx/2) (q′′′
g ) = ρ A(Δx/2) Cp ( 0 0
)
Δx 0 Δt

Solving for 𝑇0𝑘+1 we get

𝐡𝟎 𝚫𝐱 𝐡𝟎 𝚫𝐱 𝜟𝒙𝟐 𝒌
𝐓𝟎𝐤+𝟏 = (𝟏 − 𝟐𝛕 − 𝟐𝛕 ) 𝐓𝟎𝐤 + 2τ𝐓𝟏𝐤 + 2τ T∞ + τ (𝒒′′′
𝒈 ) (5.51)
𝐤 𝐤 𝒌 𝟎

Similarly for the other boundary node M subjected to convection boundary condition the
finite difference equation can be written as

𝐡𝐌 𝚫𝐱 𝐡𝐌 𝚫𝐱 𝜟𝒙𝟐 𝒌
𝐓𝐌𝐤+𝟏 = (𝟏 − 𝟐𝛕 − 𝟐𝛕 𝐤
) 𝐓𝐌𝐤 + 2τ𝐓𝐌−𝟏 + 2τ T∞ + τ (𝒒′′′
𝒈 ) (5.52)
𝐤 𝐤 𝒌 𝟎

k
For stability of the scheme the coefficients of T0k and TM should be ≥ 0. i.e.

𝐡𝟎 𝚫𝐱 𝐌𝚫𝐱
(𝟏 − 𝟐𝛕 − 𝟐𝛕 )≥0 and (𝟏 − 𝟐𝛕 − 𝟐𝛕 )≥0
𝐤 𝐤

𝟏
Or τ≤ (5.53a)
𝟐( 𝟏+𝐡𝟎 𝚫𝐱/𝐤 )

262
𝟏
and τ≤ (5.53b)
𝟐( 𝟏+𝐡𝐌 𝚫𝐱/𝐤 )

Example 5.10.A marble slab[k = 2W/(m – 0C) and α = 1×10 −6 m2/s],2 cm thick is initially at
a uniform temperature of 200 0C. Suddenly one of its surfaces is lowered to 00C and is
maintained at that temperature, while the other surface is kept insulated. Develop an explicit
finite difference scheme to determine the temperature distribution in the slab as a function of
position after 80 seconds have elapsed by dividing the slab into 5 sub-regions and also find
the heat flux at the boundary surface which is maintained at 00C after 80 seconds.

Schematic: Δx

Insulated T5

0
0 1 2 3 4 5 x

Known: L = 2=0.02 m; k = 2 W/(m-K); α = 1× 10 −6 m 2/s; T5 = 00C; M = 5;T5 = 00C

Find: (i) Finite difference equations to determine temperatures at nodes 0 to 4at different
time steps; (ii) 𝑞5𝑘

Assumptions: (i)solid is having constant thermal properties;(ii) one dimensional conduction


across the thickness of the slab

Solution: The finite difference equations for internal nodes from 1 to 4 are given by
Equation (5.48) and (5.49) with 𝑞𝑔′′′ = 0 and M = 5. With these conditions we have

𝑇𝑖𝑘+1 = τ (𝑇𝑖−1
𝑘 𝑘
+ 𝑇𝑖+1 ) + (1 − 2𝜏)𝑇𝑖𝑘 i = 1,2,3,and 4.

Therefore for node 1 we have 𝑇1𝑘+1 = τ (𝑇0𝑘 + 𝑇2𝑘 ) + (1 − 2𝜏)𝑇1𝑘 (a)

For i = 2 𝑇2𝑘+1 = τ (𝑇1𝑘 + 𝑇3𝑘 ) + (1 − 2𝜏)𝑇2𝑘 (b)

For i = 3 𝑇3𝑘+1 = τ (𝑇2𝑘 + 𝑇4𝑘 ) + (1 − 2𝜏)𝑇3𝑘 (c)

For i = 4 𝑇4𝑘+1 = τ (𝑇3𝑘 + 𝑇5𝑘 ) + (1 − 2𝜏)𝑇4𝑘 (d)

Since the surface at x = 0 is insulated then

263
𝑇0𝑘 − 𝑇1𝑘
( ) = 0 for all k.
𝛥𝑥

Therfore we have 𝑇0𝑘+1 = 𝑇1𝑘+1 (e)

Now Δx = L/M = 0.02/5 = 0.004 m ;

For stability it follows that 1 − 2τ ≥ 0.

αΔt
Choosing 1 − 2τ = 0 we have τ = ½ = Δx2

𝛥𝑥 2 0.0042
Or Δt = = = 8 s.
2𝛼 2∗1∗ 10 −6

With τ = ½ Eqs. (a) to (e) reduce to

𝑇1𝑘+1 = 0.5× (𝑇0𝑘 + 𝑇2𝑘 ) (i)

𝑇2𝑘+1 = 0.5× (𝑇1𝑘 + 𝑇3𝑘 ) (ii)

𝑇3𝑘+1 = 0.5× (𝑇2𝑘 + 𝑇4𝑘 ) (iii)

𝑇4𝑘+1 = 0.5× (𝑇3𝑘 + 𝑇5𝑘 ) (iv)

T0k+1 = T1k+1 (v)

Eqs. (i) to (v) are used to calculate the temperatures at different nodes for each time step as
shown in the table below :

Table E5.10 Finite difference calculations of temperatures for example 5.10

k t (s) i=0 1 2 3 4 5
x(m) = 0 00.004 0.008 0.012 0.016 0.020
0 0 200 200 200 200 200 200
1 8 200 200 200 200 200 0.00
2 16 200 200 200 200 100 0.00
3 24 200 200 200 150 100 0.00
4 32 200 200 175 150 75 0.00
5 40 187.5 187.5 175 125 75 0.00
6 48 181.2 181.2 156.25 125 62.5 0.00
7 56 168.7 168.7 153.1 109.4 62.5 0.00
8 64 160.9 160.9 139.1 107.8 54.7 0.00
9 72 150 150 134.4 96.9 53.9 0.00
10 80 142.2 142.2 123.5 94.2 48.5 0.00

264
Heat flux at the boundary surface at x = L after t = 80 s i.e after 10 time steps is given by

10 𝑇410 − 𝑇510 48.5 −0.0


𝑞𝑀 =k( )=2×( ) = 24,250W/m 2
𝛥𝑥 0.004

Example 5.11. A large uranium plate of thickness 4 cm, thermal conductivity 28 W/(m-K) and
thermal diffusivity of 12.5× 10 −6 m2/s is initially at a uniform temperature of 200 0C.Heat is
generated at a constant rate of 5×10 6 W/m3.Suddenly at time t = 0, one side of the plate is
brought into contact with iced water and is maintained at 0 0C at all times, while the other side
is subjected to convection to an environment at 30 0C with a heat transfer coefficient of 45
W/(m2-K). Considering a total of three equally spaced nodes, two at the boundaries and one
at the middle of the plate, estimate the temperature distribution in the plate 150 s after the start
of the cooling process using explicit method.

Sketch for example 5.11

Node 1
L

T0 h,T∞

0 2 x

Δx/2
Δx
Known: L = 4 cm =0.04 m; k = 28 W/(m-K); α = 12.5 × 10 −6 m 2/s; T0 = 00C; M = 3;
T∞ = 300C;h2 = 45 W/(m 2-K); 𝑞𝑔′′′ = 5 × 10 6 W/m3;

Find: (i) Finite difference equations to determine temperatures at nodes 1 and 2 after time
150 s

Assumptions: (i)the plate is having constant thermal properties and constant rate of heat
generation (ii) one dimensional conduction across the thickness of the plate

Solution: The finite difference equations for internal node 1 is given by Equation (5.48) and
(5.49) with i = 1 and M = 2. With these conditions we have
′′′ 𝛥𝑥 2
𝑞𝑔
𝑇1𝑘+1 = τ (𝑇0𝑘 + 𝑇2𝑘 ) + (1 − 2𝜏)𝑇1𝑘 + τ (a)
𝑘

𝛼𝛥𝑡
Where τ = 𝛥𝑥 2

The finite difference equation for boundary node 2 is given by Equation(5.52) with M = 2.

265
h2 Δx h2 Δx 𝛥𝑥 2 𝑘
i.e. T2k+1 = (1 − 2τ − 2τ ) T2k + 2τT1k + 2τ T∞ + τ (𝑞𝑔′′′ ) (b)
k k 𝑘 0

h2 Δx 45∗0.02
Δx = L / M = 0.04/2 = 0.02 m ; = = 0.032
k 28

From Equation(a) for stability of the difference scheme 1 − 2𝜏 ≥ 0 or τ ≤ 0.5


(c)

h2 Δx
And from Equation(b) for stability (1 − 2τ − 2τ )≥0
k

1
Or (1 - 2τ – 2×0.032τ ) ≥ 0 i.e. τ ≤ i.e. ≤ 0.484 (d)
2.064

Therefore we have to choose a value of τ which satisfies both Eqs. (c) and (d)

0.484∗ (0.02)2
Let τ = 0.484. Then Δt = = 15.5 s.
12.5∗ 10 −6

Any time step ≤ 15.5 s can be used. Since we want the temperature after 150 s it will be
convenient to choose Δt = 15 s so that we get the required temperatures at nodes 1 and 2 after
10 time steps.Therefore

12.5∗ 10 −6 ∗15
τ= (0.02)2
= 0.46875

Substituting this value of τ in Eqs. (a) and (b) we get

5∗ 106 (0.02)2
𝑇1𝑘+1 = 0.46875× (𝑇0𝑘 + 𝑇2𝑘 ) + (1 − 2 ∗ 0.46875))𝑇1𝑘 + 0.46875× 28
Or T1k+1 = 0.0625T1k + 0.46875 T2k + 33,482 (i)

and T2k+1 = 0.9375T1k + 0.032366 T2k + 34,386 (ii)

The temperatures at nodes 1 and 2 at each time step are calculated using the above two
equations and substituting the temperatures at the previous time step. The results are shown
in Table E5.11

Table E5.11 Finite difference calculations of temperatures in 0C for example 5.11

k t (s) i=0 1 2
x(m) = 0 0.02 0.04

266
0 0 0 200 200
1 15 0 139.7 228.4
2 30 0 149.3 172.8
3 45 0 123.8 179.9
4 60 0 125.6 156.3
5 75 0 114.6 157.1
6 90 0 114.3 146.9
7 105 0 109.5 146.3
8 120 0 108.9 141.8
9 135 0 106.7 141.1
10 150 0 106.3 139.0

5.8.3(b). Explicit Method for One-dimensional Transient Radial conduction in a cylinder

(i) Finite difference Equations for internal nodes i (i = 1,2,3, M-1):

The energy balance equation for the volume element surrounding node i (see Figure 5.5) for
a time step Δt can be written as follows.
𝛥𝐸
Qi-1,i+ Qi+1,i + (Qg) i = 𝛥𝑡

Or
𝛥𝑟 Tk k
i−1 − Ti
k [2𝜋 (𝑖𝛥𝑟 − 2
) ∗ 1] [
Δr
] + (2𝜋𝑖𝛥𝑟 ∗ 𝛥𝑟 ∗ 1)𝑞𝑖′′′

𝛥𝑟 Tk k
i+1 − Ti Tk+1 − Tk
+ k [2𝜋 (𝑖𝛥𝑟 + ) ∗ 1] [ ] = ρ ×2πiΔr2×1×Cp( i i
)
2 Δr Δt
i = 1,2,3……M-1

After simplification the above equation reduces to

𝟏 𝟏 𝚫𝐫 𝟐
𝐓𝐢𝐤+𝟏 = τ [(𝟏 − 𝐤
) 𝐓𝐢−𝟏 + (𝟏 + 𝐤
) 𝐓𝐢+𝟏 + 𝐪′′′
𝐢 ] + [𝟏 − 𝟐𝛕]𝐓𝐢𝐤
𝟐𝐢 𝟐𝐢 𝐤
i = 1,2,3……M-1
𝛼𝛥𝑡
where τ = 𝛥𝑟 2 and α = k/(ρCp ) (5.54)

For stability of numerical solution (1 - 2τ) ≥ 0 or τ ≤ ½ (5.55)

(ii) Finite difference Equations for innermost node 0:

Referring to Figure 5.5, the energy balance equation for node 0 can be written as follows:

Δr T0 − T1 Δr 2 Δr 2 (Tk+1 − Tk
0)
k (2π ∗ 1) [ ] + q′′′
0 π ( 2 ) ×1 = ρCpπ ( 2 ) ∗ 1 ∗
0
2 Δr Δt

267
After simplification the above equation reduces to

𝐪′′′
𝟎 (𝚫𝐫)
𝟐
𝐓𝟎𝐤+𝟏 = 4τ [𝐓𝟏𝐤 + 𝐤
] + [𝟏 − 𝟒𝛕]𝐓𝟎𝐤 (5.56)

For stability of numerical solution (1 - 4τ) ≥ 0 or τ ≤ ¼ (5.57)

(iii) Finite difference Equation for outermost node M when outer surface is subjected to
prescribed heat flux: The nomenclature for the volume element surrounding the outer most node M
is shown in Figure 5.21.Energy balance equation for the volume element can be written as follows:

M qs
M-1

MΔr

Figure5.21: Nomenclature for volume element surrounding the node M with


prescribed heat flux
𝑘 𝑘
(𝑇𝑀 −1 − 𝑇𝑀 ) ′′′
k ×2π [ MΔr – Δr/2] ×1 + 2π MΔr×1 qs + π [(MΔr)2 − (MΔr – Δr/2)2]×1× 𝑞𝑀
𝛥𝑟

= π [(MΔr)2 − (MΔr – Δr/2)2]×1ρCp[𝑇𝑀𝑘+1− 𝑇𝑀𝑘 ] / Δt

After simplification the above equation reduces to


𝟏 𝟏
(𝟏 − ) (𝟏 − ) 𝟒𝛕 𝐪𝐬 𝚫𝐫 𝛕𝐪′′′
𝐌 (𝚫𝐫)
𝟐
𝐓𝐌𝐤+𝟏 = 𝟒𝛕 𝟐𝐌
𝟏
𝐤
𝐓𝐌−𝟏 + [𝟏 − 𝟒𝛕 𝟐𝐌
𝟏 ] 𝐓𝐌𝐤 + 𝟏 + (5.58)
(𝟐 − ) (𝟐 − ) 𝐤 (𝟐 − ) 𝐤
𝟐𝐌 𝟐𝐌 𝟐𝐌

𝟏
(𝟏 − )
For stability, [𝟏 − 𝟒𝛕 𝟐𝐌
𝟏 ]≥0 (5.59)
(𝟐 − )
𝟐𝐌

268
(iii) Finite difference Equation for outermost node M when outer surface is in contact with a
fluid at a uniform temperature T∞ with a surface heat transfer coefficient h (convective
boundary condition):

The nomenclature for the volume element surrounding the outer most node M is shown in Figure
5.22.Energy balance equation for the volume element can be written as follows:

𝑘 𝑘
(𝑇𝑀 −1 − 𝑇𝑀 ) k
k ×2π [ MΔr – Δr/2] ×1 + 2π MΔr×1×h [T∞ − TM ]
𝛥𝑟

′′′
+ π [(MΔr)2 − (MΔr – Δr/2)2]×1× 𝑞𝑀

= π [(MΔr)2 − (MΔr – Δr/2)2]×1ρCp[𝑇𝑀𝑘+1− 𝑇𝑀𝑘 ] / Δt

After simplification the above equation reduces to

M h,T∞
M-1

MΔr

Figure5.21: Nomenclature for volume element surrounding the node M


subjected to convective condition
𝟏 𝟏
(𝟏 − ) (𝟏 − ) 𝟒𝛕 𝐡𝚫𝐫 𝟒𝛕 𝐡𝚫𝐫𝐓∞ 𝛕𝐪′′′
𝐌 (𝚫𝐫)
𝟐
𝐓𝐌𝐤+𝟏 = 𝟒𝛕 𝟐𝐌
𝟏
𝐤
𝐓𝐌−𝟏 + [𝟏 − 𝟒𝛕 𝟐𝐌
𝟏 − 𝟏 ] 𝐓𝐌𝐤 + 𝟏 +
(𝟐 − ) (𝟐 − ) 𝐤 (𝟐 − ) 𝐤 (𝟐 − ) 𝐤
𝟐𝐌 𝟐𝐌 𝟐𝐌 𝟐𝐌
(5.60)
𝟏
(𝟏 − ) 𝟒𝛕 𝐡𝚫𝐫
For stability, [𝟏 − 𝟒𝛕 𝟐𝐌
𝟏 − 𝟏 ]≥0 (5.61)
(𝟐 − ) 𝐤 (𝟐 − )
𝟐𝐌 𝟐𝐌

Example 5.12.A long chrome steel cylinder[k = 40 W/(m-K);α = 1.1 × 10 −5m2/s] 4 cm in


diameter is initially at a uniform temperature of 1000C. It is suddenly subjected to a convection
environment at 500C with a surface heat transfer coefficient of 400 W/(m2- K). Assuming one
dimensional radial unsteady state conduction determine the radial temperature distribution in

269
the cylinder 9 seconds after the exposure to the environment using explicit finite difference
scheme.

Schematic : The cylinder is divided equally as shown in the figure with Δr = R/2, where

Δr 2 h,T∞

Δr/2 1

0 R
.
R is the radius of the cylinder.

Known: k = 40 W/(m-K); α = 1.1 × 10 – 5 m2/s;


R = 0.02 m; Δr = 0.02/2 = 0.01 m; M = 2; Initial temperatures at the three nodes, 𝑇00 =
𝑇10 = 𝑇20 = 100 0C; T∞ = 50 0C; h = 400 W/(m2-K) ; Length of cylinder = 1 m

Find: Temperatures at the nodes after 9 seconds.

Assumptions: (i) Constant physical properties for aluminium; (ii) Radial unsteady state
conduction

Solution: Finite difference equation for the internal node 1:

The finite difference equation for the internal node 1 is given by Equation(5.54) with i = 1.

i.e., T1k+1 = τ [0.5 ∗ T0k + 1.5 ∗ T2k ] + [1 − 2τ]T1k …………………………….(1)

𝛼𝛥𝑡
where τ = 𝛥𝑟 2 .

For stability τ ≤ ½ ……………………………(2)

Finite difference equation for the innermost node 0:

The finite difference equation for the internal node 0 is given by Equation(5.54) with 𝑞0′′′ = 0.

i.e. T0k+1 = 4τ T1k + [1 − 4τ]T0k …………………………………(3)

For stability τ≤¼ ………………………………(4)

Finite difference equation for the outermost node 2:

270
The finite difference equation for the outermost node 2 is given by Equation(5.60) with 𝑞2′′′ =
0.
1 1
(1 − ) (1 − ) 4τ hΔr 4τ hΔrT∞
i.e., T2k+1 = 4τ 4
1 T1k + [1 − 4τ 4
1 − 1
k
] TM + 1
(2 − ) (2 − ) k (2 − ) k (2 − )
4 4 4 4

4τ∗ 400∗ 0.01 4τ∗400∗0.01∗ 50


Or T2k+1 = 1.714τ T1k + [1 − 1.714τ − ] T2k +
40∗ 1.75 40∗1.75

Or T2k+1 = 1.714τ T1k + [ 1 – 1.943τ] T2k + …………………….(5)


1
For stability 1 – 1.943τ ≥ 0 Or τ ≤ 1.943 ………………………….(6)
The numerical value of τ has to be selected such that all the three stability conditions viz.,
Eqs.(2),(4) and (6) are satisfied. The condition τ ≤ ¼ will satisfy all the three stability
condtions.Hence choose τ ≤ ¼ : i.e. τ ≤ 0.25.

𝛼𝛥𝑡 0.25∗ (0.01)2


Then ≤ 0.25 or Δt ≤
𝛥𝑟 2 1.1∗ 10−5

Or Δt ≤ 2.27 s
𝑡 9
Selecting Δt = 2.25 s, τ = 0.247 no of time steps required = 𝛥𝑡 = =4
2.25

With the value of τ = 0.247 the nodal equations (1), (3) and (5) reduce to

T0k+1 = 0.012 T0k + 0.988 T1k (a)

T1k+1 = 0.123T0k + 0.506 T1k + 0.37 T2k (b)

And T2k+1 = 0.423 T1k + 0.52 T2k + 2.82 (c)

The temperatures at the three nodes at different time steps are calculated using Eqs. (a), (b)
and (c) and are shown in the table below.

Time step Δt (s) t = kΔt (s) Temperature at different nodes (0C)


(k) T0 T1 T2
0 2.25 0 100 100 100
1 2.25 2.25 100 99.9 97.10
2 2.25 4.50 99.9 98.78 95.57
3 2.25 6.75 98.79 97.63 94.30
4 2.25 9.00 97.64 96.44 93.15

271
5.8.4.Implicit Method for Transient conduction

In the explicit finite difference scheme, the temperature at any node at time (t + Δt)
is calculated from knowledge of temperatures at the same and neighboring nodes for the
preceding time t. Hence determination of temperature at a node at some time is independent of
temperatures at other nodes for the same time. This method, though offers computational
convenience, it suffers from the limitations on the time step Δt. For a given Δx, the time step
is limited by the stability requirements as illustrated in examples 5.10 and 5.11. This dictates
the use of smaller Δt resulting in large number of time steps to determine the transient
tenmpeartures of the nodes after certain time.By employing an implicit scheme the restriction
on Δt can be completely eliminated.In the implicit scheme the space derivatives in finite
difference form are expressed using temperatures of the all the nodes at time t+∆t, while the
backward finite difference form of the derivative of temeperature iwthrespect to time is
used.

Refering to Fig 5.18, for a plane wall the energy balance equation in implicit form for any
internal node can be written as
𝑘
𝑇𝑖−1 − 𝑇𝑖𝑘+1 𝑘
𝑇𝑖+1 − 𝑇𝑖𝑘+1 𝑇𝑖𝑘+1 − 𝑇𝑖𝑘
k AΔx ( ) + k AΔx ( ) + AΔx 𝑞𝑔′′′ = ρAΔxCp ( )
𝛥𝑥 𝛥𝑥 𝛥𝑡

i = 1,2,3….. M – 1

The above equation simplifies to

𝒒′′′
𝒊 𝝉 ∆𝒙
𝟐
(𝟏 + 𝟐𝝉)𝑻𝒌+𝟏
𝟏 – 𝝉 [𝑻𝒌+𝟏 𝒌+𝟏
𝟏−𝟏 − 𝑻𝒊+𝟏 ] = + 𝑻𝒌𝒊 i = 1,2,3….. M – 1
𝒌

Similar equations for the boundary nodes 0 and M can be obtained depending on the boundary
conditions specified.

272
Chapter 6

Basic Concepts of Convective Heat Transfer


6.1. Definition of Convective Heat Transfer:- When a fluid flows over a body or inside a
channel and if the temperatures of the fluid and the solid surface are different, heat transfer
will take place between the solid surface and the fluid due to the macroscopic motion of the
fluid relative to the surface. This mechanism of heat transfer is called as “convective heat
transfer”.If the fluid motion is due to an external force (by using a pump or a compressor) the
heat transfer is referred to as “forced convection”. If the fluid motion is due to a force generated
in the fluid due to buoyancy effects resulting from density difference (density difference may
be caused due to temperature difference in the fluid) then the mechanism of heat transfer is
called as “natural or free convection”.For example, a hot plate suspended vertically in
quiescent air causes a motion of air layer adjacent to the plate surface because the temperature
gradient in the air gives rise to a density gradient which in turn sets up the air motion.

6.2. Heat Transfer Coefficient:- In engineering application, to simplify the heat transfer
calculations between a hot surface say at temperature Tw and a cold fluid flowing over it at a
bulk temperature T∞ as shown in Figure 6.1 a term called “heat transfer coefficient,h” is
defined by the equation

q = h(Tw – T∞) 6.1(a)

where q is the heat flux (expressed in W / m2) from the surface to the flowing fluid.
Alternatively if the surface temperature is lower than the flowing fluid then the heat transfer
takes place from the hot fluid to the cold surface and the heat flux is given by

q = h(T∞ – Tw) 6.1(b)

273
The heat flux in this case takes place from the fluid to the cold surface.If in equations 6.1(a)
and 6.1(b) the heat flux is expressed in W / m2, then the units of heat transfer coefficient will
be W·m−2 ·K −1 or W·m−2 ·℃−1.

The heat transfer coefficient is found to vary with (i) the geometry of
the body, (ii) the type of flow (laminar or turbulent), (iii) the transport properties of the fluid
(density, viscosity,specific heat and thermal conductivity),(iv) the difference in temperature
between the average temperature of the fluid and the surface with which the fluid is in
contact , (v) the position along the surface of the body, and (vi) whether the heat transfer is
by forced convection or free convection. For convection problems involving simple
geometries like flow over a flat plate or flow inside a circular tube, the heat transfer
coefficient can be

u∞, T∞ T∞

Fluid Temperature Profile


y

x Tw

Figure 6.1: Temperature distribution of the fluid at any x for Tw >


T∞
determined analytically. But for flow over complex configurations, experimental / numerical
approach is used to determine h.There is a wide difference in the range of values of h for
various applications.Typical values of heat transfer coefficients encountered in some
applications are given in Table 6.1.

Table 6.1: Typical Values of heat transfer coefficients

Type of flow h [W·𝐦−𝟐 ·𝐊 −𝟏 ]

Free convection air 5 – 15

-----do --------- oil 25 – 60

-----do--------- water 400 –800

Forced Convection air 15 –300

274
-------do------------ oil 50 –1700

-------do----------- water 300 – 12000

Boiling water 3000 – 55000

Condensing steam 5500 – 120000

6.3. Basic concepts for flow over a body:- When a fluid flows over a body, the velocity and
temperature distribution at the vicinity of the surface of the body strongly influence the heat
transfer by convection.By introducing the concept of boundary layers (velocity boundary layer
and thermal boundary layer) the analysis of convective heat transfer can be simplified.

6.3.1.Velocity Boundary Layer:- Consider the flow of a fluid over a flat plate as shown in
Figure 6.2 The fluid just before it approaches the leading edge of the plate has a velocity u∞
which is parallel to the plate surface.As the fluid moves in x-direction along the plate,

u∞
y

u(x, y)
δ(x)
u(x, y)

x
xcr Turbulent Region
Laminar Region Transition

Figure 6.2: Velocity boundary layer for flow over a flat plate

those fluid particles that makes contact with the plate surface will have the same velocity as
that of the plate. Therefore if the plate is stationary, then the fluid layer sticking to the plate
surface will have zero velocity (This is called no-slip condition).But far away from the plate
(y = ∞) the fluid will have the velocity u∞.Therefore starting from the plate surface (y = 0) there
will be retardation of the fluid in x-direction component of velocity u(x,y).This retardation
effect is reduced as we move away from the plate surface.At distances sufficiently long from
the plate(y = ∞) the retardation effect is completely reduced: i.e. u → u∞ as y → ∞. This means
that there is a region surrounding the plate surface where the fluid velocity changes from zero
at the surface to the velocity u∞ at the outer edge of the region. This region is called the velocity
boundary layer. The variation of the x-component of velocity u(x,y) with respect to y at a

275
particular location along the plate is shown in Figure 6.2.The distance measured normal to the
surface from the plate surface to the point at which the fluid attains 99% of u∞ is called “velocity
boundary layer thickness” and denoted by δ(x)
Thus for flow over a flat plate, the flow field can be divided into two distinct regions, namely,
(i) the boundary layer region in which the axial component of velocity u(x,y) varies rapidly
with y with the result the velocity gradient (∂u /∂y) and hence the shear stress are very large
and (ii) the potential flow region which is outside the boundary layer region, where the velocity
gradients and shear stresses are negligible.

The flow in the boundary layer, starting from the leading edge of the plate
will be initially laminar in which the fluid particles move along a stream line in an orderly
manner. In the laminar region the retardation effect is due to the viscosity of the fluid and
therefore the shear stress can be evaluated using Newton’s law of viscocity.The laminar flow
continues along the plate until a critical distance ‘xcr’ is reached. After this the small
disturbances in the flow begin to grow and fluid fluctuations begin to develop. This
characterizes the end of the laminar flow region and the beginning of transition from laminar
to turbulent boundary layer. A dimensionless parameter called Reynolds number is used to
characterize the flow as laminar or turbulent. For flow over a flat plate the Reynolds number
is defined as
u∞ x
Rex = ……………………….6.2
ν

where u∞ = free-stream velocity of the fluid, x = distance from the leading edge of the plate
and ν = kinematic viscosity of the fluid.

For flow over a flat plate it has been found that the transition from
laminar flow to turbulent flow takes place when the Reynolds number is ≈ 5 x 10 5.This number
is called as the critical Reynolds number Recr for flow over a flat plate.Therefore
u∞ xcr
Recr = = 5 x 10 5 ………….6.3
ν

The critical Reynolds number is strongly dependent on the surface roughness and the
turbulence level of the free stream fluid. For example, with very large disturbances in the free
stream, the transition from laminar flow to turbulent flow may begin at Rex as low as 1 x 10 5
and for flows which are free from disturbances and if the plate surface is smooth transition
may not take place until a Reynolds number of 1 x 10 6 is reached.But it has been found that
for flow over a flat plate the boundary layer is always turbulent for Rex ≥ 4 x 10 6.In the
turbulent boundary layer next to the wall there is a very thin layer called “the viscous sub-
layer”, where the flow retains its viscous flow character. Next to the viscous sub-layer is a
region called “buffer layer” in which the effect of fluid viscosity is of the same order of
magnitude as that of turbulence and the mean velocity rapidly increases with the distance from
the plate surface.Next to the buffer layer is “the turbulent layer” in which there is large scale
turbulence and the velocity changes relatively little with distance.

276
Boundary Layer Concept for flow over a Curved Body: Figure 6.3 illustrates the boundary
layer growth for flow over a curved body.Here ‘x’ coordinate is measured along the curved
surface of the body. Starting from the stagnation point, and at each location along x, the y
coordinate is measured normal to the surface of the body.The free stream velocity U∞, in this
case is not a constant as in the case of flow over a flat plate but is a function of x. The

Figure 6.3: Boundary layer growth for flow over a curved body

boundary layer thickness δ(x) increases with the distance x along the surface. But, because of
the curvature of the body, after some distance, the velocity profile u(x,y) exhibits a point of
inflexion; that is at this location (ðu / ðy) becomes zero at the wall surface. Beyond the point
of inflexion, reversal of flow takes place, and the boundary layer is said to be detached from
the surface.Beyond the point of flow reversal the flow become very complicated and the boundary
layer analysis is no longer valid.

6.3.2. Drag coefficient and Drag force:- If the velocity distribution u(x,y) in the boundary
layer at any ‘x’ is known then the viscous shear stress at the wall can be determined using
Newton’s law of viscosity. Thus the wall-shear stress, τw(x) at any location x can be written
as :

∂u
τw(x) = μ ( ) ………………….6.4
∂y y=0

where μ is the absolute viscosity of the fluid.The drag coefficient is dimensionless wall shear
stress. Therefore the local drag coefficient, Cx at any ‘x’ is defined as

τw (x)
Cx = 1 ……………………….6.5
ρ u2∞
2

Substituting for τw(x) in the above equation from Equation 6.4 and simplifying we get

2ν (∂u / ∂y)y = 0
Cx = ---------------------- ……………….6.6
u∞2

277
Therefore if the velocity profile u(x,y) at any x is known then the local drag coefficient C x at
that location can be determined from Equation 6.6.The average value of Cx for a total length L
of the plate can be determined from the equation
L
Cav = (1/L) ∫Cx dx ……………………6.7
0

Substituting for Cx from Equation 5.5 we have


L
∫ τw(x)dx
Cav = ------------------------
L (1/2) ρu∞2

_
τw
Or Cav = -------------------- ……………….6.8
(1/2) ρu∞2
_
Where τw is the average wall-shear stress for total length L of the plate.

The total drag force experienced by the fluid due to the presence of the plate can be written as
_
FD = As τw …………………………….6.9

Where As is the total area of contact between the fluid and the plate. If ‘W’ is the width of the
plate then As = LW if the flow is taking place on one side of the plate and As = 2LW if the
flow is on both sides of the plate.

6.3.3.Thermal boundary layer:- Similar to the velocity boundary layer one can visualize the
development of a thermal boundary layer when a fluid flows over a flat plate with the
temperature of the plate being different from that of the free stream fluid.Consider that a fluid
at a uniform temperature T∞ flows over a flat plate which is maintained at a uniform
temperature Tw.Let T(x,y) is the temperature of the fluid at any location in the flow field.Let
the dimensionless temperature of the fluid θ(x,y) be defined as
T(x,y) – Tw
θ(x,y) = ------------------- …………………………….6.10
T∞ − Tw

The fluid layer sticking to the plate surface will have the same temperature as the plate surface
[T(x,y)y = 0 = Tw] and therefore θ(x,y) = 0 at y = 0.Far away from the plate the fluid temperature
is T∞ and hence θ(x,y) → 1 as y → ∞. Therefore at each location x along the plate one can
visualize a location y = δt(x) in the flow field at which θ(x,y) = 0.99. δt(x) is called “the
thermal boundary layer thickness” as shown in Figure 6.3. The locus of such points at which
θ(x,y) = 0.99 is called the edge of the thermal boundary layer. The relative thickness of the
thermal boundary layer δt(x) and the velocity

278
boundary layer δ(x) depends on a dimensionless number called “Prandtl number” of the
fluid.It is denoted by Pr and is defined as
μCp (μ/ρ) ν
Pr = --------- = ---------- = -------- ………..6.11
k (k/ρCp) α

Where μ is the absolute viscosity of the fluid, Cp is the specific heat at constant pressure

k is the thermal conductivity,υ is the kinematic viscosity and α is the thermal diffusivity of the
fluid.The Prandtl number for fluids range from 0.01 for liquid metals to more than 100,000 for
heavy oils. For fluids with Pr = 1 such as gases δt(x) = δ(x), for fluids with Pr << 1,such as
liquid metals, δt(x) >> δ(x) and for fluids with Pr >> 1, like oils δt(x) << δ(x).

6.3.4. General expression for heat transfer coefficient:- Let us assume that Tw > T∞.Then heat
is transferred from the plate to the fluid flowing over the plate.Therefore at any ‘x’ the heat
flux is given by
q = − k (∂T /∂y)y=0 ………………………..6.12(a)

In terms of the local heat transfer coefficient hx, the heat flux can also be written as

q = hx (Tw − T∞) …………………………..6.12(b)

From equations 5.12(a) and 5.12(b) it follows that


− k (∂T /∂y)y=0
hx = ------------------ ………………………6.13
(Tw − T∞)

From equation 6.10 we have (∂T /∂y)y=0 = [T∞ − Tw] (∂θ /∂y)y=0. Substituting this expression
in Equation6.13 and simplifying we get the general expression for hx as

279
hx = k (∂θ /∂y)y=0 …………………………….6.14

The same expression for hx could be obtained even when Tw < T∞. Equation 6.14 can be used
to determine the local heat transfer coefficient for flow over a flat plate if the dimensionless
temperature profile θ(x,y) is known.

Average heat transfer coefficient:- For a total length L of the plate the average heat transfer
coefficient is given by
L
hav = (1 /L) ∫hxdx ……………………………..6.15a
0

Substituting for hx from Equation 6.14 we get


L
hav = (1 /L) ∫ k (∂θ /∂y)y=0 dx ………………….6.15b
0

Since (∂θ /∂y)y=0 at any x depends on whether the flow at that section is laminar or turbulent
the expression for hav can be written as

xcr L
hav = (1 /L) { ∫ k [(∂θ /∂y)y=0]laminar dx + ∫ k [(∂θ /∂y)y=0]turbulent dx }……5.16
0
xcr

Relation Between Fluid Friction and Heat Transfer (Reynolds – Colburn Analogy):
For laminar flow over a flat plate an analogy between momentum transfer and heat transfer
exists; that is, it is possible to establish a relation between the local drag coefficient, Cx and the
local Nusselt number Nux as shown below.
The excat expression for the local drag coefficient is given by

(Cx / 2 ) = 0.332 Rex─ ½ ……………………….6.17a

The exact expression for the local Nusselt number, Nux is given by

Nux = 0.332 Pr 1/3 Rex ─ ½ …………………..6.17b

Another dimensionless number called “STANTON NUMBER” is defined as


hx
Stx = -------------- .
ρ Cp u∞
The expression for Stx can be written as
hx x /k Nux
Stx = -------------------------- = ---------------
(ρ x u∞ /μ) (μ Cp/k) Rex Pr

280
Substituting for Nux from Equation 6.17b and rearranging we get

Stx Pr 2/3 = 0.332 Rex ─1/2

Or Stx Pr 2/3 = Cx / 2 ………………………………..6.18

Equation 6.18 is also applicable to turbulent flow over a flat plate, but it does not apply to flow
through tubes.This analogy is referred to as “Reynolds – Colburn Analogy”.

Illustrative examples on flow over a flat plate:

Example 6.1:- Assuming the transition from laminar to turbulent flow takes place at a
Reynolds number of 5 x 10 5, determine the distance from the leading edge of a flat plate at
which transition occurs for the flow of each of the following fluids with a velocity of 2 cm/s at
40 0 C.(i) Air at atmospheric pressure;(ii)Hydrogen at atmospheric pressure;(iii) water;(iv)
Engine oil;(v) mercury.Comment on the type of flow for the 5 fluids if the total length of the
plate is 1 m.

Solution: Data:- Recr = 5 x 10 5; u∞ = 2 m/s ; T∞ = 40 0 C

(i)Air at atmospheric pressure :- At 40 0 C, ν = 17 x 10 − 6 m2/s.

u∞ xcr Recr ν 5 x 10 5 x 17 x 10 − 6
Recr = ----------- or xcr = -------------------- = ---------------------------- = 4.25 m.
ν u∞ 2

(ii) Hydrogen :- For hydrogen at 40 0 C, ν = 117.9 x 10 − 6 m2/s.

5 x 10 5 x 117.9 x 10 − 6
Therefore xcr = ------------------------------- = 29.5 m
2

(iii) Water :- For water at 40 0 C, ν = 0.658 x 10 − 6 m2/s.

5 x 10 5 x 0.658 x 10 − 6
Therefore xcr = ------------------------------- = 0.1645 m
2

(iv) Engine oil :- For engine oil at 40 0 C, ν = 0.24 x 10 − 3 m2/s.

5 x 10 5 x 0.24 x 10 − 3
Therefore xcr = ------------------------------- = 60 m
2

281
(v) Mercury :- For mercury at 40 0 C, ν = 0.107 x 10 − 6 m2/s.

5 x 10 5 x 0.107 x 10 − 6
Therefore xcr = ------------------------------- = 0.027 m
2

Comments on the type of flow

Sl.No Type of fluid xcr xcr vs L Type of Flow

1 Air 4.25 xcr > L Flow is Laminar for entire length

2 Hydrogen 29.5 xcr>> L Flow is laminar for entire length

3 Water 0.1645 xcr < L Flow is partly Laminar & Partly Turbulent

4 Engine oil 60 xcr >> L Flow is laminar for entire length

5 Mercury 0.027 xcr << L Flow is turbulent for almost entire length

Example 6.2:- An approximate expression for the velocity profile u(x,y) for laminar boundary
layer flow along a flat plate is given by

u(x, y)/ u∞ =2[y / δ(x)] − 2[y / δ(x)] 3+ [y / δ(x)] 4

where δ(x) is the velocity boundary layer thickness given by the expression

δ(x) / x = 5.83 / (Rex)1/2

(a) Develop an expression for the local drag coefficient.


(b) Develop an expression for the average drag coefficient for a length L of the plate.
(c) Determine the drag force acting on the plate 2 m x 2 m for flow of air with a free stream
velocity of 4 m /s and a temperature of 80 0C.

Solution:- (a) The velocity profile u(x,y) is given as

u(x, y) = u∞ {2[y / δ(x)] − 2[y / δ(x)] 3+ [y / δ(x)] 4 }

Therefore (∂u / ∂y)y=0 = 2u∞ / δ(x)

282
τw(x) = μ (∂u / ∂y)y=0 = (2 μu∞) / δ(x)

(2 μu∞) Rex (2 μu∞) [(u∞x) / ν]1/2


= ----------------- = ------------------------ = 0.343 (μu∞) [u∞ /(x ν)]1/2 ….(1 )
5.83 x 5.83 x

The local drag coefficient at any x is given by

τw(x) 0.343 (μu∞) [u∞ /(x ν)]1/2


Cx = ----------------- = -------------------------------------------
(1/2) ρu∞2 (1/2) ρu∞2

0.686 0.686
= ---------------------- = -----------------
{(u∞x) / ν}1/2 (Rex) ½

(b) The average drag coefficient is given by


L L
Cav = (1/L) ∫ Cx dx = (1/L) ∫ 0.686 (Rex) − ½ dx
0 0

{ 0.686 (u∞/ν) − ½} L
= ----------------------- ∫ x − ½ dx
0
L

Or 2 x 0.686 1.372
Cav = --------------- = ------------------
(u∞L / ν) ½ (ReL) ½

(c) At 80 0 C for air ν = 20.76 x 10 − 6 m2 / s ; ρ = 1.00 kg / m 3

u∞L 4x2
ReL = ----------------- = ------------------- = 3.793 x 10 5
ν 21.09 x 10 − 6

1.372 1.372
Average drag coefficient = Cav = ------------------- = ------------------------ = 2.228 x 10 − 3
ReL0.5 (3.793 x 10 5)0.5

Drag force assuming that the flow takes place on one side of the plate is given by
_
FD = τw LW = (1/2)ρ u∞2 Cav LW for flow over one side of the plate

283
= (1/2) x 1.00 x 42 x 2.2228 x 10 − 3 x 2 x 2

= 0.071 N

Example 6.3:- An approximate expression for temperature profile θ(x,y) in the thermal
boundary layer region for flow over a flat plate is given by

θ(x,y) = 2y / δt −[y / δt] 2

where the thermal boundary layer thickness δt is given by


5.5
δt / x = ----------------- ; Rex is the Reynolds number based on ‘x’ and
Rex0.5Pr 1/3
Pr is the Prandtl number of the fluid. Develop an expression for (i) the local heat transfer
coefficient hx and (ii) the average heat transfer coefficient for total length L of the plate.

Solution: (i) The local heat transfer coefficient hx is given by

hx = k (∂θ / ∂y)|y = 0.

Now θ(x,y) = 2(y / δt )− (y / δt)2

2 Rex0.5Pr 1/3
Hence (∂θ / ∂y)|y = 0. = 2 / δt = ---------------------
5.5.x
2 k Rex0.5Pr 1/3
Or hx = ------------------------ = 0.364 (k / x) Rex0.5 Pr 1/3
5.5.x
hx x
----- = 0.364 Rex0.5 Pr 1/3
K
hx x /k is a dimensionless number involving local heat transfer coefficient and is called

“local Nusselt number”.

(ii) The average heat transfer coefficient for a total length L of the plate is given by

L
hav = (1 / L) ∫hx dx = (1 / L) ∫ 0.364 (k / x) Rex0.5 Pr1/3 dx
0

= (1 / L) (L 0.5 /0.5) 0.364 Pr1/3 k (U∞ / ν)0.5 = 0.728 (k / L) (U∞L / ν) 0.5 Pr 1/3

Or hav L / k = 0.728 ReL0.5 Pr1/3

284
hav L / k is a dimensionless number involving the average heat transfer coefficient and is called
the “average Nusselt number”.

Example 6.4:- The heat transfer rate per unit width from a longitudinal section x2 ─ x1 of a
flat plate can be expressed as q12 = h12 (x2 – x1)(Ts - T∞), where h12 is the average heat transfer
coefficient for the section length of (x2 – x1). Consider laminar flow over a flat plate with a
uniform temperature Ts. The spatial variation of the local heat transfer coefficient is of the
form hx = C x ─ 0.5, where C is a constant.
(a) Derive an expression for h12 in terms of C,x1 and x2.
(b) Derive an expression for h12 in terms of x1, x2, and the average coefficients h1 and h2
corresponding to lengths x1 and x2 respectively.

Solution:
u∞, T∞ q12
Ts

x1

x2

Figure P6.4: Schematic for problem 6.4

(a) hx = C x − 0.5
_ 1 x2
Therefore h12 = -------------- ∫hx dx
(x2 – x1) x1
1 x2
= ------------------- ∫ C x − 0.5 dx
(x2 – x1) 0

2C
= ------------------ [ x2 0.5 – x1 0.5]
(x2 – x1)

_ x1
(b) h1 = (1/x1) ∫C x − 0.5 dx
0

= 2C / √ x1
_ ___

285
Similarly h2 = 2C / √ x2

x1 __ _ 1 x2 x1
Since ∫ hxdx = x1h1, h12 = -------------- [ ∫ hxdx - ∫ hxdx ]
0
( x 2 – x 1) 0 0

_ _
_ h2x2 - h1x1 {2C / √ x2 } ─ {2C / √ x1 }
h12 = ----------------- = ----------------------------------- = 2C [√ x2 ─ √ x1]
x2 – x1 (x2 – x1)

6.4. Basic Concepts For Flow Through Ducts :- The basic concepts developed on the
development of velocity and thermal boundary layers for flow over surfaces are also applicable
to flows at the entrance region of the ducts.

6.4.1.Velocity Boundary Layer:- Consider the flow inside a circular tube as shown in
Figure6.5. Lat uo be the uniform velocity with which the fluid approaches the tube. As the fluid
enters the tube, a “velocity boundary layer” starts to develop along the wall-surface. The
velocity of the fluid layer sticking to the tube-surface will have zero velocity and the fluid layer
slightly away from the wall is retarded. As a result the velocity in the central portion of the
tube increases to satisfy the continuity equation (law of conservation of mass).The thickness
of the velocity boundary layer δ(z) continuously grows along the tube-surface until it fills the
entire tube. The region from the tube inlet up to little beyond the hypothetical location where
the boundary layer reaches the tube centre is called “hydrodynamic entrance region or
hydrodynamically developing region” and the
corresponding length is called “hydrodynamic entrance length Lh”. In the hydrodynamically
developing region the shape of the velocity profile changes both in axial and radial direction,
i.e., u = u(r,z). The region beyond the hydrodynamic entry length is called “Hydrodynamically
developed region”, because in this region the velocity profile is invariant with distance along
the tube,i.e., u = u(r).
If the boundary layer remains laminar until it fills the tube, then laminar flow will prevail
in the developed region. However if the boundary layer changes to turbulent before its
thickness reaches the tube centre, fully developed turbulent flow will prevail in the
hydrodynamically developed region. The velocity profile in the turbulent region is flatter
than the parabolic profile of laminar flow. The Reynolds number, defined as

Red = (um Dh) / ν …………………………………(6.19)

is used as a criterion for change from laminar flow to turbulent flow. In this definition, um is
the average velocity of the fluid in the tube, Dh is the hydraulic diameter of the tube and ν is
the kinematic viscosity of the fluid. The hydraulic diameter is defined as

286
Hydrodynamic Entrance Hydrodynamically developed
uo Region Region
δ(z) r
R

Lh
z Fully developed profile
u = u(r)
Figure 6.5: development of velocity boundary layer at entrance region of a
tube

4 x Area of flow
Dh = ------------------------------ ……………………(5.20)
Wetted Perimeter

For flows through ducts it has been observed that turbulent flow prevails for

Red ≥ 2300 ………………………………………..(6.21)

But this critical value is strongly dependent on the surface roughness, the inlet conditions and
the fluctuations in the flow. In general, transition may occur in the range 2000 < Red < 4000.
It is a common practice to assume a value of 2300 fro transition from laminar flow to turbulent
flow.

6.4.2. Friction Factor and Pressure Drop Relations For Hydrodynamically Developed
Laminar Flow

In engineering applications, the pressure gradient (dp / dz) associated with the flow is a
quantity of interest, because this decides the pumping power required to overcome thr frictional
losses in the pipe of a given length.
Consider a differential length dz of the tube at a distance z from the entrance and let
this length be in the fully developed region. The various forces acting on the fluid element in
the direction of flow are shown in Figure6.6.

287
(PA)z
(pA)z + d/dz(pA) dz

τw Sdz

Figure 6.6: Forces acting on a fluid element in fully developed floe region

Resultant force in the direction of motion = Fz = (pA)z –(pA)z+dz – τw Sdz

where S is the perimeter of the duct.


Using Taylor’s series expansion and neglecting higher order terms we can write

(pA)z+dz = (pA)z + d/dz(pA) dz

Therefore Fz = d/dz(pA) dz − τw Sdz

Rate of change of momentum in the direction of flow = 0 because the velocity u does not vary
with respect to z in the fully developed region.

Hence d/dz(pA) dz − τw Sdz = 0

For duct of uniform cross section A is constant. Therefore the above equation reduces to

dp/dz = − τw S /A ……………………………….(6.22)

For laminar flow τw = − μ (du / dr)|wall. Hence Equation (5.22) reduces to

dp μS
------ = -------- (du/dr)|wall ……………………….(6.23)
dz A

Equation(6.23) is not practical for the determination of (dp/dz), because it requires the
evaluation of the velocity gradient at the wall. Hence for engineering applications a parameter
called “friction factor, f ” is defined as follows:

(dp/dz) Dh
f = − ------------------- ………………………….(6.24a)
½ (ρum2)

288
Substituting for (dp/dz) from Equation (6.23) we have

(μS/A) (du/dr)|wall Dh
f = − ---------------------------- …………………..(6.24b)
½ (ρum2)

For a circular tube S = πDi, and A = πDi2 / 4. Hence Dh = Di

Hence for a circular tube Equation (6.24b) reduces to


f = − ------- (du/dr)|wall …………………………(6.24c)
(ρum2)

Also from Equation (5.24a) we have


(½ ) (ρum2) f
dp = − ----------------- dz
Dh

Integrating the above equation over a total length L of the tube we have

p2 (½ ) (ρum2) f L
∫dp = − ------------------- ∫dz
p1 Dh 0

or pressure drop = Δp = (p1 – p2) = ( ½ ) (L/Dh) f ρum2 …………………………(6.25)


.
Pumping power is given by P = V Δp ………………………………………...(6.26)
.
where V = volume flow rate of the fluid.

6.4.3. Thermal Boundary Layer for Flow Through Tubes: In the case of temperature
distribution in flow inside a tube, it is more difficult to visualize the development of thermal
boundary layer and the existence of thermally developed region. However under certain
heating or cooling conditions such as constant wall-heat flux or constant wall-temperature it
is possible to have thermally developed region.
Consider a laminar flow inside a circular tube subjected to uniform heat flux at
the wall. Let ‘r’ and ‘z’ be the radial and axial coordinates respectively and T(r,z) be the local
fluid temperature. A dimensionless temperature θ(r,z) is defined as

T(r,z) – Tw(z)
θ(r,z) = ------------------- ………………………………..(6.27a)
Tm(z) – Tw(z)

where Tw(r,z) = Tube wall-temperature and Tm(z) = Bulk mean temperature of the fluid.

289
The bulk mean temperature at any cross section ‘z’ is defined as follows:

∫ ρ(2πrdr) u(r,z) Cp T(r,z) ∫ rdr u(r,z)T(r,z)


Tm(z) = --------------------------------- = ---------------------- ………………..(6.27b)
∫ ρ(2πrdr) u(r,z) Cp ∫ rdr) u(r,z)

At the tube wall it is clear that θ(r,z) = 0 and attains some finite value at the centre of the tube.
Thus we can visualize the development of thermal boundary layer along the tube surface as
shown in Figure 6.7.The thickness of the thermal boundary layer δt continuously grows along
the tube surface until it fills the entire tube. The region from the tube inlet to the hypothetical
location where the thermal boundary layer thickness reaches the tube centre is called the
“thermal entry section”. In this region the shape of the dimensionless temperature profile θ(r,z)
changes both in axial and in radial directions. The region beyond the thermal entry section is
called as the “thermally developed region”, because in this region the dimensionless
temperature profile θ remains invariant with respect to z. That is in this region θ = θ(r).It is
difficult to explain qualitatively why θ should be independent of z even though the temperature
of the fluid T depends both on r and z. However it can be shown mathematically that ,for both
constant wall-heat flux and constant wall-temperature conditions, θ depends only on r for large
values of z.For constant wall-heat flux condition the wall-temperature Tw(z) increases with z.

Tw(z) Tw(z) Tw(z)


Tfi

Thermally Developing Region Thermaly


Developed Region
Thermal Entrance Length Lth
θ = θ(r)

Figure 6.7: Development of Thermal Boundary Layer In a Flow Through


A Tube Subjected to Constant Wall-Heat Flux Condition

The variation of wall-temperature and the bulk fluid temperature as we proceed along the
length of the tube for constant wall-heat flux conditions is shown in Figure 6.8. It can be shown
that for constant wall-heat flux condition the temperature difference ΔT between the tube wall
and the bulk fluid remains constant along the length of the tube in the thermally developed
region
The growth of the thermal boundary layer for constant wall-temperature conditions is
similar to that for constant wall-heat flux condition except that the wall temperature does not

290
vary with respect to z. Therefore the temperature profile T(r,z) becomes flatter and flatter as
shown in Figure 6.9 as we proceed along the length of the tube and eventually the fluid
temperature becomes equal to the wall temperature. Since the wall-temperature remains
constant and the bulk fluid temperature varies along the length the temperature difference
between the tube wall and the bulk fluid varies along the length of the tube as shown in Figure
5.10.

Tw(z)|z=L

ΔT = Tw(z) – Tm(z) = Const


Tm(z)|z=L = Tfo

0 L

Lth Thermally developed region


z θ = θ(r)

Figure 6.8: Variation of tube wall-temperature and bulk fluid


temperature along the length of the tube

291
Tw Tw Tw
Tw
Tfi

z
Thermally developing region Thermally developed
Thermal entrance length Lth region
Figure6.9: Growth of thermal boundary layer for flow through a tube with
constant wall-temperature

Tw
ΔTo
Tfo

ΔTi

Tfi

L
0
z
Figure 6.10: Variation of bulk fluid temperature along the length of the
tube
for tube with constant wall-temperature

6.4.4. Mean Temperature Difference, ΔTm: If Q is the total heat transfer rate between the
fluid and the tube surface , As is the area of contact between the fluid and the surface, hm is the
average heat transfer coefficient for the total length of the tube then we can write

Q = hm As ΔTm …………………………….(6.28)

292
Where ΔTm = mean temperature difference between the tube wall and the bulk fluid.An
expression for ΔTm can be obtained in terms of ΔTi and ΔTo where ΔTi is the temperature
difference between the tube-wall and the bulk fluid at inlet and ΔTo is the temperature
difference between the tube-wall and the bulk fluid at exit as shown below.

dQ

mC
mmpTm mCp(Tm+dTm)

dz
z

Consider a control volume consisting an elemental length dz of the tube at a distance z from
the inlet as shown in Figure5.11. Let Tm be the bulk temperature of the fluid entering the control
volume and Tm + dTm be the bulk temperature of the fluid leaving the control volume. Let dQ
be the heat transfer rate from the tube wall to the fluid over the length dz.

Energy balance for the control volume neglecting the changes in kinetic and potential energies
in comparison with change in enthalpy gives

mCpTm + dQ = mCp(Tm+dTm) (6.29 a)

and dQ = h S dz [Tw(z) – Tm(z)] (6.29b)

where S is the perimeter of the tube.From the above two equations wer have

dTm hSdz
-------------------- = -------------- (6.29c)
[Tw(z) – Tm(z)] m Cp

For a tube with constant wall-heat flux condition, since the temperature difference between the
fluid and the tube surface remains constant along the length of the tube it follows that
ΔTm = [Tw(z)|z=0 − Tfi] = [Tw(z)|z=L − Tfo] …………………(6.29d)

For a tube with constant wall-temperature condition the mean temperature difference is given
by

ΔTm = (ΔTi – ΔTo) / ln (ΔTi / ΔTo) ……………………… (6.29e)

293
6.4.5. General expression for heat transfer coefficient : Let the fluid be heated as it flows
through the tube. Then at any z the heat flux from the tube surface to the fluid is given by
Fourier’s law as
qw(z) = k (∂T / ∂r)|wall ……………………(6.30)

[Note that when the fluid is heated Tw> Tm so that (∂T / ∂r)|wall will be positive).
If hz is the heat transfer coefficient then

qw(z) = hz [Tw(z) – Tm(z)] …………….(6.31)

Therefore from Equation (6.30) and (6.31) we have

k (∂T / ∂r)|wall
hz = ------------------- ……………………(6.32)
[Tw(z) – Tm(z)]

[T(r,z) – Tw(z)]
Now θ(r,z) = --------------------
[Tm(z) – Tw(z)]

Therefore (∂T / ∂r)|wall = [Tm(z) – Tw(z)] (∂θ / ∂r)|wall

Substituting this expression in Equation (6.32) and simplifying we get

hz = − k (∂θ / ∂r)|wall ………………………….(6.33)

In the thermally developed region θ depends only on r. Hence

hz = − k (dθ / dr)|wall …………………………(6.34)

Since (dθ / dr)|wall is independent of z it follows that the heat transfer coefficient hz is
independent of z. This is true both for constant wall-temperature and constant wall-heat flux
conditions.

Illustrative Examples on Flow Through Ducts:

Example 6.5:- The velocity profile for hydrodynamically developed laminar flow inside a
circular tube of radius R is given by

u(r) = 2um[ 1 – (r/R)2]

294
where um is the average velocity of the fluid in the tube.Develop an expression for the friction
factor f and express it in terms of the Reynolds number Red where Red is defined as Red = (umD)
/ ν.

Solution:
u(r) = 2um[ 1 – (r/R)2]

Therefore (du / dr) = 2um [ 0 – 2r /R2]

Or (du / dr)|wall = (du / dr)|r = R = − 4um / R = − 8um / D

Therefore (dp/dz) = − (S/A)τw = −[(πD) /(πD2/4)] {−μ(du / dr)|wall}

= (4μ/D)( −8um / D ) = − 32 μum /D2

− (dp/dz) D 32 μum /D 64
Friction factor = f = ----------------- = --------------- = --------------
½ (ρum2) ½ (ρum2) (ρumD / μ)

Or f = 64 / Red

Example 6.6:- The velocity profile u(y) for hydro dynamically developed laminar
flow between two parallel plates a distance 2L apart is given by u(y) / um = (3 / 2)
[1– (y /L) 2]where um is the mean flow velocity and the coordinate axis y is as shown
in Figure P6.6.

2L
o x

H
Figure P 6.6 : Schematic for problem 5.6

(a)Develop an expression for the friction factor f.


(b) Write the expression for calculating the pressure drop Δp over a length H of the channel.

295
y
dz

z τw1

p
z p+ (dp/dz)dz
2L
τw2
H

Figure P6.6a : Schematic for problem


6.6
Solution:

Consider a fluid element of length dz at a distance z from the origin as shown in the figure.
Resultant force acting on the fluid element is given by

F = p ( 2L x 1) – [p + (dp/dz) dz] (2L x 1) – τw1 (dz x 1) – τw2 (dz x 1)

= − 2L (dp/dz)dz − (τw1 + τw2) dz

For fully developed flow there is no change in the momentum of the fluid in z-direction.
Hence by Newton’s second law F = 0.

Therefore we have − 2L (dp/dz)dz − (τw1 + τw2) dz = 0

Or (dp/dz) = − (τw1 + τw2) / 2L ………………………….(1).

It is given that,
u = (3/2) um [ 1 – (y/L)2 ]

Therefore (du / dy) = − (3/2)(2y / L2)

By Newton’s law of viscosity τw1 = − μ(du/dy)|y=L = − μ [ − 3um / L]

= (3μum)/L

Similarly τw2 = + μ(du/dy)|y=−L = + μ [ + 3um / L] = (3μum)/L

Substituting these expressions for τw1 and τw2 we have

(dp/dz) = − [(3μum)/L + (3μum)/L ] / 2L

= − [(3μum)/ L2]

296
(a) The friction factor f is given by
− (dp/dz) dh [(3μum)/ L2] dh
f = --------------------- = ------------------------
(1/2) ρum2 (1/2) ρum2

4 x 2L
as ( dp/dz) = (1/2) ρ um2 f (1/dh), dh = hyd. Diameter = -------------- = 4L
2
12 x 2 24
= ------------------- = ----------------
(ρumL)/μ ReL

b) The total pressure drop for length H of the plate is given by

p2 H
Δp = p1 − p2 = − ∫dp = − ∫ −[(3μum) / L2]dz = 3 (H/L)(μum / L)
p1 0

3 (H/L) (ρum2)
= --------------------- = 3 (H/L) (ρum2) / ReL
(ρumL/μ)

Example 6.7:-The friction factor for hydro dynamically developed laminar flow
through a circular tube is given by

f = 64 / Red ; Red = (umd) / ν.

Water at a mean temperature of 60 0 C and a mean velocity of 10 cm/s flows inside a tube of 1
cm ID. Calculate the pressure drop for a length of 10 m of the tube and also the corresponding
pumping power required.

Solution:
Properties of water at 60 0 C are : ρ = 985 kg/m3 ; μ = 0.78 x 10 − 3 kg / (m – s);
Mean velocity of water = um = 0.1 m/s ; Di = 0.01 m ; L = 10 m.

985 x 0.1 x 0.01


Red = (ρumDi ) / μ = ------------------------- = 2060.7 or 2061
0.78 x 10 − 3

Friction factor = f = 64/Red = 64 / 2061 = 0.031

0.031 x 10 x 985x (0.1)2


Pressure drop = Δp = f (L/Di) (1/2)ρum2 = ----------------------------------
2 x 0.01

297
= 152.68 N / m2.

. π x (0.01)2 x 0.1
Volume flow rate = V = (πDi2/4) um = ------------------------ = 7.85 x 10 − 6 m3/s
4
.
Pumping power = Δp V = 152.68 x 7.85 x 10 − 6 = 1198.5 x 10 − 6 J / s.

Example 6.8:- Engine oil [ν = 0.8 x 10 ─ 4 m2/s and k = 0.14 W /(m-K)] is in


laminar flow between two parallel plates a distance 3 cm apart and subjected to a
constant heat flux of 2500 W/m2. The average heat transfer coefficient for the hydro
dynamically and thermally developed flow is given by

(hm 4L)/k = 8.235,

where 2L is the distance between the plates. Calculate the temperature difference between the
plate surface and the mean fluid temperature.

Solution:

2L = 0.03 m ; ν = 0.8 x 10 − 4 m2 / s ; k = 0.14 W /(m-K) ; q = 2500 W / m2;

(hm4L) / k = 8.235

Therefore hm = (8.235 k) / (2 x 2L)

= (8.235 x 0.14)/(2 x 0.03) = 19.215 W / (m2 – K)

Temperature difference = ΔT = q / hm = 2500 / 19.215 = 130.11 0 C.

6.5. Concepts of Turbulent Flow:

6.5.1.Introduction: Osborne Reynolds conducted experiments on flow through a transperent


circular tube and showed that at low flow rates the flow was streamlined; but as the flow rate
was increased, the streamlines become unstable and the laminar motion changed to turbulent
flow: i.e. the fluid motion become chaotic in nature and involves crosswise mixing or eddying
superimposed on the motion of the main stream.
The crosswise mixing in turbulent flow is advantageous in the sense that it assists
greatly in improving the heat transfer, but it has the disadvantage of increasing the resistance
to flow. The flow pattern in turbulent flow is quite complex as they consist of a spectrum of
coexisting vortices which are varying in size from the larger ones to those of microscopic
dimensions. For example, in the case of turbulent flow through a tube , the size of the vortices

298
varies from a significant fraction of the tube radius at the central region of the tube to
microscopic sizes near the wall where the velocity approaches zero.
In turbulent flow, the fluid properties such as velocity, pressure, and temperature are
subject to fluctuations both with the location in the flow field and with time. Hence the
instantaneous values of these properties can be represented as a sum of a time averaged mean
part and a fluctuating part as follows:
_ _ _ _
u = u + u’; v = v + v’ ; T = T + T’ ; p = p + p’ …………………………………..6.35
_ _ _ _
where u,v,T and p are the instantaneous values ; u,v,T and p are the time-averaged values and
u’,v’,T’ and p’ are the fluctuations. If the temperature of a flowing fluid is recorded at a location
in the flow field and if the flow is turbulent at that location, then the plot of temperature versus
time will be as illustrated in Figure6.11. In turbulent flow, the fluctuations are considered
superimposed on local average flow.The time averaged value of temperature, for example, can
be defined as

__ 1 t + Δt
T = ------ ∫ T dt …………………………6.36
Δt t

T
T’ = Temperature Fluctuation

__
T = Time –Averaged Temperature

Time,t
Fig: 6.11. Temperature fluctuations with time in turbulent flow

where Δt is a very small time interval which is large enough to record the turbulent fluctuations
but small for the temperature to be unaffected by the external disturbances on the
system.Similar averaging process can be defined for velocity and pressure (according to
Equation5.36, the time average of fluctuating quantities are all equal to zero).The effects of
these fluctuations occurring in velocity and temperature in turbulent flow are to increase heat
transfer and resistance to fluid flow.

299
Consider turbulent flow along a flat plate as shown in Figure 6.12.The y coordinate
is measured normal to the plate surface. Let u, and v be the instantaneous

u∞
y

u(x, y)
u(x, y)

x
xcr Turbulent Region
Turbulent Layer
Laminar Region Transition
Buffer Layer

Viscous sublayer
Figure 6.12: Turbulent boundary layer for flow over a flat plate

values of the components of of velocities along x and y directions and T be the instantaneous
value of the temperature. Let u’,v’ and T’ be the corresponding fluctuations respectively.Let τ
be the shear stress in x direction at a location y from the plate surface.This shear stress can be
considered as to be composed of two parts:
_
viscous shear stress resulting from the mean flow velocity u and turbulent shear stress
resulting from the velocity fluctuations u’ and v’. Then we can write

τ = τviscous + τturbulent …………………………..6.37a


_
where τviscous = μ(ðu / ðy) …………………………….6.37b
___
τturbulent = ─ ρ u’v’ …………………………….6.37c

Here the viscous shear stress term given by Equation 6.37b is the laminar flow contribution.
The physical significance of the turbulent shear stress term given by Equation 6.37c is not quite
apparent,but it can be derived by introducing the instantaneous velocities u and v in the x
momentum equation (See Ref.1) and by applying the rules of averaging to terms involving the
cross products of fluctuations.
Now if q is the heat flux in the y direction, then q can be considered as composed of two
parts: the diffusive or conductive heat flux due to gradient of mean temperature and the
turbulent heat flux resulting from the temperature fluctuations T’ and the velocity fluctuations
v’ in the y direction. With this consideration, we can write

300
q = qdiffusive + qturbulent ………………………….6.38a
_
Where qdiffusive = ─ k (ðT / ðy)………………………...6.38b
____
qturbulent = ρcp v’T’ ……………………………...6.38c.

Here the diffusive heat flux given by Equation 6.38b is similar to that encountered in
conduction heat transfer, but the significance of turbulent heat flux given by Equation 6.38c is
not quite apparent.It is derived by introducing the instantaneous velovities u and v and the
instantaneous temperature T in the energy equation (See Ref.1) for forced convection and time
averaging these equations.The relations for turbulent shear stress and turbulent heat flux given
by equations 6.37c and 6.38c are not useful for computational purposes unless they are related
to the mean quantities ū and T. A number of models heve been proposed to develop such
relations by semi-empirical hypothesis. For example the ‘mixing length model” proposed by
Prandtl and von Karman have been used extensively and successively in relating turbulent
shear stress and turbulent heat flux to the gradients of average velocity and average
temperature.

6.5.2. Prandtl’s Mixing Length Model: The basic idea behind this model is analogus to the
concept of “mean free path” for the motion of molecules in kinetic theory of gases. The main
difference is that in turbulent flow macroscopic lumps are envisioned.That is, in turbulent flow
in the x-direction along the surface, Prandtl postulated that fluid particles at a distance y from
the wall surface coalesce into macroscopic lumps and then travel, on the average a distance l
in the direction normal to the main flow while retaining their x-direction momentum before
they are dispersed.Thus , if the slow moving lumps enter the fast-moving layer, they act as
drag on the fast moving layer, and the momentum is transferred between layers as a result of
transeverse mixing. It is to be noted that l is an unknown quantity, and in reality there is no
such clear defined distance. Though the concept lacks generality, it has been found useful in
most engineering applications.According to Prandtl’s theory, the velocity fluctuations can be
related to
(ðū / ðy) by the following equations.

u’ ≈ l1(ðū / ðy) ; v’ ≈ l2(ðū / ðy) ……………………………6.39a,b

where u’ and v’ are of opposite sign and l1 and l2 are the “mixing lengths for momentum
transport”.Similarly the temperature fluctuation is related to the gradient of mean
tempaerature as _
T’ = l3 (ðT / ðy) ……………………………..6.39c

Where l3 is the “mixing length for energy transport”. Now the turbulent shear stress can be
written as:

___
τturbulent = ─ ρ u’v’ = ρl1l2 (ðū / ðy)2 = ρlm2 (ðū / ðy)2

301
or τturbulent = ρεm (ðū / ðy) ……………………………………..6.40a

where l1l2 = lm2 and εm = lm2 (ðū / ðy) …………………………….6.40b,c

Similarly the turbulent heat flux can be written as:


___ __
qturbulent = ─ ρ T’v’ = ρl3l2 (ðū / ðy) (ðT / ðy) = ρlh2 (ðū / ðy)(ðT / ðy)
__
qturbulent = ρεh(ðT / ðy) …………………………………….5.41a

where l2l3 = lh2 and εh = lm2 (ðū / ðy) …………………………..5.41b,c

εm and εh are called as “eddy diffusivity for momentum transfer” and “eddy difussivity for
heat transfer” respectively. Therefore for turbulent flow the total shear stress and the total
heat heat flux can be written as follows:

(τ / ρ) = (υ + εm) (ðū / ðy) ……………………………….6.42a


__
(q / ρcp) = ─ (α + εh) (ðT / ðy) …………………………..6.42b

Where υ is the kinematic viscosity and α is the thermal diffusivity of the fluid. Equations 6.42a
and 6.42b clearly demonstrates the effects of turbulent flow in enhancing the drag as well as
the heat transfer. Depending on the level of turbulence in the flow field, the turbulent transport
properties εm and εh may be an order of magnitude larger than the diffusive properties υ and α.
In general εm and εh are not the same. Their ratio is defined as “turbulent Prandtl number, Prt”.
Thus

Prt = εm / εh ………………………….6.43.

This definition is analogus to the definition of Prandtl number Pr:

Pr = υ / α ……………………………..6.44.

The physical significance of Pr and Prt should be distinguished. The Prandtl number is a
physical property of the fluid. It varies from a value much less than 1 for liquid metals to values
much greater than 1 for liquds and ois. It is about 1 for gases. The turbulent Prandtl number is
a property of the flow field and various models have been proposed for its determination. The
simplest model is due to Reynolds who assumed Prt = 1, which implies that heat and
momentum transfer in turbulent flow takes place exactly by the same process. The numerical
value of Prt may vary between 1 and 2.

6.5.3.Velocity Distribution in Turbulent Flow: Velocity distribution in turbulent flow has


been investigated extensively due to its importance in practice.But no fundamental theory is
yet available to determine this distribution by purely theoretical approaches. Therefore,
empirical and semi-empirical relations are used to correlate the velocity in turbulent flow.

302
One of the velocity distribution which has been used widely is based on the concept
of dividing the turbulent flow field into three distinct layers as illustrated in Figure5.10.A very
thin layer adjacent ot the wall where laminar or viscous shear stress dominate is called the
“viscous sublayer”. Adjacent to this layer is the “buffer layer”, where the viscous and
turbulent shear stresses are of the same order of magnitude and the third layer next to the
buffer layer is called “turbulent layer”, where the turbulent shear stress dominates.For the
steady, turbulent flow of an incompressible, constant property fluid over a smooth surface the
velocity distribution is expressed using the following dimensionless parameters:

ū
+
u = ------------ = Dimensionless velocity
√(τw / ρ)

y
y = ------ √(τw / ρ) = Dimensionless distance
+

where ρ is the density, τw is the shear stress at the wall, υ is the kinematic viscosity and ū
component of mean velocity parallelto the wall surface.
Experiments have shown that for region where y + < 5, turbulent shear stress
contribution is negligible and the region is viscous sublayer.Therefore the shear stress takes
the form τw = μ(ðū / ðy). Integration of this expression for constant τw with ū = 0 at y = 0 gives
the following distribution for viscous sublayer:

u + = y + for 0 < y + < 5 ……………………6.45.

The buffer layer is considered to be in the region between y + = 5 and y + = 30. In this region
the velocity distribution is assumed to follow the logarithmic law of the form
u + = A ln y + + B. The constants A and B are determined from requirement that at
y + = 5 ,velocity u + be equal to that of viscous sublayer and be equal to that of turbulent layer
at y + = 30.Hence for the buffer region the velocity distribution will be of the form

u + = 5.0 ln y + ─ 3.05 for 5 ≤ y + ≤ 30 ………. 6.46

The region y + > 30 is considered to be the “turbulent layer” where the laminar shear
stress is negligible in comparison to the turbulent shear stress. By using the mixing length
concept and assuming that the mixing length varies linearly with the distance from the wall in
the form l = κy, it can be shown that the velocity distribution in the turbulent layer has a
logarithmic profile in the form

u + = (1/κ ) ln y + + C …………………………….6.47

where κ is called the universal constant. Experiments have shown that κ = 0.4 and the constant
C has been determined by the correlation of Equation 6.47 with the measured velocity profile.
For turbulent flow inside a smooth pipe, C = 5.5. Then the velocity distribution in the turbulent
layer is given as

303
u + = 2.5 ln y + + 5.5 for turbulent layer y + > 30 ……..6.48

Though the above velocity distribution [Equation6.45, Equation 6.46 and Equation 6.48] by
dividing the turbulent flow field into three distinct regions appears to be in reasonably good
agreement with the experimental data, the transition from a viscous to turbulent flow regime
takes place gradually.Therefore , he representation of of velocity distribution by three different
curves having discontinuous slopes at locations where they join is not realistis.A more serious
inconsistency of the logarithmic velocity distribution given by Equation5.48 is that it does not
give zero velocity gradient at the tube centre. For this reason the average velocity for flow
inside a pipe as determined using the above equations overestimates the velocity.But these
equations have been used extensively in literature to study the relation between momentum
and heat transfer in turbulent flow.

6.5.4.Effects of Surface Roughness on Velocity Distribution: The velocity distribution


discussed above is applicable to turbulent flow over “hydrodynamically smooth surfaces”. A
surface is considered as hydrodynamically smooth if the heights ‘λ’ of the protrusions are much
smaller than the thickness of the viscous sublayer. Surfaces encountered in engineering
applications generally are not smooth and since viscous sublayer is very thin, the protrusions
may penetrate it.Nikuradse made extensive experiments with turbulent flow inside artificially
roughned pipe over a wide range of “relative roughness, λ /Di” from about 1 / 1000 to 1 / 30.
The sand – grain roughness used in these experiments has been adopted as standard for the
effects of roughness. The experiments showed that to study the effects of roughness, it is
desirable to introduce a “roughness Reynolds number λ+” defined as

λ
λ = ---- √(τw / ρ) …………………………….6.49
+

υ
If λ < 5, the roughness has no effect on the friction factor.With this consideration three
+

distinct situations are envisioned for the effects of roughness:

(i) Hydrodynamically smooth surface: 0 ≤ λ + ≤ 5 ……………………..6.50a

(ii) Transitional: 5 ≤ λ + ≤ 70 …………………….6.50b

(iii) Fully rough: λ + > 70 ………………………6.50c

For the hydrodynamically smooth case, the heights of roughness are so small that all
protrusions are covered by the viscous sublayer and hence roughness has no effect on friction
factor. For the transitional case, the protrusions are partly outside the viscous sublayer and
cause some additional rsistance to flow. For the fully rough case, the heights of protrusions are
so large that all protrusions penetrate the viscous sublayer; hence the viscous sublayer no
longer exists and protrusions influence the turbulent mixing.
For the fully turbulent region (y + > 30),the logarithmic velocity profile guven by
Equation 6.47 is applicable if y + is replaced by y / λ. In that acse the velocity distribution is
given by

304
u + = (1/κ ) ln (y / λ )+ C ……………………….6.51

where κ = 0.4 as in the case of smooth wall but the constant C is different. A correlation of this
relation with Nikuradse’s experiments has shown that C = 8.5. Then Equation 5.51 takes the
form

u + = (1/κ ) ln (y / λ )+ 8.5 ………………………6.52

Equation 6.52. is called logarithmic velocity distribution for turbulent flow in rough pipes in
the fully rough region.
A significant difference between laminar and turbulent flow lies in the fact that, in
turbulent flow the velocity profile is affected by the surface roughness, where as in laminar
flow, the velocity profile is not affected by the surface roughness. Since the velocity
distribution affects the shear stress at the wall and hence the friction factor, it is expected that
in turbulent flow the friction factor will depend not only on the Reynplds number but also on
the relative roughness, where as in laminar flow the friction factor is a function of Reynolds
number only.

Illustrative Examples on Concepts of Turbulence:


Example 6.11.Starting with the definition of the total shear stress in the turbulent flow given
by ___ ___
τ = μ(ðu / ðy) ─ ρ u’v’ , u’v’ = ─ lm2 (ðū / ðy)2 and lm = κy

derive the logarithmic velocity distribution, u + = (1/κ ) ln y + + C for the fully turbulent
region.
ū y
Where u + = ------------ and y + = ------ √(τw / ρ)
√(τw / ρ) ν

Example 6.12. Show that the velocity distribution in the viscous sublayer is given by

u+ = y +
with u+ and y+ are as defined in example 5.11.

6.6. GOVERNING EQUATIONS FOR FLUID FLOW

We have seen that, in order to determine the drag force for flow over a flat plate or friction
factor for fully developed flow through tubes, it is necessary to know the velocity distribution
in the flow field. The velocity distribution can be found from the solution of the governing
equations of motion subject to appropriate boundary conditions. The physical significance of
various parameters affecting the drag coefficient and friction factor can be understood better if
we have some knowledge of the equations governing fluid flow. Therefore the governing
equations of continuity, momentum and energy for two-dimensional, constant property steady
flow system is derived in this section using rectangular coordinate system in x and y variables.

305
The equivalent equations in the cylindrical coordinate system in (r,z) variables are also
presented.
Assumptions made while deriving the governing equations:

(i) Flow is steady and two dimensional


(ii) Fluid properties are constant

6.6.1.Continuity Equation

The continuity equation is obtained by applying the law of conservation mass to a differential
volume element of dimensions Δx and Δy in x and y directions with unit dimension in z
direction as shown in Figure6.13.

y
𝜕
ρv(Δx.1) + [ρv(Δx. 1) ]𝛥𝑦
𝜕𝑦

𝜕
ρu(Δy.1) ρu(Δy.1) + [ρu(Δy. 1) ]𝛥𝑥
𝜕𝑥
x

y ρv(Δx.1)

306
EXERCISE PROBLEMS:

6.1.Air at 27 0C and 1 atm. flows over a flat plate at a speed of 2 m/s. Calculate the velocity
boundary layer thickness at distances of 20 cm and 40 cm from the leading edge of the plate.
Also calculate the mass flow of air that enters the boundadry layer between x = 20 cm and x
= 40 cm.Assume that the viscosity of air at 270C is 1.85 × 10 -5 kg/(m-s).Assume unit width of
the plate.The velocity boundary layer thickness δ at any x is given by
𝛿 4.64
=
𝑥 √𝑅𝑒𝑥

Where Rex is the Reynolds number at any x. Assume the velocity profile at any x to be of
the form
𝑢(𝑦) 3 𝑦 1 𝑦 3
𝑈∞
= 2 (𝛿 )− 2 (𝛿 )
Where U∞ is the free stream velocity of air.

6.2.A 1.0 kW heater is constructed of a glass plate with an electrically conducting film which
produces a constant heat flux. The plate dimensions are 60 cm X 60 cm and is placed in an
air stream at 270C and 1 atm. The free stream velocity of air is 5 m/s. Calculate the average
temperature difference along the plate and the temperature difference at the trailing edge of
the plate. Assume that for the plate with constant heat flux the local Nusselt number is given
by
Nux = 0.453 √𝑅𝑒𝑥 (𝑃𝑟)1/3

Where Pr is the fluid Prandtl number.

6.3. Air at 270C and 1 atm. flows over a 20 cm square plate at a free stream velocity of 20
m/s. The last half of the plate is heated at maintained at a uniform temperature of 770C.
Calculate the heat lost by the plate. Assume the following correlation to calculate the local
Nusselt number for the heated portion of the plate:

𝑥 3/4 − 1/3
Nux = 0.332 (𝑃𝑟)1/3 (𝑅𝑒𝑥 )1/2 [1 − ( 𝑥0 ) ]
where x0 is the unheated starting length of the plate. Take the properties of air at the
arithmetic mean temperature of the plate and the free stream.

307
Chapter 7

Forced Convection Heat Transfer For Flow


Through Ducts
7.1. Introduction:

Heat transfer and pressure drop calculations in flow through ducts are required in many
industrial applications. Laminar flow through pipe is encountered generally in compact heat
exchangers, cryogenic systems, heating or cooling of heavy fluids like oils.For such flows
analytic expressions are available for the prediction of friction factor and heat transfer
coefficient.But most of the available correlations to determine heat transfer and friction factor
for turbulent flows are either empirical or semi-empirical in nature.

7.2. Analysis of Hydrodynamically and Thermally Developed Laminar Flow

7.2.1. Couette Flow: Consider a special case of parallel flow of an incompressible fluid
between two parallel plates where one plate is stationary and the other plate is
moving with an uniform velocity U.A distance L separates the two plates (refer Figure
7.1). The stationary plate is maintained at temperature To and the moving plate at
temperature TL. This type of flow is referred to as COUETTE flow and occurs, for
example, in journal bearing. The continuity, momentum and energy equations for
such a flow are given as follows:
For parallel flow along the plate the continuity equation (6.54b) reduces to

du / dx = 0 (7.1)
The momentum equation (6.59a) reduces to

d2u / dy 2 = 0 (7.2)

And the energy equation reduces to

k (d 2T / dy 2) + μ (du / dy) 2 = 0 (7.3)

308
y
TL
y
u(y) = U

To, u(y) = 0
Figure 7.1: Schematic for Couette Flow

The boundary conditions for the momentum equation are :

at y = 0, u(y) = 0 (7.4)

and at y = L, u(y) = U. (7.5)

Integrating equation (7.2 ) twice, we have

u(y) = C1y + C2 ………………………(7.6).

Condition (7.4) in equation (7.6) gives C2 = 0.

Condition (7.5) in equation (7.6) gives U = C1L or C1= U / L.

Substituting these values of C1 and C2 in equation (2) we get the velocity distribution as

u(y) = (Uy) / L ………………………(7.7)

From equation (7.7) (∂u / ∂y) = U / L substituting this in the energy equation (7.3)

We have k(d2T / dy2) = − μ(U/L)2

Or (d2T / dy2) = − (μ/k)(U/L)2

Integrating twice with respect to y we get

T(y) = − (μ/k)(U/L)2 (y2/2) + C1y + C2 …………….(7.8)

The boundary conditions for the enrgy equation are :

309
at y = 0 T = T0 (7.9)

and at y = L, T = TL. (7.10)

Condition (7.9) in equation (7.8) gives C2 = T0.

Condition (7.10) in equation (7..8) gives TL = − (1/2)(μU2/k) + C1L + T0

Or C1 = (1/L)[(TL – T0) + (1/2)(μU2/k)]

Substituting the expressions for C1 and C2 in Equation (7.8 ) we get the temperature as

T(y) = − (μ/k)(U/L)2 (y2/2) + (y /L) [(TL – T0) + (1/2)(μU2/k)] + T0

𝑦 𝜇𝑈 2 𝑦
Or T(y) – T0 = 𝐿 [(𝑇𝐿 − 𝑇0 ) + (1 − 𝐿 )] (7.11)
2𝑘

T(y)− T0 y 1 μ U2 y
= L [1 + (1 − L)] (7.12)
TL − T0 2 k (TL − T0 )

μ U2
Now the term k (T can be written as follows:
L − T0 )

μ U2 𝜇𝐶𝑝 μ U2
= = Pr × E where Pr is the Prndtl number and E is
k (TL − T0 ) 𝑘 Cp (TL − T0 )

Eckert number. Hence Wq.(7.11) can be written as follows:

T(y)− T0 y 1 y
= L [1 + Pr E (1 − L)] (7.13)
TL − T0 2

T(y)− T0 y
Figure 7.2 shows the plot of as a function of L for different values of the parameter
TL − T0
Pr×E.The case when Pr×E = 0 corresponds to no flow condition. Hence heat transfer from the
hot plate to cold plate is by conduction in y direction. Therefore the temperature distribution
is linear.The physical significance for other values of Pr×E can be understood better if heat
transfer at the wall (either at the upper wall or at the lower wall) is considered.The heat flux at
the upper wall is given by

q(y)| y = L = − k (dT /dy)| y = L

TL − T0 1
From Equation(7.12) we have (dT /dy)| y = L = (1 − Pr ∗ E)
L 2

TL − T0 1
Therefore q(y)| y = L = − k (1 − Pr ∗ E) (7.14)
L 2

310
The above result shows that the direction of heat flow at the upper wall for the case TL >
T0 depends on the value of the parameter Pr*E.

Chart Title
1.2

0.8

0.6

0.4

0.2

0
0 0.2 0.4 0.6 0.8 1 1.2

Series1 Series2 Series3

0.2 0.4 0.6 0.8 1.0 1.2 1.4 1.6

For Pr × E > 2, the right side of Equation(7.14) is positive. Therfore the heat flows in
the positive y direction, i.e., from the fluid into the wall though the upper wall is at higher
temperature than the lower wall.
For Pr × E < 2, the right side of Equation (7.14) is negative. Hence the heat flows in the
negative y direction,i.e., from the upper wall to the fluid.
For Pr × E = 2, the right hand side of Equation(7.14) reduces to zero. Hence there is no
heat flow at the upper wall.

When TL = T0, both plates are at the same temperature, Therfore from Equation (7.11)
the temperature distribution reduces to

𝑦 𝜇𝑈 2 𝑦
T(y) – T0 = 𝐿 [ (1 − 𝐿 )] (7.15)
2𝑘

It can be seen from the aboe equation that the maximum temperature occurs at y = L/2 and
the maximum temperature will be

𝜇𝑈 2
Tmax – T0 = (7.16)
8𝑘

Combining Equation (7.15) and Equation (7.16) the the temperature distribution can be
expressed in terms of Tmax as
T(y)− T0 y y
= 4 L (1 − L) (7.17)
T −T L 0

311
Illustrative examples on Couette flow:

Example7.1:A heavy lubricating oil[μ = 0.25 kg/(m-s), k = 0.125 W/(m-K)] at room


temperature flows in the clearance between a journal andits bearing. Assuming that both the
journal and the bearing are at the same temperature, determine the maximum temperature rise
in the fluid for velocity U = 6 m/s.

Schematic: See Figure7.1

Known: For the oil, μ = 0.25 kg/(m-s), k = 0.125 W/(m-K); U = 6 m/s; T0 = TL

Find: Tmax – T0

Assumptions: (i)The oil properties are constant; (ii) the flow of oil in the space between the
journal and its bearing is considered as flow between two parallel plates; i.e. the flow is couette
flow.

Solution: For Couette flow when T0 = TL, the maximum temperature rise is given by
Equation(7.16): i.e.

𝜇𝑈 2 0.25∗ 62
ΔTmax = Tmax – T0 = = = 9 0C
8𝑘 8∗0.125

Example 7.2.A lubricating oil of viscosity μ and thermal conductivity k fills the clearance L
between two rotating cylinders, which can be regarded as two parallel plates in motion for
the purpose of analysis. The velocity of the inner cylinder is U1 and that of the outer cylinder
is U2.Find the relation for the velocity distribution in the oil layer for (a) the inner and outer
cylinders are rotating in the same direction but U2 > U1 and (b) the inner and outer cylinders
are rotatibg in the opposite directions. Also find an expression for the shear stress in the fluid
resulting from the rotation.

Schematic: Case (a) :- U2 > U1 :See FigureE7.2a

Known: Viscosity of the oil = μ; Thermal conductivity of oil = k ; Clearnace between the two
rotating cylinders = L;velocity of lower plate=U1; Velocity of upper plate = U2

Find: (a) u(y) when both U1 and U2 are in same direction with U2 > U1; (b) u(y) and shear
stress when U2 is in opposite direction with respect to U2.

Assumptions: (i) Fluid properties are constant; (ii) The flow between the two rotating
cylinders is considered as flow between two parallel plates (Couette flow)

Solution: The momentum equation for Couette flow is

d2u / dy 2 = 0 (1)

312
y
y
u(y) = U2

L
u(y) = U1

Figure E7.2a: Schematic for Example 7.2,case(a)


Integrating Equation(1) twice we have

u(y) = C1y + C2 (2)

Substituting the condition that at y = 0, u = U1 we get

C2 = U1 (3)

Substituting the condition that at y = L, u = U2 in Equation (2) we get

U2 = C1L + U1

(U2 − U1 )
Or C1 = (3)
L

Substituting the expressions for C1 and C2 in Equation (2) we get the velocity distribution as
y
u(y) = (U2 − U1 ) + U1
L

𝐮(𝐲)− 𝐔𝟏 𝐲
or = (4)
𝐔𝟐 − 𝐔𝟏 𝐋

𝐝𝐮 (𝐔𝟐 − 𝐔𝟏 )
Shear stress is given by τ = μ (𝐝𝐲 ) = μ 𝐋
Schematic for Case (b) :- U2 = − U1 :See FigureE7.2b

313
y
y
u(y) = U2

L
u(y) = U1

Figure E7.2b: Schematic for Example 7.2,case(b)

The general solution Equation(7.2) holds good for this case also.

For this case at y = 0, u(y) = U1, Hence from Equation(7.2) C2 = U1

At y = L, u(y) = − U2. Substituting this in Equation(7.2) we have

− U2 = C1L + U1

(U1 + U2 )
Or C1 = − L

Hence the velocity distribution is given by


y
u(y) = − (U1 + U2 ) + U1
L

U1 − u(y) y
= (5)
U1 + U2 L

du (U1 + U2 )
Shear stress is given by τ = μ (dy) = − μ (6)
L

Example 7.3.Consider Couette flow with heat transfer for which the lower plate moves with a
velocity of U = 15 m/s and is perfectly insulated (see Figure E7.3). The upper plate is
stationary and is made of material with kup = 1.5 W/(m-K) and thickness Lup = 3 mm. Its outer
surface is maintained at Tup = 40 0 C. The plates are separated by a distance of L0 = 5 mm
which is filled with an engine oil of viscosity μ = 0.8N-s/m2 and thermal conductivity k0
=0.145W/(m- K).
(a) On T(y) ─ y coordinates, sketch the temperature distribution in the oil film and in the
moving plate.

314
(b) Obtain an expression for the temperature at the lower surface of the film T0 in terms of the
plate speed U, the stationary plate parameters Tup,kup,Lup and the oil parameters μ, k0, L0.
Calculate this temperature for the prescribed conditions.

Schematic for the example:

y Tup
Stationary Plate

Lup

oil L0

Moving plate
Figure E7.3: Schematic for example 7.3

Solution:

For Couette flow the momentum equation is :

d2u /dy2 = 0.

Integrating twice with respect to y we have

u(y) = C1y + C2 ……………………………………(1)

The boundary conditions are (i) at y = 0, u(y) = U ;

And (ii) at y = L0, u(y) = 0.

Condition (i) in equation (1) gives C2 = U and condition (ii) in equation (1) gives

C1 = − U / L0.

Substituting the expressions for C1 and C2 in equation (1) we get the velocity distribution as

u(y) = U [1 – (y/L0)] ……………………………..(2)

315
The energy equation for Couette flow is

k(d2T / dy2) = −μ (du/dy)2 ……………………………(3)

From equation (2) we have (du/dy) = − (U / L0).

Therefore equation (3) reduces to


k0(d2T / dy2) = −μ (U / L0)2

or (d2T / dy2) = −μ (U / L0)2 / k0

Integrating once with respect to y we have

dT / dy = −(μ/k0) (U / L0)2 y + C1 ………………(4)

Integrating once again with respect to y we have

T = −(μ/k0) (U / L0)2 (y2/2) + C1y + C2 ………………………...(5)

At y = 0 the surface is insulated i.e. (dT/dy) = 0. Substituting this condition in equation (4) we
have
C1= 0.
At y = L0, the condition is

− k0 (dT/dy)|y=L0 = kup[ T|y = L0 − Tup] /Lup………………(6)

From equation (4) we have (dT / dy )|y = L0 = −(μ/k0) (U / L0)2 L0 = − μU2/(L0k0).

From equation (5) we have T|y = L0 = −(μ/k0)(U2/2) + C2.

Therefore equation (6) reduces to μU2/L0 = kup [−(μ/k0)(U2/2) + C2 – Tup] / Lup

Or C2 = μ U2 / k0 [ ( ½) + ( k0 Lup/ kup L0) ] + Tup

Therefore T|y = L0 = Tup + { μ U2 / k0 [ ( ½) + ( k0 Lup/ kup L0) ] } − (μ/k0)(U2/2)

= Tup + ( k0 Lup/ kup L0)

Or Temperature distribution is given by :

T = -(μ/k0) ( U/ L0)2 (y2/2) + { μ U2 / k0 [ ( ½) + ( k0 Lup/ kup L0) ] } + Tup

At lower surface y = 0

Therefore T|y =0 = Tup + { μ U2 / k0 [ ( ½) + ( k0 Lup/ kup L0) ] }

316
7.2.2.Hydrodynamically and thermally developed laminar flow through a circular pipe:

7.1. Determine the friction factor, the pressure drop and pumping power for fully
developed laminar flow of water at 21 0 C [μ = 9.8 kg/(m-s) ; ρ = 997.4
kg/m3]through a 2.5 cm diameter, 100 m long tube for a mass flow rate of 0.015
kg/s. What are the mean and maximum velocities of flow?

7.2. Determine the friction factor, the pressure drop and pumping power required for
the flow of water at 0.5 kg/s and 40 0 C through a tube of square cross section of
2 cm x 2 cm and 12 m long. What would be the corresponding values if the pipe is
of equilateral-triangular cross section of side 2 cm and length 5 m ?

7.3. Water at 30 0 C with a mass flow rate of 2 kg/s enters a 2.5 cm-ID tube whose wall
is maintained at a uniform temperature of 90 0 C. Calculate the length of the tube
required to heat the water to 70 0 C.

7.4. Water at 20 0 C with a mass flow rate of 5 kg/s enters a circular tube of 5 cm-ID
and 10 m long. If the tube surface is maintained at 80 0 C, determine the exit
temperature of water.

7.5. Air at 27 0 C with a flow rate of 0.01 kg/s enters a rectangular tube 0.6 cm x 1.0
cm in cross section and 2 m long. The duct wall is subjected to a uniform heat flux
of 5 kW/m2. Determine the outlet temperature of air and the duct surface
temperature at the exit assuming that the flow is hydro dynamically and thermally
developed.

7.6. Three kg/min of liquid sodium is heated from a bulk mean temperature of 400 0 C
to 500 0 C, as it flows through a stainless steel tube of 5 cm-ID and 2 mm thick.
The sodium is heated by a constant wall-heat flux, which maintains the tube-wall
temperature at 30 0 C above the bulk mean temperature of sodium all along the
length of the tube. Calculate the length of the tube required. Assume the following
properties for liquid sodium.
ρ = 846.7 kg/m3 ; k = 68.34 W/(m-K) ; Cp = 1.274 kJ/(kg-K) ; Pr = 0.00468 ;
ν =0.2937 x 10 ─ 6 m2 /s.

7.7. Consider hydro dynamically and thermally developed turbulent flow of water with
a mass flow rate of M kg/s inside a circular tube of inside diameter ‘D’. The Dittus-
Boelter equation can be used to determine the heat transfer coefficient. If the
tube’s inside diameter is changed from D to D/2 while the mass flow remains same,
determine the resulting change in the heat transfer coefficient.

317
7.8. Mercury at a temperature of 100 0 C and with a velocity of 1 m/s enters a 1.25 cm
ID tube, which is maintained at a uniform temperature of 250 0 C. Determine the
length of the tube required to heat mercury to a temperature of 200 0C.

B. Hydrodynamic and thermal entry lengths:

7.9. Determine the hydro dynamic entry lengths for flow at 60 0 C and at a rate of 0.015
kg/s of water, ethylene glycol and engine oil through a circular tube of 2.5 cm ID.

7.10. Determine the hydro dynamic entry length, thermal entry length and the heat
transfer coefficient for fully developed flow for engine oil at 60 0 C flowing at a rate
of 0.01 kg/s through a square duct 1 cm x 1cm cross section and subjected
to a uniform wall-temperature. Assume the following physical properties for the engine
oil:
ρ = 864 kg/m3 ; Cp = 2047 J/(kg-K) ; k = 0.14 W/(m-K) ; μ = 0.0725 kg/(m-s) ;
Pr = 1050

C. Flow over a flat plate:

7.11. Atmospheric air at 25 0 C flows over both the surfaces of a flat plate 1 m long with
a velocity of 5 m/s. The plate is maintained at a uniform temperature of 750C.
(a) Determine the velocity boundary layer thickness, the surface shear stress and
the heat flux at the trailing edge of the plate.
(b) Determine the drag force on the plate and the total heat transfer from the plate to
air.

7.12. Air at 30 0 C flows with a velocity of 10 m/s along a flat plate 4 m long. The plate
is maintained at a uniform temperature of 130 0 C. Assuming a critical Reynolds
number of 2 x 10 5 and width of plate to be 1 m determine (a) the heat flux at the
trailing edge of the plate, (b) the heat transfer from the laminar portion of the
plate, (c) the total heat transfer from the plate and (d) the heat transfer from the
turbulent portion of the plate.

7.13. A highly conducting thin wall 2 m long separates the hot and cold air streams
flowing on both sides parallel to the plate surface. The hot stream at 250 0 C is
flowing with a velocity of 50 m/s while the cold stream at 50 0 C is flowing with a
velocity of 15 m/s. Calculate (a) the average heat transfer coefficients for both the
air streams and the heat transfer between the two streams per metre width of the
plate and (b) the local heat flux at the mid point of the plate. Assume that the wall
is at the arithmetic mean of the temperature of the two streams for the purposes
of calculating the physical properties of the two streams and the critical Reynolds
number to be 2 x 10 5.

7.14. A flat plate of width 1 metre is maintained at a uniform temperature of 150 0 C by


using independently controlled heat generating rectangular modules of thickness
10 mm and length 50 mm. Each module is insulated from its neighbours, as well
as its back side.(see Figure P 6.14). Atmospheric air at 25 0 C flows over the plate

318
at a velocity of 30 m/s. The thermo-physical properties of the module are : k = 5.2
W/(m-K) ; Cp = 320 kJ/(kg-K) ; ρ = 2300 kg / m3.

150 0 C
T∞ = 25 0 C
u∞ = 30 m/s

10 mm

700 mm

50 mm

Figure P 7.14 : Schematic for problem 7.14

(a) Find the required power generation in W/m 2 in a module positioned at a distance
of 700 mm from the leading edge of the plate.
(b) Find the maximum temperature in the heat generating module.

D. Flow across a cylinder:

7.15. A circular pipe of 25 mm OD is placed in an air stream at 25 0 C and 1 atm pressure.


The air moves in cross flow over the pipe at 15 /s, while the outer surface of the
pipe is maintained at 115 0 C. What is the drag force exerted on the pipe per unit
length of the pipe? What is the rate of heat transfer per unit length of the pipe?

7.16. A long cylindrical heating element [k = 240 W/(m-K), ρ = 2700 kg/m3 and Cp =
900 kJ/(kg-K)] of diameter 10 mm is installed in a duct in which air moves in cross
flow over the heating element at a temperature of 27 0 C with a velocity of 10 m/s.
(a) Estimate the steady state surface temperature of the heater when electrical energy
is being generated at a rate of 1000 W per metre length of the cylinder.
(b) If the heater is activated from an initial temperature of 27 0 C, estimate the time
required for the surface temperature to come to within 10 0 C of its steady state
value.

7.17. Air at 40 0 C flows over a long 25 mm diameter cylinder with an embedded electrical
heater. Measurements of the effect of the free stream velocity V on the power per
unit length P, required to maintain the cylinder surface temperature at 300 0 C
yielded the following results:

V (m/s) : 1 2 4 8 12

P (W/m) : 450 658 983 1507 1963

319
(a) Determine the convection coefficient for each of the above test conditions. Display
your results graphically.
(b) For the corresponding Reynolds number range, determine the suitable constants
C and m for use with an empirical correlation of the form Nu m = C Redm Pr1/3.

7.18. A thermocouple is inserted into a hot air duct to measure the air temperature. The
thermocouple (T1) is soldered to the tip of a steel thermocouple well of length 15
cm and inner and outer diameters of 5 mm and 10 mm respectively. A second
thermocouple (T2) is used to measure the duct wall temperature (see Figure P
6.18).

T2
Air
at T∞, u∞ L Steel well
Di

Do

Figure P 7.18 : Schematic for problem 7.18

Consider the conditions for which the air velocity in the duct u ∞ = 3 m/s and the
two thermocouples register temperatures of T1 = 450 K and T2 = 375 K. Neglecting
radiation determine the air temperature T∞. Assume that for steel k = 35 W/(m-
K), and for air ρ = 0.774 kg / m3, μ = 251 x 10 ─ 7 N-s / m2, k = 0.0373 W/(m-K),
and Pr = 0.686

E. Flow across tube bundles:

7.19 Air at atmospheric pressure and 30 0 C flows over a bank of tubes consisting 1 cm
OD tubes, 10 rows deep. The velocity of air before it enters the bundle is 1 m/s.
(a) Determine the friction factor and the pressure drop and (b) the average heat
transfer coefficient for the following cases:
(i) Tubes are in equilateral-triangular arrangement with ST / D = SD / D = 1.25
(ii) Tubes are in square arrangement with ST / D = SL / D = 1.25

7.20. Hot flue gases at 375 0 C flow across a tube bank consisting of 1.25 cm OD tubes,
which are maintained at a uniform surface temperature of 30 0 C by flowing water
through the tubes. The tube bundle is 10 rows deep in the direction of flow and
contains 40 tubes in each row. The tubes are 1 m long and have an in-line
arrangement with SL / D = ST / D = 2. he velocity of the flow gases entering the
tube matrix is 7 m/s. Determine the average heat transfer coefficient and the total
heat transfer rate. Assume that thermo-physical properties of the flow gases to be
same as that of air at any temperature.

320
A.Hydro-dynamically and Thermally developed flow through ducts
7.1. Solution:-

Water di
At 200C

Mass flow rate = m = 0.015 kg / s ; di = 0.025 m ; L = 100 m ;

Properties of water at 20 0C are: ρ = 1000 kg / m3 ; ν = 1.006 x 10 − 6 m2 / s;

Reynolds Number = Red = uav dh / ν, where uav = average velocity of the fluid in the pipe
and dh = hydraulic diameter for the pipe.

Now m = ρ(πdi2/4)uav.

4 x 0.015
Or uav = (4m) / ρ(πdi2) = --------------------------- = 0.0305 m / s
1000 x π x 0.0252

dh = di for a circular pipe.


0.0305 x 0.025
Therefore Red = ----------------------- = 757.95
1.006 x 10 − 6
Since Red < 2300, flow is laminar. For hydro-dynamically developed laminar flow we
have friction factor as

f = 64 / Red = 64 / 745.5 = 0.084.

Pressure drop for a total length L is given by Δp = (1/2)f (L/dh) ρuav2

= ½ x 0.084 x (100 / 0.025) x 1000 x 0.03052 = 156.28 N/m2.

Pumping power = Δp (m/ρ) = 156.28 x 0.015 / 1000 = 2.34 x 10− 3 W

321
The velocity of the fluid at any radius for fully developed laminar flow through a circular
tube is given by

u(r) = 2uav [1 – (r/R)2]

Therefore umax = u(r)r=0 = 2 uav = 2 x 0.0305 = 0.061 m/s

7.2. Solution

Water
At 400C a

Mass flow rate = m = 0.5 kg / s ; a = b = 0.02 m ; L = 12 m ;

Properties of water at 40 0C are: ρ = 994.59 kg / m3 ; ν = 0.658 x 10 − 6 m2 / s;

0.5
uav = m / (ρab) = -------------------- = 1.26 m / s.
994.59 x 0.022

Hydraulic diameter = dh = 4ab / 2(a + b) = 2ab / (a + b) = 2a2/2a = a = 0.02 m.

1.26 x 0.02
Reynolds number Red = uav dh / ν = ------------------- = 38299
0.658 x 10 − 6

Since Red > 2300 flow is turbulent.

For fully developed turbulent flow through a pipe of square cross section the friction factor f
is given by (Moody chart, smooth pipe)

f = 0.02175.

Pressure drop = Δp = (1/2)f (L/dh) ρuav2 = 0.5 x 0.02175 x (12 / 0.02) x 994.59 x 1.262

= 10303 N / m2

Pumping power = Δp x m / ρ = 10303 x 0.5 / 994.59 = 5.18 W

For a tube of equilateral triangular cross section, dh = 4 {√3 x a2/4} / 3a, where a is the side of
the triangle.

322
Hence dh = a /√3 = 0.02 / √3 = 0.0115 m

0.5
Average velocity = uav = ----------------------------- = 2.9 m/s
994.59 x (√3/4) x 0.02 2

2.9 x 0.0115
Reynolds number = Red = --------------------- = 50684
0.658 x 10 − 6

Since Red > 2300, flow is turbulent. Hence from Moody chart we have

f = 0.02

Pressure drop = Δp = 0.5 x 994.59 x 2.9 2 x 0.02 x (5 / 0.0115) = 36367.4 N / m2

Pumping power = 36367.4 x 0.5 / 994.59 = 18.28 W.

7.3. Solution:

Tw

Tfi di Tfo

L
Data :- Tfi = 30 C ; Tfo = 70 0C ; Tw = 90 0C ; m = 2 kg / s ; di = 2.5 cm = 0.025 m.
0

To find L , assuming flow is hydrodynamically and thermally developed.

For pipe of circular cross section hydraulic diameter = dh = di = 0.025.

Bulk mean temperature of water = Tm = ½(Tfi + Tfo) = 0.5 x (30 + 70) = 50 0 C.

Properties of water at 50 0 C are : ρ = 990 kg/m3 ; cp = 4181 J/kg-K ;

k =0.644 W / (m-K) ; μ = 0.547 x 10 − 3 kg / (m-s) ; Pr = 3.55

323
Since nothing has been specified in the problem regarding the type of flow, it is assumed that
the flow is hydro dynamically and thermally developed.

4x2
Average velocity = uav = m / (ρx πdi2/4) = ---------------------------- = 4.11 m/s.
990 x π x 0.025 2

990 x 4.11 x 0.025


Reynolds number = Red = ρ uav dh / μ = ----------------------------- = 1.86 x 10 5
0.547 x 10 − 3

Since Red >2300, flow is turbulent. For fully developed turbulent flow the Nusselt number is
given by

Nud = 0.023 Red0.8 Pr n with n = 0.4 for Tw > Tf.

Therefore Nud = 0.023 x (1.86 x 10 5)0.8 x (3.55) 0.4

= 628

Hence the heat transfer coefficient, h = Nud k / dh

= 628 x 0.644 / 0.025 = 16177 W/(m2-K)

To find the length of the tube L, we write the energy balance equation for the entire length of
the tube as

Heat supplied to fluid from the tube wall = Increase of energy of the fluid

Therefore h (πdiL) ΔTm = mcp(Tfo – Tfi)

L = mcp(Tfo – Tfi) / hπdi ΔTm …………………..(1)

Where ΔTm = logarithmic mean temperature difference = [ΔTi – ΔTo] / ln (ΔTi / ΔTo)

ΔTi = Tw – Tfi = 90 – 30 = 60 0 C and ΔTo = Tw – Tfo = 90 – 70 = 20 0 C.

Therefore ΔTm = [60 – 20] / ln(60 / 20) = 36.41 0 C.

[2 x 4181x (70 – 30)]


Hence L = -------------------------------- = 7.23 m.
16171 x π x 0.025 x 36.41

7.4. Solution:

324
Tw

water Tfi di Tfo

Data :- Tfi = 20 0C ; m = 5 kg / s ; di = 0.05 m ; L = 10 m ; Tw = 80 0 C.

To find Tfo.

Since Tfois not known we cannot determine the bulk fluid mean temperature to know the
properties of the fluid. Hence this problem has to be solved by trial and error method as
shown below

Trial No. 1:- Assume suitable value for Tfo noting that Tfo < Tw.

Let Tfo = Tw = 60 0 C. Hence bulk mean temperature = Tm = ½(Tfi + Tfo)

= 0.5 x (20 + 60) = 40 0 C.

Properties of water at 40 0 C are : ρ = 994.59 kg/m3 ; cp = 4178.4 J/kg-K ; Pr = 4.34 ;

ν = 0.658 x 10 − 6 m2 / s ; k = 0.628 W /(m-K).

4x5
Average velocity of water = uav = 4m /( πdi ρ) = --------------------------
2

π x (0.05)2 x 994.59

= 2.56 m/s.

For a circular tube dh = di = 0.05 m.


2.56 x 0.05
Reynolds number = Red = uavdh / ν = ------------------------ = 1.945 x 10 5
0.658 x 10 − 6

Since Red > 2300, flow is turbulent.

Assuming the flow to be thermally and hydrodynamically developed,

Nud = 0.023 Red0.8 Pr n with n = 0.4 (as the fluid is heated)

= 0.023 x (1.945 x 10 5) 0.8 (4.34) 0.4

325
= 704.5
704.5 x 0.628
Heat transfer coefficient = h = Nud k / dh = ---------------------- = 8848.5 W /(m2 – K).
0.05

Heat balance equation for the total length of the tube can be written as

h πdiL ΔTm = mcp[Tfo – Tfi]

or h πdiL [ΔTi – ΔTo]


---------------------------- = mcp [ΔTi – ΔTo]
ln[ΔTi / ΔTo]

or ΔTo = ΔTi / exp {( h πdiL)/(mcp)}

[80 – 20]
= ----------------------------------------------------
exp{(8848.5 x π x 0.05 x 10) / (5 x 4178.4)}

= 30.85 0 C.

Therefore Tfo = 80 – 30.85 = 49.15 0 C. This temperature is quite away from the
assumed value and hence one more iteration is required.

Trial 2:- Assume Tfo = 49 0 C. Therefore Tm = (49+20)/2 = 34.5 0 C.

Properties of water at 34.5 0 C are : ρ = 996.22 kg/m3; cp = 4179.3 J/kg-K; Pr = 5.077;

k = 0.6195 W/(m-K); ν = 0.7537 x 10 − 6 m2/s.

4x5 2.556 x 0.05


uav = ------------------ ------ = 2.556 m/s; Red = -------------------- = 1.696 x 10 5
π x 0.05 2 x 996.22 0.7537 x 10 − 6

Therefore Nud = 0.023 x (1.696 x 10 5) 0.8 x (5.077) 0.4 = 672.2

Hence h = 672.2 x 0.6195 / 0.05 = 8328.6 W/(m2 – K)

[80 – 20]
Therefore Tfo = 80 − ---------------------------------------------------------
exp {(8328.6 x π x 0.05 x 10) / (5 x 4179.3)}
0
= 48 C which is very close to the assumed value of Tfo. Hence the
iteration can be stopped.

326
7.5. Solution:
qw
a

Tfi Tfo b

L
Data :- Fluid is air ; Tfi = 27 0 C ; m = 0.01 kg/s; a = 0.01 m; b = 0.006 m; L = 2 m;

qw = 5000 W / m2.

find (i) Tfo ; (ii) Tw|z = L

Energy balance equation for total length of the tube can be written as

mcp(Tfo – Tfi) = qw 2(a+b)L

Or Tfo = Tfi + [qw 2(a+b)L] / (mcp)

Since Tfo is not known the property cp is read at Tfi.

Therefore cp = 1005.7 J/kg-K.


2 x (0.01 + 0.006) x 2 x 5000
Therefore Tfo = 27 + --------------------------------------- = 58.8 0 C.
0.01 x 1005.7

Therefore Tm = ½(Tfi + Tfo) = 0.5 x (27 + 58.8) = 42.9 0 C.

Properties of air at 42.9 0 C are : ρ = 1.12 kg / m3; 1006.8 J/kg-K; ν = 17.30 x 10 − 6 m2/s;

Pr = 0.7045 ; k = 0.02745 W/(m-K).(It should be noted that the variation of cp with


temperature between Tfi and Tm is very negligible and hence this problem does not require
trial and error solution)

4ab 2ab 2 x 0.01 x 0.006


Hydraulic diameter = dh = ------------ = ------------- = ------------------------- = 0.0075 m.
2[a + b] [a + b] [0.01 + 0.006]

0.01
Average velocity = uav = m / [ρab] = ------------------------- = 149 m /s.
1.12 x 0.01 x 0.006
149 x 0.0075

327
Reynolds number = Red = uavdh / ν = --------------------- = 64595
17.3 x 10 − 6

Since Red >2300 flow is turbulent. Assuming that the flow is hydrodynamically and
thermally developed we have
Nud = 0.023 Red 0.8Pr n, with n = 0.4 as air is being heated.

Therefore Nud = 0.023 x [64595] 0.8 x [0.7045] 0.4 = 140.9

140.9 x 0.02745
Heat transfer coefficient = h = Nud k / dh = ---------------------- = 515.7 W/(m2 – K).
0.0075

At the exit of the tube we have qw = h [Tw|z=l – Tfo]

Therefore Tw|z=l = qw / h + Tfo = 5000 / 515.7 + 58.8 = 68.5 0 C

7.6.Solution:

Data: Fluid is liquid sodium; m = 3/60 =0.05 kg/s; Tfi = 400 0 C; Tfo = 500 0C;di = 0.05 m

ΔTi = ΔTm = ΔTo= 30 0 C; ρ = 846.7 kg/m3; k = 68.34 W/(m-K); Pr = 0.00468

cp = 1274 J/kg-K; ν = 0.2937 x 10 − 6 m2/s.

4 x 0.05
Average velocity = uav = 4m / ( ρπdi2) = ------------------------ = 0.03 m/s.
846.7 x π x 0.05 2

0.03 x 0.05
Reynolds number = Red = uavdh / ν = ----------------------- = 5107
0.2937 x 10 − 6

Since Red > 2300, flow is turbulent.Assuming the flow to be hydrodynamically and thermally
developed and since Pr << 1 (Liquid metal), the Nusselt number for constant wall heat flux
condition is given by

Nud = 4.82 + 0.0185 (Red Pr) 0.827

= 4.82 + 0.0185 x [5107 x 0.00468] 0.827 = 5.075

5.075 x 68.34
Heat transfer coefficient = h = Nud k / dh = ------------------------ = 6936.5 W/(m2 – K).

328
0.05

Energy balance equation for the total length of the tube can be written as

h (πdiL)ΔTm = mcp(Tfo – Tfi)

mcp(Tfo – Tfi) 0.05 x 1274 x (500 – 400)


or L = -------------------- = ----------------------------------- = 0.195 m
h (πdi)ΔTm 6936.5 x π x 0.05 x 30
7.7. Solution:

The Dittus-Boetler correlation for hydrodynamically and thermally developed flow is given
by
Nud = hdh/k = 0.023 Red0.8 Pr n ………………..(1)

4M
For a circular tube of diameter D, Red = uavD / ν = --------------
ρπDν

Hence Equation(1) can be written as h1D


----- = 0.023 [4M/( ρπDν)] 0.8 Pr n
k

Or h1 = 0.023k [4M/( ρπν)] 0.8 Pr nD − 1.8………….(2)

Similarly when the diameter of the tube is reduced to D/2, for the same mass flow rate the
heat transfer coefficient is given by

h2 = 0.023k [4M/( ρπν)] 0.8 Pr n(D/2) − 1.8……..(3)

Dividing Equation(3) by Equation(2) we get


1.8
h2/ h1 = 2 = 3.5 or h2 = 3.5 h1.
7.8. Solution:

329
Tw

di
Tfi Tfo

Data:- Fluid is mercury(Liquid metal) ; Tfi = 1000C; Tfo = 2000C;

di = dh = 0.0125m;uav = 1 m/s;Tw = 2500C;

To find L

Bulk mean temperature of mercury = Tm = ½(Tfi + Tfo) = 0.5 x (100 + 200) = 150 0C.

Properties of mercury at 150 0C are:- ρ = 13230 kg/m3; cp = 137.3 J/kg-K;

ν = 0.0865 x 10 − 6 m2 / s; k = 9.65 W/(m-K) ; Pr = 0.0162

1.0 x 0.0125
Reynolds number = Red = uavdh / ν = ----------------------- = 1.445 x 10 5
0.0865 x 10 − 6

Therefore Peclet number = Pe = Red Pr = 1.445 x 10 5 x 0.0162 = 2341


Since Red > 2300, flow is turbulent. Therefore for liquid metal flow subjected to uniform
wall-temperature Nusselt number is given by

Nud = 5.0 + 0.025 Pe 0.8

= 5.0 + 0.025 x 2341 0.8 = 17.4

17.4 x 11.425
Heat transfer coefficient, h = Nudk/dh = -------------------- = 15904.5 W/(m2-K)
0.0125

ΔTi = Tw – Tfi = 250 – 100 = 150 0 C ; ΔTo = Tw – Tfo = 250 – 200 = 50 0 C.

Mean temperature difference = ΔTm = [ΔTi − ΔTo] / ln(ΔTi / ΔTo)

330
= [150 – 50] / ln(150 / 50) = 91 0 C

Mass flow rate of mercury = m = ρ(πdi2/4)uav = 15904.5 x (π x 0.0125 2/4) x 1.0

= 1.952 kg/s

Energy balance equation for the total length of the pipe is given by

h πdiL ΔTm = mcp(Tfo – Tfi)

mcp(Tfo – Tfi) 1.952 x 137.3 x (200 – 100)


Therefore L = ------------------ = ------------------------------------
h πdi ΔTm 15904.5 x π x 0.0125 x 91

= 0.4715 m

B Hydrodynamic and Thermal Entry Lengths

6.9. Solution:
Data:- Tfi = 60 0C ; m = 0.015 kg / s; di = dh = 0.025 m.

(i) Fluid is water. Hence at 60 0 C, ρ = 985.46 kg / m3 ; ν = 0.478 x 10 − 6 m2 / s.

4 x 0.015
Average velocity = uav = 4m / ρπdi2 = -------------------------- = 0.032 m/s.
985.46 x π x 0.025 2

0.032 x 0.025
Reynolds number = Red = uavdh / ν = --------------------- = 1728
0.478 x 10 − 6

Since Red < 2300, flow is laminar. Hence the hydrodynamic entrance length Lh for a circular
pipe is given by

Lh / dh
-------- = 0.056
Red

Therefore Lh = 0.056 Red dh = 0.056 x 1728 x 0.025

= 2.419 m
(ii)Fluid is ethylene glycol: ρ = 1087.66 kg/m3 ; ν = 4.75 x 10 − 6 m2/s.

4 x 0.015
Average velocity = uav = 4m / ρπdi2 = -------------------------- = 0.0281 m/s.
1087.66 x π x 0.025 2

331
0.0281 x 0.025
Reynolds number = Red = ---------------------------- = 147.9
4.75 x 10 − 6

Since Red < 2300, flow is laminar.

Therefore Lh = 0.056 Red dh = 0.056 x 147.9 x 0.025

= 0.21 m.
(iii) Fluid is engine oil; ρ = 864.04 kg/m3 ; ν = 0.839 x 10 − 4 m2/s.

4 x 0.015
Average velocity = uav = 4m / ρπdi2 = -------------------------- = 0.0354 m/s.
864.04 x π x 0.025 2

0.0354 x 0.025
Reynolds number = Red = ---------------------------- = 10.55
0.839 x 10 − 4

Therefore Lh = 0.056 Red dh = 0.056 x 10.55 x 0.025 = 0.013 m.

6.10.Solution: Data: Fluid is engine oil ; Tfi = 60 0C; m = 0.01 kg/s;


square duct with a = 0.01m ; ρ = 864 kg/m 3; cp = 2047 J/kg-K; k = 0.14 W/(m-K);
μ = 0.0725 kg/(m-s); Pr = 1050.

To find (i) Lh ;(ii)Lt ; (iii) h for fully developed flow.

(i) Hydraulic diameter = dh = 4a 2 /(4a) = a = 0.01 m.

0.01
Average velocity = uav = m / (ρa ) = --------------------- = 0.1157 m/s.
2

864 x (0.01) 2

864 x 0.1157 x 0.01


Reynolds number = Red = (ρuavdh) / μ = ------------------------- = 13.8
0.0725

Since Red < 2300, flow is laminar. Hence (Lh / dh)


---------- = 0.09 for a tube of square section.
Red

Therefore Lh = 0.09 Red dh = 0.09 x 13.8 x 0.01 = 0.0124 m.

(ii) For constant wall temperature condition we have

332
(Lt / dh)
---------- = 0.041
Pe

Hence Lt = 0.041 Pe dh = 0.041 x (13.8 x 1050) x 0.01 = 5.94 m

(iii)For fully developed laminar flow through a square tube Nusselt number is given by

Nu T = hdh / k = 2.976

Hence h = 2.976 k / dh = 2.976 x 0.14 / 0.01 = 41.66 W/(m2-K)

Consider the flow of water at a rate of 0.01 kg/s through an equilateral triangular duct of
sides 2 cm and whose walls are kept at a uniform temperature of 100 0C. Assuming the flow
to be hydrodynamically and thermally developed, determine the length of the tube required to
heat the water from 200C to 70 0C.

Solution:
Tw = 100 0C

Water
Tfo=700C
Tfi = 20 0C
m = 0.01 kg/s

L=?

a = 0.02 m

Bulk mean temperature of water = ½(Tw + T∞) = 0.5x (20 + 70) = 45 0C.

Properties of water at bulk mean temperature are: ρ = 992.3075 kg/m3; Pr = 4.01


ν = 0.598 x 10 − 6 m2/s; k = 0.63375 W/(m-K); cp = 4179.9J/kg-K

For an equilateral triangular tube, area of flow = A = (√3 / 4)a 2 = (√3 / 4) x 0.02 2

= 1.732 x 10 − 4 m 2

333
4 [(√3 / 4)a 2] a 0.02
Hydraulic diameter = dh = --------------------- = ------------- = ------------ = 0.01155 m
3a √3 √3
m 0.01
Average velocity of water = uav = ----------------- = -------------------------------
ρA 992.3075 x 1.732 x 10 − 4

=0.0582 m/s

0.0582 x 0.01155
Reynolds number = Red = uavdh / ν = -------------------------- = 1124
0.598 x 10 − 6

Since Red <2300 flow is laminar.For thermally developed laminar flow with constant wall-
temperature the Nusselt number is given by

Nud = hav dh / k = 2.47

2.47 x 0.63375
Therefore hav = ------------------ = 135.53 W/(m2-K)
0.01155

Mean temperature difference between the surface and the bulk fluid is given by

ΔTm = [ΔTi – ΔTo] / ln[ΔTi / ΔTo]

Now ΔTi = Tw – Tfi = 100 – 20 = 80 0C; ΔTo = Tw – Tfo = 100 – 70 = 30

Hence ΔTm = [80 – 30] / ln{80/30} = 50.1 0C.

Rate of heat transfer to water = Q = mcp(Tfo – Tfi) = 0.01 x 4179.9 x (70 – 20)

= 2090 W

But Q = havPL ΔTm, where P is the perimeter of the duct = 3a

Hence Q = hav 3aL ΔTm

Q 2090
Or L = -------------------- = -------------------------------- = 5.13 m
hav 3a ΔTm 135.53 x 3 x 0.02 x 50.1

C Flow over a flat plate:

6.11. Solution:

334
u∞, T∞
Tw

x
L

Data:- Fluid is air; u∞ = 5 m/s; T∞ = 25 0 C; L = 1 m; Tw = 75 0 C.

Mean temperature = ½(Tw + T∞) = 0.5 x (75 + 25) = 50 0 C.

Properties of air at 50 0 C are : ρ = 1.093 kg/m3; ν = 18.02 x 10 − 6 m 2 /s; Pr =0.703

k = 0.028 W/(m-K).

a) 1) Reynolds number at the trailing edge = ReL = (u∞L) / ν = 5 x 1 / (18.02 x 10 − 6)

= 2.775 x 10 5

Assuming the critical Reynolds number to be 5 x 10 5, the flow is laminar at the trailing edge.
Therefore from heat transfer data hand book we have

δ(x)|x=L = 5 L ReL− 0.5

= 5x 1 x (2.775 x 10 5)− 0.5

= 0.0088 m.

2) Local drag coefficient at the trailing edge is given by

Cx|x=L = 0.664 ReL− 0.5 = 0.664 x (2.775 x 10 5) − 0.5

= 1.26 x 10 − 3

Therefore τw(x)|x= L = ½(ρu∞2) Cx|x=L = ½ x 1.093 x 5 2 x 1.26 x 10 − 3

Or τw(x)|x= L = 0.0173 N/m 2.

3) Local Nusselt number at the trailing edge is given by

335
Nux|x=L = 0.332 ReL 0.5 Pr 0.333

= 0.332 x (2.775 x 10 5) 0.5 x (0.703) 0.333

= 155.5

Hence local heat transfer coefficient at the trailing edge is given by

hx|x=L = (Nux|x=L k) / L = 155.5 x 0.028 / 1

= 4.354 W/(m2-K)

Heat flux at the trailing edge = qw|x=L = [hx|x=L] (Tw – T∞)

= 4.354 x (75 – 25) = 217.7 W/(m2 – K)

(4) Drag force: Fd = {τw} avg .A= 2 x τw(x)|x= L x 1= 2x 0.0173= 0.0346 N

Therefore average heat transfer coefficient is havg = 2 hx|x=L = 4.354 x 2

=8.708 W/(m2 – K) and

Average heat flus = qw = [havg] (Tw – T∞)= 8.708 x 50 = 435.4 W/(m2 – K)

6.12. Solution:-

u∞ = 10 m/s (air)

T∞ = 30 0 C Tw = 130 0 C

x
W=1 m
L=4m

To find :- (a) qw(x)|x=L; (b) Qlaminar; (c) Qtotal assuming Recr = 2 x 10 5 (d) Qturbulent

Mean film temperature = ½(Tw + T∞) = 0.5 x (130 + 30) = 80 0C.

Properties of air at mean film temperature are: ν =21.48 x 10 − 6 m2/s; Pr = 0.692


k = 0.03047 W /(m-K).

336
10 x 4
ReL = u∞L / ν = -------------------------- = 1.86 x 10 6
21.48 x 10 − 6

Since ReL > Recr flow is turbulent at the trailing edge.

For turbulent flow of air over a flat plate the local Nusselt number is given by

Nux = 0.0296 Rex 0.8 Pr 1/3.

Hence Nux|x=L = 0.0296 x [1.86 x 10 6] 0.8 (0.692) 1/3 = 2714

2714 x 0.03047
Therefore [hx|x=L] L / k = 2803 or hx|x=L = ---------------------- = 20.67 W /(m2-K)
4

Heat flux at the trailing edge = qw(x)|x=L = [hx|x=L] (Tw – T∞)

= 20.67 x (130 – 30) = 2067 W / m2.

(b) Recr = u∞ xcr / ν.

Recr ν 2 x 10 5 x 21.48 x 10 − 6
Or xcr = --------- = ------------------------------- = 0.4296 m
u∞ 10

Hence flow is laminar up to xcr.

Average Nusselt number for the laminar region is given by

[Nuav] laminar = 0.664 Recr0.5 Pr 1/3

= 0.664 x [2 x 10 5] 0.5(0.697) 1/3 = 263.3

Hence average heat transfer coefficient for the laminar region is


263.3 x 0.03003
[hav]laminar = [Nuav] laminar k / xcr = -----------------------
0.4296
= 18.40 W /(m –K)2

Heat transfer rate from laminar portion = Qlaminar = [hav]laminar [x cr W] (Tw – T∞)

= 18.4 x [ 0.4296 x 1] x (130 – 30)

= 790.5.0 W

337
(c)Average Nusselt number for the entire length of the plate is given by

Nuav = Pr 1/3 [ 0.037ReL0.8 – A]

Where A = 0.037Recr 0.8 – 0.664Recr0.5.

For this problem A = 0.037 x [2 x 10 5] 0.8 – 0.664 x [2 x 10 5] 0.5 = 356

Therefore Nuav = (0.697) 1/3 [0.037 x {1.86 x 10 6} 0.8 – 356] = 3085

3085 x 0.03047
Hence hav = Nuav k / L = ----------------------- = 23.5 W/(m2 – K)
4

Qtotal = hav (LW)(Tw – T∞) = 23.5 x (4 x 1) x (130 – 30) = 9400 W

Qturbulent = Qtotal – Qlaminar = 9400 – 790.5 = 8609.5 W

6.13. Solution:
Hot air at T∞1 =250 0 C, u∞1 = 50 m/s
Tw = ½( T∞1+ T∞2)
= 150 0 C

L=2m

Cold air at T∞2 = 50 0C, u∞2 = 15 m / s

Additional data:- Recr = 2 x 10 5

To find (i) hav for the hot surface ; (ii) hav for the cold surface ; (iii) qw(x)|x=L/2

(i) Mean film temperature for the hot fluid = ½(Tw + T∞1) = 0.5 x (150 + 250) = 200 0C.

Properties of air at 200 0C are: ν = 34.85 x 10 − 6 m2 / s;k = 0.03931 W/(m-K);Pr = 0.68

Reynolds number at the trailing edge of the plate = ReL = u∞1L / ν

50 x 2
Or ReL = ------------------- = 2.869 x 10 6
34.85 x 10 − 6

Since ReL > Recr, flow is partly laminar and partly turbulent. Therefore the average Nusselt
number is given by

Nuav = Pr 1/3[0.037 ReL0.8 – A]

338
Where A = 0.037 Recr0.8 – 0.664 Recr 0.5.

Or A = 0.037 x [2 x 10 5]0.8 – 0.664 x [2 x 10 5] 0.5 = 347.25

Hence Nuav = (0.68) 1/3 [0.037 x (2.869 x 10 6) 0.8 – 347.25] = 4465

4465 x 0.0391
Hence for the hot surface [hav] hot = Nuav k / L = ---------------------- = 87.29 W/(m2-K)
2

(ii) Mean film temperature for the cold surface = ½(150 + 50) = 100 0C.

Properties of air at the mean film temperature are: ν = 23.33 x 10 − 6 m2/s; Pr = 0.693
k = 0.03184 W/(m-K)
15 x 2
ReL = u∞2L / ν = -------------------------- = 1.286 x 10 6
23.33 x 10 − 6

Therefore Nuav = (0.693) 1/3[0.037 x (1.286 x 10 6) 0.8 – 347.25] = 2219

2219 x 0.03184
Hence for the cold surface [hav]cold = -------------------- = 35.33 W/(m2-K).
2

(iii) The rate of heat transfer from the hot air stream to cold air stream is given by

Q = (T∞1 - T∞2) / [Rc1 + R + Rc2]

Where Rc1 = Thermal resistance offered by hot surface for convection,

Rc2 = Thermal resistance offered by cold surface for convection,

and R = Thermal resistance offered by the plate for conduction.

1
Now Rc1 = 1 / [hav]hot A = ------------------- = 0.00573 m2 – K / W.
87.29 x (2 x 1)
1
Similarly Rc2 = 1 / [hav]cold A = -------------------- = 0.01415 m2 – K / W.
35.33 x (2 x 1)

R = L / Ak . Since k is not given it is assumed that k is very large i.e R = 0.

Therefore Q = [250 – 50] / [ 0.00573 + 0.01415] = 10,060 W.

339
(iv) At mid point of the plate x = L / 2.

Therefore for the hot fluid Re|x=L/2 = ½ ReL = 0.5 x 2.865 x 10 6

= 1.4325 x 10 6 which is > Recr.

Therefore flow is turbulent at mid point of the plate.

Hence [Nu|x=L/2]hot = 0.037 [Re|x=L/2] 0.8 Pr 1/3

= 0.037 x [1.4325 x 10 6] 0.8 x (0.6815) 1/3 = 2739

Hence [hx|x=L/2]hot = 2739 x 0.03931 / 1 = 107.7 W/(m2-K).

Similarly [Nu|x=L/2]cold = 0.037 x [0.5 x 1.286 x 10 6] 0.8 x (0.693) 1/3 = 1451

Hence [hx|x=L/2]cold = 1451 x 0.03184 / 1 = 46.2

Heat flux at the mid point of the plate is given by

qw(x)|x=L/2 = [hx|x=L/2]hot [T∞1 – Tw] = [hx|x=L/2]cold [Tw - T∞2]

CT∞1 + T∞2 [hx|x=L/2]hot 107.7


Therefore Tw = ----------------------- ; C = ---------------- = ---------- = 2.33
1+C [hx|x=L/2]cold 46.2

(2.33 x 250) + 50
Hence Tw = -------------------------- = 189.94 0C
1 + 2.33

Therefore qw(x)|x=L/2 = 107.7 x [250 –189.94] = 6468.5 W/m2

Also check for qw(x)|x=L/2 ; qw(x)|x=L/2 = 46.2 x (189.94 – 50) = 6465.2 W/m2

6.14. Solution:

340
Air with u∞ = 30 m/s,T∞ = 25 0C
Tw = 150 0 C

10 mm

X1 = 700 mm Insulated

X2 = 750 mm

Properties of the module: km = 5.2 W/(m-K); cpm = 320 kJ/kg-K;


ρm = 2300 kg/m3.

Mean film temperature = ½(Tw + T∞) = 0.5 x(25 + 150) = 87.5 0C.
Properties of air at 87.5 0C are : ν = 21.79 x 10 − 6 m2/s; k = 0.03075 W/(m-K);
Pr = 0.695. Assume Recr = 5 x 10 5

Reynolds number at x1 = Rex1 = u∞x1/ ν = 30 x 0.7 / (21.79 x 10 − 6) = 9.637 x 10 5.

Similarly Rex2 = u∞x2 / ν = 30 x 0.75 / (21.79 x 10 − 6) = 10.325 x 10 5

Since Rex1 > Recr, the flow is turbulent at x1 and the flow will be turbulent over the module
under consideration. Therefore the average heat transfer coefficient for the module can be
written as x2
[hav]m = {1 / (x2 – x1)} ∫ hxdx
x1

= {1 / (x2 – x1)} ∫(Nux k/x)dx

x2
= {1 / (x2 – x1)} ∫{k[ 0.037 (Rex)0.8 – 871]Pr 1/3 /x}dx
x1

k Pr 1/3 x2

= ---------------- { ∫ [0.037 (u∞/ν)0.8 x − 0.2 – 871/x]dx }


(x2 – x1) x1

k Pr 1/3
= ---------------- {0.037 (u∞/ν)0.8 ∫ x − 0.2 dx - 871∫ (dx /x) }
(x2 – x1)
k Pr 1/3
= ---------------- { 0.04625[Rex20.8 – Rex10.8] – 871 ln (x2 / x1)}
(x2 – x1)

341
0.03075 x (0.695)1/3{0.04625[(10.325 x105)0.8 –(9.325 X 105)0.8] – 871 ln (0.75/0.7)}
= -------------------------------------------------------------------------------------------------------
(0.75 – 0.7)

= 95.16 W / (m2-K)

For the module, power generation = qw = [hav]m {Tw - T∞} = 95.16 x (150 – 25)

= 11895 W/m2 = 11.895 kW/ m2

(b) Since the bottom surface of the module is insulated, all the heat generated in the module
is transferred to air from the top surface of the module. Hence if q’’’ is the heat generated per
unit volume then
q’’’ (x2 – x1)δW = qw(x2 – x1)W,

where δ is the thickness of the module.

Therefore q’’’ = qw / δ = 11895 / 0.001 = 11.895 x 10 6 W/m3.

For the module the governing conduction equation is

d2T/dy2 + q’’’ / k = 0 ………………………….(a)

where y is the coordinate measured in the direction of the thickness of the module.

The boundary conditions are (i) at y = 0, the surface is insulated i.e. dT/dy = 0

and at y = δ, T = Tw. The solution of Equation(a) subject to the boundary conditions is given
by

T(y) + q’’’y2 / 2k = Tw + q’’’δ2 / 2k

Since the bottom surface is insulated, the maximum temperature of the module will be at the
bottom surface (y = 0) and is therefore given by

Tmax = Tw + q’’’δ2 / 2k

11.895 x 106 x (0.001)2


= 150 + ----------------------------- = 151.1 0C
2 x 5.2

D) Flow across a cylinder

342
6.15. Solution:

u∞ = 15 m/s; T∞ = 25 0 C
Tw = 1150C

L d = 0.025 m

Mean film temperature = ½ (Tw + T∞) = 0.5 x (115 + 25) = 70 0C.

Properties of air at 70 0C are: ρ = 1.0231 kg/m3; ν = 20.05 x 10 −6m2/s; Pr = 0.699;

k = 0.0295 W/(m-K); To find (i) Drag force FD ; (ii) Q

(i) Reynolds number = Red = u∞d / ν = 15 x 0.025 / 20.05 x 10 −6

= 18703

From the chart of drag coefficient versus Reynolds number we get


the drag coefficient ,CD = 1.2.

Therefore drag force = FD = ½(ρu∞2)LD CD = 0.5 x 1.0231 x 152 x 1 x 0.025 x 1.2 = 3.453 N

(ii) From Heat transfer data hand book for gases the average Nusselt number is given by
(μw = μ∞ can be considered for air )

Nuav = havD/k = (0.4Red0.5 + 0.06 Red 2/3) Pr0.4

= (0.4 x 18703 0.5 + 0.06 x 18703 2/3) x (0.699) 0.4 = 84.04

84.04 x 0.0295
Therefore hav = Nuav k/d = -------------------- = 99.17 W/(m2-K)
0.025

Heat transfer = Q = havπdL(Tw – T∞) = 99.17 x π x 0.025 x 1 x (115 – 25) = 701 W

6.16 Solution:

343
u∞ = 10 m/s; T∞ = 27 0C

Qw

L D = 0.01 m

Given:- Qw = 1000 W/m; ks = 240 W/(m-K); ρs = 2700 kg/m3; Cps = 900 kJ/kg-K

To find (i) Tw under steady state condition

(ii) time ‘t’ required for the surface to reach a temperature of (Tw – 10) 0C

Since Tw is not known it is not possible to read the properties at the mean film temperature.
Hence the problem has to be solved by trail and error procedure.

Trial 1:- Calculations are started using the properties of air at T∞.
Properties of air at 270C are:

ρ = 1.1774 kg/m3; cp = 1.0057 kJ/kg-K; ν = 15.68 x 10 − 6 m2/s; k = 0.02624 W/(m-K);

Pr = 0.708

10 x 0.01
Red = u∞D / ν = ---------------------- = 6377.5
15.68 x 10 − 6

( μw = μ ∞ can be considered for air )

Therefore Nuav = havD/k = (0.4Red0.5 + 0.06 Red 2/3) Pr0.4

= (0.4 x 6377.50.5 + 0.06 x 6377.5 2/3) x (0.708) 0.4 = 45.79

45.79 x 0.02624
Therefore hav = Nuav k/D = ------------------------- = 120.15 W/(m2- K)
0.01

Now Qw = hav πDL (Tw – T∞)

344
Or Tw = T∞ + Qw /hav πDL = 27 + 1000 / (120.15 x π x 0.01 x 1) = 291.930 C

Trial 2:- Assume Tw = 291.93 0C .Mean film temperature = ½(291.93 +27) = 159.5 0C.

Properties of air at 159.50C are: ν = 30.09 x 10 − 6 m2/s; k = 0.03640 W/(m-K); Pr = 0.682.

10 x 0.01
Red = u∞D / ν = ---------------------- = 3323.4
30.09 x 10 − 6

Nuav = (0.4 x 3323.4 0.5 + 0.06 x 3323.4 2/3) x (0.682) 0.4 = 31.25

Hence hav = 31.25 x 0.03640 / 0.01 = 113.75 W/(m2-K)

Therefore Tw = 27 + 1000 / (113.75 x π x 0.01 x 1) = 306.8 0C. Since this value of Tw is


considerably different from the value got in the first trial, one more iteration is required.

Trial 3:- Assume Tw = 303 0 C. Mean film temperature = 0.5 (303 + 27) = 165 0 C.

Properties of air at 165 0 C are: ν = 30.88 x 10 − 6 m2/s; k = 0.0369 W/(m-K);

Pr = 0.682
10 x 0.01
Red = u∞D / ν = ---------------------- = 3238.34
30.88x 10 − 6

Nuav = (0.4 x 3238.34 0.5 + 0.06 x 3238.34 2/3) x (0.682) 0.4 = 30.8

Hence hav = 30.8 x 0.0369 / 0.01 = 113.65W/(m2-K)

Therefore Tw = 27 + 1000 / (113.65 x π x 0.01 x 1) = 307 0C which is very close to the


assumed value of 303 0 C and hence the iteration can be stopped.
Therefore Tw = 307 0C.

(ii) To determine the time ‘t’ required for the surface of the cylinder to reach a temperature of
(307 – 10 = 2970C).

If dT/dt represents the rate of change of temperature of the surface w.r.t. time during
unsteady state, then energy balance equation for the cylinder can be written as follows:

Rate of increase of energy of the cylinder = Heat transfer rate by convection from
the surroundings to the cylinder +

E. Flow across tube bundles

345
6.19.Solution: Case (i) Square Arrangement

Row 1 Row 2 Row 3


u∞,T∞

D
ST

SL

Data:- T∞ = 30 0 C; D = 0.01 m; N = 10; u∞ = 1 m/s; ST / D = SL / D = 1.25

To find (i) friction factor,f; (ii) pressure drop Δp ; (iii) hav

Since the surface temperature of the tubes is not known properties of air are evaluated at T∞.
Hence properties of air at 30 0C are:

ρ = 1.1774 kg/m3; μ = 1.983 x 10 − 5 kg/(m-s); cp = 1005.7 J/kg-K; k = 0.02624 W/(m-K);


Pr = 0.708

(i) For square arrangement the maximum velocity is given by

ST / D 1.25
Umax = u∞ ------------------ = 1 x ---------------- = 5 m/s
[ST / D – 1] [ 1.25 – 1]

Gmax = ρUmax = 1.1774 x 5 = 5.887 kg/(m2-s)

0.01 x 5.887
Reynolds number = Re = DGmax / μ = ------------------------ = 2969
1.983 x 10 − 5

From the graph friction factor f = 0.55 and Z = 1 for square arrangement as ST = SL.

N (Gmax) 2 10 x (5.887) 2
Now Δp = f ---------------- Z = 0.55 x ------------------------- x 1.0 = 161.9 N/ m2
ρ 1.1774

(ii) For N ≥ 20 the average Nusselt number is given by

346
Nuav = c2Rem Pr0.36 (Pr/Prw)n

Here c2 = 0.27 ; m = 0.63 ; n = 0.

Therefore Nuav = 0.27 x (2969) 0.63 (0.708)0.36 = 36.74

Since N <20, the above value of Nuav has to be multiplied by a correction factor.

Hence [ Nuav] N=10 = c3[Nuav]N≥ 20

For Re = 2969 and N = 10 from the graph c3 = 0.98

Therefore [ Nuav] N=10 = 0.98 x 36.74 = 36

Therefore hav = 36 x 0.02624 / 0.01 = 94.5 W/(m2-K)

Case (ii); Equilateral Triangular Arrangement:

u∞,T∞

ST
SD

Given: ST / D = SD / D = 1.25

ST / D
Maximum velocity Umax = u∞ ------------------ = 5 m/s as calculated above
[ST / D – 1]

ST / D 1.25
Or Umax = (½ )u∞ ------------------ = 0.5 x 1 x ------------- = 2.5 m/s
[SD / D – 1] (1.25 – 1)

347
We have to choose the higher of the two maximum velocities to calculate the Reynolds
number.Hence Umax and Reynolds number will be same as the above case.

From chart for staggered tube arrangement, f = 0.7and Z = 1

10 x 5.887 2
Therefore Δp = 0.7 x ----------------- x 1 = 206.1 N/m2
1.1774

From data hand book the average Nusselt number is given by

[Nuav]N≥20 = c2 Re m Pr 0.36(Pr /Prw) n

For staggered arrangement c2 = 0.35 x (ST / SL) 0.2

Now SL2 = SD2 − (ST/2)2 or SL / D = [(SD/D)2 – (ST/2D)2] ½

= (√3/2)(ST/D)

Or ST/SL = 2 / √3 = 1.155

Hence c2 = 0.35 x [1.155]0.2 = 0.36 ;

For staggered arrangement m = 0.6 ; for air n = 0.

Hence [Nuav]N≥20 = 0.36 x( 2969)0.6 (0.708)0.36

= 38.54

[ Nuav] N=10 = c3[Nuav]N≥ 20

From the graph c3 = 0.98. Hence [ Nuav] N=10 = 0.98 x 38.54 = 37.8

Therefore hav = 37.8 x 0.02624 / 0.01 = 99.2 W/(m2-K).

6.20. Solution:

348
Hot gases at ST
u∞,T∞ D

SL

Data:- T∞ = 375 0C; u∞ = 7 m/s; Tw = 30 0C; Number of rows = N = 10 ;

D = 0.0125 m; number of tubes in each row = m = 40; L = 1 m ;

In-line arrangement; SL/D = ST/D = 2.

To find:- (i) hav ; (ii) Q

Mean film temperature = 0.5 x (375 + 30) = 202.5 0C.

Properties of air at 202.5 0C are: ρ = 0.7424 kg/m3; cp = 1026.6 J/(kg-K);

μ = 2.606 x 10 – 5 N-s / m2 ; k = 0.03948 W/(m-K); Pr = 0.6799

For inline arrangement Umax = u∞ (ST/D) / [(ST/D) − 1] = 7 x 2 /(2 – 1) = 14 m/s.

Mass velocity = Gmax = ρ Umax = 0.7424 x 14 = 10.394 kg/(m2-s)

0.0125 x 10.394
Reynolds number = Re = DGmax / μ = ---------------------- = 4985.6
2.606 x 10 − 5

The average Nusselt number for N ≥ 20 is given by [Nuav]N≥20 = c2 RemPr0.36

For in-line arrangement from data hand book, c2 = 0.27 and m = 0.63

Therefore [Nuav]N≥20 = 0.27 x (4985.6) 0.63 x (0.6799)0.36 = 50.19

[ Nuav] N=10 = c3[Nuav]N≥ 20 with c3 = 0.96

Hence [ Nuav] N=10 = 0.96 x 50.19 = 48.18

48.18 x 0.03948
Average heat transfer coefficient = hav = [ Nuav] N=10 k / D = ----------------------
0.0125

349
= 152.175 W/(m2-K).

Energy balance between the tubes surfaces and hot gases can be written as

Heat transfer from hot gases to the tube surfaces = Q = (πDLNm)hav(T∞ - Tw)

Or Q = π x 0.0125 x 1 x 10 x 40 x 152.175 x (375 – 30) = 82.779 x103 W = 82.779 kW

350
Chapter 7 : Free Convective Heat Transfer

7.1. Introduction:

A. Free convection from/to plane surfaces:

7.1. A vertical plate 30 cm high and 1 m wide and maintained at a uniform temperature
of 120 0 C is exposed to quiescent air at 30 0 C.Calculate the average heat transfer
coefficient and the total heat transfer rate from the plate to air.
7.2. An electrically heated vertical plate of size 25 cm x 25 cm is insulated on one side
and dissipates heat from the other surface at a constant rate of 600 W/m 2 by free
convection into quiescent atmospheric air at 30 0 C. Determine the surface
temperature of the plate.
7.3. Determine the heat transfer by free convection from a plate 30 cm x 30 cm whose
surfaces are maintained at 100 0 C and exposed to quiescent air at 20 0 C for the
following conditions: (a) the plate is vertical. (b) Plate is horizontal
7.4. A circular plate of 25 cm diameter with both surfaces maintained at a uniform
temperature of 100 0 C is suspended in horizontal position in atmospheric air at 20
0
C. Determine the heat transfer from the plate.
7.5. Consider an electrically heated plate 25 cm x 25 cm in which one surface is
thermally insulated and the other surface is dissipating heat by free convection into
atmospheric air at 30 0 C. The heat flux over the surface is uniform and results in
a mean surface temperature of 50 0 C. The plate is inclined making an angle of 50
0
from the vertical. Determine the heat loss from the plate for (i) heated surface
facing up and (ii) heated surface facing down.
7.6. A thin electric strip heater of width 20 cm is placed with its width oriented vertically.
It dissipates heat by free convection from both the surfaces into atmospheric air
at 20 0 C. If the surface temperature of the heater is not to exceed 225 0 C,
determine the length of the heater required in order to dissipate 1 kW of energy
into the atmospheric air.
7.7. A plate 75 cm x 75 cm is thermally insulated on the one side and subjected to a
solar radiation flux of 720 W/m2 on the other surface. The plate makes an angle of
60 0 with the vertical such that the hot surface is facing upwards. If the surface is
exposed to quiescent air at 25 0 C and if the heat transfer is by pure free convection
determine the equilibrium temperature of the plate.

B. Free convection from/to Cylinders:

7.8. A 5 cm diameter, 1.5 m long vertical tube at a uniform temperature of 100 0C


is exposed to quiescent air at 20 0 C. calculate the rate of heat transfer from
the surface to air. What would be the heat transfer rate if the tube were kept
horizontally?
7.9. A horizontal electrical cable of 25 mm diameter has a heat dissipation rate of 30
W/m. If the ambient air temperature is 27 0 C, estimate the surface temperature
of the cable.
7.10. An electric immersion heater, 10 mm in diameter and 300 mm long is rated at 550
W. If the heater is horizontally positioned in a large tank of water at 20 0 C, estimate
its surface temperature. What would be its surface temperature if the heater is
accidentally operated in air.

351
A.Free Convection to or from plane surfaces

7.1. Solution:

Tw = 120 0 C

 T∞ = 30 0C
L = 0.3m
Mean film temperature of air = 0.5 x (120 +30) = 750C
x
Properties of air at 750C are:

β = 1/ (273 + 75) = 2.874 x 10 − 3 1/K; Pr = 0.693

k = 0.03 W/(m-K) ; ν = 20.555 x 10 − 6 m 2/s ;

First we have to establish whether the flow become turbulent within the given length of the
plate by evaluating the Rayleigh number at x = L.

9.81 x 2.874 x 10 − 3 x (120 – 30) x 0.3 3


GrL = (gβΔTL 3) / ν 2 = ----------------------------------------------------
20.555 x 10 − 6

= 1.62 x 10 8

Rayleigh number = RaL = GrLPr = 1.62 x 10 8 x 0.693 = 1.12 x 10 8.

Since RaL < 10 9 flow is laminar for the entire height of the plate. Hence the average Nusselt
number is given by (from data hand book)

Nuav = 0.59 x (RaL) 0.25 = 0.59 x (1.12 x 10 8) 0.25 = 60.695

60.6 x 0.03
Therefore hav = Nuav k / L = ---------------- = 6.069 W /(m2 – K).
0.3

352
Total heat transfer fro both sides of the plate per unit width of the plate is given by

Qtotal = hav(2LW) ΔT = 6.06 x (2 x 0.3 x 1) X (120 – 30) = 327.726 W/m.

7.2. Solution:

Insulated

qw = 600 W/m2
 T∞ = 30 0C

L = 0.25 m

Since Tw is not known, it is not possible to determine the mean film temperature at which fluid
properties have to be evaluated. Hence this problem requires a trial and error solution either by
assuming Tw and then calculate Tw by using the heat balance equation and check for the
assumed value or assume a value for hav ,calculate Tw and then calculate hav and check for the
assumed value of hav.Since it is difficult to guess a reasonable value for Tw to reduce the number
of iterations, it is preferable to guess a reasonable value for hav for air as we know that for air
hav varies anywhere between 5 and 15 W/(m2-K).

Trial 1:- Assume hav = 10 W/(m2-K).

Now qw = hav[Tw – T∞] or Tw = T∞ + qw / hav = 30 + 600 / 10 = 900C.

Hence mean film temperature = 0.5 x [90 + 30] = 60 0C.

Properties of air at 60 0C are: β = 1 / (60 + 273 ) = 3.003 x 10 − 3 1/K; Pr = 0.696;

k = 0.02896 W/(m-K); ν = 18.97 x 10 − 6 m2/s.

9.81 x 3.003 x 10 − 3 x 600 x (0.25)4


RaL× = GrL×Pr =[(gβqwx4) /(kν2)]Pr = ----------------------------------------------x 0.696
0.02896 x (18.97 x 10 − 6) 2

Or RaL× = 4.61 x 10 9.

353
Since RaL× >10 9 flow is turbulent for the entire length of the plate

Hence Nuav = 1.25 Nux|x=L = 1.25 x 0.17 x (4.61 x 10 9) 0.2 = 55.37


Therefore hav = 55.37 x 0.02896 / 0.25 = 6.41 W/(m2 – K)

Since the calculated value of hav deviates from the assumed value by about 34 %, one more
iteration is required.

Trial 2:- Assume hav = 6.41 W/(m2-K)

Hence Tw = 30 + 600 / 6.41 = 123.6 0C 120 0 C

Mean film temperature = 0.5 x (120 + 30) = 75 0 C

Properties of air at 75 0C are:- β = 1/(75 + 273) = 2.873 x 10 − 3 1/K. Pr = 0.686

k = 0.03338 W /(m-K); ν = 25.45 x 10 − 6 m 2 /s.

9.81 x 2.873 x 10 − 3 x 600 x 0.25 4


RaL× = ------------------------------------------- x 0.686 = 2.06 x 10 9
0.03338 x (25.45 x 10 − 6) 2

Flow is turbulent for the entire length of the plate.

Hence Nuav = 1.25 Nux|x=L = 1.25 x 0.17 x (2.06 x 10 9) 0.25 = 45.27

Therefore hav = 45.27 x 0.03338 / 0.25 = 6.04 W/(m2 – K).

Since the calculated value of hav is very close to the assumed value (error < , the iteration is
stopped. The surface temperature of the plate is therefore given by

Tw = 30 + 600 / 6.04 = 129.3 0 C.

7.3. Solution:- Case(i) When the plate is vertical

Data:- Characteristic length = L = height of the plate = 0.3 m; Tw = 100 0C;T∞= 20 0C;

Mean film temperature = 0.5 x (100 + 20) = 60 0C.

Properties of air at 60 0C are: β = 1 / (60 + 273 ) = 3.003 x 10 − 3 1/K; Pr = 0.696;

k = 0.02896 W/(m-K); ν = 18.97 x 10 − 6 m2/s.

RaL = GrLPr =( gβΔTL 3/ ν 2) Pr

354
9.81 x 3.003 x 10 − 3 x (100 – 20) x (0.3) 3
= ---------------------------------------------------- x 0.696 = 1.23 x 10 8
(18.97 x 10 − 6 ) 2

From data hand book corresponding to this value of RaL have

Nuav = 0.59 x (1.23 x 10 8) 0.25 = 62.13

Therefore hav = 62.13 x 0.02896 / 0.3 = 5.99 W/(m2-K).

Rate of heat transfer = Q = hav(2LW)(ΔT) = 5.99 x (2 x 0.3 x 0.3) x (100 – 20)

= 86.256 W

Case (ii) When the plate is horizontal

Heated surface facing upwards with heat


 T∞ transfer coefficient htop

 T
Heated surface facing downwards with
Heat transfer coefficient hbottom

Data:- T∞ = 20 0C; Temperature of both the surfaces = Tw = 100 0C;


Mean film temperature = 0.5 x (100 + 20) = 60 0C;L = W = 0.3 m
Properties of air at 60 0C are: β = 1 /(60 + 273) = 3.003 x 10 − 3 1/K; Pr = 0.696
ν = 18.97 x 10 − 6 m2/s; k = 0.02896 W/(m-K)

(a) To find htop:- Characteristic length = L = A/P = LW /{2(L+W)}

=L2 / 4L = L/4 = 0.3 / 4 = 0.075 m

gβΔTL 3 9.81 x (3.003 x 10 − 3) x (100 – 20) x (0.075) 3


RaL = ----------- Pr = ----------------------------------------------------------- x 0.696
ν2 (18.97 x 10 − 6) 2

Or RaL = 1.923 x 10 6.

355
From data hand book for heated surface facing upwards with constant surface temperature
the average Nusselt number is given by

Nutop = htopL/k = 0.54 x (RaL) 0.25 = 0.54 x ( 1.923 x 10 6) 0.25 = 20.11


Hence htop = 20.11 x 0.02896 / 0.075 = 7.76 W/(m2-K)

(b)To find hbottom:- From data hand book for heated surface facing downwards with constant
surface temperature, the average Nusselt number is given by

Nubottom = hbottomL/k = 0.27 x(1.923 x10 6) 0.25 = 10.05

Hbottom = 10.05 x 0.02896 / 0.075 = 3.88 W/(m2-K)

Total heat loss to air = Qtotal = Qtop + Qbottom = (LW)htop ΔT + (LW)hbottomΔT

= (0.3 x 0.3) x (100 – 20) x (7.76 + 3.88) = 83.808 W

7.4. Solution: Data:- Horizontal circular plate with D = 0.25 m;Tw = 100 0C ; T∞ = 20 0C

This problem is similar to the previous problem except for the characteristic length. For a
horizontal circular plate of diameter D the characteristic length is given by

L = A/P = (πD2/4) / (πD) = D/4 = 0.25 / 4 = 0.0625 m

Mean film temperature = 0.5 x (100 + 20) = 60 0C;L = W = 0.25 m

Properties of air at 60 0C are: β = 1 /(60 + 273) = 3.003 x 10 − 3 1/K; Pr = 0.696

ν = 18.97 x 10 − 6 m2/s; k = 0.02896 W/(m-K).

gβΔTL 3 9.81 x (3.003 x 10 − 3) x (100 – 20) x (0.0625) 3


RaL = ----------- Pr = ----------------------------------------------------------- x 0.696
ν2 (18.97 x 10 − 6) 2

Or RaL = 1.112 x 10 6.

From data hand book for heated surface facing upwards with constant surface temperature
the average Nusselt number is given by

Nutop = htopL/k = 0.54 x (RaL) 0.25 = 0.54 x ( 1.112 x 10 6) 0.2 = 17.53

Hence htop = 17.53 x 0.02896 / 0.0625 = 8.12 W/(m2-K)

356
(b)To find hbottom:- From data hand book for heated surface facing downwards with constant
surface temperature, the average Nusselt number is given by

Nubottom = hbottomL/k = 0.27 x (1.112 x10 6) 0.25 = 8.76

hbottom = 8.76 x 0.02896 / 0.0625 = 4.059 W/(m2-K)

Total heat loss to air = Qtotal = Qtop + Qbottom = (πDL)htop ΔT + (πDL)hbottomΔT

= (π x 0.25 x 1)x (100 – 20) x (8.76 + 4.059) = 805.44 W

7.5. Soloution: Data:- L = W = 0.25 m; T∞ = 30 0C; Tw = 50 0C;


Mean film temperature = 50 − 0.25 x (50 - 30) = 45 0C;Properties of air at 45 0C are:
Pr = 0.6835; k = 0.02791 W/(m-K); ν = 17.455 x 10 − 6 m2/s.
β = 1/[{30 +0.25x(50-30)} +273] =3.25 x 10 − 3

(i) Inclined plate with heated surface facing upwards:

Characteristic length = L = 0.25 m


gβΔTL3
GrL = -----------
ν2
θ = − 50 0
9.81 x (50 – 30) x 3.25 x 10−3 x(0.25)3
= -------------------------------------------------
(17.455 x 10 − 6)2
Insulated
= 3.27 x 10 7

Hence RaL = 3.27 x 10 7 x 0.6835 = 2.23x107

From data hand book, for inclined plate with


heated surface facing upwards the Nusselt number
is given by

Nuav = 0.145 [RaL1/3 − (GrcPr)1/3] + 0.56(GrcPrcosθ)1/4

The above correlation is valid only if GrL > Grc.

From data hand book for θ = − 50 0, Grc = 4 x 10 8 which is more than GrL.Hence the above
correlation cannot be used. Instead the following correlation has to be used.

Nuav = 0.59 (GrLcos θ Pr) ¼ = 0.59 x (2.23 x 107 x cos 500) ¼ = 36.3

Hence hav = 36.3 x 0.02791 / 0.25 = 4.05 W/(m2 – K).

357
Therefore Q = 4.05 x (0.25 x 0.25) x (50 – 30) = 5.062 W

(ii) Inclined plate with heated surface facing downwards:

The correlation for Nusselt number when the heated surface is facing downwards is given by
Nuav = 0.56 (GrLcos θ Pr) ¼ = 0.56 x (2.23 x 107 x cos 500) ¼

= 34.45
hav = 34.45 x 0.02791 / 0.25 = 3.84 W/(m2-K).

Hence Q = 3.84 x (0.25 x 0.25) x (50 – 30) = 4.8 W

7.6. Solution:

Tw = 225 0 C
W

 T∞=200C

L=0.2 m

Mean film temperature = 0.5 x (225 + 20) = 122.5 0C. Properties of air at 122.50C
are: β = 1/(122.5 + 273) = 2.5 x 10 − 3 1/K; ν = 25.90 x 10 − 6 m2/s; Pr = 0.6865;
k = 0.03365 W/(m-K).

9.81 x 2.5 x 10 − 3 x(225 – 20) x 0.2 3


GrL = ---------------------------------------------- = 5.99 x 10 7
(25.9 x 10 − 6) 2

RaL = 5.99 x 10 7 x 0.6865 = 4.11 x 10 7

Hence Nuav = 0.59 x (4.11 x 10 7) 0.25 = 47.24

Therefore hav = 47.24 x 0.03365/0.2 = 7.94 W/(m2-K)

1000
Now Q = havLWΔT or W = Q / (havLΔT) = -----------------------------
7.94 x 0.2 x (225 – 20)

358
= 3.0718 m

7.7. Solution:

qw = qsolar =720 W/m2

θ = − 60 0 T∞ = 25 0 C

Insulated

Since Tw is not known, mean film temperature to evaluate the fluid properties cannot be
determined. Hence the problem requires a trial- and – error method by suitably assuming a
value for hav and then check for this assumpotion.

Trial 1:- Assume hav = 5 W/(m2-K):

Tw = T∞ + qw / hav = 25 + 720 / 5 = 169 0C.

Tm = Tw − 0.25[Tw – T∞] = 169 – 0.25 x [169 – 25] = 133 0C.

Properties of air at 133 0C are: ν = 26.62 x 10 − 6 m2/s; k = 0.03413 W/(m-K); Pr = 0.685

Mean temperature to evaluate β is given by Tβ = T∞ + 0.25[Tw – T∞]

= 25 + 0.25 x(169 – 25) = 61 0C


Therefore β = 1/[61 + 273] = 2.994 x 10 − 3 1/K.

9.81 x 2.994 x 10 − 3 x [169 – 25] x 0.75 3


GrL = ------------------------------------------------------ = 2.51 x 10 9
(26.62 x 10 − 6) 2

For θ = − 60 0, Grc = 10 8. Since GrL > Grc the average Nusselt number is given by

Nuav = 0.145 [(GrLPr) 1/3 – (GrcPr) 1/3] + 0.56[GrcPr]1/4

= 0.145 x [(2.51 x 10 9x 0.685)1/3 – (10 8 x 0.685) 1/3] + 0.56 x ( 10 8 x 0.685)1/4

359
= 164.45

Therefore hav = 164.45 x 0.03413 / 0.75 = 7.48 W/(m2-K).

Since the calculated value of hav is quite different from the assumed value one more iteration
is required.

Trial 2: Assume hav = 7.48 W/(m2-K).

With this assumption, following the steps shown in trial 1 we get

Tw = 121.25 0C ;Tm = 97.18 0C C;Tβ = 49.06 0C; GrL = 2.311 x 10 9;Nuav = 159.96

Hence hav = 6.88 W/(m2-K). This value agrees with the assumed value within 8 %. Hence the
iteration is stopped and the equilibrium temperature of the plate surface is calculated as

Tw = 25 + 720 /{0.5(7.48 + 6.88)} = 125.27 0C.

B Free convection from/to cylinders

7.8. Solution: (i) When the tube is vertical:

Tw = 100 0C Mean film temperature = 0.5x(100+20)

= 60 0C.
0
Properties of air at 60 C are:Pr = 0.696

k = 0.0290 W/(m-K); ν = 18.97x10−6

m2/s;β = 1/(60 + 273) =3.003x10−31/K.


L = 1.5 m  T∞ = 20 0C

gβΔTL 3 9.81 x 3.003 x 10 − 3 x (100 – 20) x 1.5 3


GrL = ----------- = -------------------------------------------------- = 2.21 x 10 10
ν2 (18.97x10−6) 2

360
(L/d) [ 1.5 / 0.05 ]
Now ------------ = --------------------------- = 0.0078
GrL1/4 [2.21 x 10 10] ¼

(L/d)
Since ------- < 0.025, the vertical tube/cylinder can be treated as a vertical flat surface
GrL1/4
For calculating the Nusselt number.
Now RaL = GrLPr = 2.21 x 10 10 x 0.696 = 1.538 x 10 10

Hence Nuav = 0.1 x (1.538 x 10 10) 1/3 = 248.7

Therefore hav = 248.7 x 0.029 / 1.5 = 4.81 W/(m2 – K).

Rate of heat transfer = Q = πdLhavΔT = π x 0.05 x 1.5 x (100 – 20) x 4.81 = 90.67 W

(ii) When the pipe is horizontal:- When the pipe is horizontal, the characteristic length is the
diameter. Hence

gβΔTd 3 9.81 x 3.003 x 10 − 3 x (100 – 20) x 0.05 3


Grd = ----------- = -------------------------------------------------- = 8.185 x 10 5
ν2 (18.97x10−6) 2

Rad = 8.185 x 10 5 x 0.696 = 5.697 x 10 5

From data hand book the average Nusselt number is given by


1/6

Rad
[Nuav] ½ = 0.60 + 0.387 -----------------------------------
[1 + (0.559/Pr)9/16] 8/27
1/6

(5.697x 105)
½
[Nuav] = 0.60 + 0.387 ----------------------------------- = 4.01 or Nuav = 16.08
[1 + (0.559/0.696)9/16] 8/27

Therefore hav = 16.08 x 0.029 / 0.05 = 9.32W/(m2 – K).

Q = π x 0.05 x 1.5 x (100 – 20) x 9.32 = 175.67 W

7.9. Solution:

361
 T∞ = 27 0C Tw = ?

qw = 30 W/m d =.025 m

Since Tw is not known, it is not possible to evaluate the fluid properties at the mean film
temperature. Hence the problem has to be solved by trial and error solution by assuming a
suitable value for hav and check for the assumed value.

Trial 1:- Assume hav = 10 W/(m2-K)

qw = πdhav[Tw – T∞] or Tw = T∞ + qw /(πdhav) = 27+ 30 /(π x 0.025 x 10) = 65 0C.

Mean film temperature = 0.5 x (27 + 65) = 46 0C.

Properties of air at 46 0C are: β = 1 /(46 + 273) = 3.135 x 10 − 3 1/K; k = 0.0280 W/(m-K)

Pr = 0.684; ν = 17.45 x 10 − 6 m2/s.

9.81 x 3.135 x 10 − 3 x (65 – 27) x 0.025 3


Grd = ---------------------------------------------------- = 6.0 x 10 4.
[17.45 x 10 − 6 ] 2

(6.0 x 10 4 x 0.684) 1/6


½
Hence [Nuav] = 0.60 + 0.387 ------------------------------------ = 2.8
[ 1 + (0.559/0.6985) 9/16] 8/27

Hence Nuav = 7.84 or hav = 7.84 x 0.028 / 0.025 = 8.78 W/(m2 – K).

Since the calculated value of hav deviates very much from the assumed value one more
iteration is required.

(ii) Trial 2: Assume hav = 8.78 W/(m2-K).


Proceeding in the same way as in trial 1 we have Tw = 70.5 0C. Hence Tm = 48.75 00C
Properties of air at 50 0C are: β = 1/(50+273) = 3.05 x 10 − 3 1/K; Pr = 0.698;
k = 0.02826 W/(m-K); ν =17.95 x 10 − 6 m2/s.

9.81 x 3.05 x 10 − 3 x (70.5 – 27) x 0.025 3


Grd = ---------------------------------------------------- = 6.4 x 10 4.
[17.95 x 10−6 ] 2

362
1/6

(6.4 x 104x 0.698)


½
[Nuav] = 0.60 + 0.387 ----------------------------------- = 2.89 or Nuav = 8.35
[1 + (0.559/0.698)9/16] 8/27

Hence hav = 8.35 x 0.02865 / 0.025 = 9.5691 W/(m2-K).

The calculated value of hav agrees with the assumed value within 5 % iteration is stopped.

The equilibrium temperature of the surface = Tw = 27 + 30 /(π x 0.025 x 9.5691) = 67 0C.

7.10. Solution:
Qw

d
L

Data:- L = 0.3 m; d = 0.01 m; Qw = 550 W; T∞ = 20 0C;

550
Wall heat flux = qw = Qw /(πdL) = ---------------------- = 58357 W/m2
(π x 0.01 x 0.3)

Since Tw is not known, fluid properties cannot be evaluated at the mean


temperature and hence the problem has to be solved by trial and error procedure
by assuming a suitable value for hav and then check for the assumed value.

Case(i):- When the heater is immersed in water

For free convection in liquids the order of heat transfer coefficient is around 10 to 1000
W/(m2-K).Let us assume hav = 1000 W/(m2-K).

Hence Tw = T∞ + qw / hav = 20 + 58357 / 1000 = 78.4 0C.

Mean film temperature = 0.5 x (20 + 78.4) = 49.2 0C.

Properties of water at 49.2 0C are: β = 3.103 x 10 − 3 1 / K;

Pr = 3.68; k = 0.639 W/(m-K) ; ν = 0.5675 x 10 − 6 m2/s;

9.81 x 3.103 x 10 − 3 x (78.4 – 20) x (0.01) 3


Grd = ------------------------------------------------------- = 5.519 x 10 6
(0.5675 x 10 − 6) 2

363
Rad = 5.519 x 10 6 x 3.68 = 2.03 x 10 7.

For horizontal cylinders Nusselt number is given by

Nud = C Rad n with C = 0.125 and n = 1/3 for this value of Rad.

Hence Nud = 0.125 x (2.03 x 10 7) 1/3 = 33.908

Hence hav = 33.908 x 0.639 / 0.01 = 2166.21 W/(m2 –K)

Trial 2:- Assume hav = 2166.21 W/(m2-K)

Tw = 20 + 58357 / 2166.21 = 46.940C.

Hence mean film temperature =0.5x(20 + 46.94) = 33.47 0C.


Properties of water at 33 0C 0C are : k = 0.6129 W/(m-K); Pr = 5.68; β = 3.3 x 10 − 3; ν =
0.831 x 10 − 6 m2 /s.

9.81 x 3.3 x 10 − 3 x (46.94 – 20) x (0.01) 3


Grd = ----------------------------------------------------- = 2.2 x 106.
(0.831 x 10 − 6) 2
Rad = 2.2 x 10 x 5.68 = 1.24 x 10 7
6

Hence, Nuav = 0.125 x (1.24 x 10 7) 0.333 = 28.77


Therefore, hav = 28.77 x 0.6129 / 0.01 = 1763.3 W/(m2 – K)

Trial 3:- Assume hav =1763 W/(m2-K)

Tw = 20 + 58357 / 1763 = 53.1 0C. Mean film temperature = 0.5 x (20 + 53.1) = 36.55 0C.

Properties of water at 400C are: k = 0.6280 W/(m-K); Pr = 4.34; β = 3.19 x 10 −3 1/K;


ν = 0.657 x 10 − 6 m2/s.

9.81 x 3.19 x 10 − 3 x (53 – 20) x (0.01) 3


Rad = ------------------------------------------------- x 4.34 = 1.03 x 10 7
(0.657 x 10 − 6) 2

Hence, Nuav = 0.125 x (1.03 x 10 7) 0.333 = 27.05


Therefore, hav = 27.05x 0.6280 x / 0.01 = 1698.74 W/(m2-K)

Since the calculated value of hav agrees with the assumed value within 4%, iteration is
stopped and the equilibrium temperature of the heater is calculated as

Tw = 20 + 58357 / 1698.74 = 54.35 0C


Case (ii):- When the heater is exposed to air

364
When a heated surface is exposed to air the order of heat transfer coefficient varies between 5
and 20 W/(m2-K).

Trial 1:- Assume hav = 20 W/(m2-K)

Tw = 20 + 58357 / 20 = 2938 0C. Mean film temperature = 0.5 x (20 + 2938) = 1479 0C

Properties of air at 1479 0C are : β = 1/(1479 +273) = 5.71 x 10 − 4 1/K; Pr = 0.7045;

k = 0.108 W/(m-K); ν = 294.3 x 10 − 6 m2/s.

9.81 x 5.71 x 10 − 4 x (2938 – 20) x (0.01) 3


Rad = ------------------------------------------------- --- x 0.7045 = 133
(294.3 x 10 − 6) 2

Nuav = 0.850 x (133) 0.188 = 2.13

Hence hav = 2.13 x 0.108 / 0.01 = 23 W/(m2-K) This is 13% away from the assumed value
and hence one more iteration is required.

Trial2:- Assume hav = 23 W/(m2-K)

Tw = 20 + 58357 / 23 = 2557 0C ; Mean film temperature = 0.5 x ( 20 + 2557) = 1289 0C

Properties of air at 1289 0C are: β = 6.4 x 10 − 4 1/K; ν = 244.34 x 10 − 6 m2/s;Pr = 0.705;

k = 0.0978 W/(m-K)

9.81 x 6.4 x 10 − 4 x (2557 – 20) x (0.01) 3


Rad = ------------------------------------------------- --- x 0.705 = 188
(244.34 x 10 − 6) 2

Nuav = 0.850 x (188) 0.188 = 2.275.Hence hav = 2.275 x 0.0978 / 0.01 = 22.25 W/(m2 – K).

This value of hav agrees with the assumed value within 4% and hence the iteration is stopped.
The equilibrium temperature of the heater is therefore given by

Tw = 20 + 58357 / 22.25 = 2643 0C.

365
Chapter 8 : Condensation & Boiling
8.1. Introduction: Knowledge of heat transfer occurring during change of phase i.e. during
condensation and boiling is very useful in a number of ways. For example in all power and
refrigeration cycles, it is necessary to convert a liquid into a vapour and vice-versa. This is
accomplished in boilers or evaporators and condensers.

Heat transfer coefficients in both condensation and boiling are generally


much higher than those encountered in single phase processes. Values greater than 1000
W/(m2-K) are almost always obtained. This fact has been used in several recent
applications where it is desired to transfer high heat fluxes with modest temperature
differences. An example is the “heat pipe” which is a device capable of transferring a large
quantity of heat with very small temperature differences.

8.2. Film-wise and Drop-wise condensation:- Condensation occurs whenever a vapour


comes into contact with a surface at a temperature lower than the saturation temperature of
the vapour corresponding to its vapour pressure. The nature of condensation depends on
whether the liquid thus formed wets the solid surface or does not wet the surface. If the liquid
wets the surface, the condensate flows on the surface in the form of a film and the process is
called “film-wise condensation”. If on the other hand, the liquid does not wet the surface, the
condensate collects in the form of droplets, which either grow in size or coalesce with
neighboring droplets and eventually roll of the surface under the influence of gravity. This
type of condensation is called “drop-wise condensation”.

The rate of heat transfer during the two types of condensation processes
is quite different. For the same temperature difference between the vapour and the surface,
the heat transfer rates in drop-wise condensation are significantly higher than those in film-
wise condensation. Therefore it is preferable to have drop-wise condensation from the
designer’s point of view if the thermal resistance on the condensing side is a significant part
of the total thermal resistance. However it is generally observed that, although drop-wise
condensation may be obtained on new surfaces, it is difficult to maintain drop-wise
condensation continuously and prolonged condensation results in a change to film-wise
condensation. Therefore it is still the practice to design condensers under the conservative
assumption that the condensation is of film type.

8.3. Nusselt’s theory for laminar film-wise condensation on a plane vertical surface:-
The problem of laminar film-wise condensation on a plane vertical surface was first
analytically solved by Nusselt in 1916.He made the following simplifying assumptions in his
analysis.
(i) The fluid properties are constant.
(ii) The plane surface is maintained at a uniform temperature, Tw which is less than the
saturation temperature Tv of the vapour.

(iii) The vapour is stationary or has a very low velocity and so it does not exert any drag on
the motion of the condensate: i.e., the shear stress at the liquid-vapour interface is zero.

366
(iv) The flow velocity of the condensate layer is so low that the acceleration of the
condensate is negligible.
(v) The downward flow of the condensate under the action of gravity is laminar.
(vi) Heat transfer across the condensate layer is purely by conduction; hence the liquid
temperature distribution is linear.

[τ + (∂τ/∂y)dy]dx
x pdy τdx
pdy
δ

dx

(p+dp)dy (p+dp)dy

ρLdxdyg ρvdxdyg
(a) Force balance on a condensate (b) Force balance on a vapour element
element at the same distance x from top
Figure 8.1 : Laminar film condensation on a vertical plate

Consider the film-wise condensation on a vertical plate as illustrated in Figure8.1.


Here ‘x’ is the coordinate measured downwards along the plate, and ‘y’ is the coordinate
measured normal to the plate from the plate surface. The condensate thickness at any x is
represented by δ [ δ = δ(x)]. The velocity distribution u(y) at any location x can be
determined by making a force balance on a condensate element of dimensions dx and dy in x
and y directions as shown in Figure 8.1(a). Since it is assumed that there is no acceleration of
the liquid in x direction, Newton’s second law in x direction gives

ρLdxdyg + pdy + [τ + (∂τ/∂y)dy]dx −τdx − (p + dp)dy = 0

or (∂τ/∂y) = (dp/dx) −ρLg …………………………………..(8.1)

Expression for (dp/dx) in terms of vapour density ρv can be obtained by making a force

balance for a vapour element as shown in Figure 8.1(b). The force balance gives

ρvdxdyg + pdy = (p + dp) dy

367
or (dp/dx) = ρvg

Substituting this expression for dp/dx in Equation (8.1) we have

(∂τ/∂y) = (ρv−ρL)g

Since the flow is assumed to be laminar, τ = μL(∂u/∂y)

Therefore ∂/∂y{μL (∂u/∂y)} = (ρv−ρL)g

Integrating with respect to y we have μL (∂u/∂y) = (ρv−ρL)g y + C1

(ρv−ρL)g y C1
Or (∂u/∂y) = --------------- + ------- ……(8.2)
μL μL

Integrating once again with respect to y we get


(ρv−ρL)g y2 C1 y
u(y) = ---------------- + ------------- + C2 ......(8.3)
2 μL μL

The boundary conditions for the condensate layer are: (i) at y = 0, u = 0;

(ii) at y = δ, (∂u/∂y) = 0.

Condition (i) in Equation (8.3) gives C2 = 0 and condition (ii) in Equation (8.2) gives

(ρv−ρL)g δ C1
0 = ------------- + ---------
2 μL μL

(ρv−ρL)g δ
Therefore C1 = − --------------
2

Substituting for C1 and C2 in Equation(8.3) we get the velocity distribution in the condensate
layer as

g(ρL − ρv)
u(y) = --------------- [ δy – (y2/2)] …………………(8.4)
μL

If ‘m’ is the mass flow rate of the condensate at any x then


δ
m = ∫ρLudy
0

368
δ
m = ∫ ρL{ g(ρL − ρv) / μL}[ δy – (y2/2)]dy
0

g ρL (ρL − ρv) δ 3
= ------------------ ………………………………..(8.5)
3 μL

g ρL (ρL − ρv) δ 2 dδ
Hence dm = ----------------------
μL

Amount of heat transfer across the condensate element = dq = dm hfg

g ρL (ρL − ρv) δ 2 dδ hfg


Or dq = ------------------------- ………………………….(8.6)
μL

Energy balance for the condensate element shown in the figure can be written as

dq = kL(Tv – Tw)dx / δ

g ρL (ρL − ρv) δ 2 dδ hfg


Or ------------------------------- = kL(Tv – Tw)dx / δ………….(8.6)
μL

kL μL (Tv – Tw)dx
or δ dδ = ----------------------
3

g ρL (ρL − ρv) hfg

Integrating we get
δ4 kL μL (Tv – Tw)x
----- = --------------------- + C 3
4 g ρL (ρL − ρv) hfg

At x = 0, δ = 0. Hence C3 = 0.

Therefore δ4 kL μL (Tv – Tw)x


----- = ---------------------
4 g ρL (ρL − ρv) hfg

4 kL μL (Tv – Tw)x
or δ = [-------------------------- ] 1/ 4……………………(8.7)
g ρL (ρL − ρv) hfg

369
Now kL (Tv – Tw)dx
----------------- = hx dx [Tv – Tw]
δ
kL g ρL (ρL − ρv) hfg kL3
Therefore hx = --------- = [ --------------------------] 1 / 4
δ 4 μL (Tv – Tw)x

g ρL (ρL − ρv) hfg kL3


Or hx = 0.707[ --------------------------] 1 / 4 ...............................(8.8)
μL (Tv – Tw)x

The local Nusselt number Nux can therefore be written as

hxx g ρL (ρL − ρv) hfg x3


Nux = ----- = 0.707[ --------------------------] 1 / 4 ...............................(8.8)
kL μL (Tv – Tw)kL

The average heat transfer coefficient for a length L of the plate is given by
L
hav = (1/L) ∫ hxdx ………………………………………(8.9)
0

It can be seen from Equation (8.8) that hx = C x − ¼ , where C is a constant given by

g ρL (ρL − ρv) hfg kL3


Or C = 0.707[ --------------------------] 1 / 4 …………………(8.10)
μL (Tv – Tw)

L
Hence hav = (1/L) C ∫ x − ¼ dx = (C / L) (4/3) L− ¼ = (4/3)C L− ¼
0

Substituting for C from Equation (8.10) we have

g ρL (ρL − ρv) hfg kL3


hav = 0.943[ --------------------------] 1 / 4 = (4/3)hx|x = L.........................(8.11)
μL (Tv – Tw)L

8.4. Condensation on Inclined Surfaces : Nusselt,s analysis given above can readily be
extended to inclined plane surfaces making an angle θ with the horizontal plane as shown in
Figure 8.2.

370
y

Figure 8.2 : Condensation on


an inclined plane surface
θ
g

The component of the gravitational force along the length of the pate is g sin θ.The
expressions for local and average heat transfer coefficients can therefore be written
as

g sin θρL (ρL − ρv) hfg kL3


hx = 0.707[ ------------------------------------] 1 / 4
μL (Tv – Tw)x
…………….......(8.12)

g sin θρL (ρL − ρv) hfg kL3


and hav = 0.943[ ----------------------------------] 1 / 4 = (4/3)hx|x = L
μL (Tv – Tw)L

…………………………(8.13)

8.5. Condensation on a horizontal tube: The analysis of heat transfer for condensation on
the outside surface of a horizontal tube is more complicated than that for a vertical
surface. Nusselt,s analysis for laminar film-wise condensation on the surface of a horizontal
tube gives the average heat transfer coefficient as

gρL (ρL − ρv) hfg kL3


hav = 0.725 [ --------------------------------- ] 1 / 4 ……………(8.14)
μL (Tv – Tw) D

where D is the outside diameter of the tube. A comparison of equations (8.11) and (8.14) for
condensation on a vertical tube of length L and a horizontal tube of diameter D gives

[hav]vertical 0.943
--------------- = ------------(D/L) ¼ = 1.3 (D/L) 1/4 .....................................(8.15)
[hav]horizontal 0.725

This result implies that for a given value of (Tv – Tw), the average heat transfer coefficient for
a vertical tube of length L and a horizontal tube of diameter D becomes equal when L =

371
2.856 D.For example when L = 100 D, theoretically [hav]horizontal would be 2.44 times
[hav]vertical. Therefore horizontal tube arrangements are generally preferred to vertical tube
arrangements in condenser design.

8.6. Condensation on horizontal tube banks: Condenser design generally involves


horizontal tubes arranged in vertical tiers as shown in Figure 8.3 in such a way that the

Figure 8.3 : Film-wise condensation on


horizontal tubes arranged in a vertical
tier.

condensate from one tube drains on to tube just below. If it is assumed that the drainage from
one tube flows smoothly on to the tube below, then for a vertical tier of N tubes each of
diameter D, the average heat transfer coefficient for N tubes is given by

gρL(ρL – ρv)hfg kL3 1


[hav]N tubes = 0.725 [ ----------------------- ] ¼ = ------------ [hav] 1 tube ……………(8.16)
μL(Tv – Tw) N D N 1/ 4

This relation generally gives a conservative value for the heat transfer coefficient. Since
some turbulence and some disturbance of condensate are unavoidable during drainage, the
heat transfer coefficient would be more than that given by the above equation.

8.7. Reynolds number for condensate flow: Although the flow hardly changes to turbulent
flow during condensation on a single horizontal tube, turbulence may start at the lower
portions of a vertical tube. When the turbulence occurs in the condensate film, the average
heat transfer coefficient begins to increase with the length of the tube in contrast to its
decrease with the length for laminar film condensation. To establish a criterion for transition
from laminar to turbulent flow, a “Reynolds number for condensate flow” is defined as
follows.
ρL uav Dh
Re = ----------------- …………………..(8.17)
μL

372
where uav is the average velocity of the condensate film and Dh is the hydraulic diameter for
the condensate flow given by
4 x (Cross sectional area for condensate flow) 4A
Dh = --------------------------------------------- --------- = -------
Wetted Perimeter P
4A ρL uav 4M
Therefore Re = -------------------- = ---------------……………..(8.18)
P μL P μL

where M is mass flow rate of condensate at the lowest part of the condensing surface in kg/s.
The wetted perimeter depends on the geometry of the condensing surface and is given as
follows.
πD …..For vertical tube of outside diameter D ………….(8.19 a)
P = 2L …...For horizontal tube of length L …………………(8.19 b)
W ….. For vertical or inclined plate of width W………...(8.19 c)

Experiments have shown that the transition from laminar to turbulent condensation takes
place at a Reynolds number of 1800. The expression for average heat transfer coefficient
for a vertical surface [Equation(8.11)] can be expressed as follows.

g ρL(ρL – ρv) kL3 hfg


hav = 0.943 [ ----------------------------- ] 1 / 4
μL(Tv – Tw)

Generally ρL >> ρv. Therefore

g ρL2 kL3 hfg


hav = 0.943 [ ----------------------------- ] 1 / 4 ………………..(8.20)
μL(Tv – Tw)

The above equation can be arranged in the form

hav [νL2 / (gkL3) ] 1 / 3 = 1.47 Re L− 1/ 3 ………………………(8.21)

The above equation is valid for ReL < 1800.


It has been observed experimentally that when the value of the film Reynolds number is
greater than 30, there are ripples on the film surface which increase the value of the heat
transfer coefficient. Kutateladze has proposed that the value of the local heat transfer
coefficient be multiplied by 0.8(RE / 4)0.11 to account for the ripples effect. Using this
correction it can be shown that

ReL
(hav / kL)( νL2 / g) 1 / 3 = ------------------------ ………………(8.22)
[1.08 ReL1.22 – 5.2]

373
8.8. Turbulent film condensation: For turbulent condensation on a vertical surface,
Kirkbride has proposed the following empirical correlation based on experimental data.

hav [νL2 / (gkL3) ] 1 / 3 = 0.0077 (ReL) 0.4 ……………………(8.23)

In the above correlation the physical properties of the condensate should be evaluated at the
arithmetic mean temperature of Tv and Tw.

8.9. Film condensation inside horizontal tubes: In all the correlations mentioned above, it
is assumed that the vapour is either stationary or has a negligible velocity. In practical
applications such as condensers in refrigeration and air conditioning systems, vapour
condenses on the inside surface of the tubes and so has a significant velocity. In such
situations the condensation phenomenon is very complicated and a simple analytical
treatment is not possible. Consider, for example, the film condensation on the inside surface
of a long vertical tube. The upward flow of vapour retards the condensate flow and causes
thickening of the condensate layer, which in turn decreases the condensation heat transfer
coefficient. Conversely the down ward flow of vapour decreases the thickness of the
condensate film and hence increases the heat transfer coefficient.
Chato recommends the following correlation for condensation at low vapour
velocities inside horizontal tubes:

g ρL(ρL – ρv) kL3 h×fg


hav = 0.555 [ --------------------------- ] 1 / 4 ……………..(8.24 –a)
μL(Tv – Tw) D

where h×fg = hfg + (3/8)cp,L(Tv – Tw) ………………………….(8.24 –b)

This result has been developed for the condensation of refrigerants at low Reynolds number
[Rev = (ρvuvD) / μv < 35,000 ; Rev should be evaluated at the inlet conditions.]

For higher flow rates, Akers, Deans and Crosser propose the following
correlation for the average condensation heat transfer coefficient on the inside surface of a
horizontal tube of diameter D:

hav D
------ = 0.026 Pr 1 / 3 [ReL + Rev(ρL / ρv) ½ ] 0.8 ………..(8.25)
k

where ReL = (4ML) / (πDμL) : Rev = (4Mv) / (πDμv) …………………….(8.26)

The above equation correlates the experimental data within 50 % for ReL > 5000 and

Rev > 20,000.

8.10. Illustrative examples on film wise condensation:

374
Example 8.1: Saturated steam at 1.43 bar condenses on a 1.9 cm OD vertical tube which is
20 cm long. The tube wall is at a uniform temperature of 109 0C . Calculate the average heat
transfer coefficient and the thickness of the condensate film at the bottom of the tube.

Solution: Data:- Tv = Saturation temperature at 1.43 bar = 110 0 C (from steam tables)

Tw = 109 0C ; Characteristic length = L = 0.2 m ; D = 0.019 m ;

To find : (i) hav ; (ii) δ(x)|x=L;

Mean film temperature of the condensate (water) = 0.5 x (110 + 109) = 109.5 0C.

Properties of water at 109.5 0C are: ρL = 951.0 kg/m3; μL = 258.9 x 10 − 6 N-s / m2;

k = 0.685 W/(m-K); ν = 0.2714 x 10 − 6 m2/s; hfg = 2230 kJ/kg. Also ρL >>> ρv.

Let us assume that the condensate flow is laminar and later check for this assumption.

g ρL2 kL3 hfg


hav = 0.943 [ ----------------------------- ] 1 / 4
μL(Tv – Tw) L

9.81 x (951)2x (0.685)3 x 2230 x 103


Hence hav = 0.943 x [--------------------------------------------] 1/ 4
258.9 x 10 − 6 x (110 – 109) x 0.2

= 17,653 W / (m2-K)

(ii) hav = (4 / 3)hx|x=L or hx|x=L = ¾ x hav = 0.75 x 17,653 = 13,240 W/(m2-K).

Therefore δ(x)|x=L = kL / hx|x=L = 0.685 / 13240 = 5.174 x 10 − 5 m = 0.0517 mm.

Check for Laminar flow assumption:- The relation between hav and Reynolds number at the
bottom of the tube is given by

hav [νL2 / (gkL3) ] 1 / 3 = 1.47 Re L− 1/ 3 or ReL = (1.47 / hav)3(gkL3 / νL2)

Hence ReL = (1.47 / 17,653) 3 [9.81 x 0.685 3 / {0.2714 x 10 − 6}2]

= 24.72

Since ReL < 1800, our assumption that condensate flow is laminar is correct.

375
Example 8.2. Saturated steam at 80 0C condenses as a film on a vertical plate 1 m high. The
plate is maintained at a uniform temperature of 70 0C. Calculate the average heat transfer
coefficient and the rate of condensation. What would be the corresponding values if the effect
of ripples is taken into consideration.

Solution:Data:- Tv = 80 0C; Tw = 70 0C; Mean film temperature =0.5 x (80 + 70) = 75 0C.

Properties of condensate (liquid water) at 75 0C are: ρL = 974.8 kg/m3;

kL = 0.672 W /(m-K) ; μL = 381 x 10 − 6 N-s/m2; hfg at 80 0C = 2309 kJ/kg-K;

νL = 0.391 x 10 − 6 m2/s.Charecteristic length = L = 1.0 m.

Assuming laminar film condensation the average heat transfer coefficient is given by

g ρL2 kL3 hfg


hav = 0.943 [ ----------------------------- ] 1 / 4
μL(Tv – Tw)

9.81 x (974.8)2 x (0.672)3 x 2309 x 103


= 0.943 x [ ------------------------------------------------] 1/ 4 = 6066.6 W /(m2 – K).
381.6 x 10 − 6 x (80 – 70 ) x 1.0

hav L (Tv – Tw) 6066.6 x 1.0 x (80 – 70)


Condensate rate = M = --------------------- = ------------------------------- 0.0263 kg/s.
hfg 2309 x 10 3

4M
Check for laminar flow assumption :- ReL = --------------, where P = width of the plate for
μL P

4 x 0.0263
vertical flat plate. Hence ReL = ------------------------- = 276
381 x 10 − 6
Since ReL < 1800, the condensate flow is laminar.

Since ReL > 30, it is clear that the effects of ripples have to be considered.
4M 4 hav L (Tv – Tw)
Now ReL = ------------ = -----------------------
μL P μL P hfg

ReL μL P hfg
Hence hav = ------------------ …………………………………………….(1)
4L(Tv – Tw)

376
When the effects of ripples are considered the relation between ReL and hav is given by
Equation(8.22) as follows:
ReL
1.08 ReL1.22 – 5.2 = ----------------------- Substituting for hav from Equation(1) we have
(hav/kL)(νL2 /g)1 /3

4L (Tv – Tw) kL (g / νL2) 1/3


1.08 ReL1.22 – 5.2 = --------------------------------
μL P hfg

4 x 1 x (80 – 70) x 0.672 x {9.81 /( 0.391 x 10 − 6)2}1/3


1.08 ReL1.22 – 5.2 = ----------------------------------------------------------------------
381.6 x 10 − 6 x 1.0 x 2309 x 10 3

1.08 ReL1.22 – 5.2 = 1221.3. Or ReL = 319.4

319.4 x 381.6 x 10 − 6 x 1.0 x 2309 x 10 3


Hence from Equation(1) hav = ---------------------------------------------------- = 7036 W /(m2 –
K).
4 x 1.0 x (80 – 70)

hav L (Tv – Tw) 7036 x 1.0 x (80 – 70)


Hence M = --------------------- = ---------------------------- = 0.03047 kg / s.
hfg 2309 x 10 3

[ It can be seen that the ripples on the surface increase the heat transfer coefficient by about
15 %].

Example 8.3. Air free saturated steam at 65 0C condenses on the surface of a vertical tube of
OD 2.5 cm. The tube surface is maintained at a uniform temperature of 35 0C. Calculate the
length of the tube required to have a condensate flow rate of 6 x 10 −3kg/s.

Solution:Data:- Tv = 65 0C; Tw = 35 0C; D0 = 0.025 m; M = 6 x 10 − 3 kg/s.

To find length of the tube, L.

Mean film temperature = 0.5 x (65 + 35) = 50 0C.Properties of condensate

(liquid water) at 50 0C are: kL = 0.640 W/(m-K); μL = 0.562 x 10 − 3 N-s/m2; ρL = 990

kg/m3; At 65 0C, hfg = 2346 x 10 3 J/(kg-K).

4M 4 x 6 x 10 − 3
Reynolds number = Re = --------------- = ------------------------------- = 544

377
μL πDo 0.562 x 10 − 3 x π x 0.025

Since Re < 1800 flow is laminar. It is more convenient to use Equation(8.21)

hav [νL2 / (gkL3) ] 1 / 3 = 1.47 Re L− 1/ 3

or (gkL3) 1.47 x (544)− 1/3 x [9.81 x 0.643] 1/3


hav = 1.47 Re L− 1/ 3 [ ---------- ] 1/3 = ----------------------------------------------
νL2 (0.562 x 10 − 3/ 990)2

= 3599 W/(m2 – K)

Heat balance equation gives M hfg = hav πDoL [Tv – Tw]

M hfg 6 x 10 − 3 x 2346 x 10 3
Therefore L = ---------------------- = ------------------------------------
hav πDo [Tv – Tw] 3599 x π x 0.025 x (65 – 35)

= 1.66 m

Example 8.4. Air free saturated steam at 85 0C condenses on the outer surfaces of 225
horizontal tubes of 1.27 cm OD, arranged in a 15 x 15 array. Tube surfaces are maintained
at a uniform temperature of 75 0C. Calculate the total condensate rate per one metre length
of the tube.

Solution: Data:- Tv = 85 0C; Tw = 75 0C; Do = 0.0127 m; L = 1 m;

Number of tubes in vertical tier = N = 15 ; Total number of tubes = n = 225;

Mean film temperature = 0.5 x (85 + 75) = 80 0C. Properties of the condensate (liquid

water) are: kL = 0.668 W/(m-K); μL = 0.355 x 10 − 3 N-s/m2; ρL= 974 kg/m3;

At 85 0C, hfg = 2296 x 10 3 J/(kg-K).

For N horizontal tubes arranged in a vertical tier, hav is given by

g ρL2 hfg kL3


hav = 0.725 [ --------------------- ] 1 / 4
μL(Tv – Tw)NDo

0.725 x [9.81 x (974)2 x (0.668)3] 1/4


hav = ------------------------------------------------------ = 7142 W/(m2 – K)
[0.355 x 10 − 3 x (85 – 75) x 15 x 0.0127] ¼

378
Q = hav Atotal (Tv – Tw) = hav n π DoL (Tv – Tw)

= 7142 x 225 x π x 0.0127 x 1 x (85 – 75) = 641.14 x 10 3 W

Mass flow rate of condensate = M = Q / hfg = 641.14 x 10 3 / 2296 x 10 3 = 0.28 kg/ (s-m)

Example 8.5. Superheated steam at 1.43 bar and 200 0C condenses on a 1.9 cm OD vertical
tube which is 20 cm long. The tube wall is maintained at a uniform temperature of 109 0C.
Calculate the average heat transfer coefficient and the thickness of the condensate at the
bottom of the tube. Assume cp for super heated steam as 2.01 kJ/(kg-K).

Solution: With a superheated vapour, condensation occurs only when the surface
temperature is less than the saturation temperature corresponding to the vapour pressure.
Therefore for a superheated vapour, the amount of heat to be removed per unit mass to
condense it is given by

Q / M = hfg + cpv(Tv – Tsat)

Where cp is the specific heat of superheated steam and Tsat is the saturation temperature
corresponding to the vapour pressure. If it is assumed that the liquid – vapour interphase is at
the saturation temperature, then Equation(8.20 ) still holds good with hfg replaced by
hfg + cpv(Tv – Tsat).

Hence g ρL2 kL3 { hfg + cpv(Tv – Tsat)}


hav = 0.943 [ --------------------------------------- ] 1/ 4
μL(Tsat – Tw)L

At 1.43 bar, Tsat = 110 0C.Mean film temperature = 0.5 x (110 + 109) = 109.5 0C.

Properties of the condensate at 109.5 0C are: kL = 0.685W/(m-K); μL = 0.259 x 10 − 3 N-

s/m2; ρL= 951 kg/m3;At 1.43 bar, hfg = 2230 x 10 3 J/(kg-K).

9.81 x (951)2 x (0.685)3x {2230 x 103 + 2010 x (200 – 110)}


hav = 0.943 x [ -------------------------------------------------------------------------- ] 1/ 4
0.259 x 10 − 3 x (110 – 109) x 0.2

= 18,000 W /(m2 – K).

Hence hx|x=L = (¾) x 18000 = 13,500 W / (m2 – K).

δ(x)|x=L = kL / hx|x=L = 0.685 / 13,500 = 5.07 x 10 − 5m

Example 8.6. Air free saturated steam at 70 0C condenses on the outer surface of a 2.5 cm
OD vertical tube whose outer surface is maintained at a uniform temperature of
50 0C. What length of the tube would produce turbulent film condensation?

379
Solution: Data:- Tv = 70 0C; Tw = 50 0C; Do = 0.025 m; Vertical tube.

To find L such that Re = 1800.

Mean film temperature = 0.5 x (70 + 50) = 60 0C. Properties of the condensate (liquid

water) are : kL = 0.659W/(m-K); μL = 0.4698 x 10 − 3 N-s/m2; ρL= 983.2 kg/m3;

At 70 0C hfg = 2358 x 10 3 J/(kg-K).

Re (μLπDo) 1800 x 0.4698 x 10 − 3 x π x 0.025


Re = 4M / (μLπDo) or M = ------------------- = ---------------------------------------------
4 4

= 0.0166 kg / s.

For turbulent flow hav [νL2 / (gkL3) ] 1 / 3 = 0.0077 (ReL) 0.4

Or hav = 0.0077 (ReL) 0.4[νL2 / (gkL3) ] − 1 / 3

Hence hav = 0.0077 x (1800)0.4 x [ (0.4698 x 10 − 3/983.2)2 / (9.81 x 0.6593) ] − 1 / 3

= 3563.4 W / (m2 – K).

Heat balance equation is M hfg = hav π DoL (Tv – Tw)

M hfg 0.0166 x 2358 x 10 3


Hence L = ----------------------- = -------------------------------------
hav π Do (Tv – Tw) 3563.4 x π x 0.025 x (70 – 50)

=7m

Example 8.7. Saturated steam at 100 0C condenses on the outer surface of a 2 m long
vertical plate. What is the temperature of the plate below which the condensing film at the
bottom of the plate will become turbulent?

Solution: Data:- Tv = 100 0C; L = 2 m. Since Tw is not known, properties of the condensate
at the mean film temperature cannot be determined and therefore the problem has to be
solved by trial and error procedure as follows:

Trial 1:- The properties of the condensate are read at Tv = 100 0C. The properties are

kL = 0.683 W/(m-K); μL = 0.2824 x 10 − 3 N-s/m2; ρL= 958.4 kg/m3;

At 100 0C, hfg = 2257 x 10 3 J/kg-K.

380
Since the flow has to become turbulent at the bottom of the plate we have

hav = 0.0077 (ReL) 0.4[νL2 / (gkL3) ] − 1 / 3 with ReL = 1800

9.81 x 0.683 3
Hence hav = 0.0077 x (1800)0.4 x [ ------------------------------ ] 1 / 3
(0.2824 x 10 − 3 / 958.4) 2

= 5098 W / (m2 – K)

Now M hfg = hav L W (Tv – Tw)

Or Tw = Tv – (M/W)hfg / (hav L). But ReL = 4M / (μLW) or M/W = ReL μL / 4.

ReL μL hfg 1800 x 0.2824 x 10 − 3 x 2257 x 10 3


Therefore Tw = Tv − -------------------- = 100 − ----------------------------------------------
4 hav L 4 x 5098 x 2

= 72 0C

Trial 2:- Assume Tw = 72 0C. Mean film temperature = 0.5 x (100 + 72) = 86 0C.

Properties of the condensate at 86 0C ; kL = 0.677 W/(m-K); μL = 0.3349 x 10 − 3 N-s/m2;

ρL= 968.5kg/m3; At 100 0C, hfg = 2257 x 10 3 J/(kg-K).

9.81 x 0.677 3
Hence hav = 0.0077 x (1800)0.4 x [ ------------------------------ ] 1 / 3
(0.3349 x 10 − 3 / 968.5) 2

= 4541 W /(m2 – K).

1800 x 0.3349 x 10 − 3 x 2257 x 10 3


Therefore Tw = 100 − ----------------------------------------------- = 60 0C
4 x 4541 x 2

Since the calculated value of Tw is quite different from the assumed value, one more iteration
is required.

Trial 3:- Assuming Tw = 60 0C and proceeding on the same lines as shown in trial 2 we

get hav = 4365 W /(m2 – K) and hence Tw = 590C. This value is very close to the value
assumed (difference is within 2 % ). The iteration is stopped. Hence Tw = 59 0C.

381
Example 8.8. Air free saturated steam at 90 0C condenses on the outer surface of a 2.5 cm
OD, 6 m long vertical tube, whose outer surface is maintained at a uniform temperature of
60 0C. Calculate the total rate of condensation of steam at the tube surface.

Solution: Data:- Tv = 90 0C; Tw = 30 0C; Do = 0.025 m; L = 6 m. Vertical tube.

Mean film temperature = 0.5 x (90 + 60) = 75 0C. Properties of the condensate at 75 0C

are: kL = 0.671 W/(m-K); μL = 0.3805 x 10 − 3 N-s/m2; ρL= 974.8 kg/m3; At 90 0C, hfg =

2283 x 10 3 J/(kg-K).

We do not know whether the condensate flow is laminar or turbulent. We start the
calculations assuming laminar flow and then check for laminar flow condition.
For laminar flow
g ρL2 kL3 hfg
hav = 0.943 [ ----------------------------- ] 1 / 4
μL(Tv – Tw) L

9.81 x (974.8)2 x (0.671)3 x 2283 x 10 3


Hence hav = 0.943 x [--------------------------------------------------] 1 / 4
0.3805 x 10 − 3 x (90 – 60) x 6

= 2935.3 W /(m2 – K).

For laminar flow hav [νL2 / (gkL3) ] 1 / 3 = 1.47 Re L− 1/ 3

Or ReL = (1.47 / hav)3(gkL3 / νL2)

= (1.47 / 2935.3)3 x [9.81 x 0.6713 x 974.82 / (0.3805 x 10 − 3)2]

= 2443

Since ReL > 1800, flow is turbulent.

For turbulent flow hav [νL2 / (gkL3) ] 1 / 3 = 0.0077 (ReL) 0.4

Or hav = 0.0077 (ReL) 0.4 / [νL2 / (gkL3) ] 1 / 3……….(1)

ReL = 4M / (μLπDo). But Mhfg = havπDoL (Tv – Tw) or M / (πDo) = havL (Tv – Tw) / hfg

4 havL (Tv – Tw)


Therefore ReL = ----------------------
hfg μL

382
Substituting this expression for ReL in equation (1) we have

4 havL (Tv – Tw)


hav = 0.0077 [------------------------- ] 0.4 [νL2 / (gkL3) ] − 1 / 3
hfg μL

4 L (Tv – Tw)
(hav )0.6 = 0.0077 [------------------------- ] 0.4 [νL2 / (gkL3) ] − 1 / 3
hfg μL

4 x 6 x (90 – 60) (0.3805 x 10 − 3)2


= 0.0077 x [--------------------------------------- ] x [ --------------------------------- ] − 1 / 3
0.4

2283 x 10 3 x 0.3805 x 10 − 3 974.82 x 9.81 x (0.671)3

= 192. Hence hav = [192] 1 / 0.6 = 6390 W /(m2 – K).

havπDoL (Tv – Tw) 6390 x π x 0.025 x 6 x(90 – 60)


Therefore M = ----------------------- = ------------------------------------------ = 0.0396 kg/s
hfg 2283 x 10 3

8.11.Dropwise Condensation: Experimental investigations on condensation have indicated


that, if traces of oil are present in steam and the condensing surface is highly polished , the
condensate film breaks into droplets. This type of condensation is called “drop wise
condensation”. The droplets grow, coalesce and run off the surface, leaving a greater portion
of the condensing surface exposed to the incoming steam. Since the entire condensing
surface is not covered with a continuous layer of liquid film, the heat transfer rate for ideal
drop wise condensation is much higher than that for film wise condensation. The heat
transfer coefficient may be 2 to 3 times greater for drop wise condensation than for film wise
condensation. Hence considerable research has been done with the objective of producing
long lasting drop wise condensation. Various types of chemicals have been tried to promote
drop wise condensation. Continuous drop wise condensation, obtainable with different
promoters varies between 100 to 300 hours with pure steam and are shorter with industrial
steam. Failure occurs because of fouling or oxidation of the surface, or by the flow of the
condensate or by a combination of these effects.
It is unlikely that long lasting drop wise condensation can be produced under
practical conditions by a single treatment of any of the promoters currently available.
Therefore in the analysis of a heat exchanger involving condensation of steam, it is
recommended that film wise condensation be assumed for the condensing surface.

8.12. Boiling Types: When evaporation occurs at a solid-liquid interface, it is called as


“boiling”. The boiling process occurs when the temperature of the surface Tw exceeds the
saturation temperature Tsat corresponding to the liquid pressure. Heat is transferred from the
solid surface to the liquid, and the appropriate form of Newton’s law of cooling is

qw = h [Tw – Tsat] = h ∆Te …………………………(8.27)

383
Where ∆Te = [Tw - Tsat] and is termed as the “excess temperature”. The boiling process is
characterized by the formation of vapour bubbles which grow and subsequently detach from
the surface. Vapour bubble growth and dynamics depend, in a complicated manner, on the
excess temperature ∆Te, the nature of the surface, and the thermo-physical properties of the
fluid, such as its surface tension. In turn the dynamics of vapour bubble growth affect fluid
motion near the surface and therefore strongly influence the heat transfer coefficient.
Boiling may occur under varying conditions. For example if the liquid is
quiescent and if its motion near the surface is due to free convection and due to mixing
induced by bubble growth and detachment, then such a boiling process is called “pool
boiling”. In contrast in “forced convection boiling”, the fluid motion is induced by an
external means as well as by free convection and bubble induced mixing. Boiling may also
be classified as “sub-cooled boiling” and “saturated boiling”. In sub-cooled boiling, the
temperature of the liquid is below the saturation temperature and the bubbles formed at the
surface may condense in the liquid. In contrast, in saturated boiling, the temperature of the
liquid slightly exceeds the saturation temeperature, Bubbles formed at the surface are then
propelled through the liquid by buoyancy forces, eventually escaping from a free surface.

8.13. Pool Boiling Regimes: The first investigator who established experimentally the
different regimes of pool boiling was Nukiyama. He immersed an electric resistance wire
into a body of saturated water and initiated boiling on the surface of the wire by passing
electric current through it. He determined the heat flux as well as the temperature from the
measurements of current and voltage. Since the work of Nukiyama, a number of
investigations on pool boiling have been reported. Figure 8.4 illustrates the characteristics of
pool boiling for water at atmospheric pressure. This boiling curve illustrates the variation of
heat flux or the heat transfer coefficient as a function of excess temperature ∆Te. This curve
pertains to water at 1 atm pressure.From Equation (8.27) it can seen that qw depends on the
heat transfer coefficient h and the excess temperature ∆Te.

Free Convection Regime(up to point A):- Free convection is said to exist if ∆Te ≤ 5 0 C. In
this regime there is insufficient vapour in contact with the liquid phase to cause boiling at the
saturation temperature. As the excess temperature is increased, the bubble inception will
eventually occur, but below point A (referred to as onset of nucleate boiling,ONB), fluid
motion is primarily due to free convection effects.Therefore,according to whether the flow is
laminar or turbulent, the heat transfer coefficient h varies as ∆Te1/4 or as ∆Te1/3 respectively
so that qw varies as ∆Te5/4 or as ∆Te4/3.

Nucleate Boiling Regime(Between points A and C):- Nucleate boiling exists in the range 5 0
C ≤ ∆Te ≤ 30 0 C. In this range, two different flow regimes may be distinguished. In the
region A – B, isolated bubbles form at nucleation sites and separate from the surface,
substantially increasing h and qw. In this regime most of the heat exchange is through
direct transfer from the surface to liquid in motion at the surface, and not through vapour
bubbles rising from the surface. As ∆Te is increased beyond 10 0C (Region B-C), the
nucleation sites will be numerous and the bubble generation rate is so high that continuous
columns of vapour appear. As a result very high heat fluxes are obtainable in this region. In
practical applications, the nucleate boiling regime is most desirable, because large heat fluxes

384
are obtainable with small temperature differences. In the nucleate boiling regime, the heat
increases rapidly with increasing excess temperature

120
107

106
qw, W/m2

105

104

103

∆Te until the peak heat flux is reached. The location of this peak heat flux is called the burnout
point, or departure from nucleate boiling (DNB), or the critical heat flux (CHF). The reason
for calling the critical heat flux the burnout point is apparent from the Figure 8.4. Such high
values of ∆Te may cause the burning up or melting away of the heating element.
Film Boiling Regime:- It can be seen from Figure 8.4 that after the peak heat flux is reached,
any further increase in ∆Te results in a reduction in heat flux. The reason for this curious
phenomenon is the blanketing of the heating surface with a vapour film which restricts liquid
flow to the surface and has a low thermal conductivity. This regime is called the film boiling
regime. The film boiling regime can be separated into three distinct regions namely (i) the
unstable film boiling region, (ii) the stable film boiling region and (iii) radiation dominating
region. In the unstable film boiling region, the vapour film is
unstable, collapsing and reforming under the influence of convective currents and the
surface tension. Here the heat flux decreases as the surface temperature increases, because the
average wetted area of the heater surface decreases. In the stable film boiling region, the heat
flux drops to a minimum, because a continuous vapour film covers the heater surface.In the
radiation dominating region, the heat flux begins to increase as the excess temperature
increases, because the temperature at the heater surface is sufficiently high for thermal
radiation effects to augment heat transfer through the vapour film.

385
8.14. Pool Boiling Correlations:

Correlation for The Nucleate Boiling Regime:- The heat transfer in the nucleate boiling
regime is affected by the nucleation process, the distribution of active nucleation sites on the
surface, and the growth and departure of bubbles.Numerous experimental investigations have
been reported and a number of attempts have been made to correlate the experimental data
corresponding to nucleate boiling regime.The most successful and widely used correlation
was developed by Rohsenow. By analyzing the significance of various parameters in relation
to forced - convection effects. He proposed the following empirical relation to correlate the
heat flux in the entire nucleate boiling regime:

Cpl ∆Te qw _______________


------------- = Csf [ --------- √ σ* / {g (ρl – ρv)} ] 0.33 ………………. (8.28)
hfg Prln (μl hfg)

where Cpl = specific heat of saturated liquid, J /(kg -0C)

Csf = constant to be determined from experimental data depending upon


Heating surface – fluid combination

hfg = latent heat of vapourization, J / kg

g = acceleration due to gravity, m / s2

Prl = Prandtl number of saturated liquid

qw = boiling heat flux, W / m2

∆Te = excess temperature as defined in Equation (8.27)

μl = viscosity of saturated liquid, kg / (m – s)

ρl, ρv = density of liquid and saturated vapour respectively, kg / m3

σ* = surface tension of liquid – vapour interface, N / m.

In Equation (8.28) the exponent n and the coefficient Csf are the two provisions to adjust the
correlation for the liquid – surface combination. Table 8-1 gives the experimentally
determined values Csf for a variety of liquid – surface combinations. The value of n should be
taken as 1 for water and 1.7 for all other liquids shown in Table 8 – 1.

386
Table 8 – 1: Values of Csf of Equation (8.28) for various liquid – surface combinations

Liquid – surface combination Csf

Water – Copper 0.0130


Water – scored copper 0.0068
Water – chemically etched stainless steel 0.0130
Water – mechanically polished stainless steel 0.0130
Water – ground and polished stainless steel 0.0060
Water – brass 0.0060
Water – nickel 0.0060
Water – platinum 0.0130
n-Pentane – polished copper 0.0154
n-Pentane – lapped copper 0.0049
Benzene – chromium 0.1010
Ethyl alcohol – chromium 0.0027

Correlations for Peak Heat Flux:- The correlation given by Equation (8.28) provides
information for the heat flux in nucleate boiling, but it cannot predict the peak heat
flux.Based on stability considerations, Kutateladze and Zuber veloped the following
correlation to calculate the peak heat flux in pool boiling from an infinite horizontal plate
facing up.
Π σ× g (ρl – ρv)
qmax = ----- ρv hfg [ ---------------------- ] ¼ [ 1 + ρv / ρl ] ½ ………….(8.29)
24 ρv2

where σ × = surface tension of liquid – vapour interface, N / m


g = acceleration due to gravity, m / s2
ρl, ρv = density of liquid and vapour respectively, kg / m3
hfg = latent heat of vapourization, J / kg
qmax = peak heat flux, W / m2

It is apparent from this equation that large values of hfg, ρv, g and σ × are desirable for a large
value of the peak heat flux. For example, water has a large value of hfg; hence the peak heat
flux obtainable with boiling water is high. This equation also shows that a reduced
gravitational field decreases the peak heat flux. For most situations, the quantity
[ 1 + ρv / ρl ] ½ is approximately equal to unity. Hence Equation (8.29) can be written as

π
qmax = ----- ρv1/2 hfg [σ* g (ρl – ρv) ] ¼ ………………………..(8.30)
24

Lienhard and Dhir improved the analysis and considered the effect of the size of the
horizontal plate. They showed that

387
qmax = 0.149 hfg ρv1/2 [σ* g (ρl – ρv) ] ¼ ……………………….(8.31)

The above expression was shown to be valid as long as the plate is large and the
dimensionless quantity L [g (ρl – ρv) / σ* ] ½ ≥ 2.7, where L is the characteristic dimension of
the plate. For circular plate L is taken as the diameter, while for a square plate it is taken as
the side of the plate.

For the case of horizontal cylinders of radius R, Lienhard & Sun recommended the
following modified form of Equation (8.30).

π
qmax = ----- ρv1/2 hfg [σ* g (ρl – ρv) ] ¼ f(L’) ……………………..(8.32)
24

where f(L’) = 0.89 + 2.27 exp { − 3.44 √L’ }, and L’ = R [g (ρl – ρv) / σ× ] ½.

Equation (8.32) is valid for situations in which L’ ≥ 0.15. This equation cal also be used for
large spheres in which case f(L’) = 0.84 and L’ = 4.26. For small spheres
f(L’) = 1.734 / √L’, where 0.15 ≤ L’ ≤ 4.26.

Correlations for Film boiling Regime:- No satisfactory correlation exists for the unstable
film boiling region. For the stable film boiling region, Bromley has derived the following
correlation for horizontal cylinders:

kv3 ρv (ρl – ρv) g hfg*


ho = 0.62 [ ----------------------------------- ] ¼ ……………………………………(8.33)
D μv ∆Te

Where ho = average boiling heat transfer coefficient in the absence of radiation W/(m2 –K),
D = outside diameter of the tube,
hfg* = Difference between the enthalpy of the vapour at the mean film temperature,Tm
(Tm = [Tw + Tsat] / 2) and the enthalpy of the liquid at the saturation temperature

≈ hfg + 0.8 Cpv ∆Te.


In the above correlation, all the vapour properties are evaluated at the mean film temperature
while the liquid density is evaluated at the saturation temperature.
Since radiation is significant in film boiling, the radiation component has to be added
in order to obtain the total heat transfer. Brmoley has shown that the total heat transfer
coefficient h is given by the relation

h = ho [ho / h ] 1/3 + hr …………………………….(8.34)

where hr is the radiation heat transfer coefficient which is calculated from the formula for the
radiation heat exchange between parallel planes;

388
1 σ [ Tw4 – Tsat4 ]
hr = ----------------------- ------------------------- ……(8.35)
[ 1 / ε + 1 / α – 1] [Tw – Tsat

where α = absorptivity of liquid and ε = emissivity of the hot tube .

Equation(8.34) is difficult to use because a trial and error approach is needed to determine h.
When hr is smaller than ho, Equation (8.34) can be replaced by

h = ho + ¾ hr ………………………………………(8.36)

8.15. Illustrative examples on pool boiling

Example 8.9. Saturated water at 100 0C is boiled with a copper heating element having a
heating surface area of 0.04 m2which is maintained at a uniform temperature of 115 0C.
Calculate the surface heat flux and the rate of evaporation of water. Also calculate the
critical flux.

Solution: Given:- Tsat = 100 0C; Tw = 115 0C; Surface area = A = 0.04 m2;

Properties of liquid water at 100 0C are: Cpl = 4216 J/(kg – K); hfg = 2257 x 10 3 J/kg;

ρl = 960.6 kg/m3; ρv = 0.5977 kg/m3; Prl = 1.75; μl = 282.4 x 10 − 6 kg/(m – s);

σ× = 58.8 x 10 − 3 N/m ; ∆Te = 115 – 100 = 15 0C.

Since ∆Te lies between 5 0C and 30 0C, the boiling is in the nucleate regime.

Csf = 0.0130 for water – copper combination and for water n = 1.For nucleate boiling

region we can use Equation (8.28) which is as follows:

Cpl ∆Te qw _______________


------------- = Csf [ --------- √ σ× / {g (ρl – ρv)} ] 0.33 ………………. (8.28)
hfg Prln (μl hfg)

Substituting the given values we have


−3
4216 x 15 qw 58.8 x 10
----------------------- = 0.013 x [ --------------------------------- x √ { -------------------- } ] 0.33
(2257 x 10 3) x 1.75 282.4 x 10 − 6 x 2257 x 10 3 9.81 x (960.6 – 0.5977)

Solving for qw we get qw = 4.84 x 10 5 W / m2.

Hence total heat transfer = Q = Aqw = 0.04 x 4.84 x 10 5 = 19.36 x 10 3 W = 19.36 kW.

389
19.36 x 10 3
Rate of evaporation = M = Q / hfg = ----------------- = 8.58 x 10 − 3 kg / s.
2257 x 10 3

Critical heat flux can be calculated from Equation (8.31) namely

qmax = 0.149 hfg ρv1/2 [σ× g (ρl – ρv) ] ¼ ……………………….(8.31)

Hence qmax = 0.149 x 2257 x 10 3 x (0.5977) ½

x [ 58.8 x 10 − 3 x 9.81 x (960.6 – 0.5977) ] ¼

= 1.261 x 10 6 W/m2 = 1.261 MW/m2.

Example 8.10. A metal clad heating element of 6 mm diameter and emissivity equal to unity
is horizontally immersed in a water bath. The surface temperature of the metal is 255 0C
under steady state boiling conditions. If the water is at atmospheric pressure estimate the
power dissipation per unit length of the heater.

Solution: Given:- Tw = 255 0C ; Tsat = saturation temperature of water at 1 atm = 100 0C;

∆Te = 255 – 100 = 155 0C. Since ∆Te > 120 0C, film boiling conditions will prevail. The heat
transfer in this regime is given by Equation(8.33) namely

kv3 ρv (ρl – ρv) g hfg×


ho = 0.62 [ ----------------------------------- ] ¼
D μv ∆Te

Properties of water at 100 0C are: ρl = 957.9 kg/m3; hfg = 2257 x 10 3 J/kg;

ρv = 4.808 kg/m3; Cpv = 2.56 x 10 3 J/(kg-K); kv = 0.0331 W / (m-0C);

μv = 14.85 x 10 − 6 kg / (m-s).

Substituting these values in the expression for ho we have


3 3 3
(0.0331) x 4.808 x (957.9 – 4.808) x 9.81 x {2257 x 10 + 0.8 x 2.56 x 10 x 155 }
ho = 0.62 x [---------------------------------------------------------------------- ] ¼
14.85 x 10 − 6 x 0.006 x 155

= 460 W/(m2 – K)

1 σ {Tw4 – Tsat4}
hr = ---------------------- x ------------------------------
[1/ε + 1/α − 1 ] {Tw – Tsat}

390
1 5.67 x 10 − 8 x { 528 4 – 373 4}
= ------------------------ x -------------------------------------------
[1/1+1/1−1] { 528 – 373}

= 21.3 W / (m2-K).

Now h ≈ ho + ¾ hr = 460 + ¾ x 21.3 = 476 W /(m2 – K).

Hence Q = h A ∆Te = 476 x (π x 0.006 x 1)x 155 = 1.36 x 10 3 W / m.

Example 8.11. A vessel with a flat bottom and 0.1 m2 in area is used for boiling water at
atmospheric pressure. Find the temperature at which the vessel must be maintained if a
boiling rate of 80 kg/h is desired. Assume that the vessel is made of copper and the boiling is
nucleate boiling. Take ρv = 0.60 kg/m3.

Solution: Given:- A = 0.1 m2; Tsat = 100 0C; M = 80 kg/h = 0.022 kg/s; Prl = 1.75

hfg = 2257 x10 3 J /kg; Cpl = 4216 J/(kg-K); ρl = 960.6 kg/m3; σ× = 58.8 x 10 − 3 N/m;

μl = 282.4 x 10 − 6 kg / (m-s); n = 1; For water-copper combination Csf = 0.0130;

M hfg 0.022 x 2257 x 10 3


qw = Q / A = -------- = ---------------------------- = 4.965 x 10 3 W/m2
A 0.1

For nucleate boiling Equation(8.28) is used to calculate the excess temperature .∆Te

Cpl ∆Te qw _______________


------------- = Csf [ --------- √ σ× / {g (ρl – ρv)} ] 0.33
hfg Prln (μl hfg)

4216 x ∆Te
----------------------- = 0.013 x {4.965 x 10 5/(282.4 x 10− 6x 2257 x 103)
2257 x 10 3 x 1.75 _____________________________
x √ 58.8 x 10 − 3 / [9.81 x (960.6 – 0.6)] } 0.33

Or ∆Te = 15.2 0C

Hence Tw = 100 + 15.2 = 115.2 0C.

Example 8.12. Calculate the heat transfer coefficient during stable film boiling of water from
a 0.9 cm diameter horizontal carbon tube. The water is saturated and at 100 0C and the tube
surface is at 1000 0C. Take the emissivity of the carbon surface to be 0.8 and assume that at
the average film temperature, the steam has the following properties.

391
kv = 0.0616 W/(m-K); ρv = 0.266 kg/m3; μv = 28.7 x 10 −6 kg/(m-s); Cpv = 2168 J/(kg-K);
ρl = 958.4 kg/m3

Solution: Given:- D = 0.009 m; ∆Te = Tw – Tsat = 1000 – 100 = 900 0C; ε = 0.8; α = 1.0

hfg× = hfg + 0.8 Cpv ∆Te = 2257 x 10 3 + 0.8 x 2168 x 900 = 3818 x 10 3 J/kg.

For stable film boiling the convection coefficient is given by Equation(8.33)

kv3 ρv (ρl – ρv) g hfg×


ho = 0.62 [ ----------------------------------- ] ¼
D μv ∆Te

(0.0616)3 x 0.266 x (958.4 – 0.266) x 9.81 x 3818 x 10 3


ho = 0.62 x [ ---------------------------------------------------------------------- ] ¼
0.009x (28.7 x 10 − 6) x 900

= 194 W/(m2 – K)

Radiation heat transfer coefficient is given by


1 σ (Tw4 – Tsat4)
hr = ---------------------- -------------------------
[ 1/ε + 1/α – 1] (Tw – Tsat)

1 5.67 x 10 − 8 (1273 4 − 373 4)


hr = ---------------------- x --------------------------------------
[ 1/0.8 + 1/1 – 1] (1273 – 373)

= 131.4 W/(m2 – K).

Hence h = ho + ¾ hr = 194 + ¾ x 131.4 = 292.5 W/(m2-K)

392
CHAPTER 9

HEAT EXCHANGERS
9.1. Introduction: Heat exchangers are devices in which heat transfer takes place between two
or more fluids.Over the period of time many different types of heat exchangers have been
developed to suit particular applications. We find the use of heat exchangers in steam power
plants, chemical processing plants, air conditioning plants, household refrigerators,
automobiles etc.

9.2. Classification of Heat Exchangers: There are many classifications of heat exchangers.
Here we consider classifications based on (i) the heat transfer process, (ii) compactness, (iii)
construction type, (iv) flow arrangement, and (v) heat transfer mechanism.

(i) Classification based on heat transfer process : Based on the heat transfer process, heat
exchangers are classified as
(a) Direct transfer type (or recuperative),
(b) Storage type (or regenerative type)
(c) Direct mixing type.

A “direct transfer type” of heat exchanger is one in which the cold and hot fluids flow
simultaneously through the exchanger and heat is transferred through a wall separating the two
fluids. This type of heat exchanger is the one which is mostly used.
Cold fluid in

Hot
Fluid in Hot
Fluid out

Cold fluid out

Figure9.1: Direct transfer type concentric tube heat exchanger


A

simple example of this type is shown in Figure 9.1. The heat exchanger has a concentric tube
arrangement. One fluid flows through the inner tube, while the other fluid flows through the
annular space between the two tubes. The heat transfer takes place across the wall of the inner

393
tube. Although both the fluids flow through the exchanger simultaneously, there is no mixing
of the two fluid There are no moving parts.The most serious defect of this type of heat
exchanger is the fact that , with passage of time, scale and dirt tends to accumulate on the heat
transfer surface. This accumulation (called fouling) increases the thermal resistance to heat
flow so that the performance of the heat exchanger deteriorates.
A “storage type” heat exchanger or regenerator is one in which the heat transfer from the
hot fluid to the cold fluid occurs through a coupling medium in the form of a porous solid
matrix. The hot and cold fluids flow alternately through the matrix, the hot fluid giving heat
to the matrix and the cold fluid extracting heat from the matrix. The arrangement is illustrated
in Figure 9.2.In many applications, a continuous flow has to be

Hot fluid in Cold fluid in

Cold fluid out Hot fluid out

Figure 9.2: Single matrix storage type heat exchanger

maintained on both the hot and cold sides.In such cases, it is a common practice to use a
rotating disc type matrix, every element of which passes from the hot stream to the cold and
back again during each revolution.. Storage type heat exchangers are commonly used in open
–hearth steel melting furnaces and in blast furnaces.It is also used at very low temperatures in
certain cryogenic cycles where a very high degree of performance is required.
Because of alternate flow of hot and cold fluid through the matrix, the storage type of
heat exchanger is self cleaning. This type also results in a much more compact arrangement
than the direct transfer type. The major disadvantages of the storage type are that some mixing
of hot and cold fluids is inevitable and that sealing the hot side from the cold side in the rotary
design presents considerable difficulty.
A “direct contact type” of heat exchanger is one in which the two fluids come in direct contact
with one another in the heat exchanger. Therefore this type of heat exchanger can be used only

394
for two immiscible fluids.Cooling towers of steam power plants and scrubbers used in
chemical plants are examples of this type. If heat has to be transferred between a gas and a
liquid, the gas is either bubbled through the liquid or the liquid is sprayed in the form of
droplets into the gas. A schematic diagram of a direct contact type heat exchanger is shown in
Figure 9.3. Very often in this type of heat exchanger in addition to heat transfer there will be
mass transfer also.

Figure 9.3: Direct mixing type of Heat Exchanger

(ii) Classification based on compactness: The definition of compactness with respect to heat
exchangers is quite arbitrary. The ratio of the maximum heat transfer surface area of the heat
exchanger to its volume called as ‘surface area density”, is normally used as the parameter. A
heat exchanger having a surface area density greater than 700 m2 / m3 is arbitrarily referred to
as a compact heat exchanger. Automobile radiators having an area density of the order of 1100
m2 / m3 and the glass ceramic heat exchangers used in vehicular gas-turbine engines having a
surface area density of 6600 m2 / m3 are compact heat exchangers. The human lungs, with an
area density of about 20,000 m2 / m3 , are the most compact heat and mass exchanger. The very
fine matrix regenerator used in sterling engine has an area density approaching that of the
human lung.
The compactness of a heat exchanger becomes very important when heat
exchangers are to be employed in automobiles, marine applications, aircraft, space vehicles,
cryogenic systems, and refrigeration and air conditioning units.

(iii) Classification according to construction: Based on the type of construction, heat


exchangers are classified as (a) tubular type, (b) plate type , (c) plate-fin type, (d) tube-fin type
etc. The tubular type is further classified as (1) concentric tube type (double pipe heat
exchanger) and (2) shell and tube heat exchanger

395
(iv) Classification based on flow arrangement: Heat exchangers are also classified as (a)
parallel flow, (b) counter flow, (c) cross flow and (d) multi-pass flow.

Parallel flow arrangement: In parallel flow arrangement the hot and cold fluids enter the
same end of the heat exchanger, flow through in the same direction and leave together at the
other end as illustrated in Figure 9.4(a). The temperature distribution along the length of the
exchanger is shown in Figure 9.4(b) In this figure Thi and Tho are the inlet and exit temperatures

Hot fluid at Thi

Tco Figure 9.4 (a) Parallel Flow


Heat
Tci Exchanger

Tho
Hot fluid out at Tho

Thi
Fig 9.4 (b) Temperature
Tho distribution in a parallel flow
T exchanger
Tco

Thi = Inlet Temp. of Hot Fluid


Tci Tci = Inlet Temp. of cold Fluid
Tho = Exit Temp. of Hot Fluid
Tco = Exit Temp. of Cold Fluid
A

of the hot fluid and Tci and Tco are the inlet and exit temperatures of the cold fluid. It is obvious
from the temperature distribution shown that for parallel flow arrangement Tco cannot exceed
Tho in order to satisfy the second law of thermodynamics and in practice it is necessary that
there shall be a finite temperature difference between the two fluids in order to have a finite
amount of heat transfer.Hence Tco is always less than Tho for a parallel flow heat exchanger.The

396
temperature difference between the two fluids is quite large at one end of the exchanger and
decreases considerably as we proceed towards the other end.

Counter flow arrangement:The schematic and temperature distribution for a counter flow
arrangement are shown in Figure9.5(a) and Figure 9.5 (b) respectively. In this type of
exchanger both the fluids flow parallel to, but opposite to each other. It can be seen from Figure
9.5(b) that Tco has to less than Thi to satisfy the second law. However it is possible to have Tco
> Tho in a counter flow arrangement.

Hot fluid at Thi

Tci Figure 9.5 (a) Counter Flow


Tco Heat Exchanger

Tho
Hot fluid out at Tho

Thi
Fig 9.5 (b) Temperature
Tho distribution in a counter flow
Tco
T exchanger

Tci

Cross flow heat exchanger: In this type the two fluids flow through the exchanger at right
angles to each other. The temperature distribution of the two fluids depend on whether each
fluid is mixed or unmixed as it flows through the exchanger. Figure 9.6 illustrates the direction
of flow of the two fluids as well as the temperature distribution for an

397
Cold fluid
in

y
Hot
Fluid
in x

Th = Th (x,y)

Tc = Tc(x,y)
a
a Hot fluid
Outlet
Temperature
distribution

Cold fluid outlet


Figure 9.6: Unmixed-unmixed cross flow temperature distribution
Heat exchanger

Unmixed-unmixed cross flow heat exchanger. Thermodynamically, the effectiveness for the
cross flow heat exchanger falls in between that for the counter flow and parallel flow
arrangements. The largest temperature difference exists at the corner of the entering hot and

398
cold fluids such as point a in Figure 9.6.In a cross flow heat exchanger, a fluid stream is
considered unmixed when it passes through individual flow channels or tubes with no fluid-
mixing between adjacent flow channels.

Multipass flow: The multi pass flow arrangement is frequently used in heat exchanger
design as multipassing increases the overall effectiveness individual effectiveness. Figure9.7
shows some typical arrangements of multipass flow heat exchangers. Figure9.7(a) is a one
shell pass two tube pass heat exchanger, where as Figure9.7(b) is a two shell pass four tube
pass heat exchanger. Shell side fluid in

(a) (b)

Shell side fluid out


Figure9.7: Multipass flow arrangement:- (a) one shell pass two tube pass arrangement;
(b) Two shell pass four tube pass arrangement.

9.3. Overall Heat Transfer Coefficient and Fouling Factor: In the thermal analysis of heat
exchangers,various thermal resistances in the path of the heat flow from the hot fluid to the
cold fluid are combined in the form of overall heat transfer coefficient, U.For example, for a
double pipe heat exchanger, the total thermal resistance is the sum of the individual components;
i.e., resistance of the inside flow, the conduction resistance in the tube material, and the outside
convective resistance, given by

Rtotal = 1 / (Aihi) + t / (kAln) + 1 / (Aoh0) ….………………………..(9.1)

Where Ai = πdiL, ……..……………………………(9.2a)


Ao = πdoL, …………………………………..(9.2b)
t = (d0 – di) / 2 …………………………………...(9.2c)
Ai and Ao refer to inner and outer heat-transfer surface areas, respectively, t is the wall
thickness, and Aln is the logarithmic mean heat transfer area, defined as

(Ao − Ai)
Aln = ----------------- …………………………………… (9.2d)
ln (Ao / Ai)

399
and hi and ho are inside and outside heat transfer coefficients respectively.
The total heat transfer resistance can be defined in terms of overall heat transfer
coefficient based on either outer or inner areas, as long as the basis is clearly spelled out.
For example, based on outer area, we have

1 1 1
Rtotal = + + ............................. (9.3)
𝐴𝑖ℎ𝑖 𝑘𝐴 𝐴𝑜ℎ𝑜

which after simplifying yields the overall heat transfer coefficient based on inner and
outer areas, respectively as

1
Ui = --------------------------------------------------------------- ……………….(9.4a)
(1/hi)+ [di/(2k)] ln (do/di) + (di/do)( (1/hi)

1
Uo = --------------------------------------------------------------- ……………….(9.4b)
(do/di)(1/hi)+ [d0/(2k)] ln (do/di) + (1/ho)

We note from the above equations that if the wall thickness is negligible Di ≈ Do .For example, in
thin tube heat exchangers or the thermal conductivity of the tube material is very high, the
conduction resistance through the tube may be neglected in Equations(9.4a) and (9.4b) to give
1 1 1 1
𝑈𝑖
≈ 𝑈𝑜
= ℎ𝑖
+ ℎ𝑜
………………………………..(9.5)

The convection coefficients for the inlet and outlet side of the heat exchanger tube can be estimated
using empirical correlations appropriate for the flow geometry and conditions. During normal heat
exchanger operation, surfaces are subjected to fouling by fluid impurities, rust formation, and scale
depositions, which can markedly increase the resistance to heat transfer between the fluids. For
such situations, one would add the fouling resistance (inside and/or outside-side) to Equation (9.4a
and 9.4 b) to give

1
Ui = -------------------------------------------------------------------------- ………………….(9.6a)
(1/hi) + Fi + [di/(2k)] ln (do/di) +(di/do)Fo + (di/do) (1/ho)

and
1
Uo = ------------------------------------------------------------------------ ………………….(9.6b)
(do/di)(1/hi) + (do/di)Fi + [d0/(2k)] ln (do/di) + Fo + 1/ho
Where di = inside diameter of the inner tube, do = outside diameter of the inner tube,
Fi = Fouling factor for the inside surface of the inner tube,and Fo = Fouling factor for the
outside surface of the inner tube.Values of the fouling factor have been determined
experimentally for a number of situations by conducting tests with clean and fouled surfaces.

400
Some representative values obatained after running the heat exchangers for long periods of
time are given in Table 9.1.

Table 9.1 : Representative values of the fouling factor

Fluid Fouling Factor (m2 –K/W)


Distilled water 0.0001
City water (below 500C) 0.0002
City water (above 500C) 0.0004
Sea water & treated boiler feed water (below 500C) 0.0001
Sea water & treated boiler feed water (above 500C) 0.0002
Refrigerating liquids 0.0002
Fuel oil,crude oil 0.001

9.4. Mean Temperature Difference: Consider an elemental area dA of a wall across which
heat is being transferred in a direct transfer type heat exchanger (Figure9.8). The rate of heat
transfer across the area is given by

dQ = U ΔT dA ………………………………..(9.7)

where ΔT = (Th – Tc) is the temperature difference between the bulk mean temperatures of
the two fluids on the two sides of the area dA.

Hot Fluid at Th dA

dQ

Cold fluid at Tc

Figure 9.8: Heat transfer across an elemental area dA

The total heat transfer rate for the heat exchanger is obtained by integrating Equation (9.7).
A
Hence Q = ∫ U ΔT dA
0

If U is assumed to be a constant, then

A
Q = U ∫ ΔT dA …………………………………….(9.8)
0

The mean temperature difference ΔTm for a heat exchanger is defined such that

401
Q = UA ΔTm ………………………………………...(9.9)

It follows from Equation(9.8) and Equation(9.9) that

A
ΔTm = (1 / A) ∫ ΔT dA ……………………………(9.10)
0

Equation (9.10) is an important equation describing the performance of a heat exchanger.

For a circular tube the surface area of the heat exchanger can either be inside surface area A i
or the outside surface area A0. In that case Equation (9.10) and (9.11) has to be modified as
follows:

Q = U0 A0 ΔTm ………………………………………...(9.11a)

Or Q = Ui Ai ΔTm ………………………………………...(9.11b)

Where Ui and U0 are the overall heat transfer coefficients based on the outside surface area and
inside surface area respectively.
9.4.1.Mean temperature difference for a parallel flow heat exchanger: The form of the mean
temperature difference, ΔTm , may be determined by applying an energy balance to differential
control volumes (elements) in the hot and cold fluids. As shown in Figure 9.9, for the case of
parallel flow arrangement, each element is of length dx and the heat transfer surface area is
dA.Energy balance equation for a surface area dA can be written as follows:

Thi

Tho

Tco

Tci

0 A
Area

Figure9.9: Mean temperature


difference in Parallel flow

dQ = ─ mhCph = − Ch dTh ……………………………..(9.12 a)

402
The heat transfer across the surface area dA can be expressed in terms of the overall heat transfer
coefficient as:
dQ = UdA dT ……………………………….(9.12)

where dT = Th – Tc is the local temperature difference between the hot and cold fluids.

From Equation(9.11) we have dTh = − dQ / Ch and dTc = dQ / Cc.


1 1
Hence dTh – dTc = − U{ 𝐶ℎ + 𝐶𝑐
} dA (Th – Tc)

𝑑(𝑇ℎ−𝑇𝑐) 1 1
Or (𝑇ℎ−𝑇𝑐)
= − U{ 𝐶ℎ + 𝐶𝑐
} dA ………………………………….(9.13)

Integrating Equation (9.13) between A=0 and A we have

(𝑇ℎ𝑜−𝑇𝑐𝑜) 𝑑(𝑇ℎ−𝑇𝑐) 𝐴 1 1
∫(𝑇ℎ𝑖−𝑇𝑐𝑖) (𝑇ℎ−𝑇𝑐)
= ∫0 − U{ 𝐶ℎ + 𝐶𝑐
} dA …………………………. (9.14)

(𝑇ℎ𝑜−𝑇𝑐𝑜) 1 1
Or ln (𝑇ℎ𝑖−𝑇𝑐𝑖)
= −𝑈 𝐴 {𝐶ℎ + 𝐶𝑐
}. ……………………………… (9.15)

1 (𝑇𝑐𝑜−𝑇𝑐𝑖)
Now Q = Cc (Tco – Tci) or = ------------------------------- (9.16a)
𝐶𝑐 𝑄

1 (𝑇ℎ𝑖−𝑇ℎ𝑜)
Similarly 𝐶ℎ
= 𝑄
------------------------------- (9.16b)

Substituting these expressions in Equation(9.15) and rearranging we get

(𝑇ℎ𝑜−𝑇𝑐𝑜) 𝑈𝐴
ln = − {(𝑇ℎ𝑖 − 𝑇ℎ𝑜) − (𝑇𝑐𝑜 − 𝑇𝑐𝑖)}
(𝑇ℎ𝑖−𝑇𝑐𝑖) 𝑄

Or Q = UA ∆Tm ………………………(9.17)

(∆𝑇𝑜− ∆𝑇𝑖)
Where ∆Tm = ln (∆𝑇𝑜 / ∆𝑇𝑖) …………………………(9.18a)

and ∆To = Tho – Tco and ∆Ti = Thi – Tci ……………………………..(9.18b)

It can be seen from Equation(9.18a) that ∆Tm is logarithmic mean of ∆𝑇𝑜 𝑎𝑛𝑑 ∆𝑇𝑖 and therefore
it is normally referred to as “Logarithmic Mean Temperature Difference (LMTD in short”.

9.4.2.Mean temperature difference for a counterl flow heat exchanger:

403
Figure9.9: Mean temperature
difference in Counter flow

For a counterflow heat exchanger, the energy balance equations for an elemental area dA is given
by

dQ = − Ch dTh = ─ Cc dTc ………………………………..(9.20)

The heat transfer across the surface area dA can be expressed in terms of the overall heat transfer
coefficient as:
dQ = UdA dT ……………………………….(9.12)

where dT = Th – Tc is the local temperature difference between the hot and cold fluids.

From Equation(9.11) we have dTh = − dQ / Ch and dTc = ─ dQ / Cc.

Hence dTh – dTc = − U {1/Ch ─ 1/Cc} dA (Th – Tc)

Or d(Th – Tc) / (Th – Tc) = ─ U{1/Ch ─ 1/Cc} dA

Integrating Equation (9.13) between A=0 and A we have

ln {(Tho ─ Tci) / (Thi ─ Tco)} = ─UA{1/Ch ─ 1/Cc}………………………………...(9.21)

404
Substituting for 1/Ch and 1/Cc from Equation(9.16a) and (9.16b) and rearranging we get

ln{(Tho ─ Tci) / (Thi ─ Tco)} = ─ UA {(Thi ─ Tho) / Q ─ (Tco ─ Tci) / Q)

Or Q = UA {( Tho ─ Tci) ─ {(Thi ─ Tco)} / ln{(Tho ─ Tci) / (Thi ─ Tco)} ………………...(9.22)

Or Q = UA ∆Tm

(∆𝑇𝑜− ∆𝑇𝑖)
Where ∆Tm = ln …………………………(9.23a)
(∆𝑇𝑜 / ∆𝑇𝑖)

and ∆To = Tho – Tci and ∆Ti = Thi – Tco ……………………………..(9.23b)

The expression for ∆Tm for counter flow [Equation(9.23)] is identical with that for parallel flow
except that the the symbols ∆Ti and ∆To represent different quantities in each case.

9.4.3.Mean temperature difference for special cases:

Case (i): When one of the fluids is undergoing a change of phase in the heat exchanger:

This situation may arise in the case of a boiler or a condenser. In the boiler the cold fluid will be
undergoing the change of phase at constant pressure so that Tco = Tci as shown in Fig 9.10(a). In
this case the mean temperature difference is same both for parallel flow and counter flow
arrangements.Similarly in a condenser the hot fluid will be undergoing a change of phase so that
Tho = Thi as shown in Fig 9.10(b) and hence in this case also the mean temperature difference is
same both for parallel flow and counter flow arrangements.

405
Figure9.10(a): Mean temperature difference when Figure9.10(b) Meantemperature difference
when
cold fluid is undergoing a change of phase hot fluid is undergoing a change of phase

Case (ii ): When ∆Ti = ∆To: This situation arises only for counterflow arrangement. When ∆Ti =
∆To , Equation (9.23a) gives

∆Tm = 0 / 0 (Indeterminate).

Hence ∆Tm has to be obtained using L’Hospital’s rule.

Let ∆Ti / ∆To = p. Then ∆Tm = Lt {∆To (p – 1) /ln p}


p→1

Differentiating the numerator and denominator with respect to ‘p’ we get

∆Tm = Lt {∆To ( 1) / (1/p)} = ∆To


p→1

Thus the mean temperature difference is equal to the difference in temperature between the hot and
cold side at either end of the heat exchanger. It can be shown that in this case both T h and Tc vary
linearly with the same slope.Consequently the value of ∆T is the same all along the length of the
heat exchanger.

9.4.4.Mean temperature difference for a cross flow heat exchanger: In both parallel flow and
counter flow arrangements, the temperature on both sides of the wall separating the two fluids

406
vary along the length of the heat exchanger,x and therefore are functions of a single variable.
That is both Th and Tc are functions of x only. This is not the case for cross flow heat exchanger.
Case(i): consider a single pass cross flow situation in which both fluids are unmixed (see
Figure9.6). Here the area A across which the heat transfer takes place is in the plane of the
paper with two fluids flowing on either side, one in the x-direction and the other in the y
direction. It is obvious that both Th and Tc are functions of x and y and the exit profile
temperatures are not uniform.
Consider an elemental surface area dA (dA = dx dy). The rate of heat transfer across this area
is given by
dQ = U(Th ─ Tc)dA = U(Th ─ Tc) dx dy ………………..(9.24)

If U is assumed to be constant then

Q = ∫dQ = U ∫ ∫ (Th ─ Tc) dx dy ………………………(9.25)

Comparing Equation (9.25) with Equation(9.9) we have

ΔTm = (1 / A) ∫ ∫ (Th ─ Tc) dx dy …………………….(9.26)

The determination of ΔTm therefore involves double integration and is rather complicated.

Case(ii): If mixing occurs on one side or on both sides the situation will be different from that
in case(i).By mixing we mean a secondary flow transverse to the flow direction takes place
and this permits a fluid to mix with itself and consequently evens out the temperature of the
fluid in the transverse direction. Therefore if the cold fluid is mixed and the hot fluid is unmixed
and if we assume the cold fluid is flowing in x direction and hot fluid is flowing in y direction
then Tc = f(x) and Th = f(x,y).

Case(iii): If both the fluids are mixed then Tc = f(x) and Th = f(y).

The computation for the mean temperature difference for the three cases of cross flow
have ben done by Bowman, Mueller and Nagle.They have represented their results in the form
of a correction factor,F which is defined as follows:

Mean temperature difference in cross flow


F = ------------------------------------------------------------------------------
Mean temperature difference if the arrangement is counter flow

ΔTm
i.e. F = --------------------------- ………………………………………………(9.27)
(ΔTm)counter flow

Thus the rate of heat transfer for a cross flow heat exchanger can be written as :

Q = UAF (ΔTm)counter flow ……………………………………(9.28)

407
F is expressed as a function of two parameters R and P where R and P are given by the
following expressions:

(T1 ─ T2) (t2 ─ t1)


R = --------------- and P = ----------------
(t2 ─ t1) (T1 ─ t1)

The parameter R is the ratio of the change in temperatures of the two fluids which is also equal
to the ratio of the heat capacities of the two fluids. P is the ratio of the change in temperature
of one of the fluids to the difference of inlet temperatures of the two fluids.By definition it
follows that R ≥ 0 and 0 ≤ P ≤ 1. Foa cases (i) and (iii) it is immaterial whether T stands for
temperature for hot fluid or cold fluid , but for case (ii) where one fluid is mixed and the other
unmixed,T stands for the temperature of the mixed fluid whether it is hot fluid or cold fluid
and t stands for the temperature of the unmixed fluid

Example 9.1: Water at 25 0 C and a velocity of 1.5 m/s enters a brass condenser tube 6 m long, 1.34
cm ID, 1.58 cm OD and k = 110 W/(m-K). Steam is condensing on the outer surface of the tube with a
heat transfer coefficient of 12,000 W/(m2 – K). The fouling factors for the inner and outer surfaces are
both equal to 0.00018 (m2 – K) / W. Calculate the overall heat transfer coefficient based on (i) the
inside surface area and (ii) the outside surface area.

Solution:

408
ho= 12000 W/m2k
Fo= 0.00018 m2-k/W

water hi , Fi = 0.00018 m2-k/W do


di
Tfi = 25oC
Uav = 1.5 m

k=110 W/m-k
di = 0.0134m
do= 0.0158m

To find : (i) Ui ; (ii) Uo

(i) Overall heat transfer coefficient based on inside surface area is given by

1
Ui = -----------------------------------------------------------------------------------
(1/hi )+Fi +[di/2k]loge(do/di)+(di/do)Fo + (di/do)(1/ho)

To find hi: Properties of water at 250C are:

k = 0.6805 W/(m-K) ; ν = 0.945x 10 − 6 m 2/s ; Pr = 6.22

Red = Uavdi/ ν = 1.5x0.0134/0.945x10 − 6 = 21270

Since Red > 2300, flow is turbulent.Assuming that the flow is hydrodynamically and
thermally developed we have

Nud = hidi/k = 0.023(Red)0.8 x (Pr)0.4 = 138.5

hi = 138.5 x 0.6085 / 0.0134 = 6289.5 W/(m2-K)

1
Ui = ------------------------------------------------------------------------------------------------------

409
(1/6289.5)+0.00018+[0.0134/2x110]loge(0.0158/0.0134)+(0.0134/0.0158)x0.00018

Ui = 1747 W/(m2-K)

(ii) Ui Ai = Uo Ao

Hence Ui π di L = Uo π do L

Or Uo = Ui (di/do)

= 1747 x (0.0134/0.0158)

= 1481.6 W/(m2-K)

Example 9.2. In a heat exchanger hot fluid enters at 60 0 C and leaves at 48 0 C, where as the cold
fluid enters at 35 0 C and leaves at 44 0 C. Calculate the mean temperature difference for (a) parallel
flow, (b) counter flow, (c) single pass cross flow (both fluids unmixed), (d) single pass cross flow
(hot side fluid mixed, cold side fluid unmixed), and (e) single pass cross flow (hot side fluid unmixed,
cold side fluid mixed).

Given: Thi = 60 0C ; Tho = 480C ; Tci = 350C ; Tco = 440C

To find: (a) MTD for parallel flow ; (b) MTD for counter flow; (c) MTD for single pass cross flow
with both fluids mixed; (d) single path cross flow with hot fluid mixed and cold fluid unmixed.

(a) MTD for parallel flow:

ΔTo = Tho - Tco


ΔTi = Thi - Tci

Tco

ΔTi = Thi - Tci = 60 – 35 = 250C ;

ΔTo = Tho - Tco = 48 – 44 = 40C

410
(ΔTi ─ ΔTo) (25 ─ 4)
For parallel flow, MTD = ------------------- = ---------------- = 11.5 0C
ln(ΔTi / ΔTo) ln (25/4)

(b) MTD for counter flow:

∆Ti = Thi - Tco = 60 – 44 = 16o C

∆To = Tho - Tci = 48 – 35 = 13o C

(ΔTi ─ ΔTo) (16 – 13)


Hence MTD = ----------------- = ---------------- = 14.45 o C
ln(ΔTi / ΔTo) loge (16/13)

(c) MTD for cross flow with both fluids mixed: MTD = F (MTD)counter flow.

The correction factor F is obtained from the chart as follows:


Since both the fluids are mixed, it is immaterial whether T stands for hot fluid or cold fluid.We take
that T stands for hot fluid and t stands for cold fluid. Then T1 = Thi, T2 = Tho, t1 = Tci and t2 = Tco.

T1

t1 t2

T2
(T1 ─ T2) (60 ─ 48) (t2 - t1) (44 – 35)
Therefore R = ------------- = --------------- = 1.33 ; P = ---------- = -------------- = 0.36
(t2 ─ t1) (44 ─ 35) (T1 – t1) (60 – 35)

From chart , F = 0.90.

Hence MTD = 0.90 x 14.45 = 13.010C

(d) MTD for cross flow with cold fluid mixed and hot fluid unmixed:

Since cold fluid is mixed, T stands for temperature of cold fluid and t stands for temperature of hot
fluid. Therefore T1 = Tci = 35 0C , T2 = Tco = 44 0C, t1 = Thi = 60 0C and t2 = Tho = 48 0C.

(35 ─ 44) (48 ─ 35)


Therfore R = ---------------- = 0.75 ; P = --------------- = 0.52
(48 ─60) (35 – 60)

From chart F = 0.92. Therefore MTD = 0.92 x 14.45 = 13.3 0C

411
Example 9.3: A simple heat exchanger consisting of two concentric flow passages is used for heating
1110 kg/h of oil (Cp = 2.1 kJ/kg-K) from a temperature of 27 0 C to 49 0C.The oil flows through the
inner pipe made of copper (k = 350 W/m-K) with2.54 cm ID and 2.86 cm OD. The surface heat
transfer coefficient on the oil side is 635 W/m2 –K. The oil is heated by water supplied at a rate of 390
kg/h and at an inlet temperature of 93 0 C. The water side heat transfer coefficient is 1270 W/m2-K.
the fouling factors o the oil side and water side are 0.0001 and 0.0004 m2-K/W respectively. What is
the length of the heat exchanger required for (i) parallel flow, (ii) counter flow arrangement.

Given : di = 0.0254m ; do= 0.0286m ; k = 350 W/(m – K) ;

Cold fluid : oil mc = (1110 /3600) = 0.3055 kg/s; Cpc = 2100 J/(kg-K) ; Tci = 27 0C ;

Tco = 49 0C; hi = 635 W / (m2 – K); Fi = 0.0001 m2-K/W;

Hot fluid : water ; mh = 390/3600 = 0.1083 kg/s; Cph = 4200 J/kg-K ;Thi = 93oC ;

ho = 1270 W/ (m2-K) ; Fo = 0.0004 (m2-K) / W

ho, Fo
water

oil hi , Fi do
di

To find : To find L for (i) parallel flow (ii) counter flow:

Solution: Overall heat transfer coefficient based on outside area of the inner tube is given by
1
Uo = ----------------------------------------------------------------------------------------------------
(do/di) (1/hi) + (do/di) Fi + (do/2k) loge(do/di) + Fo + (1/ho)

1
= ----------------------------------------------------------------------------------------------------- -----------------------
(0.0286/0.0254)(1/635)+(0.0286/0.0254)0.0001+(0.0286/700)loge(0.0286/0.0254)+0.0004+ (1/1270)

= 325 W/(m2-K)

412
Heat balance equation can be written as

Q = mc x Cpc (Tco – Tci) = 0.3083 x 2100 ( 49 – 27 ) = 14243.5 W

Also Q = mh Cph (Thi – Tho)

Or Tho = Thi – Q/mh Cph = 93 – [14243.5/(0.1083x4200)] = 61.7oC

(i) Parallel flow arrangement :

∆Ti = Thi – Tci = 93 – 27 = 66oC

∆To = Tho – Tco = 61.7 – 49 = 12.7oC

hence MTD = ∆Tm = ∆Ti - ∆To / loge (66/12.7) = 32.34 o C

Q = Uo π do L π ∆Tm

Or L = Q / (Uo π do∆Tm) = 14243.5/(325 x π x 0.0286 x 32.34) = 15.1 m.

(ii) Counter flow arrangement : In this case ∆Ti = Thi – Tco = 93 – 49 = 44 0C.

∆To = Tho – Tci = 61.7 – 27 = 34.7 0C.

∆Ti – ∆To (44 – 34.7)


Hence ∆Tm = ------------------- = -------------------- = 39.20C.
ln (∆Ti / ∆To) ln (44 / 34.7)

Hence L = Q / (Uo π do∆Tm) = 14243.5 / (325 x π x 0.0286 x 39.2) = 12.5 m

9.5.Effectiveness – NTU method:

9.5.1. Limitations of Mean Temperature Difference method:- There are generally two types of
problems encountered with heat exchangers. The first type (known as the design type) is the design of
heat exchanger for a given heat load i.e.,for the two fluids, the flow rates, the inlet and exit temperatures
are specified and the heat transfer area required for the heat exchanger is to be calculated. This type of
problem has been solved in Example 9.3 using the mean temperature difference method. In the second
type of problem(known as the rating problem of heat exchanger), for a given heat exchanger and the
two given fluids , only the flow rates and the inlet tempetratures of the two fluids are specified and the
exit temperatures of the two fluids have to be determined.In the mean temperature difference
method,for this type of problem, the exit temperatures of the two fluids have to be determined by
solving two simulataneous equations for both parallel flow and counter flow arrangements, where as
for cross flow and multi pass arrangements the exit temperatures have to be determined using a trial
and error method as illustrated below:

Case (i):Solution for Parallel Flow arrangement: From Equation (9.15) we have

Tho ─ Tco = exp {─ UA (1 / Ch + 1 / Cc)}……………………………9.29(a)

413
From heat balance equation we have

(Ch/Cc)Tho + Tco = (Ch/Cc)Thi + Tci ………………………………….9.29(b)

The RHS of both the equations 9.29(a) and 9.29(b) can bedetermined from the given information and
hence these two rquations can be solved for Tho and Tco

Case(ii) Solution for counter Flow arrangement:

Equation(9.22) can be written as follows:

(Tho ─ Tci)
-------------- = exp {─ UA [(1 / Ch) ─ (1 / Cc)]
(Thi ─ Tco)

or

Tho + exp {─ UA [(1 / Ch) ─ (1 / Cc)] Tco = exp {─ UA [(1 / Ch) ─ (1 / Cc)] Thi + Tci ………9.29(c)

Equation9.29(b) holds good for counter flow arrangement also. Hence Equation9.29(b) and 9.29(c) can
be solved simultaneously to get Tho and Tco.

Case (iii): Solution for crossflow and multi pass arrangements: Unfortunately the solution for these
type of heat exchangers are not as straight forward as the above two types ,as the mean temperature
difference involves the correction factor, F which requires the knowledge of the exit temperatures.
Hence the problem has to be solved by trial and error method[ ] as shown below.

Calculate R ,assume a value for F and read the value of P from the chart for the particular
arrangement.Knowing P, calculate Tho, Tco and Q . Then calculate ΔTm for counterflow arrangement
and check for theassumed value of F by calculating F from the equation

Q
F = -------------------------
UA (ΔTm)counter flow

The procedure is repeated until the solution converges. The method is illustrated for all the three cases
mentioned above with the following example.

Example 9.4. In an oil to water heat exchanger, the oil[Cp = 2000 J/(kg-k)] enters the exchanger at
1000C with a heat capacity rate of 3700 W/K. Water(Cp= 4190 J/kg-K) enters the exchanger at 150C
with a mass flow rate of 0.6 kg/s.Determine the exit temperatures of the two fluids for (i) parallel flow
arrangement, (ii) counterflow arrangement and (iii)on shell pass two tube pass arrangement.Assume
the total surface area of the heat exchanger is 10 m2 and the overall heat transfer coefficient is 500
W/(m2-K)

Given: Hot fluid is oil with: Thi = 100 0C; Ch = 3700 W / K;

Cold Fluid is water with : Tci = 150C ; mc = 0.6 kg/s ; Cpc = 4190 J/(kg-K)

U = 500 W/(m2 – K) ; A = 10 m2

414
Solution: (a) Parallel flow arrangement: Cc = mc Cpc = 0.6 x 4190 = 2514 W/K

From Equation9.29(a) we have Tho ─ Tco = (100 – 15) exp {─ 500 x 10 (1 / 3700 + 1 / 2514)}

Or Tho ─ Tco = 3.01 0C ………………………………………….(i)

From Equation9.29(b) we have (3700/2514) Tho + Tco = (3700/2514) x100 + 15

Or 1.47Tho + Tco = 162.20 ……………………………………….(ii)

Solving for Tho and Tco from Equation(i) and Equation(ii) we get

Tho = 66.89 0C , Tco = 63.88 0C and Q = 3700 x (100 ─ 66.89) = 122,507 W

(b) Counter flow arrangement:

From Equation9.29 (c) we have

Tho + exp {─ 500x10 [(1 / 3700) ─ (1 / 2514)} Tco = exp {─ 500x10 [(1 / 3700) ─ (1 / 2514)}x 100

+ 15

Or Tho + 1.892 Tco = 204.2 0C ………………………………………………………..(iii)

Solving for Tho and Tco from (ii) and (iii) we get Tho = 57.6 0C ,Tco = 77.470 C and

Q = 2514 x (77.47 ─ 15) = 157,050 W

(c) One shell pass two tube pass arrangement:

Trial 1: Assume F = 0.8 (This generally is a mimum value for good design)

T1 – T2 Tco ─ Tci Ch 3771


R = -------- = --------------- = -------- = ------------ = 1.50
t2 ─ t1 Thi ─ Tho Cc 2514

From chart, corresponding to F = 0.8 and R = 1.50, P = 0.40

Hence ( t2 – t1) / (T1 – t1) = 0.40

t2 = t1 + 0.40(T1 – t1) = 100 + 0.40 x (15 – 100)

Or t2 = 66.0 0C = Tho

Q = 3771 x (100 – 66.0) = 128,214 W

Hence Tco = Tci + Q / Cc = 15 + 128214 / 2514 = 66.00 0C

415
(100 – 66.0) – (66 – 15)
(ΔTm)counter flow = -------------------------------------------------- = 41.93 0C
ln {(100 – 66.0) / (66 – 15)}

Q 128214
Hence F = ------------------------ = --------------------- = 0.612.
U A (ΔTm)counter flow 500 x 10 x 41.93

The calculated value of F is about 31 % lower than the assumed value. Hence further iteration has to
be carried out. The Table 2 shows the difference between the assumed value of F and calculated
value in % for first 16 iterations.

Table 2: Iterative solution for 1-2 heat exchanger of Example 9.4

Trial F P Tho Tco F (calculated) ΔF in %


(Assumed)
1 0.80 0.4052 65.56 65.69 0.6080 + 31.58
2 0.6080 0.4513 61.64 71.46 0.7701 ─ 21.05
3 0.7701 0.4163 64.61 67.09 0.6435 + 19.67
4 0.6435 0.4456 62.13 70.75 0.7477 ─ 13.94
5 0.7477 0.4234 64.01 67.98 0.6672 + 12.07
6 0.6672 0.4417 62.46 70.26 0.7328 ─ 8.95
7 0.7328 0.4273 63.68 68.46 0.6806 + 7.67
8 0.6806 0.4394 62.65 69.98 0.7243 ─ 6.03
9 0.7243 0.4291 63.53 68.68 0.6868 + 5.46
10 0.6868 0.4375 62.81 69.74 0.7173 ─ 4.25
11 0.7173 0.4314 63.33 68.98 0.6949 + 3.12
12 0.6949 0.4363 62.92 69.58 0.7125 ─ 2.47
13 0.7125 0.4322 63.26 69.08 0.6979 + 2.09
14 0.6979 0.4359 62.95 69.54 0.7113 ─ 1.88
15 0.7113 0.4329 63.20 69.17 0.7005 + 1.54
16 0.7005 0.4355 62.98 69.49 0.7099 ─ 1.32

It can be seen from the above table that the convergence of the iterations is slow and F continues to
fluctuate, but it will converge. This rating problem can be guaranteed convergent using the Newton-
Raphson method (i.e. the solution of f(x) = 0 where function f is non linear in the present case).

The above example illustrates the limitations of the mean temperature difference method for solving
the rating problem of a given heat exchanger. Though the method is straight forward for parallel flow
and counterflow arrangement, but it requires solving two simultaneous equations to find Tho and Tco..For
crossflow and multi pass type heat exchangers the method involves a trial and error solution wherein
the solution converges very slowly iterating on F.This problem can be solved relatively straight forward
by effectiveness-NTU method.

9.5.2 Definitions of Terms Used in Effectiveness – NTU Method:

416
(i) Capacity ratio,C: It is defined as the ratio of minimum heat capacity to the maximum capacity. i.e.

Cmin
C = ---------- …………………………………………………………….(9.30)
Cmax

Cc (Thi – Tho)
If Cc < Ch then C = ------------- = -------------------- ……………………………………..(9.31a)
Ch (Tco – Tci)

Ch (Tco – Tci)
If Cc > Ch then C = ------------- = -------------------- ……………………………………..(9.31a)
Cc (Thi – Tho)
(ii) Number of transfer units, NTU:
UA
NTU = --------------- …………………………………..(9.32)
Cmin

UA
If Cc < Ch, then NTU = ----------------- …………………………………(9.33a)
Cc

UA
If Cc > Ch, then NTU = ----------------- …………………………………(9.33b)
Ch

(iii) Effectiveness, ε: This defined as the ratio of the actual rate of heat transfer in the exchanger to
the maximumpossible rate of heat transfer.

Exercise Problems:

A. Overall heat transfer coefficient:

9.1. A stainless steel tube [k = 45 W/(m-K)] of inner and outer diameters of 22 mm and
27 mm respectively, is used in a cross flow heat exchanger (see Figure P 9.2). The
fouling factors for the inner and outer surfaces are estimated to be 0.0004 and
0.0002 (m2–K) /W respectively

417
Water at
75 0C & Di = 22 mm D0 = 27 mm
0.5 m/s

Air at 15 0 C & 20 m/s

Figure P 9.2 : Schematic for problem 9.2

Determine the overall heat transfer coefficient based on the outside surface area
of the tube. Compare the thermal resistances due to convection, tube-wall
conduction and fouling and make comments.

B. Mean Temperature Difference Method:

9.2. A one-shell pass, two-tube pass exchanger is to be designed to heat 0.5 kg/s of
water entering the shell side at 10 0 C. The hot fluid oil enters the tube at 80 0 C
with a mass flow rate of 0.3 kg/s and leaves the exchanger at 30 0 C. The overall
heat transfer coefficient is 250 W/m2-K. Assuming the specific heat of oil to be 2
kJ/kg-K, calculate the surface area of the heat exchanger required.

9.3. A single pass cross flow heat exchanger uses hot gases (mixed) to heat water
(unmixed) from 30 0 C to 80 0 C at a rate of 3 kg/s.The exhaust gases, having
thermo-physical properties similar to air enter and leave the exchanger at 225 and
100 0 C respectively. If the overall heat transfer coefficient is 200 W/m 2-K,
determine the required surface area of the exchanger.

9.4. A two-shell pass, four-tube pass heat exchanger is used to heat water with oil.
Water enters the tubes at a flow rate of 2 kg/s and at 20 0 C and leaves at 80 0 C.
Oil enters the shell side at 140 0 C and leaves at 90 0 C. If the overall heat transfer
coefficient is 300 W/m2-K, calculate the heat transfer area required.

9.5. A shell and tube heat exchanger is to be designed for heating water from 25 0 C
to 50 0 C with the help of steam condensing at atmospheric pressure. The water
flows through the tubes (2.5 cm ID, 2.9 cm OD and 2 m long) and the steam
condenses on the outside of the tubes. Calculate the number of tubes required if
the water flow rate is 500 kg/min and the individual heat transfer coefficients on

418
the steam and water side are 8000 and 3000 W/ m2-K respectively. Neglect all
other resistances.

C. Effectiveness – NTU method:

9.6. Show that for counter flow heat exchanger with capacity ratio C = 1, the
effectiveness is given by

ε = NTU / (1 + NTU)

9.7. The following data refer to a heat exchanger.


Mass flow rate of the hot fluid = 4 kg/min.
Mass flow rate of the cold fluid = 8 kg/min.
Specific heat of hot fluid = 4.20 kJ/kg-K.
Specific heat of the cold fluid = 2.52 kJ/kg-K.
Inlet temperature of hot fluid = 100 0 C.
Inlet temperature of cold fluid = 20 0 C.

What is the maximum possible effectiveness if the arrangement is (i) parallel flow
and (ii) counter flow?

9.8. Calculate the exit temperature of the hot fluid and inlet temperature of the cold
fluid for a counter flow heat exchanger having the following specifications.
Mass flow rate of hot fluid = 3 kg/s.
Mass flow rate of cold fluid = 0.75 kg/s.
Cp for hot fluid = 1.05 kJ/kg-K.
Cp for cold fluid = 4.2 kJ/kg-K.
Inlet temperature of hot fluid = 500 0 C.
Exit temperature of cold fluid = 85 0 C.
Overall heat transfer coefficient = 450 W/m2-K.
Total surface area of the exchanger = 1 m2.

9.9. In a gas turbine power plant, heat is transferred in an exchanger from the hot
gases leaving the turbine to the air leaving the compressor. The air flow rate is
5000 kg/h and the fuel-air ratio is 0.015 kg/kg. The inlet temperatures on the air
side and the gas side are 170 0 C and 450 0 C respectively. The overall heat transfer
coefficient for the exchanger is 52 W/m 2-K and the surface area of the exchanger
is 50 m 2. If the arrangement is cross flow with both fluids unmixed determine the
exit temperatures of both the fluids and the rate of heat transfer. Take the specific
heats of both the fluids as 1.05 kJ/kg-K.

9.10. A concentric-tube heat exchanger operates on the counter flow mode. The fluid
flowing in the annular space enters the exchanger at 20 0 and leaves at 70 0 C. The
fluid flowing through the inner tube enters at 110 0 C and leaves at 65 0 C. The
length of the exchanger is 30 m. It is desired to increase the outlet temperature of
the cold fluid to 80 0 C by increasing only the length while maintaining the same
mass flow rates, inlet temperatures and tube diameters. Make any justifiable
assumption and calculate the new length.

419
9.11. It is proposed to cool 1000 kg/h of oil from150 0 C to 50 0 C in a heat exchanger
using 1667 kg/h of water at an inlet temperature of 30 0 C. Calculate the surface
required assuming a single pass cross flow arrangement in which the oil is mixed
and the water unmixed. Assume Cp for oil to be 2.087 kJ/kg-K and the overall heat
transfer coefficient to be 550 W/m2-K. Solve the problem by the mean temperature
difference method as well as by the ε – NTU method.

9.12. Two identical counter flow heat exchangers are available. Water at the rate of 5000
kg/h and at 30 0 C is to be heated by cooling an oil (Cp = 2.1 kJ/kg-K) at 90 0 C.
The oil flow rate is 2000 kg/h. The heat transfer area is 3 m 2. From the point of
view of maximizing the heat transfer rate, which of the following is the best
arrangement?
(a) Both the fluids flow in series.
(b)The oil flow is split up equally between the two exchangers, while the water flows
in series.
(c) Both oil and water flows are split up equally in both the exchangers.

9.13. A counter flow double pipe heat exchanger is used to heat 1.25 kg/s of water from
35 0 C to 80 0 C by cooling an oil (Cp = 2.0 kJ/kg-K) from 150 0 C to 85 0 C. The
overall heat transfer coefficient is 850 W/m2-K. A similar arrangement is to be built
at another location, but it is desired to compare the performance of the single
counter flow heat exchanger with two smaller counter flow heat exchangers
connected in series on the water side and in parallel on the oil side as shown in
Figure P 9.16. The oil flow is split equally between the two exchangers and it may
be assumed that the overall heat transfer coefficient for the smaller exchangers is
the same as for the large exchanger. If the smaller exchanger costs 20 % more
per unit surface area, which would be the most economical arrangement – the one
large exchanger or the two equal-sized small exchangers?

A.Overall heat transfer coefficient:

9.1 Solution:

420
ho
Fo= 0.0002 m2-k/W
Di = 0.022m
Do= 0.027m

water hi , Fi = 0.0004 m2-k/W Di Do


Q
Tfi = 75oC
Uav = 0.5 m

k=45 W/m-k

Air at T∞ = 15oC
Uav = 20m/s

To find hi: Properties of water at 750C are:

k = 0.6715 W/(m-K) ; ν = 0.39x 10 − 6 m 2/s ; Pr = 2.38

Red = Uavdi/ ν = 0.5x0.022/0.39 x 10 − 6 = 28205

Since Red > 2300, flow is turbulent.

Nud = hidi/k = 0.023(Red)0.8 x (Pr)0.4 = 118.2

hi = 118.2 x 0.6715/ 0.022 = 3608 W/(m2-K)

Rci = 1/(hiAi) = 1/ hi π di L = 1/ ( π x 0.022 x 1 x 3608)

= 4.01 x 10− 3 k/W

To find ho:

Air is flowing across the cylinder.

Properties of air at 150C are :

421
k = 0.0255 W/(m-K) ; ν = 14.61x 10 − 6 m 2/s ; Pr = 0.704

Red = U∞do/ ν = 20 x 0.027/14.61 x 10 − 6 = 36961

From data hand book,

Nud = hodo/k = [ 0.4 Red 0.5 + 0.06 Red 0.6667


] x Pr0.4 x ( µ ∞/µ w )0.25

For gases, ( µ ∞/µ w ) = 1

Hence hodo/k = [ 0.4 x (36961)0.5 + 0.06 (39961) 0.6667


] x 0.7040.4 = 130.4

ho = 130.4 x 0.0255/0.027 = 123 W/(m2-K)

Rco = 1/(h0A0) = 1/ h0 π d0 L = 1/ ( π x 0.027 x 1 x 123)

= 0.096 k/W

Rfi = Fi / Ai = Fi / π di L = 0.004 / (π x 0.022 x 1) = 0.0579 k/W

R = ( 1/ 2 π L k ) x log e (do/di ) = ( 1 / 2 x π x 1 x 45 ) log e (0.027/0.022)

= 7.24 x 10-3 k/W

Rfo = Fo / Ao = Fo /( π do L) = 0.0002 / (π x 0.027 x 1)= 2.36 x 10-3 k/W

Total thermal resistance = ∑R = 4.01 x 10-3 + 0.0579 + 7.24 x 10-3 + 2.36 x 10-3
+ 0.096

= 0.1675 k/W

If Uo is the overall heat transfer coefficient based on outside area then,

Uo Ao = 1/ ∑R

Uo = 1 / Ao ∑R

= 1 / ( π x 0.027 x 0.1675)

= 70.4 W/(m2-K)

Comparison between various resistances:

Rci Thermal resistance for 4 x 10-3


convection at the inside
surface.

422
Rfi Resistance due to fouling at 0.0579
the inside surface
R Resistance of the tube wall 7.24 x 10-3
for conduction
Rfo Resistance due to fouling at 2.36 x 10-3
the outside surface
Rco Thermal resistance for 96 x 10-3
convection at the outside
surface

The comparison shows that the thermal resistance for convection heat transfer from the outer
surface of the tube due to air is very large compared to the other resistances i.e,

∑R ≈ Rco

9.2 Solution:

Shell side fluid : water :

mc = 0.5 kg/s ; Cpc = 4200 J/kg-K ;Tci = 10oC ;

Tube side fluid : Oil :

mh = 0.3 kg/s ; Cph = 2000 J/kg-K ;Thi = 80oC ; Tho = 30oC

U = 250 W/(m2-K)

Heat balance equation is

Q = mh Cph (Thi – Tho) = 0.3 x 2000 x (80 – 30)

= 30000 W
Also Q = mc x Cpc (Tco – Tci)

Or Tco = Tci + Q/ mc Cpc = 10 + 30000/(0.5 x 4200)

= 24.3oC

(∆Tm)c.f = (80 – 24.3) – (30 – 10) / loge [(80 – 24.3) / (30 – 10)]

= 34.35oC

Single shell pass - two tube pass HE :

423
T1 = 10 oC ; T2 = 24.3 oC ; t1 = 80 oC ; t2 = 30 oC

R = T1 – T2 / t2 – t1 = 10 – 24.3 / 30 – 80 = 0.29

P = t2 – t1 / T1 – t1 = 30 – 80 / 10 – 80 = 0.714

From chart , F = 0.875

Hence (∆Tm) = F x (MTD)c.f = 0.875 x 34.85 = 30.5oC

Q = U A (∆Tm)

Or A = Q / U (∆Tm) = 30000/(250 x 30.5) = 3.93 m2

9.3 Solution:

Cross flow HE : Cold water (Unmixed) : mc = 3 kg/s ; Cpc = 4200 J/kg-K ;Tci = 30oC ;
Tco = 80oC ;

Hot gases (Mixed) : Cph = 1005 J/kg-K ;Thi = 225oC ; Tho = 100oC ; U = 200 W/(m2/K)

Q = mc x Cpc (Tco – Tci) = 3 x 4200 (80 – 30 ) = 630000 W

Also

P = t2 – t1 / T1 – t1 = 80 – 30 / 225 – 30 = 0.256

R = T1 – T2 / t2 – t1 = 225 – 100 / 80 – 30 = 2.50

From chart, F = 0.93

(∆Tm)c.f = (225 – 80) – (100 – 30) / loge [(225 – 80) / (100 – 30)] = 54.62oC

(∆Tm) = 0.93 x 54.62 = 50.8oC

Q = U A (∆Tm) or A = Q / A (∆Tm) = 630000 / (200 x 54.62) = 57.7 m2.

9.4 Solution:

Water : mc = 2 kg/s; Cpc = 4200 J/kg-K ; Tci = 20oC ;Tco = 80oC ;

Oil : Thi = 140oC ; Tho = 90oC ; U = 300 W / (m2 – K )

424
Q = mc x Cpc (Tco – Tci) = 2 x 4200 (80 – 20 ) = 504000 W

Tube side fluid is water. Hence t1 = 20oC ; t2 = 80 oC

Shell side fluid is oil. Hence T1 = 140 oC ; T2 = 90 oC

P = t2 – t1 / T1 – t1 = 80 – 20 / 140 – 20 = 0.5

R = T1 – T2 / t2 – t1 = 140 – 90 / 80 – 20 = 0.83

From chart F = 0.97

(∆Tm)c.f = (140 – 80) – (90 – 20) / loge [(140 – 80) / (90 – 20)] = 64.87oC

(∆Tm) = 0.97 x 64.87 = 60.9oC

Q = U A (∆Tm) or A = Q / A (∆Tm) = 504000 / (300 x 62.9) = 26.7 m2.

9.5 Solution:

Shell and Tube HE : Cold fluid : Water : mc = 8.33 kg/s ;Tci = 25oC ; Tco = 50oC ;

Hot fluid : Steam condensing at atmospheric pressure.

Hence Thi = Tho = Tsat at atmospheric pressure = 99.6o C (from steam tables)

hfg = 2257 x 103 J / kg – K

Tube side fluid is water. Hence hi = 3000 W / m2 – K

Shell side fluid is oil. Hence ho = 8000 W / m2 – K

Inside dia of tube = di = 0.025 m

Outside dia of tube = do = 0.029 m

Length of the tube = L = 2 m

Overall heat transfer coefficient based on outside surface area is given by


1
Uo = ----------------------------------------------------------------------------------------------------
(do/di) (1/hi) + (do/2k) loge(do/di) + (1/ho)

425
1
= ------------------------------------------------------------------ = 1954 W/(m2-K)
(0.029 / 0.025) (1/3000) + 0 + ( 1 / 8000 )

Since Thi = Tho , both parallel flow and counter flow arrangement will give the same

value of (∆Tm).

Hence (∆Tm) = (99.6 – 25) – (99.6 – 50 ) / loge [(99.6 – 25) / (99.6 – 50 ) ] = 61.2 oC

Q = mc x Cpc (Tco – Tci) = 8.33 x 4200 (50 – 25 ) = 874650 W

Q = Uo Ao (∆Tm) or Ao = Q / Uo (∆Tm) = 874650 / (1954 x 61.2) = 7.314 m2.

Surface area of each tube = ao = π do L = π x 0.029 x 2 = 0.1822 m2

Hence number of tubes = n = Ao / ao = 7.314 / 0.1822 = 40.14 ≈ 41

B. Effectiveness - NTU method:

9.6 Solution:

For a counter flow HE effectiveness is given by

1 - ℮[ - (1 – c ) NTU ]
ε= ------------------------------
1 - c℮[ - (1 – c ) NTU]

when c = 1 , the above expression gives

1 - ℮0
ε= ------------------ = 0 / 0 = indeterminate.
1 – 1 x ℮0

Hence we have to find ε using L’hospital’s rule.

d/dc [1 - e[ - (1 – c ) NTU ] ]
ε= Ltc--> 1 -------------------------------------
d/dc [1 - ce[ - (1 – c ) NTU] ]

0 – NTU ℮ - (1 – c ) NTU
= Ltc--> 1 ------------------------------------------------------------------
0 – {℮ - (1 – c ) NTU x 1 + c (NTU)℮ - (1 – c ) NTU }

426
0 – NTU NTU
ε = --------------------------- = --------------------
0 – { 1 + NTU } 1 + NTU

9.7 Solution:

mh = 0.067 kg/s ; Cph = 4200 J/kg-K ;Thi = 100oC ;

mc = 0.133 kg/s; Cpc = 2520 J/kg-K ; Tci = 20oC ;

mh Cph = 0.067 x 4200 = 281.4 J / s – K

mc Cpc = 0.133 x 2520 = 335.16 J / s – K

mh Cph 281.4
Since mh Cph < mc Cpc , hence c = -------------- = ------------ = 0.84
mc Cpc 335.16

For a parallel flow HE ,

1 - e[ - (1 + c ) NTU ]
ε= ------------------------------ ………………………………………………….. (1)
1+c

For a given value of c , ε will be max if [d ε/d(NTU)] = 0

From (1),

1 x [ 0 + (1+c)e –(1+c)NTU]
[d ε/d(NTU)] = ------------------------------------- = 0
1+c

Therefore, e –(1+c)NTU = 0 or NTU = ∞

Substituting this condition in eqn (1) we have

1 - ℮[ - (1 + c ) ∞ ] 1–0 1 1
εmax = ---------------------- = --------------- = ----------- = --------------- = 0.5435
1+c 1+c 1+c 1.84

427
For a counter flow HE,

1 - ℮[ - (1 – c ) NTU ]
ε = ------------------------------ ……………………………………… (2)
1 - c℮[ - (1 – c ) NTU]

dε [1 - c℮[ - (1 – c ) NTU]] [(1-c)℮ - (1 – c ) NTU] – [1- ℮[ - (1 – c ) NTU]][c℮- (1 – c ) NTU(1-c)]


------------ = ------------------------------------------------------------------------------------------- =0
d(NTU) [1 - c℮[ - (1 – c ) NTU]]2

Evaluating, we get 1 – c = 0 or c = 1 ; substituting this value of c, we have

ε = 1 [ from (2) ]

9.8 Solution:

Counter flow HE :

mh = 3 kg/s ; Cph = 1050 J/kg-K ;Thi = 500oC ;

mc = 0.75 kg/s; Cpc = 4200 J/kg-K ; Tco = 85oC ;

U = 450 W / (m2- K )

mh Cph = 3 x 1050 = 3150 J / s – K

mc Cpc = 0.75 x 4200 = 3150 J / s – K

mh Cph = mc Cpc Hence Thi – Tho = Tco – Tci and c = 1

UA UA
Also NTU = ------------------ or ---------------------
mc Cpc mh Cph

450 x 1
= -------------- = 0.1428
3150

For a counter flow HE,

428
1 - ℮[ - (1 – c ) NTU ]
ε = ------------------------------
1 - c℮[ - (1 – c ) NTU]

NTU 0.1428
Since c = 1, ε = ---------------- = ------------------- = 0.1250
1 + NTU 1.1428

Thi – Tho Tco – Tci


But ε = --------------- or ------------------ when c = 1
Thi – Tci Thi – Tci
85 - Tci
Hence 0.1250 = -----------------
500 - Tci

Or 62.5 – 0.125 Tci = 85 - Tci

85 – 62.5
Or Tci = ---------------------- = 25.7oC
(1 – 0.125)

Also Thi – Tho = Tco – Tci when c = 1

Hence 500 - Tho = 85 – 25.7

Tho = 440.7oC

9.9 Solution:

Hot gases : mh = mc (1 + 0.015) = 1.015 mc = 1.015 x (5000/3600) = 1.41 kg/s

Thi = 450oC ;

Cold fluid : Air : mc = 1.39 kg/s; Cpc = Cph = 1050 J/kg-K ; Tci = 170oC ;

U = 52 W / (m2- K ) ; A = 50 m2

mh Cph = 1.39 x 1050 = 1459.5 J / s – K

mc Cpc = 1.41 x 1050 = 1480.5 J / s – K

429
mc Cpc 1459.5
Since mh Cph > mc Cpc , hence c = -------------- = ------------ = 0.986
mh Cph 1480.5

UA 52 x 50
NTU = ------------------ = -------------- = 1.78
mc Cpc 1459.5

From chart for cross flow with both fluids unmixed,

ε = 0.6

when mh Cph > mc Cpc , Tco – Tci


ε = ------------------
Thi – Tci

Or Tco = Tci + ε (Thi – Tci )

= 170 + 0.6 x [450 – 170 ]

= 338 oC

Also mc x Cpc (Tco – Tci) = mh x Cph (Thi – Tho)

mc Cpc
Hence Tho = Thi – ------------- x [ Tco – Tci ]
mh Cph

= 450 - 0.986 [ 338 – 170 ] = 284.35 oC

9.10 Solution:

Counter flow HE :

case (i) Thi = 110o C ; Tho = 65o C ; Tci = 20o C ; Tco = 77oC ; L = 30 m ; U1

case (ii) Tci = 20o C ; Tco = 80oC ; Thi = 110o C ; U2 = U1 (assumed)

For case (i) Q1 = mc x Cpc (Tco – Tci) = mc x Cpc ( 70 – 20 )

For case (ii) Q2 = mc x Cpc ( 80 – 20 )

430
Q1 mc x Cpc ( 70 – 20 ) 5
Hence, ------ = -------------------------------- = ------
Q2 mc x Cpc ( 80 – 20 ) 6

Also for case (i) mc x Cpc ( 70 – 20 ) = mh x Cph ( 110 – 65 )

Also for case (ii) mc x Cpc ( 80 – 20 ) = mh x Cph [ ( 110 – (Tho)2 ]

110 – 65 70 – 20
Hence --------------------- = --------------
110 - (Tho)2 80 – 20

hence (Tho)2 = 110 – (6/5) [ 110 – 65 ] = 56 o C

(110 – 70 ) – (65 – 20 )
Hence for case (i) (∆Tm)1 = ----------------------------------------- = 42.45oC
loge [(110 – 70 ) / (65 – 20 )]

(110 – 80 ) – (56 – 20 )
Hence for case (ii) (∆Tm)2 = ----------------------------------------- = 32.9 oC
loge [(110 – 80 ) / (56 – 20 )]

Q1 U1 π d L1 (∆Tm)1 5
hence , --------- = ---------------------- = -------
Q2 U2 π d L2 (∆Tm)2 6

L1 (∆Tm)1 5
Hence --------------------------- = ----------
L2 (∆Tm)2 6

Or L2 = (6/5) x [ (∆Tm)1 / (∆Tm)2 ] x L1

= (6/5) x (42.45/32.90) x 30 = 46.45 m

9.11 Solution:

Solution by MTD method:

431
Cross flow HE with oil mixed and water unmixed;

Hot fluid : oil : mh = 0.278 kg/s ; Cph = 2087 J/kg-K ;Thi = 150oC ; Tho = 50oC

Cold fluid : water : mc = 0.463 kg/s ; Cpc = 4200 J/kg-K ;Tci = 30oC;

U = 550 W / (m2 – K )

Q = mc x Cpc (Tco – Tci) = 0.278 x 2087 [150 – 50 ] = 58019 W

Also Q = mh x Cph (Thi – Tho)

Or Tco = Tci + (Q / mc Cpc)

= 30 + [58019 / (0.463 x 4200 ) ] = 59.8 oC

(∆Tm)c.f = (150 – 59.8) – (50 – 30) / loge [(150 – 59.8) / (50 – 30) ]

= 46.6 oC

R = (150 – 50 ) / (59.8 – 30 ) = 3.35

P = (59.8 – 30 ) / ( 150 – 30 ) = 0.25

From chart F = 0.80

(∆Tm) = 0.8 x 46.6 = 37.3 oC

Q 58019
A = ------------- = --------------- = 2.83 m2
U (∆Tm) 550 x 37.3

Solution by ε – NTU method :

mh Cph = 0.278 x 2087 = 580.2 J / s – K

mc Cpc = 0.463 x 4200 = 1944.6 J / s – K

mh Cph 580.2
Since mh Cph < mc Cpc , hence c = -------------- = ------------ = 0.29
mc Cpc 1944.6
when mh Cph < mc Cpc ,

Thi – Tho 150 – 50

432
ε = ------------------ = ---------------- = 0.83
Thi – Tci 150 – 30

From chart, NTU = 2.8 = UA / (mCp)min = UA / mh Cph

Hence A = 2.8 x 530.2 / 550 = 2.95 m2

9.12. Solution:
Cold: water: mc= 5000/3600 = 1.39 kg/s ; Tci= 30 oC ;Cpc= 4200 J/(kg-K) assumed

Hot : oil : mh = 2000/3600 = 0.555 kg/s ;Cph = 2100 J/(kg-K) ;Thi= 90oC

A= 3m2 ; U = 465 W/(m2-K)

Case (1) Both the fluids flow in series.


The arrangement for this case will be as shown with figure below:

Tco Thi , mhcph


EXCHANGER 2

Tc×
Th×
Tci , mhcph

EXCHANGER 1
Tho
As mcCpc = (5000/3600) x 4200 = 5833.3 J/(s-K)

And mhCph= (2000/3600) x 2100 = 1166.7 J/(s-K)

therefore mcCpc< mhCph

Therefore C= 1166.7/5833.3 = 0.20

433
UA 465*3
NTU    1.196
mhC ph 1166.7
1  exp[(1  C ) NTU ]

1  C exp[(1  C ) NTU ]
1  exp[(1  0.2)1.196] 1  0.3841
 
1  0.2 exp[(1  0.2)1.196] 1  0.2*0.3841
 0.667
Case (2) When oil is split up equally between the two heat exchangers:

In this case mh= 1000/3600 = 0.278 kg/s

Therefore mh Cph = 0.278x2100= 583.8 J/(s-K)

Therefore C = 583.8/5833.3 = 0.10

UA 465*3
NTU    2.4
mhC ph 583.8
1  exp[(1  C ) NTU ]

1  C exp[(1  C ) NTU ]
1  exp[(1  0.1)2.4] 1  0.1153
 
1  0.1exp[(1  0.1)2.4] 1  0.1*0.1153
 0.895

Case (3) Both the oil and water flows are split up equally:

In this case mcCpc = (2500/3600) x 4200 = 2917 J/(s-K)


mhCph= 0.278 x 2100 = 583.8 J/(s-K)

Therefore C= 583.8/2917 = 0.20


UA 465*3
NTU    2.4
mhC ph 583.8
1  exp[(1  C ) NTU ]

1  C exp[ (1  C ) NTU ]
1  exp[(1  0.2)2.4] 1  0.1467
 
1  0.2 exp[ (1  0.2)2.4] 1  0.0293
 0.879

Since ε is the highest in case (2), this arrangement is the best for ” maximizing heat transfer”

434
9.13 Solution:

Oil (mh), Cph= 2000 J/kg-k


EXCHANGER 1 Thi= 150oC

mh/2 mh/2
Tci = 35oC
mc =1.25 kg/s
Tco=80oC

T c×
Thi Th1× Th2×

EXCHANGER 2
Tho=85oC

U1 = U2 = U = 850 W / (m2 – K )

Case (i) single counter flow HE :

Q = mc x Cpc (Tco – Tci) = 1.25 x 4200 [80 – 35 ] = 236250 W

Q = mh x Cph (Thi – Tho)

Q 236250
Hence mh = ------------------------ = ---------------------------- = 1.817 kg/s
Cp [Thi – Tho] 2000 x [150 – 85 ]

mh Cph = 1.817 x 2000 = 3634.6 J / s – K

mc Cpc = 1.25 x 4200 = 5250 J / s – K

mh Cph 3634.6
Since mh Cph < mc Cpc , hence c = -------------- = ------------ = 0.7
mc Cpc 5250

Thi – Tho 150 – 85


ε = ------------------ = ------------------- = 0.565
Thi – Tci 150 – 35

From chart, NTU = 1.0 = UA / (mCp)min = UA / mh Cph

Hence A = 1 x 3634.6 / 850 = 4.276 m2

435
Case (ii) When two smaller heat exchangers are used:

For this case mh = 1.317 / 2 = 0.9085 kg/s

mh Cph = 0.9085 x 2000 = 1817 J / s – K

mc Cpc = 1.25 x 4200 = 5250 J / s – K

Hence c = 1817 / 5250 = 0.35

To find ‘ε’ , we should know the exit temperatures of hot and cold fluids for at least one

HE. Since UA and (mCp)min is the same for both the exchangers NTU should be same

for both the exchangers.

Thi – Th1× Thi – Th2×


Thus ε1 = ------------------ = ε2 = ----------------
Thi – Tci Thi – Tc×

150  Thi 150  Th 2*


Therefore   1   2 ..............................(1)
150  35 150  Tc*

Since the oil flow is the same in each exchanger and the average exit oil temperature must be
85oC …..it follows that
Th1*  Th 2*
 85o C........................................(2)
2
Energy balance on the second heat exchanger gives

5250(80  Tc* )  1817(150  Th 2* )..................................(3)

Equations 1,2 ,3 may be solved for the three unknowns Th1× , Th2× , Tc× . The solutions are as
follows:
Eqn 1can be rearranged after cross multiplying as:

150Th1*  115Th 2*  150Tc*  Th1*Tc*  5250........................(4)


Eqn 2……………… Th1*  Th 2*  170........................................(5)
Eqn 3……………….. Th 2*  2.9Tc*  82.....................................(6)

From eqn 6………………


T *  82
Tc*  h 2  0.345Th 2*  28.3
2.9

436
From eqn 5………………. Th1*  170  Th 2*
Substituting these expressions in Eqn 4 we have:
150(170  Th 2* )  115Th 2*  150(0.345Th 2*  28.3)  (170  Th 2* )(0.345Th 2*  28.3)  5250

Or Th 2*2  706Th 2*  57055  0


706  7062  4*57055
Therefore Th 2   93or 613
*

2
Th2× cannot be 613 oC.

Therefore………. Th 2*  93o C

Therefore……….. Th1*  170  93  77o C

And…………….. Tc*  0.345*93  28.3  60.4o C

150  77
Therefore……………. 1   2   0.635
150  33

Therefore from chart, NTU  1.16

1817 *1.16
Therefore…………….. A1   2.48m 2  A2
850

Therefore total area required to meet the heat load = 2.482×2= 4.92m2

This is more than the 4.276 m2 required in the one larger heat exchanger. In addition the cost
per unit area is greater so that the most economical choice from the heat transfer point of
view would be the single large heat exchanger.

437
CHAPTER: 10

RADIATION HEAT TRANSFER

10.1. INTRODUCTORY CONCEPTS AND DEFINITIONS

10.1.1 THERMAL RADIATION

When a body is placed in an enclosure whose walls are at temperatures below that of
the body, the temperature of the body will decrease even if the enclosure is evacuated. This
process by which heat is transferred from a body by virtue of its temperature, without the aid
of any intersecting medium is called “THERMAL RADIATION”. The actual mechanism of
radiation is not yet completely understood. There are at present two theories by which radiation
phenomenon is explained. According to Maxwell’s electromagnetic theory, Radiation is
treated as electromagnetic waves, while Max Planck’s theory treats radiation as “photons” or
“quanta of energy”. Neither theory completely describes all observed phenomena. It is
however known that radiation travels with the speed of light, c (c = 3x108 m/s) in a vacuum.
This speed is equal to the product of the frequency of the radiation and the wavelength of this
radiation,

OR c = λν ………………………………….(10.1)

Where λ = wavelength of radiation (m) and ν = frequency (1/s).

Usually, it is more convenient to specify wavelength in micrometer, which is equal to


10-6 m.

From the viewpoint of electromagnetic wave theory, the waves travel at the speed of
light, while from the quantum theory point of view, energy is transported by photons which
travel at the speed of light. Although all the photons have the same velocity, there is always a
distribution of energy among them. The energy associated with a photon, ep = hν where h is
the Planck’s constant equal to 6.6256 x 10-34 Js. The entire energy spectrum can also be
described in terms of the wavelength of radiation.

Radiation phenomena are usually classified by their characteristic wavelength, λ. At


temperatures encountered in most engineering applications, the bulk of the thermal energy
emitted by a body lies in the wavelengths between λ= 0.1 and 100 μm. for this reason, the
portion of the wavelength spectrum between λ= 0.1 and 100 mm is generally referred to as
“THERMAL RADIATION”. The wavelength spectrum in the range l= 0.1 and 0.7 mm is
visible to the naked eye, and this is called light rays. The wavelength spectrum of radiation is
illustrated in Fig 10.1

438
−2
Ultra violet radiation (10 to 0.4 μm) 3
Infrared Radiation (0.7 to 10 μm )

Thermal Radiation
( 0.1 μm to 100 μm )

−5 −4 −3 −2 −1 0 2 3 4 5
10 10 10 10 10 10 10 10 10 10 10

Light Rays (0.1 μm to 0.3 μm)

λ , μm

Figure 10.1 Typical Spectrum of electromagnetic radiation

In the study of radiation transfer, a distinction should be made between bodies “semi-
transparent” to radiation and those which are “opaque”. If the material is semitransparent to
radiation, such as Glass, salt crystals, and gases at elevated temperatures, then the radiation
leaving the body from its outer surfaces results from emissions at all depths within the material.
The emission of radiation for such cases is a BULK or a VOLUMETRIC PHENOMENON. If
the material is opaque to thermal radiation, such as metals, wood, rock etc. then the radiation
emitted by the interior regions of the body cannot reach the surface. In such cases, the radiation
emitted by the body originates from the material at the immediate vicinity of the surface (i.e.
within about 1mm) and the emission is regarded as a SURFACE PHENOMENON. It should
also be noted that a material may behave as a semi transparent medium for certain temperature
ranges, and as opaque for other temperatures. Glass is a typical example for such behaviour. It
is semi transparent to thermal radiation at elevated temperatures and opaque at intermediate
and low temperatures.

10.1.2 DEFINITIONS OF TERMS USED IN THERMAL RADIATION

Monochromatic Emissive Power (Eλ): The monochromatic emissive power of a surface at


any temperature T and wavelength λ is defined as the quantity which when multiplied by
dλ gives the radiant flux in the wavelength range to λ + dλ.

Emissive Power (E): The emissive power of a surface is the energy emitted by the surface at
a given temperature per unit time per unit area for the entire wavelength range, from λ=0 to ∞

439

E = ∫Eλdλ ……………………………………………(10.2)
0

Absorptivity, Reflectivity and Transmissibility of a body:

Qt Qρ

Figure10.2: Effects of radiation incident on a surface

When a radiant energy strikes a material surface, part of the radiation is reflected, part is
absorbed, and part is transmitted, as shown in Figure 10.2. Reflectivity (ρ) is defined as the
fraction of energy which is reflected, absorptivity (α) as the fraction absorbed, and
transmissivity (τ) as the fraction transmitted. Thus ρ + α + τ = 1.

Most solid bodies do not transmit thermal radiation, so that for many applied problems, the
transmissivity may be taken as zero. Then

ρ + α = 1 ……………………………………(10.3)

440
Specular Radiation and Diffuse Radiation:

Φ1 Ф2

Ф2 = Φ1

(a) Specular Radiation (b) Diffuse Radiation

Figure10.3: Specular and Diffuse Radiation

When radiation strikes a surface, two types of reflection phenomena may be observed. If
the angle of incidence is equal to the angle of reflection, the radiation is called Specular.
On the other hand, when an incident beam is distributed uniformly in all directions after
reflection, the radiation is called Diffuse Radiation. The two types of radiation are depicted
in Figure 10.3. Ordinarily, no real surface is either specular or diffuse. An ordinary mirror
is specular for visible light, but would not necessarily be specular over the entire
wavelength range.A rough surface exhibits diffuse behaviour better than a highly polished
surface. Similarly, a highly polished surface is more specular than a rough surface.

Black Body:

A body which absorbs all incident radiation falling on it is called a blackbody. For a
blackbody, α = 1, ρ = τ = 0. For a given temperature and wavelength, no other body at the
same temperature and wavelength, can emit more radiation than a blackbody. Blackbody
radiation at any temperature T is the maximum possible emission at that temperature. A
blackbody or ideal radiator is a theoretical concept which sets an upper limit to the emission
f radiation. It is a standard with which the radiation characteristics of other media are
compared.

Emissivity of a Surface (ε): The emissivity of a surface is the ratio of the emissive power
of the surface to the emissive power of a black surface at the same

441
temperature. It is denoted by the symbol ε.

i.e. ε = [E/Eb]T.

Monochromatic Emissivity of a Surface (ελ):

The monochromatic emissivity of a surface is the ratio of the monochromatic emissive


power of the surface to the monochromatic emissive power of a black surface at the same
temperature and same wavelength.
ελ = [Eλ /Ebλ ] λ,T.
Gray Body:A gray body is a body having the same value of monochromatic emissivity
for all wavelengths. i.e.
ε = ελ, for a gray body.
Radiosity of A Surface (J):

This is defined as the total energy leaving the surface per unit time per unit area of the
surface. This definition includes the energy reflected by the surface due to some radiation
falling on it.

Irradiation of a surface(G):

This is defined as the radiant energy falling on a surface per unit time, per unit area of the
surface.

Therefore if E is the emissive power, J is the radiosity, e is the irradiation and r the
reflectivity of a surface, then,
J = E + ρG
For an opaque surface ρ + α = 1 or ρ = (1 – α)

J = E + (1-α)G …………………………………………. (10.4)

10.2 LAWS OF RADIATION

10.2.1 STEFAN – BOLTZMANN LAW:

This law states that the emissive power of a blackbody is directly proportional to the
fourth power of the absolute temperature of the body. i.e.

Eb α T4
Or Eb = σT4 ---------------------------------- (10.5)
where σ is called the Stefan – Boltzmann constant.
In SI units σ = 5.669x10-8 W/(m2-K4).

10.2.2 PLANCK’S LAW:

442
This law states that the monochromatic power of a blackbody is given by

C1
Ebλ = ------------------------------ ………………………..(10.6)
λ 5 [ exp (C2 / λT) – 1]
where C1 and C2 are constants whose values are found from experimental data; C1 = 3.745 x
10-16 Wm2 and C2 = 1.4388 x 10-2 m-K. λ is the wavelength and T is the absolute temperature
in K.

10.2.3 WEIN’S DISPLACEMENT LAW:

It can be seen from Equation 10.6 that at a given temperature, Ebλ depends only on λ.
Therefore this vale of λ which gives max value for Ebλ can be obtained by differentiating
Eq(10.6) w.r.t λ and equating it to zero.

Let C2/λT = y . Then Equation (10.6) reduces to

C1
Ebλ = -----------------------------
[C2 / (yT)] 5 [ e y – 1]

Then dEbλ C1 d / dy {[C2 / (yT)] 5 [ e y – 1]}


------ = ------------------------------------------
dy {[C2 / (yT)] 5 [ e y – 1]} 2

or d / dy {[C2 / (yT)] 5 (e y – 1)} = 0

Or ey(5 – y) = 5

By trial and error, y = 4.965

Therefore, if λm denotes the value of λ which gives maximum Ebλ, then

C2/λmT = 4.965

Or λmT = C2/4.965 = 1.4388x10-2 /4.965

λmT = 0.002898 m-K ………………………………. (10.7)

Equation (10.7) is called the Wein’s displacement law. From this equation it can be seen that
the wavelength at which the monochromatic emissive power is a maximum decreases with
increasing temperature. This is also illustrated in Fig 10.4(a). Fig 10.4(b) gives a

443
comparison of monochromatic emissive powers for different surfaces at a particular
temperature for different wavelengths.

Ebλ

Wave length, λ (μm)

Figure 10.4 (a) Black body emissive power as a function of wave length
and Temperature

Figure 10.4 (b) Comparison of emissive powers of different types o


surfaces as a function of wavelength at a given temperature

444
10.2.4 KIRCHOFF’S LAW:

This law states that the emissivity of a surface is equal to its absorptivity when the
surface is in thermal equilibrium with the surroundings.

Proof: Consider a perfect black enclosure i.e. the one which absorbs all the incident radiation
falling on it (see Fig 10.5). Now let the radiant flux from this enclosure per unit area arriving
at some area be qi W/m2.

Black Enclosure
EA

Sample

qi Aα

Figure 10.5 : Model used for deriving Kirchoof law

Now suppose that a body is placed inside the enclosure and allowed to come to thermal
equilibrium with it. At equilibrium, the energy absorbed by the body must be equal to the
energy emitted; otherwise there would be an energy flow into or out of the body, which would
raise or lower its temperature. At thermal equilibrium we may write

EA = qiA α ………………………………….(10.8)

If we now replace the body in the enclosure with a black body of the same size and shape and
allow it to come to thermal equilibrium with the enclosure,

EbA = qiA……………………………………(10.9)

Since α = 1 for a blackbody, if Equation 10.8 is divided by Equation 10.9 we get

E/Eb = α

445
But by definition E/Eb = e, the emissivity of the body, so that

ε = α ……………………………………….(10.10)
Equation 10.10 is called Kirchoff’s law and is valid only when the body is in thermal
equilibrium with the surroundings. However, while analyzing radiation problems in practice
we assume that Kirchoff’s law holds good even if the body is not in thermal equilibrium with
the surroundings, as the error involved is not very significant.

10.3 ILLUSTRATIVE EXAMPLES ON BASIC CONCEPTS

Example 10.1: The emission of radiation from a surface can be approximated as blackbody
radiation at 1000K.

(a) What fraction of the total energy emitted is below λ = 5μm


(b) What is the wavelength below which the emission is 10.5% of the total emission at
1000K.
(c) What is the wavelength at which the maximum spectral emission occurs at 1000K.

Solution: The radiation flux emitted by the blackbody over the wavelength interval 0 – l is
given by
λ
[Eb]0 – l = ∫Ebλdλ
0
The integration required in the above equation has been done numerically and the results are
presented in the form of a table. The table gives the value of D 0-λ where
λ
∫Ebλdλ 1 λ
D0-λ = ---0-∞----------- = ------- ∫ Ebλ dλ
∫Ebλdλ σ T4 0
0

(a) From Table of Radiation properties, for λT = 5 x 1000 = 5000, D0-λ = 0.6337. This
means that 63.37 % of the total emission occurs below λ = 5 μm.

(b) From the same table, for D0-λ = 0.105, λT = 2222.


Hence λ = 2222/1000 = 2.222 μm.

(c) From Wein’s displacement law, λmT = 0.002898.


Hence for T = 1000 K, λm =0.002898 / 1000 = 2.898 x 10 −6 m = 2.898 μm.

Example 10.2: The monochromatic emissivity of a surface varies with the wavelength in the
following manner:

446
ελ = 0 for λ < 0.3μm
= 0.9 for 0.3μm < λ < 1μm
= 0 for λ > 1μm
Calculate the heat flux emitted by the surface if it is at a temperature of 1500 K

Solution:
Eλ = ε λ Ebλ
∞ 0.3 μm 1 μm ∞
Therefore E = ∫ ε λ Ebλ = ∫ 0.0 Ebλ dλ + ∫ 0.9 Ebλ dλ + ∫ 0.0 Ebλ dλ
0 0 0.3 μm 1 μm

1 μm 1 μm 0.3 μm
= 0.9 ∫Ebλ dλ = 0.9 [ ∫ Ebλ dλ − ∫ Ebλ dλ ]
0.3 μm 0 0

= 0.9 σ T 4 [ D0-1 – D0 – 0.3]

For λ = 1μm, λT = 1500 μm-K, therefore D0-1 = ½ (0.01972 + 0.00779) = 0.93755

For λ = 0.3μm, λT = 450 μm-K, therefore D0-3 = 0

Thus E = 0.9x5.67x10-8x15004 [0.013755 – 0] = 3553 W/m2

Example 10.3: Calculate the heat flux emitted due to thermal radiation from a black surface
at 60000 C. At what wavelength is the monochromatic emissive power maximum and what is
the maximum value?

Solution:Temp of the black surface = 6273K

Heat Flux emitted Eb = σT4 = 5.67x10-8x62734 = 87798 KW/m2

Wavelength corresponding to max monochromatic emissive power is given by

λmT = 0.002898 m-K

λm = 0.002898/6273 = 4.62x10-7 m
The maximum monochromatic emissive power is given by

2 π C1
(Ebλ)max = ------------------------------------
λmax [ exp {C2 / (λmaxT)} – 1]

2 x π x 0.596 x 10 − 16
= ---------------------------------------------------------------
(4.62 x 10 − 7) 5 x [ exp{ 0.014387 / 0.002898} – 1]

447
= 1.251 x 10 14 W / m2

Example 10.4. The spectral hemispherical emissivity (monochromatic emissivity) of fire brick
at 750K as a function of wavelength is as follows:

ε1 = 0.1 for 0 < λ < 2μm


ε2 = 0.6 for 2μm < λ < 14μm
ε3 = 0.8 for14 < λ < ∞
Calculate the hemispherical emissivity, ε for all wavelengths.

Solution:

E ∫ελ Ebλ dλ 1 λ1 λ2 λ3
ε = ------ = - ------------- = ------ [ ε1 ∫ Ebλ dλ + ε2 ∫ Ebλ dλ + ε3 ∫ Ebλ dλ ]
0

Eb σ T4 σ T4 0 λ1 λ2

Where λ1 = 2μm, λ2 = 14μm, λ3 = ∞

Thus = ε1D0-λ1 + ε2[D0-λ2 – D0-λ1] + ε3[D0-∞ – D0-λ2]

Now, λ1T = 2x750 = 1500; D0-λ1 = 0.013

λ2T = 14x750 = 10500; D0-λ2 = 0.924 λ3T = ∞; D0-λ3 = 1

Hence ε = 0.1 x 0.013 + 0.6 x [ 0.924 – 0.013] + 0.8 x [1 – 0.924] = 0.609

Example 10.5: the filament of a light bulb is assumed to emit radiation as a black body at
2400K. if the bulb glass has a transmissivity of 0.90 for radiation in the visible range, calculate
the percentage of the total energy emitted by the filament that reaches the ambient as visible
light.Assume the wave length rabge corresponding to the visible range as0.38 μm to 0.76 μm.

Solution: The wavelength range corresponding to the visible range is taken as


λ1 = 0.38μm to λ2 = 0.76μm. Therefore the fraction F of the total energy emitted in this range
is given by
λ2
∫ Ebλ dλ λ2 λ1
F = τ [λ1--------- ] = τ [ ∫Ebλ dλ − ∫Ebλ dλ ] / Eb
0 0
Eb
= τ [D0-λ2 – D0-λ1].

Now λ1T = 0.38 x 2400 = 912. Hence D0-λ1 = 0.0002

448
and λ2T = 0.76 x 2400 = 1824. Hence D0-λ2 = 0.0436

Therefore F = 0.9 x [0.0436 – 0.0002] =0.039 .

Only 3.9 % of the total energy enters the ambient as light. The remaining energy produces
heating.

10.4 RADIATION HEAT EXCHANGE BETWEEN INFINITE PARALLEL


SURFACES IN THE PRESENCE OF NON PARTICIPATING MEDIUM

Assumptions:
(i) The medium does not participate in radiation heat exchange between the two
surfaces.
(ii) The surfaces are flat and are at specified uniform temperatures.

10.4.1: RADIATION EXCHANGE BETWEEN TWO PARALLEL BLACK SURFACES

J1 = Eb1
T1 G1 = J2

G2 = J1 J2 = Eb2
T2

Fig: 10.6 Radiation heat exchange between two parallel black surfaces.

Since both surfaces are parallel, flat and infinite, radiosity of surface 1 = irradiation of
surface 2 and vice versa. i.e. J1 = G2 and J2 = G1. Since both the surfaces are black, J1 = Eb1 =
σT14 and J2 = Eb2 = σT24

Net radiation leaving A1 = Qr1 = A1(J1 – G1) All this energy will reach A2.

449
Net radiation leaving A1 and reaching A2 is given by

Q1-2 = Qr1 = A1(J1 – G1) = A1[J1 – J2]

Or Q1-2 = A1[Eb1 – Eb2]

Or Q1-2 = σA1[T14 – T24] (10.11)

10.4.2 RADIATION HEAT EXCHANGE BETWEEN TWO PARALLEL INFINITE


GRAYSURFACES:

J1
T1, α1, ε1, A1 G1 = J2

G2 = J1 J2
T2, α2 ,ε2, A2

Fig: 10.7 Radiation Heat Exchange Between 2 Parallel Infinite Gray Surfaces.

Since the net radiation leaving A1 will reach A2,


Q1-2 = Qr1 = A1[J1 – G1] (10.12a)

J1 = E1 + (1-α1)G1 (10.12b)

J2 = E2 + (1-α2)G2 (10.12c)

J1 = G2 (10.12d)

J2 = G1 (10.12e)

Equation (10.12b) can be written as

J1 – (1 – α1)G1 = E1 ……………………………..(4.12f)

450
Equation (4.12c) with the help of Eqns. (10.12d) and Eqns. (10.12e) can be rewritten as

– (1 – α2)J1 + G1 = E2 (10.12g)

Solving for J1 and G1 from Equation (10.12f) and (10.12g) we get

E1 + (1 – α1) E2
J1 = ----------------------------
1 – (1 – α1) (1 – α2)

ε1E b1 + (1 – α1) ε2E b2


Or J1 = ----------------------------- ……………………………..(10.13a)
1 – (1 – α1) (1 – α2)

ε2E b2 + (1 – α2) ε1E b1


and G1 = ----------------------------- ……………………………..(10.13b)
1 – (1 – α1) (1 – α2)

Substituting these expressions for J1 and G1 in Equation( 10.12a) we get

A1
Q1-2 = -------------------------- [ε1E b1 + (1 – α1) ε2E b2 − ε2E b2 − (1 – α2) ε1E b1]
[1 – (1 – α1) (1 – α2)]

A1 [α2 ε1Eb1 − α1 ε2Eb2 ]


Or Q1-2 = ------------------------------------
[1 – (1 – α1) (1 – α2)]

Substituting for Eb1 and Eb2 in terms of temperatures we get

σA1 [α2 ε1T14 − α1ε2T2 4]


Or Q1-2 = ------------------------------------ …………………………………….(10.14)
[1 – (1 – α1) (1 – α2)]

451
If Kirchoff’s law holds good then α1 = ε1 and α2 = ε2.

σA1 [ε1 ε2T14 − ε1ε2T2 4]


Hence Q1-2 = ------------------------------------
[1 – (1 – ε1) (1 – ε2)]

σ A1 (T14 – T24)
Or Q1-2 = --------------------------- …………………………(10.15)
[ 1 / ε1 + 1 / ε2 − 1 ]

10.4.3 PLANE RADIATION SHIELDS: It is possible to reduce the net radiation heat
exchange between two infinite parallel gray surfaces by introducing a third surface in between
them. If the third surface, known as the radiation shield is assumed to be very thin, then both
sides of this surface can be assumed to be at the same temperature.
Figure10.7 shows a scheme for radiation heat exchange between two parallel infinite gray
surfaces at two different temperatures T1 and T2 in presence of a radiation shield at a uniform
temperature, T3.

Now Q1-3 σ (T14 – T34)


------ = --------------------------- ………………………..(10.16a)
A1 [ 1 / ε1 + 1 / ε13 – 1]

Q3-2 σ (T34 – T24)


And ------ = --------------------------- ………………………..(10.16b)
A1 [ 1 / ε32 + 1 / ε2 – 1]

452
T1, α1= ε1, A1
T3, ε13, A3 = A1=A2

T3, ε23, A3 T2, α2 = ε2, A2

Fig: 10.7 Radiation Heat Exchange Between Two Parallel Infinite Gray
surfaces in presence of a radiation shield

For steady state conditions, these two must be equal..Therefore we have

(T14 – T34) (T34 – T24)


--------------------------- = ----------------------------
[ 1 / ε1 + 1 / ε13 – 1] [ 1 / ε32 + 1 / ε2 – 1]

Let X = [ 1 / ε1 + 1 / ε13 – 1] and Y = [ 1 / ε32 + 1 / ε2 – 1]

Then
(T14 – T34) (T34 – T24)
--------------- = ------------------
X Y

Solving for T3 we get


T14 + (X/Y)T24
T3 = [ ----------------------------- ] 1 / 4 ………………….(10.17)
(1 + X /Y)

Substituting this value of T3 in Equation (10.16a) we get

453
Q1-3 / A1 = Q3-2 / A1 = (Q1-2 / A)1 Rad.Shield = σ { T24 – [{T14 +(X/Y)T24}/(1 + X/Y)] } / X

…………………….(10.17a)

Special case: When ε1 = ε2 = ε13 = ε32 = ε, then X = Y = 2ε − 1

Hence T3 = [(T1 4+ T24) / 2 ] ¼ ………………………..(10.18a)

σ{ T14 − [(T14 + T24) / 2 ]


and [Q1-2 / A ]1 rad shield = -------------------------------------
[2 / ε − 1]

σ [ T14 − T24 ]
= ------------------------ ………………………………(10.18b)
2 [2 / ε − 1]

It can be seen from the above equation that when the emissivities of all surfaces are equal, the
net radiation heat exchange between the surfaces in the presence of single radiation shield is
50% of the radiation heat exchange between the same two surfaces without the presence of a
radiation shield. This statement can be generalised for N radiation shields as follows:

1
[Q1-2 / A]N shields = --------- [Q1-2 / A] without shield ……………(10.18c)
(N + 1)

10.5: ILLUSTRATIVE EXAMPLES ON PLANE RADIATION SHIELDS

Example 10.6: Two parallel infinite grey surfaces of emissivities 0.5 are at temperatures of
400K and 300K. Determine the net radiation heat flux between the two surfaces. Also
determine the reduction in radiation flux when a plane radiation shield having emissivity of
0.5 on both its surfaces is placed between the two grey surfaces. Also determine the steady
state temperature of the shield.
Solution:The radiation flux between two grey surfaces is given by

σ (T1 4 – T2 4)
q = Q1-2 / A = -------------------------- .
[ 1 / ε1 + 1 / ε2 – 1]
5.67 x 10 − 8 x (400 4 – 300 4)
Since ε1 = ε2 = ε = 0.5, we have q = -------------------------------------
(2 / 0.5 – 1)

454
Or q = 330.75 W / m2.

When a radiation shield of same emissivity is placed between two grey surfaces, the
temperature of the shield T3 is given by

T3 = [(T14 + T24) / 2 ] ¼

Hence T3 = [(400 4 + 300 4) / 2] ¼

= 360.3 K

Also, since the emissivities of the plates and shield are equal we have

(q)1 shield = q / 2 = 330.75 / 2 = 163.375 W /m2

Example 10.7: Two parallel plates are at temperatures T1 and T2 and have emissivities
ε1 = 0.8 and ε2 = 0.5. A radiation shield having the same emissivity ε3 on both sides is placed
between the plates. Calculate the emissivity ε3 of the shield in order to reduce the radiation
heat loss from the system to one tenth of that without shield.

Solution: Radiation flux between the two plates without the presence of a radiation shield is
given by

σ [ T14 − T24 ] σ [ T14 − T24 ]


q1-2 = ------------------------- = ---------------------------
[ 1 / ε1 + 1 / ε2 − 1 ] [ 1 / 0.8 + 1 / 0.5 − 1]

σ [ T14 − T24 ]
or q1-2 = -------------------- …………………………(1)
2.25

When a shield is placed between the plates, the radiation flux is given by

455

T14  ( x / y )T24 
  T14  
 1 x / y 
 q 1radshield 
x
1 1 1 1 1
x  1    1  0.25 
1  3 0.8  3 3
1 1 1 1 1
y  1   1  1
3 2 3 0.5 3
 
 (1  x / y )T1  T1  ( x / y )T2 
4 4 4

  q 1radshield 
x (1  x / y )
 T14  T24 
 q 1radshield 
x y

But (q)1radshield = 0.1 x q1-2

 T14  T24  1  T1  T2 


4 4
1 1
   x  y  22.5  0.25  1  22.5
x y 10 2.25 3 3
2 2
  21.25   3   0.094
3 21.25
Example 10.8: Two large parallel plates are at 800K and 600K have emissivities of 0.5 and
0.8 respectively. A radiation shield having emissivity of 0.1 on the surface facing 800K plate
and 0.05 on the surface facing 600K plate is placed between the plates. Calculate the heat
transfer rate per m2 with and without the shield. Also calculate the temperature of the shield.

Solution:

The radiation flux without the radiation shield is given by

σ (T14 – T24) 5.67 x 10 − 8 x (800 4 – 600 4)


q = ------------------------ = -------------------------------------- = 7056 W / m 2
[ 1 / ε1 + 1 / ε2 – 1] [ 1 / 0.5 + 1 / 0.8 – 1]

When a radiation field is placed between thick plates the radiation flux is given by

σ (T14 – T34)
(q)1 Rad. shield = ------------------------- with x = [ 1 / ε1 + 1 / ε13 – 1]
X

T14 + (x/y) T24


and T3 = [ ------------------ ] ¼ with y = [ 1 / ε32 + 1 / ε2 – 1]
1 + (x/y)

Now x = [ 1 / 0.5 + 1 / 0.1 – 1] = 11 and y = [1 / 0.05 + 1 / 0.8 – 1] = 20.25

456
800 4 + (11/20.25) x 600 4
Therefore T3 = [------------------------------- ] 1/4
1 + (11 / 20.25)

= 746.8 K

5.67 x 10 − 8 x (800 4 – 746.8 4)


Hence (q)1 Rad. shield = ---------------------------------------- = 508 W / m 2
11

Example 10.9: Find an expression for the net radiant flux between two infinite parallel diffuse
grey surfaces at temperatures T1 and T2 degrees Kelvin when an infinite opaque plate of
thickness b and thermal conductivity K is placed between them. Assume that all surfaces have
the same emissivity.

Solution: The schematic for the problem is shown in Figure E10.9. For steady state heat
transfer we have

q = Q1-2 / A = Q1-3 / A = Q3-4 / A = Q4-2 / A

σ (T14 – T34)
Now Q1-3 / A = ------------------------ …………………………………………….(1)
(2 / ε – 1)

Q3-4 / A = k (T3 – T4) / b ……………………………………………………(2)

457
T1, ε

T3 , ε

T4 , ε
Plate of thickness b &
Conductivity k T2 , ε

Figure E10.9: Schematic for example 10.9

σ (T44 – T24)
Q4-2 / A = ------------------------ …………………………………………….(3)
(2 / ε – 1)

From Equation (1), T3 = [ T14 – (q / σ)(2 / ε – 1)] ¼.

Similarly from Equation (3) we get T4 = [ T24 – (q / σ)(2 / ε – 1)] ¼.

Substituting these expressions for T3 and T4 in Equation (2) we get

q = (k / b)[{T14 – (q / σ)(2 / ε – 1)}1/4 – {T24 + (q / σ)(2 / ε – 1)}1/4]

Or (qb) / k = [{T14 – (q / σ)(2 / ε – 1)}1/4 – {T24 + (q / σ)(2 / ε – 1)}1/4]

Example 10.10: Calculate the steady heat flow through the composite slab of Fig P10.10
consisting of two large plane walls with an evacuated space in between. The thicknesses
of the walls are 20 and 30cm, they have thermal conductivities of 1.0 and 0.5 W/m-k and the
emissivities of the surfaces facing each other are 0.5 and 0.4 respectively.

Solution :

If is the heat flux through the composite slab then

Q (Ti – T1) (473 – T1) (473 – T1)


q = ------- = ---------- = ----------------------- = ------------------- ……………………(a)
A Rci + R1 (1 / 20 + 0.2 /1) 0.25

458
L1 L2

Surface in
contact surface in
with fluid T1 contact
at Ti, hi k1 k2 with fluid
T2 at T0, h0

vacuum
Figure E10.10: Schematic for example 10.10

σ (T14 – T24) 5.67 x 10 − 8 x [ T14 – T24]


Also q = ---------------------------- = -----------------------------------
(1 / ε1 + 1 / ε2 – 1) [ 1 / 0.5 + 1 / 0.4 – 1]

5.67 x 10 − 8 x [ T14 – T24]


Or q = ------------------------------------ ……………………………………………(b)
3.5
(T2 – T0) (T2 – 313) (T2 – 313)
and q = --------------------- = --------------------------- = --------------- ……………..(c)
R2 + Rco (0.3 / 0.5) + (1 /10) 0.7

From Equation (a) T1 = 473 – 0.25 q, and from Equation (c) T2 = 313 + 0.7q.

Substituing these expressions for T1 and T2 in Equation (b) we get

5.67 x 10 − 8 [ (473 – 0.25q) 4 – (313 + 0.7q) 4]


q = ------------------------------------------------------------
3.5

Solving the above equatin by trial and error method we get q = 139 W / m 2.

459
10.6 VIEW FACTOR OR CONFIGURATION FACTOR:

In engineering applications, we come across problems involving radiation heat


exchange between two or more finite surfaces. When the surfaces are separated from each
other by a non participating medium that does not absorb, emit or reflect radiation, then the
radiation heat exchange is not affected by the medium. A vacuum is a perfect non participating
medium. However, air and many gases closely approximate this condition. For any two
surfaces, the orientation of them with respect to each other affects the fraction of the radiation
energy leaving one surface and striking the other directly. The concept of “VIEW FACTOR”
(also called as CONFIGURATION FACTOR/SHAPE FACTOR) has been utilised to
formalise the effects of orientation in the radiation heat exchange between surfaces. Before the
concept of view factor is introduced, two more terms have to be defined.

10.6.1 SOLID ANGLE AND INTENSITY OF RADIATION:

Solid Angle: The solid angle dw subtended by an elemental area dA surrounding point P with
respect to any other point O in space is defined as the component of the area dA in the direction
OP divided by the square of the distance between O and P. This is illustrated in Figure 10.9.
Solid angle is measured in Steradian (Sr).
n
dA
P
n
φ

dω r
r

dω = dA / r2 dω = dA cos φ / r2

Figure 10.9 : Definition of solid angle


Based on this definition, it can be readily inferred that the solid angle subtended by a
hemispherical surface from its centre is 2π (dw = 2πxr2/r2) and by a full glass sphere from its
centre is 4π.
Intensity of Radiation: The total intensity of radiation emitted by the surface in a given
direction is equal to the radiant flux passing in that direction per unit solid angle. If I is the
intensity of radiation and E is the total emissive power, then by definition

I = dE/dw (10.19a)

E = ∫Idw (10.19b)

460
Where the integration is carried out over all directions encompassed by a hemisphere.

Consider an elemental area dA1 whose total emissive power is E1. This total radiant
energy emitted by dA1 can be intercepted by a hemisphere as shown in Fig 10.10.

dAn

P
θ
OP = r
φ
O


dA1

Fig 10.10: Radiation from a differential area dA1 into surrounding hemisphere
centered at dA1.

If I is the intensity of radiation at any point P on the surface of the hemisphere due to emission
by an elemental area dA1 at O, then

 dA 
E1  I 1 cos  dw  I 1 cos   2n 
 r 

2  I1 cos   r sin  d  rd 
E1   
2
0 0 
 r2 

Assuming that I1 is same in all directions (Lambert’s Law)

2
E1  I1  2  cos  sin  d d (10.20)
 0 0

E1   I1

If the surface is a black surface then

Eb = πIb (10.21)

461
10.6.2: VIEW FACTOR BETWEEN TWO ELEMENTAL SURFACES.

Consider 2 elemental surfaces of area dA1 and dA2 as shown in Figure 10.11. Let their
normals n1 and n2 make angles φ1 and φ2 with the line joining the centroid of the two elemental
areas. Let dw2 be the solid angle subtended by dA2 at dA1 and dw1 be the solid angle subtended
by dA1 at dA2.

n1 n2

φ2
dA2

φ1

dA1

Figure 10.11: View factor between two elemental areas

Let I1 be the intensity of radiation from dA1 striking dA2 and I2 be the intensity of radiation
from dA2 striking dA1. If dq1->2 is the radiant energy leaving dA1 and striking dA2 then
dq12  I1dA1 cos 1dw2
dA2 cos 2
Or dq12  I1dA1 cos 1 2
r
Radiation energy leaving dA1 = dqr1 = E1dA1

Fraction of energy leaving dA1 and striking dA2 is defined as the view factor of dA2 with
respect to dA1 and is denoted by dF1-2

dq1-2 I1 dA1 dA2 cos φ1 cos φ2


Therefore dF1-2 = ------------- = -----------------------------------
dqr1 r2 E1 dA1

Using the relation E1 = π I1 we have

dA2 cos φ1 cos φ2


dF1-2 = ------------------------- (10.22)

462
π r2

Similarly, the view factor of dA1 with respect to dA2 is denoted by dF2-1 and given by

dA1 cos φ1 cos φ2


dF2-1 = ------------------------- (10.23)
π r2
It follows from Equation 10.22 and 10.23 that

dA1dF1-2 = dA2dF2-1 (10.24)

10.6.3: VIEW FACTOR FOR FINITE SURFACES:

n1

n2
φ2 dA2

r A2

A1 φ1

dA1

Figure10.12: View Factor between two finite areas


Consider two finite

surfaces of areas A1 and A2 as shown in Fig 10.12. If n1 and n2 are the


Normals for elemental areas dA1 and dA2 then energy leaving dA1 and reaching dA2 is given
by

dq1→2 = I1 dA1 cos φ1 dA2 cos φ2 / r2

Hence Q1→2 = ∫ ∫[{ I1 dA1 dA2 cos φ2} / r2].

463
A1 A2

Total radiation emitted by A1 = Qr1 = E1A1 = π I1 A1

Fraction of enrgy which leaves A1 and reaches A2 is given by

Q1→2 ∫ ∫[{ I1 dA1 dA2 cos φ2} / r2].


F1-2 = -------- = ---------------------------------------
Qr1 π I1 A1

1
Or F1-2 = ------ [∫ ∫{ I1 dA1 dA2 cos φ2}/ ( πr2)] …………………(10.25a)
A1

1
Similarly F2-1 = ------ [∫ ∫{ I1 dA1 dA2 cos φ2}/ ( πr2)] …………………(10.25b)
A2

It follows from Equations (10.25a) and (10.25b) that

A1F1-2 = A2F2-1 ……………………………………………….(10.26)

Properties of view factor: Consider an enclosure consisting of N zones, each of surface area
Ai (i = 1,2,3 … N). The surface of each zone may be plane, convex or concave. For the
enclosure, the following relations hold good.

1. AiFi-j = AjFj-i , i= 1,2,3 … N, j = 1,2,3 … N

2. Fi-i = 0 if Ai is plane or convex (i.e. Ai cannot see itself)


≠ 0 if Ai is concave.
3. F1-1 + F1-2 + … + F1-N = 1
F2-1 + F2-2 + … + F2-N = 1
|
|
FN-1 + FN-2 + … + FN-N = 1
N
In short, ∑ Fi,j = 1, i = 1,2,3 … N
j=1

4. When there are two surfaces, one surface say A1 is completely enclosed by A2 and
if A1 cannot see itself then, F1-2 = 1 and F2-1 = A1/A2
5. The view factor F1-2 between surfaces A1 and A2 (Figure 10.13) is equal to the sum
of the view factors F1-3 and F1-4 if the two areas A3 and A4 together make up the
area A2.

464
A2 = A3 + A4

A3
A4

A1

Figure 10.13 : Additive relation between view factors

i.e., F1-2 = F1-3 + F1-4 . It should however be noted that

F2-1 ≠ F3-1 + F3-4

View factors for standard configurations: The determination of view factors has been the
object of considerable research. In cases where the integrals in Equation 10.25 and Equation
10.26 cannot be solved analytically, numerical methods have been used. Some of these results
are represented graphically for certain standard configurations like
(i) Shape factors between parallel rectangles of equal size.
(ii) Shape factors between rectangles perpendicular to each other and having a common
edge
(iii) Shape factor from an elemental area dA1 to a rectangular area A2
(iv) Shape factor between two coaxial parallel discs
(v) Shape factors for concentric cylinders of finite length etc.

With the help of those charts and View Factor algebra, shape factors between surfaces not
covered above can be determined.

465
10.7 ILLUSTRATIVE EXAMPLES ON VIEW FACTORS:

Example 10.11: Determine the view factor between an elemental area A1 and a circular disc
A2 of radius R. The two areas are parallel to each other and positioned at a distance L form
each other such that the perpendicular to A1 passes through the centre of A2.

R
ρ

Φ2

L Φ1
r
α

FigureE10.11: Figure for example 10.11

dA1dA2 cos 1 cos 2


A1 F1 2  
A1 A2
 r2
Since A1  A2 and 1  2   , the above exp can be written as
dA2 cos 2  2 d  cos 2 
R
A1 F1 2  A1   F1 2  
A2
 r2 0
 r2
L
Now cos 1  cos   and r  L2   2
L 
2 2

2 d  L2 2 d 
R R
 F1 2  0  L2   2    L2   2   L 0 L2   2 2
2

 
R2
 F1 2   sin 2 
L R
2 2

466
Example 10.12: Obtain an expression for the shape factor for a conical cavity with respect to
itself. The height of the cavity is H and the semi vertex angle of the cavity is α
(See Figure E10.12a)

Solution:

A2

H dz
α
z

r
ds
Figure E10.12(a) : Figure
for Example 10.12
Figure E10.12 (b):

Refering to Figure 10.12(b) we have A1 and A2 form an enclosure.

Hence F1-1 + F1-2 = 1, and F2-1 + F2-2 = 1.

Since A2 cannot see itself, F2-2 = 0. Hence F2-1 = 1.0

Also A1F1-2 = A2F2-1. Therefore F1-2 = A2 / A1.


___________
From Figure E10.12(b), dA1 = 2π r ds = 2 π √ (dr2 + dz2)
____________
Or dA1 = 2 π z tan α dz √ [(dr / dz)2 + 1]
_______________
= 2 π z tan α dz √ [ tan 2 α + 1 ]

= 2 π z dz tan α sec α
H
Therefore A1 = ∫2 π z dz tan α sec α = πH2 tan α sec α
0

πH 2 tan 2 α

467
Hence F1-1 = 1 – F1-2 = 1 − ------------------------- = 1 – sin α
πH2 tan α sec α

Example 10.13: Consider 3 small surfaces each of area dA1 = dA2 = dA3 = 2 cm2 as shown in
fig E10.13. (a) Calculate the solid angle subtended by dA2 with respect to a point on dA1 (b)
The solid angle subtended by dA3 with respect to a point on dA1 and (c) The elemental Diffuse
factors dFdA1-dA2 and dFdA1-dA3.

Φ1 = 30 0; Φ2 = 45 0 ;

r12 = 300 cm ; r13 = 200 cm ;


dA2
dA1 = dA2 = dA3 = 2 cm2 ;
Φ2
dA3

r12
n3
n2
n1 r13
Φ1
Figure E10.13: Figure for example
10.13 dA1

If dw2 1 is the solid angle subtended by dA2 w.r.t a point on dA1 then
dA2 cos 2 2 cos 450
dw2 1    1.57  105 sr
r122 3002
dA3 cos 3
Similarly dw31  2
(but 3  00 )
r13
dA3 2
 dw31  2
 2
 5  105 sr
r13 200
dA2 cos 12 cos 2 2  cos 300  cos 450
dFdA1  dA2    4.33  106
  r12 
2
  300 2

dA3 cos 13 cos 3 2  cos 2 00


Similarly dFdA1  dA3    1.59  105
 r13
2
  200 2

Example 10.14: Determine the view factor F1-2 between an elemental surface dA1 and the
finite rectangular surface A2 for the geometric arrangements shown in Fig E10.14

468
4m

8m 8m

3m A2 3m A2 A2
6m

4m 4m

3m
3m
3m

(a) (b) (c)

Figure E10.14: Figures for example 10.14

Soluiton : (a) The configuration in Figure E10.14(a) is a standard configuration for which
the analytical expression for F1-2 is given by

1 x y y x
F1-2 = ---- [ ------------ tan − 1 {------------- } + ------------- tan − 1 { ----------------} ]
2π √(1 + x2) √(1 + x2) √(1 + y2) √(1 + y2)

Where x = L1 / D = 3 /3 = 1 ; and y = L2 / D = 4 / 3 = 1.33

Substituting these values in the expression for F1-2 we get

1 1 1.33 1.33 1
F1-2 = ---- [ ------------ tan − 1 {------------- } + ------------- tan − 1 { ----------------} ]
2π √(1 + 12) √(1 + 12) √(1 + 1.332) √(1 + 1.332)

Or F1-2 = 0.154

469
(a) The area A2 is divided into two equal areas A3 and A4 both having the common edge
of width L1 = 3 m.Then

F1-2 = F1-3 + F1-4 = 2 F1-3 (Because F1-3 = F1-4)

= 2 x 0.15 v4 = 0.308

(c) In this case A2 is divided into three equal areas A3,A4, and A5 as shown below

Now F1-2 = F1-3 + F1-4 + F1-5

A4 A5 = 3 F1-3

A3 = 3 x 0.154

= 0.462

Example 10.15: Determine the Shape factor F1-2 for the configuration shown in Fig E10.15

470
3m

A1

1m

1m
A2

1m 1m

Figure E10.15: Figure for example 10.15

Solution:

A6 F6-1 A4 F4-1
F1-2 = F1-6 − F1-4 = ---------- − --------------
A1 A1

A6 A4
= ------ [ F6-5 − F6-3 ] − ---------- [ F4-5 − F4-3 ]
A1 A1

Values of F6-5, F6-3, F4-5 and F4-3 can be obtained from chart as follows.

To find F6-5:

471
A5 = A6 = 3 x 2 = 6 m 2 ;
2m
L1 / W = L2/W = 2/3 = 0.667;

3m From chart F6-5 = 0.22. Similarly we


A5
Get F6-3 = 0.16; F4-5 = 0.32 ; F4-3 = 0.27

2m
A6

6 3x1
Hence F1-2 = ------- [ 0.22 − 0.16 ] − ------- [ 0.32 − 0.27 ] = 0.07
1x3 3x1

Example 10.16: Find F1-2 for the configuration shown in Figure E10.16

472
2m

2m
2m
A2
A1

2m

Figure E10.16: Figure for example 10.16

Refering to Figure E10.16(a),let

A6 = A2 + A4 = 4 x 2 = 6 m2 and

A4 A5 = A1 + A3 4 x 2 = 6 m2.

A1 = A2 = A3 = A4 = 2 x 2 = 4 m2.
A2
A1
F1-2 = F1-6 − F1-4

= (A6F6-1) / A1 − F1-4
A3
= (A6/A1)[F6-5 − F6-3] − F1-4
Figure E10.16(a)
= (A6/A1)F6-5 −{(A3F3-6)/A1}− F1-4

Or F1-2 = (A6/A1)F6-5 − (A3/A1) [ F3-2 + F3-4] − F1-4

But F3-2 = F1-4 and F3-4 = F1-2.

Hence F1-2 = (A6/A1)F6-5 − (A3/A1)F1-4 − (A3/A1) F1-2 − F1-4

A6F6-5 − A3F1-4 − A1F1-4 2A1F6-5 − 2A1F1-4


Or F1-2 = ---------------------------------- = -------------------------- = F6-5 − F1-4
(A1 + A3) 2A1
To find F6-5:- L2 = L1 = 2 m; W = 4 m.

473
L1 / W = L2 / W = 2 / 4 = 0.5.

From chart F5-6 = 0.25 = F6-5 since A5 = A6. Similarly F1-4 = 0.2.

Hence F1-2 = 0.25 – 0.20 = 0.05

Example 10.17: Find the Shape Factor F2-1 for the configuration shown in fig E10.17

A2
2m

2m
A1

Figure E10.17: Figure for example


10.17
0.5
2m

Solution: Refer to Figure 10.17 (a)

F2-1 = F2-3 + F2-5 = F2-3 + (A5 F5-2) / A2

= F2-3 + (A5 / A2) [ F5-6 – F5-4]

= F2-3 + (A6F6-5) / A2 − (A5F5-4) / A2

= F2-3 + (A6 / A2) [F6-1 − F6-3] − (A5F5-4) / A2

= F2-3 + (A6 / A2)F6-1 − (A3F3-6) / A2 − (A5F5-4) / A2

= F2-3 + (A6 / A2)F6-1 − (A3F2-1) / A2 − (A5F5-4) / A2

Or [( 1 + (A3 / A2) ] F2-1 = F2-3 + (A6 / A2)F6-1 − (A5F5-4) / A2

474
A1 = A3 + A5; A6 = A2 + A4
A4
A2
A2 = A3 = 0.5 x 2 = 1 m 2

A4 = A5 = 1.5 x 2 = 3 m 2

A1 = A6 = 2 x 2 = 4 m2

A3 A5

Figure E10.17(a)

F2-3 + (A6 / A2)F6-1 − (A5F5-4) / A2


Or F2-1 = -----------------------------------------------------
[( 1 + (A3 / A2) ]

From chart : F2-3 = 0.06 ; F5-4 = 0.17 ; F6-1 = 0.2

0.06 + 4 x 0.2 − 3 x 0.17


Hence F2-1 = --------------------------------- = 0.175
( 1 + 1)

Example 10.18: Find the Shape factor F1-2 for the configuration shown in Fig E10.18

Solution: Refer Figure E10.18(a).

F1-2 = F1-8 + F1-4 + F1-6. But F1-4 = F1-6.

Hence F1-2 = F1-8 + 2F1-4

= F1-8 + 2 (A4F4-1) / A1

From example 10.16 we have F4-1 = F7-10 − F4-3

Hence F1-2 = F1-8 + 2 (A4 /A1) [F7-10 − F4-3]

From chart F1-8 =0.15 ; F7-10 = 0.23 ; F4-3 = 0.2

475
a/2
a

A1
a
A2

a Figure E10.18: Figure for example


10.18

A7 = A1 + A3 ;
A3
A10 = A4 + A8

A1 A4

A5 A8

A6

Figure E10.18 (a)

a2
Hence F1-2 = 0.15 + 2 x -------------- x [0.23 – 0.20] = 0.27
[(a/2) x a]

Example 10.19 A1 and A2 are two rectangular flat surfaces having a common edge and
inclined at an arbitrary angle α to each other. They are very long along the common edge and
have lengths of ab and ac respectively in the other direction. Show that

476
F1-2 = (ab +ac) – bc
2ab

Solution: Refer Figure E10.19

c A1,A2 and A3 form an enclosure.

A2 Hence F1-1 + F1-2 + F1-3 = 1.


A3 But F1-1 = 0. Hence
F1-2 + F1-3 = 1 ………………….(a)
similarly
F2-1 + F2-3 = 1 ………………….(b)
a b
A1 F3-1 + F3-2 = 1…………………..(c)
Figure E10.19
From Equation (a) we get F1-2 = 1 – F1-3
…...(d)

From (c) we get F3-1 = 1 – F3-2

Or (A1F1-3) / A3 = 1 – (A2F2-3)/A3
Or F1-3 = (A3 / A1) – (A2F2-3) / A1

= (A3 / A1) – (A2 / A1) [1 – F2-1]

= (A3 / A1) – (A2 / A1) + F1-2

Substituting this expression in Equation (d) we get

F1-2 = 1 – [(A3 / A1) – (A2 / A1) + F1-2]

Solving for F1-2 we get


(A1 + A2) – A3 (ab x 1) + (ac x 1) – (bc x 1)
F1-2 = ---------------------- = -----------------------------------
2A1 2 x (ab x 1)

(ab + ac – bc)
= -------------------
2 ab

Example 10.20: (Hottel’s cross string formula) Obtain an expression for the view factor
between two flat surfaces, which extend to infinity in one direction.

Solution: Refer Figure E10.20

477
A2 = L2 x 1

A4 = L4 x 1

A6 = L6 x 1

A3 = L3 x 1 A1 = L1 x 1

A5 = L5 x 1

Figure E10.20 : Figure for example 10.20

Consider unit width perpendicular to the plane of the paper. A1, A2, A3 and A4 form an
enclosure. Hence we have
F1-1 + F1-2 + F1-3 + F1-4 = 1 and F1-1 = 0.

Therefore F1-2 + F1-3 + F1-4 = 1 …………………………(a)

Similarly A1, A3 and A5 form an enclosure. Therefore we have

F1-3 + F1-5 = 1 ………………………………..(b)

Also F3-1 + F3-5 = 1 ………………………………..(c)

and F5-1 + F5-3 = 1 ………………………………..(d)

From Equation (b) we have F1-3 = 1 – F1-5

= 1 – (A5F5-1) / A1

= 1 – (A5 / A1) [ 1 – F5-3]

= 1 – (A5 / A1) + (A3F3-5) / A1

Or F1-3 = 1 – (A5 / A1) + (A3 / A1) [1 – F3-1]

= 1 – (A5 / A1) + (A3 / A1) – F1-3

478
1 – (A5 / A1) + (A3 / A1) A1 – A5 + A3
Hence F1-3 = --------------------------------- = ---------------------------
2 2A1

A1 – A6 + A4
Similarly F1-4 = ------------------------
2A1

Now from Equation (a) we have F1-2 = 1 – [F1-3 + F1-4]

Substituting the expressions obtained for F1-3 and F1-4 we get

A1 – A5 + A3 A1 – A6 + A4
F1-2 = 1 – ------------------- − -------------------
2A1 2A1

(A5 + A6) – (A3 + A4) (L5 + L6) – (L3 + L4)


= ---------------------------- = ----------------------------
2A1 2L1

Example 10.21: A truncated cone has top and bottom diameters of 10cm and 20cm and a
height of 10cm. Calculate the shape factor between the top surface and the side and the side
and itself.

Solution: To find (i) F2-3 and (ii) F3-3 . Refer to Figure E10.21.

(i) F2-1 = (A1F1-2) / A2. F1-2 can be directly obtained fom chart as follows:

L1 / r = 10 / 10 = 1 ; and L2 / r = 5 / 10 = 0.5.

Hence fromchart F1-2 = 0.12.

π x (10)2
Therefore F2-1 = -------------- x 0.12 = 0.48; F2-1 + F2-2 + F2-3 = 1 and F2-2 = 0
π x (5)2

Hence F2-3 = 1 – F2-1 = 1 – 0.48 = 0.52

479
A2
r2
A3

r1 = 10 cm

r2 = 5 cm
L
L = 10 cm

r1
A1

Figure E10.21: Figure for example 10.21

(ii) F1-3 = 1 – F1-2 = 1 – 0.12 = 0.88

π x (10 2)
Hence F3-1 = (A1F1-3) / A3 = ------------- x 0.88 = 0.525
526.9

Therefore F3-3 = 1 – [F3-1 + F3-2] = 1 – [0.525 + 0.0775] = 0.397

Example 10.22: Determine the shape factors for the geometries shown in Figure E10.23(a) to
E10.23(i)

480
Long groove, A1 Surroundings, A2
A1
Long duct

A2

Find F1-2, F2-1 and F1-1

Figure E 10.23 (a) Figure E 10.23 (b)

A1 1.0 Surroundings, A3
Find F1-1 and F1-3

Figure E 10.23 (c)


2.0

A1 is covered by a
A1 hemispherical surface A2 of
radius 1.5. Find F1-2, F1-3,
1.0 F2-1 and F2-2

Figure E 10.23 (d)

481
A1

A2
2.0 A2
A1

1.0

FigureE10.23(e): F1-3,F2-3,F2-2,F1-2 for FigureE10.23 (f): F1-2,F2-1 for


disc surrounded by a short cylinder sphere on infinite plane

A1
A2
A1 A3
1.0
A2
1.5

1.5
1.0
3.0

FigureE10.23 (g): F1-2,F2-1 FigureE10.23(h): F1-2,F2-1 ,F1-3, F2-3


,F2-3

A1

A2

Figure E10.23 (i) : F1-2, F2-1, F1-3;


1.0 hemispherical shell, A1 enclosing a
3.0 small sphere,A2 surrrounded by a
large enclosure A3

482
Solution: Refer Figure E10.23(a):

A1 and A2 form an enclosure. Hence F1-1 + F1-2 = 1


and F2-1 + F2-2 = 1.
Since A2 is a flat surface F2-2 = 0. Hence F2-1 = 1. But A1F1-2 = A2F2-1.
Therefore F1-2 = (A2 / A1)F2-1 = (A2 / A1) =

Refer to Figure E10.23(b) :


All the radiation from (1) which goes to the surroundings can be intercepted by an imaginary
surface 21 as shown in Figure Now 1 and 21 form an enclosure. Therefore from the above
example
1  1
F21  F21  1; F1 2  F1 2  ; F11 
 
Refer to Figure E10.23(c):

All the radiation from (1) which goes to the surroundings can be intercepted by an imaginary
surface 2 and 4 as shown in Figure Now, 1, 2 and 4 form an enclosure.

 F11  F12  F14  1


but F14  F12

483
 F11  1  2 F1 2          (a)
also F21  F22  F24  1 and F2 2  0
 F21  F24  1  F2 1  1  F2 4
from chart for two parallel coaxial discs  2  and  4 
F2 4  0.383
F21  1  0.383  0.617
A2 F21  R 2 1.0
 F1 2    0.617   0.617  0.309
A1 2 RL 2 1.0
 F11  1  2  0.309  0.383
F13  1  F11  1  0.383  0.617

Refer to Figure E10.23(d)

F11  F1 2  F13  1 and F11  0; F13  0


 F1 2  1____________________(a )
F21  F2 2  F23  1 ___________(b)
F31  F3 2  F33  1 and F33  F31  0
 F3 2  1____________________(c )
A3 F3 2  A2 F23
A3  1.52  0.52 
 F23  F3 2  1  0.444
A2 2 1.52 
A2 F21  A1 F1 2
A1   0.52 
 F21  F1 2   1  0.055
A2 2 1.52 
From eq (b), F2 2  1   F21  F23 
 1   0.055  0.444 
 F2-2 =0.5

484
Refer to Figure E10.23(e):
Surroundings (3) can be replaced by imaginary surfaces (5) and (4) as shown in
Figurebelow.Now, 1, 2, 4 and 5 form an enclosure.Hence

F1-1 + F1-2 + F1-4 + F1-5 = 1, and F1-1 = F1-4 = 0.

Hence F1-2+ F1-5 = 1…………………….(a)

A5 F2-1 + F2-2 + F2-4 + F2-5 = 1, and F2-5 = F2-4 + F2-4


A2
Hence F2-2+ 2F2-5 = 1…………………….(b)
A1

F4-1 + F4-2 + F4-4 + F4-5 = 1, and F4-1 = F4-4 = 0.

Hence F4-2+ F4-5 = 1………………………(c)


A4
F5-1 + F5-2 + F5-4 + F5-5 = 1, and F5-5 = 0.

Hence F5-1 + F5-2 + F5-4 = 1

Or F5-2 + F5-(1+4) = 1………………………..(d)

To find F1-5: L/r1 = 2.0 / 0.5 = 4 ; r5 / L = 1.5 / 2 = 0.75 ;


From chart F1-5 = 0.35.

From Equation(a) we have F1-2 = 1 – 0.35 = 0.65

To find F51 4 :
d 3
R   0.75
2L 2  2
2 R 2  1 2  0.752  1
X   3.78
R2 0.752
X   X 2  4  3.78  3.782  4 
 F1 45  F51 4   
2 2
or F51 4  0.286
 From Eqn (d) F5 2  1  0.286  0.714

485
A5 F5 2  1.52 
 F25    0.714  0.268
A2 2 1.5  2
 From (b) F2 2  1  2  0.268  0.465
A1 F15
Now F5 4  F51 4  F51  F51 4 
A5
  0.52 
 0.286   0.35  0.247
 1.52 

AF  1.5  2

 F45  5 5 4   0.247  0.278


A4  1.52  0.52 
From (c) F4 2  1  0.278  0.722
A4 F4 2  1.5  0.5 
2 2

 F2 4    0.722  0.24


A2 2 1.5  2
F21  F25  F2 4  0.268  0.240
 F21  0.028
F23  F25  F21 4  2 F25  2  0.268
 F23  0.536

10.8 RADIATION HEAT EXCHANGE BETWEEN FINITE SURFACES

10.8.1 RADIATION HEAT EXCHANGE BETWEEN FINITE BLACK SURFACES

Consider two black surfaces of area A1 and A2 and at temperatures T1 and T2 as shown in Fig
10.14. Then radiation leaving A1 and reaching A2 can be written as

486
A2,T2, Eb2

A1, T1,Eb1

Figure 10.14: Radiation Heat Exchange Between Two Black Surfaces

Q1->2 = A1J1F1-2 Since A1 is a black surface, J1 = Eb1 thus,

Q1->2 = A1Eb1F1-2

Similarly radiation leaving A2 and reaching A1 is given by


Q2->1 = A2Eb2F2-1

Net heat exchange between A1 and A2 can be written as


Q12 = Q1->2 – Q2->1
= A1F1-2Eb1 – A2F2-1Eb2

But A1F1-2 = A2F2-1

Thus Q1-2 = A1F1-2 (Eb1 – Eb2)

Q1-2 = σA1F1-2(T14 – T24) -----------------(10.27)

10.8.2 RADIATION HEAT EXCHANGE BETWEEN FINITE GREY SURFACES (NET


WORK METHOD)

The calculation of radiation heat transfer between black surfaces is relatively easy because all
the radiant energy which strikes a surface is absorbed by it. When non black bodies are
involved, the situation is much more complex because all the energy striking a surface will not
be absorbed: part will be reflected back to another heat transfer surface, and part may be

487
reflected out of the system entirely. The problem can become complicated because the radiant
energy can be reflected back and forth between heat transfer surfaces several times.
While deriving the expression for radiation exchange between any two finite grey
surfaces the following assumptions are made
i. All the surfaces are diffuse and uniform in temperature.
ii. The reflection and emissive properties are constant over all the surface
iii. Radiosity and irradiation are uniform over each surface. This assumption is not strictly
correct even for ideal grey diffuse surfaces, but the problems become exceedingly
complex when this restriction is not imposed.
iv. The surfaces are opaque (i.e. Transmissivity is zero)

Now the net radiation from a surface is given by

Qr = A(J – G).
But J = E + (1 – α)G.
Assuming α = ε, J = ε Eb + (1 – ε)G. Or G = (J - ε Eb) / (1 – ε)

Hence expression for Qr can be written as follows:

Qr = A [J – (J - ε Eb) / (1 – ε)]

(Eb – J) (Eb – J)
Qr = ---------------- = ---------------- ………………..(10.28)
(1 – ε)/(Aε) R

Equation 10.28 can be interpreted as follows… (Eb-J) can be thought of as thermal potential,
R = (1-ε)/εA can be thought of as thermal resistance offered by the surface for radiation, as Qr
is the radiation heat flow rate. Therefore a radiating surface can be replaced by an element as
shown in Fig 10.15

Qr

Eb R = (1 – ε)/(Aε) J

Fig 10.15: Element representing surface resistance in the Radiation network method

Now let us consider the radiation heat exchange between two surfaces A1 and A2. Radiation
which leaves A1 and strikes A2 is given by

Q1→2 = A1J1F1-2

Similarly Radiation which leaves A2 and strikes A1 is given by

488
Q2→1 = A2J2F2-1

Therefore net radiation heat transfer from A1 to A2 is given by

Q1-2 = Q1→2 – Q2→1 = A1J1F1-2 - A2J2F2-1

But A1F1-2 = A2F2-1.


(J1 – J2)
Hence Q1-2 = A1F1-2(J1 – J2) = ---------------- ………………(10.29)
1 / (A1F1-2)

Equation (10.29) can be represented by an element as shown in Fig 10.16

J1 J2

R12 = 1 / (A1F12)

Figure 10.16: Element representing space resistance in radiation network


method

The two network elements shown in Fig 10.15 and 10.16 represent the essentials of radiation
network method. To construct a network for a particular radiation heat transfer problem we
need only to connect a “surface resistance” (1-ε)/εA to each surface and a “space resistance”
1/(AiFi-j) between the radiosity potential potentials. This is illustrated below.

10.8.3 NETWORK METHOD FOR RADIATION HEAT EXCHANGE BETWEEN TWO


PARALLEL INFINITE GREY SURFACES

The radiation network for the above problem will be as shown in Fig 10.17

J1
T1, α1= ε1, A1 G1 = J2

G2 = J1 J2
T2, α2 = ε2, A2

Eb1 J1 J2 Eb2

Qr1 Q12 R12 Q21 R2 Qr2


R1

Fig 10.17 Radiation network for two parallel infinite grey surfaces

489
(Eb1 – Eb2)
From Figure 10.17 we can write Qr1 = Q12 = ------------------------
(R1 + R12 + R2)

σ(T14 – T24)
Or Q12 = ---------------------------------------------------------------------
{(1 – ε1) / (A1ε1) + 1 / (A1F12) + (1 – ε2) / (A2ε2)}

For two infinite parallel surfaces A1 = A2 and F12 = 1.0. Hence the above expression simplifies
to

σA1 (T14 – T24) σA1 (T14 – T24)


Q12 = ----------------------------------------- = --------------------------------
{(1 – ε1) / ε1 + 1 + (1 – ε2) / ε2} { (1 / ε1) + (1 / ε2) – 1}

an expression which has already been derived (Refer Equation 10.15).

10.8.4: NETWORK FOR RADIATION HEAT EXCHANGE BETWEEN TWO PARALLEL


INFINITE GRAY SURFACES IN PRESENCE OF A RADIATION SHIELD

T1, α1= ε1, A1


T3, ε13, A3 = A1=A2

T3, ε23, A3 T2, α2 = ε2, A2

R1 R13 R3 R3’ R32 R2


Qr1 J1 J13 Eb3 J32 J2 Eb2
Eb1

Fig: 10.18 Radiation Net work forHeat Exchange Between Two Parallel Infinite Gray
surfaces in presence of a radiation shield

From Fig 10.18 the net radiation heat transfer from A1 to A2 is given by

(Eb1 – Eb2)
Q12 = ----------------------------------------------
[R1 + R13 + R3 + R3’ + R32 + R2]

490
(Eb1 – Eb2)
= ---------------------------------------------------------------------------------------------------
[(1 – ε1) / (A1ε1) + 1 / (A1F13) + (1 – ε13) / (A3 ε13) + (1 – ε32) / (A3 ε32) + 1 / (A3F32)

+ (1 – ε2) / (A2ε2)]

But F13 = F32 = 1 and A1 = A2 = A3. Therefore we have

σA1(T14 – T24)
Q12 = ------------------------------------------------------------------------------------------
[{(1 – ε1) / ε1} + 1 + {(1 – ε13) / ε13} + {(1 – ε32) / ε32}+ 1 + {(1 – ε2) / ε2}]

σA1(T14 – T24)
= -------------------------------------------------------------------- ………………..(10.30)
[ { (1 / ε1) + (1 / ε2) – 1 } + { (1 / ε13) + (1 / ε32) – 1 }]

10.8.5: RADIATION NETWORK FOR HEAT EXCHANGE BETWEEN TWO


SURFACES, ONE SURFACE COMPLETELY ENCLOSING THE OTHER (The enclosed
surface cannot see itself)

A2, T2, ε2
A1, T1, ε1

Figure 10.19(a): A1 is completely enclosed by A2 and A1 cannot see


itself

Let A1 be completely enclosed by A2 as shown in Figure10.19(a). Then we have

F1-1 + F1-2 = 1 and F1-1 = 0.

Hence F1-2 = 1 and therefore

F2-1 = A1/A2. The net work is shown in Figure10.19(b)

491
(Eb1 – Eb2) σ(T14 – T24)
Qr1 = Q1-2 = -------------------- = -----------------------------------------------------------
(R1 + R12 + R2) [(1 – ε1)/(A1ε1) + 1/(A1F1-2) + [(1 – ε2)/(A2ε2)]

σ(T14 – T24)
= ------------------------------------- ………………………………………(10.31)
[(1/ε1) + (A1/A2){(1/ε2) – 1}]

Eb1 J1 J2 Eb2

Qr1 Q12 R12 Q21 R2 Qr2


R1

Figure 10.19 (b) : Radiation network for the configuration shown in Figure 10.19(a)

10.8.6: NETWORK METHOD FOR THREE ZONE ENCLOSURE

The network method described above can be readily generalised to enclosures


involving three or more zones. However when there are more than three zones, the analysis
becomes more involved and it is preferable to use the more direct “Radiosity Matrix” method.
The radiation network for a three zone enclosure shown in Fig 10.20(a) is shown in Fig
10.20(b)

A1, ε1, T1

A2, ε2 T2

A3, ε3 T3

Figure 10.20 (a) : Radiation in a three – zone enclosure

492
Figure 10.20 (b) : Radiation Network for a three zone enclosure

Reradiating Surface: In many practical situations one of the zones may be thermally
insulated. In such a case, the net radiation heat flux in that particular zone is zero, because that
surface emits as much energy as it receives by radiation from the surrounding zones. Such a
zone is called a “RERADIATION ZONE” or an “ADIABATIC ZONE”. Fig 10.21(a)
represents a three zone enclosure with surface (3) being the reradiating surface and Fig
10.21(b) the corresponding radiation network.

A1, ε1, T1

A2, ε2 T2

Reradiating
surface ,A3, T3

Figure 10.21 (a) : Radiation in a three – zone enclosure with A3 being a reradiating surface

493
Figure 10.21 (b) : Radiation network for the configuration shown in Figure 10.21(a)

For a three zone enclosure under steady state conditions, by I law of thermodynamics.

Qr1 + Qr2 + Qr3 = 0 …………………………………..(10.32)

If A3 is a reradiating surface, then Qr3 = 0.

Therefore Qr1 = - Qr2 = (Eb1 – Eb2) / Req …………………………………….(10.33a)

Where Req = R1 + [(1/R1) + {1/ (R13 + R23)}] ─ 1

(1 – ε1) (1 – ε2)
i.e Req = ----------- + [ A1F12 + {1/(A1F1-3) + 1/(A2F2-3)} ─ 1 ] ─ 1 + ------------
A1ε1 A2ε2

………………. (10.33b)

10.9 ILLUSTRATIVE EXAMPLES ON NETWORK METHOD:

Example 10.24: Two square plates 1m x 1m are parallel to and directly opposite to each other
at a distance of 1m. The hot plate is at 800K and has an emissivity of 0.8. The clod plate is at
600K and also has an emissivity of 0.8. The radiation heat exchange takes place between the
plates as well as the ambient at 300K through the opening between the plates. Calculate the
net radiation at each plate and the ambient.

Solution:

494
1m

A1, ε1, T1 1m

1m 3

A2, ε2, T2

Given: T1 = 800 K ; ε1 = 0.8 ; T2 = 600 K; ε2 = 0.8; T3 = 300 K


To find: Qr1 ; Qr2; Qr3

Solution: (1 – 0.8)
R1 = (1 – ε1) / (A1ε1) = ----------------- = 0.25
(1 x 1) x 0.8
(1 – 0.8)
Similarly R2 = (1 – ε2) / (A2ε2) = ----------------- = 0.25
(1 x 1) x 0.8
R3 = (1 – ε3) / (A3ε3)
A3 is the area of the surroundings which is very large. Hence R3 = 0.The radiation net work
for this problem is shown in FigureE 10.24.

From chart F1-2 = F2-1 = 0.20. But F1-1 + F1-2 + F1-3 = 1 and F1-1 = 0.
Hence F1-3 = 1 – F1-2 = 1 – 0.2 = 0.8 = F2-3.

R12 = 1 / (A1F1-2) = 1 / {(1x1) x 0.2} = 5.0. R13 = 1 / (A1F1-3) = 1 / {(1x1) x 0.8} = 1.25

R23 = 1 / (A2F2-3) = 1 / {(1x1) x 0.8} = 1.25

Eb1 = σT14 = 5.67 x 10 ─ 8 x 800 4 = 23224 W / m2 = 23.224 kW/m2.

Eb2 = σT24 = 5.67 x 10 ─ 8 x 600 4 = 7348 W / m2 = 7.348 kW/m2.

Eb3 = σT234 = 5.67 x 10 ─ 8 x 300 4 = 459 W / m2 = 0.459 kW/m2.


For steady state radiation, radiation energy cannot accumulate at nodes.Hence

Qr1 = Q1-2 + Q1-3

Or Qr1 ─ Q1-2 ─ Q1-3 = 0

495

Eb1  J1

 J1  J 2   J1  J 3  0
R1 R12 R13
23.224  J1  J1  J 2   J1  0.459 
or    0 _______  a 
0.25 5 1.25
similarly Qr2  Q21  Q23  Qr2  Q21  Q23  0
Eb2  J 2

 J 2  J1   J 2  Eb3
0
R2 R12 R23
7.348  J 2  J 2  J1   J 2  0.459 
   0 _______  b 
0.25 5 1.25
Solving Eqn (a) and (b) simultaneously we get
J1  18.921KW / m 2 ; J 2  6.709 KW / m 2
Eb1  J1 23.224  18.921
 Qr1    17.212 KW
R1 0.25
Eb2  J 2 7.348  6.709
Qr2    2.557 KW
R2 0.25
But Qr1  Qr2  Qr3  0  Qr3   Qr1  Qr2    17.212  2.557 
Qr3  19.769 KW

Example 10.25 The configuration of a furnace can be approximated as an equilateral triangular


duct which is sufficiently long that the end efforts are negligible. The hot wall is at 900K with
an emissivity of 0.8 and the cold wall is at 400K with emissivity of 0.8. The third wall is a
reradiating wall. Determine the net radiation flux leaving the hot wall.

Solution:
A1 = A2 = A3 = 1m2
T1 =900K, Є1 = 0.8
T2 = 400K, Є2 = 0.8

The radiation network for the above problem will be as shown for example 10.24

496
1  1 1  0.8
R1    0.25
A11 1 0.8
1   2 1  0.8
R2    0.25
A2 2 1 0.8
Using Hottel's cross string formula, we have

F1 2 
 A1  A2   A3  1  1  1  0.5  F  F2 3
13
2 A1 2 1
1 1
 R12    2  R23  R13
A1 F1 2 1 0.5
1
 1 1 
Req  R1      R2
 12
R R13  R23 
1
1 1 
 0.25     0.25  1.833
 2 2  2 
Eb1  Eb2  T14  T24  5.67 108  9004  4004 
Qr1   
Req Req 1.833
 19503 W / m 2
Qr1  Qr2  Qr3  0 and Qr3  0  Qr2  Qr1  19503 W / m 2

Example 10 .26 A short cylindrical enclosure is maintained at the temperatures as shown in


Fig P4.26. Assuming Є2= Є3=1; Є1=0.8 determineQr1 and Qr2

Reradiating
surface From chart F1-2 = 0.175 = F2-1
(A2=A1) ;
T1= T2 = 1000 K Also F1-1 + F1-2 + F1-3 = 1 and
2000 K 1 m dia ε2 = 1.0 F1-1 = 0.Hence F1-3 = 1 – 0.175
ε1= 0.8 = 0.825 = F2-3

1m

497
The radiation network for the above problem will be as shown for example 10.24

1  1 1  0.8
R1    0.318
A11   0.52  0.8
1   2 1 1
R2   0
A2 2 A2 1
1 1
R12    7.3
A1 F1 2   0.52  0.175
1 1
R13    1.54
A1 F1 2   0.52  0.825
1 1
R23    1.54
A2 F23   0.52  0.825
1
 1 1 
Req  R1      R2
 R12 R13  R23 
1
 1 1 
 0.318     0  2.484
 7.3 1.54  1.54 
Eb1  Eb 2  T1  T2  5.67 10   2000  1000 
4 4 8 4 4

 Qr1   
Req Req 2.484
 329.14 103 W  329.14 KW
Qr1  Qr 2  Qr 3  0 and Qr 3  0
 Qr 2  Qr1  329.14 KW

498
Example 10.27 A spherical tank with diameter 40cm fixed with a cryogenic fluid at 100K is
placed inside a spherical container of diameter 60cm and is maintained at 300K. The
emissivities of the inner and outer tanks are 0.15 and 0.2 respectively. A spherical radiation
shield of diameter 50cm and having an emissivity of 0.05 on both sides is placed between the
spheres. Calculate the rate of heat loss from the system by radiation and find also the rate of
evaporation of the cryogenic liquid if the latent heat of vaporization of the fluid is 2.1x105 W-
s/Kg

Solution: The schematic and the corresponding network for the problem will be as shown in
Fig P.4.27

499
Eb1  Eb 2
Qr1  Q12 
R1  R13  2 R3  R32  R2
 T14  T24 

1  1 1 1 3 1 1 2
 2  
A11 A1 F13 A3 3 A3 F3 2 A2 2
 A1 T14  T24 

1 A1  2  A1  1 
   1    1
1 A3   3  A2   2 
5.67 108  4   0.22   1004  3004 
 2 2
 6.83 W
1  40   2   40   1 
    1      1
0.15  50   0.05   60   0.2 
Q 6.83
Evaporation Ratio  r1   3.25  105 Kg / s
hfg 2.110 5

4.10 RADIOSITY- MATRIX METHOD FOR RADIATION IN ENCLOSURES

The network method is quite easy to apply for determining the radiation exchange in enclosures
having not more than 3 zones. As the number of zones forming the enclosure increases, the
manipulations involved in the network method becomes enormous and the method is not so
practical. Whereas the radiosity-matrix method is very straight forward and the method
transforms the problem to the solution of an algebraic matrix equation for the unknown
radiosities Ji [i=1,2,3,…….N] once these equations are solved for Ji then the net radiation flux
or the zone temperature at any zone (i) can be immediately determined. This is illustrated
below.
Consider an enclosure made up of N zones.Let Gi be the irradiation at zone i
Let Ji be the Radiosity at zone i .Net radiation heat flux from zone i is given by

qi = Qri / Ai …………………………….(10.34)

500
Radiation leaving Aj and striking Ai is given by

Qj – i = AjFj-i Jj = AiFi-j Jj

Radiation leaving all zones and reaching zone i is therefore given by


N N
∑Qj-I = Ai ∑Fi-j Jj
j=1 j=1

N
Therefore irradiation for zone i = Gi = Qj – I / Ai = ∑Fi-j Jj ………………………….(10.35)
j=1

Now qi = Ji – Gi. Substituting for Gi from Equation (10.35) we get


N
qi = Ji - ∑Fi-j Jj ………………………………….(10.36)
j=1

(Ebi – Ji)
Also qi = ----------------- ………………………………(10.36a)
{(1-εi) / εi}

From Equation (10.36) and Equation (10.36a) it follows that


N
Ji ─ (1 – εi)[Ji – ∑Fi-j Jj
j=1

Ebi = ----------------------------------------- …………..(10.37)


εi
i= 1,2,3,……N

Equations (4.36) and (4.37) provide the fundamental relations for obtaining a system of N
algebraic equations to determine the N unknown radiosities. Once the radiosities are known
the net radiation heat flux qi at any zone (i) can be computed using either Equation 4.36 or
4.36a

The solution depends on the prescribed conditions for each of the zones. Two situations
are of practical interest
1. Temperatures are prescribed for each of the N zones
2. Temperatures are prescribed for some of the zones and the net radiation flux are
prescribed for the remaining zones

i. Temperatures prescribed for all the zones

Consider Equation 10.37


N
J i  1   i   Fi  j J j
Ebi  j=1
 i = 1,2,3 ... N 
i

501
And Ebi = σTi4 is known because Ti’s are prescribed. Therefore the above set of equations can
be solved for unknown radiosities Ji  i  1,2,3 ... N  and knowing Ji the net radiation flux qi
can be determined from Equation 4.36a

Eq 10.37 can be rewritten as follows:


N
( Ji / εi) – {(1 – εi) / εi}∑Fi-jJj = σTi4 ……………………………(10.38)
j=1

Equation (10.38) is of the form


[M] {J} = {T} ……………………………………….(10.39a)

m11 m12 m13 ……….m1N


Where m21 m22 m23 …………….m2N
[M] = - ………(10.39b)
-
-
mN1 mN2 mN3 ………….mNN

J1
J2
{J} = - (10.39c)
-
JN

T1
T2
-
{T} = - (10.39d)
-
TN

The elements mij of matrix [M] can be determined from the following:
δij ─ (1 – εi) Fi-j
mij = ------------------------- , i = 1 to N and j = 1 toN.(10.39e)
εi

Where δij = 1 for i = j and = 0 for i ≠ j. …………………………(10.39f)

δij is known as Kronecker delta.

ii. Temperature prescribed for some zones and net heat flux prescribed for others
In many practical situations, temperatures are prescribed for some of the zones and net
heat fluxes for the remaining zones of an enclosure. In such problems we have to determine
the net heat fluxes for the zones for which temperatures are specified and temperatures for the

502
zones for which the net heat fluxes are prescribed. This can be done using the same equations
(10.36) and (1037) and illustrated below.
Let us assume that temperatures Ti are prescribed for zones i=1, 2, 3 ….k and the net heat
fluxes qi are prescribed for the remaining zones i=k+1, k+2….n
For zones 1 to k since temperatures are prescribed we can use Equation 10.38 that is

N
( Ji / εi) – {(1 – εi) / εi}∑Fi-jJj = σTi4; i= 1 to k……………… (10.40)
j
=1

For zones i = k+1, k+2, … N, with prescribed heat fluxes we can use Eq (4.36) namely
N
qi = Ji - ∑Fi-j Jj ; 1 = k+1, k+2, …… N (10.40a))
j=1

It is more convenient to express Equation (10.40) and (10.40a) in matrix form as follows.

[M] {J} = {S} …………………………………(10.41a)


Where [M] is given by Equation(10.39b) with

δij ─ (1 – εi) Fi-j


mij = ------------------------- , i = 1 to k and j = 1 to k…..(10.41b)
εi

mij = δij ─ Fi-j ; 1 = k+1 to N and j = k+1 to N ……….(10.41c)

{J} is as given by Equation(10.39c) . {S} is given by


σT14
σT24
-
-
{S} = σTk4 ………………………………………..(10.41d)
qk+1
qk+2
-
qN
Ones these equations are solved for unknown radiosities Ji, then the unknown radiation fluxes
can be determined from the equation

εi
qi = --------- [Ebi – Ji] ; i = 1,2,3, ……….k
(1 – εi)
…………….(10.41e)
10.10: ILLUSTRATIVE EXAMPLES ON RADIOSITY MATRIX METHOD

Example 10.28:- Solve Example 10.24 using radiosity matrix method.

Solution: Given: T1 = 800K, Є1 = 0.8; T2 = 600K, Є2 = 0.8; T3 = 300K, J3 = Eb3 = σT34

503
F1-2 = F2-1 = 0.2; F1-1 = F2-2 =0; F1-3 = F2-3 = 0.8

Since J3 is already known, we have to solve only for J1 and J2. Thus the matrix form of equation
for radiosities J1 and J2 can be written as
 m11 m12   J1   T1 
4

m      4
 21 m22   J 2   T2 
 ij  1   i  Fij
mij 
i
where  ij  1 for i = j
 0 for i  j
1  1  0.8   0
 m11   1.25;  T14  5.67 108  8004  23224W / m 2
0.8
0  1  0.8  0.2
m12   0.05;  T24  5.67  108  6004  7348W / m 2
0.8
0  1  0.8  0.2
m21   0.05
0.8
1  1  0.8   0
m22   1.25
0.8
 1.25 0.05  J1  23224 
     
 0.05 1.25   J 2  7348 
i.e,
1.25 J1  0.05 J 2  23224
0.05 J1  1.25 J 2  7348
Solving for J1 and J 2 we get J 2  6632W / m 2 and J1  18845W / m 2
Eb1  J1 23224  18845
 qr1    17516W / m 2
 1  1   1  0.8 
   
 1   0.8 
E  J 2 7348  6632
qr 2  b 2   2864W / m 2
 1 2   1  0.8 
   
 2   0.8 
qr1  qr 2  qr 3  0  qr 3   17516  2864  20380W / m 2
Example 10.29:- Solve example 10.25 by radiosity-matrix method.

Solution: Given A1 = A2 = A3 = 1 m2 ; T1 = 900 K ; T2 = 400 K ; Qr3 = 0 ; ε1 = 0.8 ;

ε2 = 0.8;
This is a case wherein temperature is specified for 2 zones and radiation flux is specified for
the remaining zone.
δij − (1 – εi) Fij
mij = ------------------------ for i = 1,2 and mij = δij − Fij for i = 3
εi

504
0 – (1 – 0.8) x 0.5
Therefore m11 = (1 – 0) / 0.8 = 1.25 ; m12 = ------------------------- = - 0.125 = m21
0.8

(1 – 0) 0 – (1 – 0.8) x 0.5
m22 = ------------ = 1.25 ; m13 = ------------------------ = − 0.125 = m23
0.8 0.8

m31= 0 – F31 = − 0.5 ; m32 = 0 – F32 = − 0.5 ; m33 = 1 – F33 = 1.0

The radiosity matrix equation can now be written as follows:

σ T1 4
[mij]{Ji} = {S} where {S} = σ T24
0
In expanded form the above equation can be written as:

m11 J1 + m12 J2 + m13 J3 = σ T14

m21 J1 + m22 J2 + m23 J3 = σ T24

m31 J1 + m32 J2 + m33 J3 = 0

Substituting the numerical values fo mij , T1, T2 and σ the above three equations can be solved
for J1, J2 and J3.

505
CHAPTER 11: MASS TRANSFER
11.1 INTRODUCTION: Mass transfer can result from several different phenomena. There
is a mass transfer associated with convection, where in, mass is transported from one place to
another in the flow system. This type of mass transfer occurs on a macroscopic level and is
normally treated in the subject of fluid mechanics. When a mixture of gases or liquids is
contained such that, there exists a concentration gradient of one or more of the constituents of
the system, there will be a mass transfer on a microscopic level as the result of diffusion from
regions of high concentration to regions of low concentration.
This chapter deals with some of the simple relations which may be used to calculate mass
diffusion and their relation to heat transfer. However it should be remembered that the
general subject of mass transfer encompasses both mass diffusion on a molecular scale and
the bulk mass transport which may result from a convection process.
Mass diffusion not only occurs on a molecular scale, but also in turbulent systems
where accelerated diffusion rates will occur as a result of the rapid-eddy mixing process.
Mass diffusion may also result from a temperature gradient in a system; this is called
thermal diffusion. Similarly, a concentration gradient can give rise to a temperature gradient
and a consequent heat transfer.

11.2. FICK’S LAW OF DIFFUSION: Consider the system shown in Figure 11.1. A thin

A thin partition separates two gases A and B.


When the partition is removed, the two gases
diffuse through each other until equilibrium
A B is established and the concentration of the
gases is uniform throughout the box. The rate
at which the diffusion rate takes place is given
Figure 11.1: Diffusion of gas A by Fick’s law of diffusion, which states
that
into gas B the mass flux off of a constituent per unit area
is proportional to the concentration gradient.

Therefore mA
------ = − D (∂CA / ∂x) ……………………………(11.1)
A

Where D = constant of proportionality and is called “diffusion coefficient”,


expressed in m2/s ;
mA = mass flux of component A per unit time, kg/s;

506
CA = mass concentration of component A per unit volume, kg/m3.

An expression similar to Equation(11.1) could also be written for the diffusion of the
constituent A in either y or z direction.

Similarity between Equation(11.1) and the Fourier law of heat conduction

(Q /A)x = − k (∂T / ∂x)

and the equation for shear stress between fluid layers,

τ = μ (∂u / ∂y)

can be noticed. The heat conduction equation describes the transport of energy, the viscous-
shear equation describes the transport of momentum across the fluid layers, and the Fick’s
diffusion equation describes the transport of mass.

To understand the physical mechanism of diffusion, consider the imaginary plane


shown by the dashed line in Figure 11.2.The concentration of component A is greater

CA

(mA / A)x = − D(∂CA/∂x)

Figure11.2: Sketch illustrating diffusion dependence on


concentration profile

on the left hand side of this plane than on the right hand side. A higher concentration means
that there are more molecules per unit volume. If the system is a gas or a liquid, the
molecules are moving in a random fashion, and the higher the concentration, more
molecules will cross a given plane per unit time. Thus, on the average, more molecules are
moving from left to right across the plane than in the opposite direction. This results in a net
mass transfer from the region of high concentration to the region of low concentration.

507
In gases the diffusion rates are clearly dependent on the molecular
speed, and consequently we should expect a dependence of the diffusion coefficient on
temperature since temperature indicates the average molecular speed.

11.3. DIFFUSION IN GASES: GILLILAND has proposed a semi-empirical equation for


the diffusion coefficient in gases:

T3/2 __________________
D = 435.7 ---------------------------- √[ (1 / MA) + (1 / MB) ]
p [ VA1/3 + VB1/3 ]

……………………….(11.2)

In the above equation D is in cm 2/s, T is in degrees Kelvin, p is the total system pressure in
pascals, and VA and VB are the molecular volumes of constituents A and B and MA and MB
are the molecular weights of constituents A and B.

Equation (11.2) offers a convenient expression for calculating the diffusion


coefficients for various compounds and mixtures, but should not be used as a substitute for
experimental values of the diffusion coefficients when they are available for a particular
system.
With reference to Figure11.1, it can be stated that the diffusion process is
occurring in two ways at the same time; that is gas A is diffusing into gas B and at the same
time gas B is diffusing into gas A. We , therefore, could refer to the diffusion coefficient for
either of these processes.
While working with Equation (11.1), one can use mass flux per unit area
and mass concentrations, or the equation may be expressed in terms of molal concentrations
and fluxes. There is no general rule to say which type of expression will be most convenient,
and the specific problem under consideration will determine the one to be used. For gases
Fick’s law may be expressed in terms of partial pressures by making use of the ideal-gas
equation of state
The ideal-gas equation of state can be written as

p = ρRT ………………………………(11.3)

The density ρ represents the mass concentration to be used in Fick’s law. The gas _
constant R for a particular gas may expressed in terms of the universal gas constant R and the
molecular weight of the gas: _
R
RA = ------- …………………………….(11.4)
MA

_
Where R = 8314 J/(kg-mol-K)

508
_
or R = 0.08205 m3.atm/(kg-mol – K)

Consequently Fick’s law of diffusion for component A into component B can be written as
follows:
_
CA = ρA = (pAMA) / (R T)
_
Hence ( ∂CA / ∂x) = [MA / (R T)] (dpA/dx)

if isothermal diffusion is considered. Substituting this expression in Equation(11.1) we have

mA _
------ = − DAB [MA / (R T)] (dpA/dx) ………………..(11.5)
A

Similarly we can also write for the diffusion of component B into component A as

mB _
------ = − DBA [MB / (R T)] (dpB/dx) ………………..(11.6)
A
Equation (11.6) also assumes isothermal diffusion process.

Equimolal counter diffusion in gases:- Consider a physical situation as shown in Figure


11.3: NA and NB represent the steady-state molal diffusion rates of components A and B,

Reservoir of A Reservoir of B

NA NB

Δx
1 2

Figure 11.3: Sketch illustrating equimolal diffusion

respectively. In this steady-state situation each molecule of A is replaced by a molecule of B,


and vice versa. The molal diffusion rates are given by

509
mA _
NA = ------ = − DAB [A / (R T)] (dpA/dx)
MA

mB _
NB = ------ = − DBA [A / (R T)] (dpB/dx)
MB

The total pressure of the system p remains constant at steady state, so that

p = pA + pB = constant

where pA and pB are partial pressures of gas A and gas B respectively.

Therefore it follows that (dpA/dx) + (dpB/dx) = 0

or (dpA/dx) = − (dpB/dx) ………………………….(11.7)

Since each molecule of A is replacing a molecule of B, the molal diffusion rates are equal:

− NA = NB

Therefore DAB [A / (R T)] (dpA/dx) = − DBA [A / (R T)] (dpB/dx)

Using Equation (11.7), the above equation can be written as

DAB = DBA = D …………………………………(11.8)

Integration of Equation(11.5) between section 1 and section 2 of Figure 11.3 we get

mA − DMA (pA2 – pA1)


----- = ---_---- ------------------ …………………..(11.9)
A RT Δx

Illustrative example 11.1:- Consider two large vessels, each containing uniform mixtures of
nitrogen and carbon dioxide at 1 atm, and 288.9 K, but at different concentrations. Vessel 1
contains 90 mole percent N2 and 10 mole percent CO2, where as vessel 2 contains 20 mole
percent N2 and 80 mole percent CO2. The two vessels are connected by a duct of 0.1524 m
inside diameter and length of 1.22 m. Determine the rate of transfer of nitrogen between the
two vessels by assuming that steady-state transfer takes place. The mass diffusivity for the N2
– CO2 mixture at 1 atm and 288.9 K is 0.16 x 10 − 4 m 2/s.

Solution: In this mass transfer process, N2 is transferred from vessel 1 containing higher
concentration of N2 to vessel 2 having lower concentration. In the early stages of mass
transfer, the partial pressure of N2 in both vessels is considered to remain constant so that

510
steady-state transfer can be assumed. Then the mass transfer process can be characterized as
an equimolal counter diffusion as described above. Hence the mass flux of N2 can be
determined by using Equation (11.9)

Data:- d = 0.1524 m ; length = Δx = 1.22 m; D = 0.16 x 10 − 4 m2/s;


_
R = 0.08205 m3.atm/(kg-mol – K) ; (pN2)1 = 0.9 x 1 = 0.9 atm; (pN2)2 = 0.2 x 1 = 0.2 atm

The mass transfer rate of N2 is given by

− DMN2 A [(pN2)2 - (pN2)1]


mN2 = ---_---- -----------------------
RT Δx

− 0.16 x 10 − 4 x 28.013 x (π/4)x 0.1524 2 x [0.2 – 0.9]


= --------------------------------------------------------------------- = 2.012 x 10 − 7 kg/s
0.08205 x 288.9 x 1.22

11.4. Diffusion of vapour through a stationary gas (STEFAN Flow):Many engineering


applications such as heat pipes, cooling ponds, and the familiar perspiration involve
condensation, evaporation, and transpiration in the presence of a non condensable gas, and
this process is nothing but diffusion of a vapour through a stationary (stagnant) gas.To
analyse such a process, consider the isothermal evaporation of water from a surface and the
subsequent diffusion through a stagnant layer as shown in Figure 11.4. The free surface of
the water is exposed to air in the tank, as shown. We assume that the system is isothermal
and the total pressure of the system remains constant. We further assume that the system is in
steady state. This requires that there be a slight air movement over the top of the tank to
remove the water vapour that diffuses to that point. Whatever

511
x
Air
2

pw pA

1
Water
p

Figure 11.4: Diffusion of water vapour into air

air movement may be necessary to accomplish this, it is assumed that it does alter the
concentration profiles in the air in the tank.We further assume that air and water vapour
behave as ideal gases.
As the water evaporates, it will diffuse upward through the air, and at
steady state this upward movement must be balanced by a downward diffusion of air so that
the concentration at any x will remain constant. But at the surface of the water there can be
no net mass movement of air downward. Consequently there must be a bulk mass movement
upward with a velocity just large enough to balance the diffusion of air
downward. This bulk mass movement then produces an additional mass flux of water vapour
upward. The diffusion of air downward is given by (see Equation 11.5)

_
mA = − DAB A [MA / (R T)] (dpA/dx) ………………..(11.10)

where A denotes the cross sectional area of the tank. This must be balanced by the bulk-mass
transfer upward. If u is the bulk-mass velocity then
pAMA
mA = − ρAAu = − -------- Au
RT

DAW
or u = -------- (dPA/dx) ……………………….(11.11)
pA

Mass diffusion of water vapour is given by

512
_
(mw)diffusion = − DWA A [MW / (R T)] (dpW/dx) ………(11.12)

and bulk transport of water vapour is given by

pWMW
(mw)bulk = ρWAu = ---_-------- Au
RT

Substituting for u we get


pWMW DAW
(mw)bulk = ρWAu = ---_------------ (dpA/dx) A
RT pA

(mW)total = (mw)diffusion + (mw)bulk

_ pWMWDAW
= − DWA A [MW / (R T)] (dpW/dx) + --------_-------------(dpA/dx) A
R T pA

Also p = pW + pA, p = constant. Hence (dpA/dx) = − (dpW/dx) ; DWA = DAW = D. Hence the
expression for (mW)total reduces to

D A MW
(mW)total = − --_-------- (dpW/dx) = [pA + pW] / pA
RT

D A MW
or (mW)total = − --_-------- (dpW/dx) p/ (p −pW) ………………………………..(11.13)
RT

Equation 11.13 is called Stefan’s law. It may be integrated to give

D p MW A D p MW A
(mW)total = --_---------------- ln [(p – pw2) / (p – pw1) = --_-------------- ln [pA2 / pA1]
R T (x2 – x1) R T (x2 – x1)

………………………………….(11.14)

Illustrative example 11.1:- Estimate the diffusion rate of water from the bottom of a test tube
10 mm in diameter and 15 cm long into dry atmospheric air at 250C.
_
Solution:Data:- d = 0.01 m; (x2 – x1) = 0.15 m; T = 25 + 273 = 298 K;R = 8314.3N-m/kMol-
K ;p = 1atm = 101.32 kPa ; pw1 = saturation pressure at 250C = 3.166 kPa

513
From tables for diffusion of water into air D = 0.256 cm 2/s;
We use Equation(11.14) to calculate the mass flux. The partial pressure of water vapour at
the bottom of the test tube is the saturation temperature of water at 250C and at
the top the partial pressure of water vapour is zero as the air is dry (i.e. pw2 = 0).
Substituting the values in Equation(11.14) we have

(0.256 x 10 − 4) x 18 x (π x 0.005 2)
(mw)total = ------------------------------------------- ln [(101.32 – 0) / (101.32 – 3.166)]
8314.3 x298 x 0.15

= 3.131 x 10 − 10 kg/s

11.5. Steady State Diffusion in Liquids:

Steady state Equimolal Counter Diffusion in Liquids:- The analysis for steady state counter
diffusion in liquids follows the same approach as that for gases.
In terms of number of mole flux of species A ,Fick’s law can be written as

NA = (mA / MA) = − D (∂CA / ∂x) ……………………..(11.15)

The integration of Equation(11.15) from x = x1 to x = x2 with CA varying from CA1 to


CA2,gives

(CA1 − CA2 )
NA = D ---------------- ………………………………….(11.16)
(x2 – x1)

The above equation can also be expressed in terms of mole fractions of species
at locations x1 and x2 as follows.

Mole fraction at x1 = (CA1 / C ) = χA1; Mole fraction at x2 = χA2 = CA2 /C.

Where C = Total molal concentration of the mixture.

Equation(11.16) can be written in terms of mole fraction as

(χA1 − χA2)
NA = DC -------------- kg-mol /(m2-s) …………………(11.17)
(x2 – x1)

Steady State Unidirectional Diffusion in liquids:- The analysis for the determination of the
concentration distribution during steady state unidirectional diffusion in liquids is exactly
same as that for gases except that in the case of liquids the equation is derived in terms of
molal concentrations CA and CB instead of partial pressures.

514
If NA = molal flux of species A per unit area, then

C ( CB2 – CB1) C ( CA1 – CA2)


NA = D ---------------- ------------------ = D ----------------- ------------------ ……(11.18 a)
( x2 – x1 ) CB ln ( x2 – x1 ) CB ln

Where the logarithmic mean concentration CB ln of component B is defined as

( CB2 – CB1)
CB ln = -------------------- ……………………..(11.18 b)
ln (CB2 / CB1)

Also C = CA + CB and CB2 – CB1 = ( CA1 – CA2). Equations (11.18) can also be written in
terms ox mole fractions:

C ( χB2 – χB1) C ( χA1 – χA2)


NA = D ---------------- ------------------ = D ----------------- ------------------ ……(11.19 a)
( x2 – x1 ) χB ln ( x2 – x1 ) χB ln

Where the logarithmic mean mole fraction χB ln of component B is defined as

( χB2 – χB1)
χB ln = -------------------- ……………………..(11.19 b)
ln (χB2 / χB1)

In Eqs.(11.18) and (11.19), the units of various quantities may be taken as

Ci = molal concentration, i = A, or B, kg-mol/m3

χi = Ci / C = mole fraction of component i

C = molal concentration of the mixture, kg-mol/m3

D = mass diffusivity, m2/s

NA = molal flux of species A, kg-mol /(m2-s)

X = distance,m

11.6. Solution to tutorial problems:

11.1 Solution: Data:- p = 1 atm; T = 273 K; (CA)1 = 0.8;

515
(Cso2)1 = 0.20; (CA)2 = 0.30; (Cso2)1 = 0.70; d = 0.1 m ; Δx = 1.8 m

Now pA1 / p = (CA)1. Hence pA1 = 0.8 x 1 = 0.8 atm. Similarly pA2 = 0.3 x 1 = 0.3 atm
_
R = 0.08205 m3 – atm / (kg mol – K); MA = 28.86; D = 0.122

− DMA A [pA2 - pA1]


mA = --- _---- -----------------------
RT Δx

0.122 x 10 − 4 x 28.86 x (π x 0.052) x [ 0.8 – 0.3]


= -------------------------------------------------------------
0.08205 x 273 x 1.8

= 3.43 x 10 − 8 kg / s.

11.2. Solution: Data:- T = 298 K; x2 – x1 = 3 m; D = 0.0962 cm 2/s; pA1 = 0.01 atm;


A = 1 m2; p = 1 atm.MB = Molecular weight of benzene = 78

D p MB A
mB = _-------------- ln [pA2 / pA1]
R T (x2 – x1)

0.0962 x 10 − 4x1.0 x 78 x 1.0


= ---------------------------------------------- ln [(1.0-0) /(1.0 – 0.01)]
0.08205 x 298 x 3

= 1.03 x 10 − 7 kg / (s – m2)

516
517

Potrebbero piacerti anche